You are on page 1of 146

INSURANCE LAW

I.

GENRAL CONCEPTS
A.

1.

INSURANCE
Definition Sec 2 (1), I.C.

Sec. 2(1). A "contract of insurance" is an agreement whereby one undertakes for a consideration to
indemnify another against loss, damage or liability arising from an unknown or contingent event.
2. Elements

a.

The insured has an insurable interest Sec 12-14, I.C.

Sec. 12. The interest of a beneficiary in a life insurance policy shall be forfeited when the beneficiary is
the principal, accomplice, or accessory in willfully bringing about the death of the insured; in which
event, the nearest relative of the insured shall receive the proceeds of said insurance if not otherwise
disqualified.
Sec. 13. Every interest in property, whether real or personal, or any relation thereto, or liability in
respect thereof, of such nature that a contemplated peril might directly damnify the insured, is an
insurable interest.
Sec. 14. An insurable interest in property may consist in:
(a) An existing interest;

(b)

An inchoate interest founded on an existing interest; or


(c) An expectancy, coupled with an existing interest in that out of which the expectancy arises.

b.

The insured is subject to a risk of loss by the happening of the designated


peril Sec 3, I.C.

Sec. 3. Any contingent or unknown event, whether past or future, which may damnify a person having
an insurable interest, or create a liability against him, may be insured against, subject to the provisions
of this chapter.
The consent of the husband is not necessary for the validity of an insurance policy taken out by
a married woman on her life or that of her children.
Any minor of the age of eighteen years or more, may, notwithstanding such minority, contract
for life, health and accident insurance, with any insurance company duly authorized to do business in
the Philippines, provided the insurance is taken on his own life and the beneficiary appointed is the
minor's estate or the minor's father, mother, husband, wife, child, brother or sister.
The married woman or the minor herein allowed to take out an insurance policy may exercise all
the rights and privileges of an owner under a policy.
All rights, title and interest in the policy of insurance taken out by an original owner on the life
or health of a minor shall automatically vest in the minor upon the death of the original owner, unless
otherwise provided for in the policy.

c.

The insurer assumes the risk Sec 2, I.C.

Sec. 2. Whenever used in this Code, the following terms shall have the respective meanings hereinafter
set forth or indicated, unless the context otherwise requires:
(1) A "contract of insurance" is an agreement whereby one undertakes for a consideration to indemnify
another against loss, damage or liability arising from an unknown or contingent event.
A contract of suretyship shall be deemed to be an insurance contract, within the meaning of this
Code, only if made by a surety who or which, as such, is doing an insurance business as hereinafter
provided.

By: Elaine Marie G. Laceda

INSURANCE LAW

(2) The term "doing an insurance business" or "transacting an insurance business", within the meaning
of this Code, shall include:
(a) making or proposing to make, as insurer, any insurance contract;

(b)

making or proposing to make, as surety, any contract of suretyship as a vocation and not as
merely incidental to any other legitimate business or activity of the surety;
(c) doing any kind of business, including a reinsurance business, specifically recognized as
constituting the doing of an insurance business within the meaning of this Code;
(d) doing or proposing to do any business in substance equivalent to any of the foregoing in a
manner designed to evade the provisions of this Code.
In the application of the provisions of this Code the fact that no profit is derived from the making of
insurance contracts, agreements or transactions or that no separate or direct consideration is received
therefor, shall not be deemed conclusive to show that the making thereof does not constitute the doing
or transacting of an insurance business.
(3) As used in this code, the term "Commissioner" means the "Insurance Commissioner".
d. Such assumption of risk is part of a general scheme to distribute actual losses
among a large group of persons bearing a similar risk; and

e.

In consideration of the insurers promise, the insured pays a premium. Sec


77, I.C.

Sec. 77. An insurer is entitled to payment of the premium as soon as the thing insured is exposed to
the peril insured against. Notwithstanding any agreement to the contrary, no policy or contract of
insurance issued by an insurance company is valid and binding unless and until the premium thereof has
been paid, except in the case of a life or an industrial life policy whenever the grace period provision
applies.
PhilamCare Health Systems, Inc. vs. Court of Appeals
G.R. No. 125678 March 18, 2002
Ernani Trinos, deceased husband of respondent Julita Trinos, applied for a health care coverage with
petitioner Philamcare Health Systems, Inc. In the standard application form, he stated that he was
never treated for high blood pressure, heart trouble, diabetes, cancer, liver disease, asthma or peptic
ulcer.
Under the agreement, respondents husband was entitled to avail of hospitalization benefits, whether
ordinary or emergency, listed therein. He was also entitled to avail of "out-patient benefits" such as
annual physical examinations, preventive health care and other out-patient services.
During the period of his coverage, Ernani suffered a heart attack and was confined at the Manila Medical
Center (MMC) for one month. While her husband was in the hospital, respondent tried to claim the
benefits under the health care agreement. However, petitioner denied her claim saying that the Health
Care Agreement was void. According to petitioner, there was a concealment regarding Ernanis medical
history. Doctors at the MMC allegedly discovered at the time of Ernanis confinement that he was
hypertensive, diabetic and asthmatic, contrary to his answer in the application form.
After Ernanis death, Julita Trinos filed an action for damages where she asked for reimbursement of her
expenses plus moral damages and attorneys fees.
RTC rendered judgment in favor of the plaintiff ordering defendants to pay and reimburse the medical
and hospital coverage of the late Ernani Trinos, and to pay moral and exemplary damages plus
attorneys fees and cost of suit. CA affirmed.
Held: SC Affirmed.
An insurance contract exists where the following elements concur:

By: Elaine Marie G. Laceda

INSURANCE LAW

1.
2.
3.
4.
5.

The insured has an insurable interest;


The insured is subject to a risk of loss by the happening of the designated peril;
The insurer assumes the risk;
Such assumption of risk is part of a general scheme to distribute actual losses among a large
group of persons bearing a similar risk; and
In consideration of the insurers promise, the insured pays a premium.

Section 10 provides that: Every person has an insurable interest in the life and health
1. of himself, of his spouse and of his children;
2. of any person on whom he depends wholly or in part for education or support, or in whom he
has a pecuniary interest;
3. of any person under a legal obligation to him for the payment of money, respecting property or
service, of which death or illness might delay or prevent the performance; and
4. of any person upon whose life any estate or interest vested in him depends.
In the case at bar, the insurable interest of respondents husband in obtaining the health care
agreement was his own health. The health care agreement was in the nature of non-life insurance,
which is primarily a contract of indemnity. Once the member incurs hospital, medical or any other
expense arising from sickness, injury or other stipulated contingent, the health care provider must pay
for the same to the extent agreed upon under the contract.
Having assumed a responsibility under the agreement, petitioner is bound to answer the same to the
extent agreed upon. In the end, the liability of the health care provider attaches once the member is
hospitalized for the disease or injury covered by the agreement or whenever he avails of the covered
benefits which he has prepaid.
Under Section 27 of the Insurance Code, "a concealment entitles the injured party to rescind a contract
of insurance." However, the right to rescind should be exercised previous to the commencement of an
action on the contract. In this case, no rescission was made. Besides, the cancellation of health care
agreements as in insurance policies require the concurrence of certain conditions, none of which was
fulfilled in this case.
1. Prior notice of cancellation to insured;
2. Notice must be based on the occurrence after effective date of the policy of one or more of the
grounds mentioned;
3. Must be in writing, mailed or delivered to the insured at the address shown in the policy;
4. Must state the grounds relied upon provided in Section 64 of the Insurance Code and upon
request of insured, to furnish facts on which cancellation is based.
The defendant Philamcare Health Systems Inc. had twelve months from the date of issuance of the
Agreement within which to contest the membership of the patient if he had previous ailment of asthma,
and six months from the issuance of the agreement if the patient was sick of diabetes or hypertension.
The periods having expired, the defense of concealment or misrepresentation no longer lie.
Lastly, petitioner alleges that respondent was not the legal wife of the deceased member considering
that at the time of their marriage, the deceased was previously married to another woman who was still
alive. However, the health care agreement is in the nature of a contract of indemnity. Hence, payment
should be made to the party who incurred the expenses.

3. Characteristics

a.

Aleatory (but not wagering) Sec 2010, Civil Code; Sec 25, I.C.

Art. 2010. By an aleatory contract, one of the parties or both reciprocally bind themselves to give or to
do something in consideration of what the other shall give or do upon the happening of an event which
is uncertain, or which is to occur at an indeterminate time. (1790)

By: Elaine Marie G. Laceda

INSURANCE LAW

Sec. 25. Every stipulation in a policy of insurance for the payment of loss whether the person insured
has or has not any interest in the property insured, or that the policy shall be received as proof of such
interest, and every policy executed by way of gaming or wagering, is void.

b.

Indemnity (except life and accident insurance where the result is death, and
valued policies) Sec 17, I.C.

Sec. 17. The measure of an insurable interest in property is the extent to which the insured might be
damnified by loss or injury thereof.
c. Personal
d. Unilateral (executed as to insured and executory as to insurer upon payment
of premiums)
e. Conditional (distinguish between property insurance, where loss may or may
not occur and may be total or partial, and life insurance, death will occur so
that time of happening is the contingent element)

4.

Perfection Art 1318-1319, Civil Code; Sec 77 and 226, I.C.

Art. 1318. There is no contract unless the following requisites concur:


(1) Consent of the contracting parties;
(2) Object certain which is the subject matter of the contract;
(3) Cause of the obligation which is established. (1261)
Art. 1319. Consent is manifested by the meeting of the offer and the acceptance upon the thing and
the cause which are to constitute the contract. The offer must be certain and the acceptance absolute. A
qualified acceptance constitutes a counter-offer.
Acceptance made by letter or telegram does not bind the offerer except from the time it came to
his knowledge. The contract, in such a case, is presumed to have been entered into in the place where
the offer was made. (1262a)
Sec. 77. An insurer is entitled to payment of the premium as soon as the thing insured is exposed to
the peril insured against. Notwithstanding any agreement to the contrary, no policy or contract of
insurance issued by an insurance company is valid and binding unless and until the premium thereof has
been paid, except in the case of a life or an industrial life policy whenever the grace period provision
applies.
Sec. 226. No policy, certificate or contract of insurance shall be issued or delivered within the
Philippines unless in the form previously approved by the Commissioner, and no application form shall
be used with, and no rider, clause, warranty or endorsement shall be attached to, printed or stamped
upon such policy, certificate or contract unless the form of such application, rider, clause, warranty or
endorsement has been approved by the Commissioner.
Enriquez vs. Sun Life Assurance Company of Canada
G.R. No. L-15895 November 29, 1920
On September 24, 1917, Joaquin Herrer applied with the Sun Life Assurance Company of Canada for a
life annuity and paid the sum of P6,000.
The application was immediately forwarded to the head office of the company at Montreal, Canada. On
November 26, 1917, the head office gave notice of acceptance by cable to Manila. On December 4,
1917, the policy was issued at Montreal.
On December 18, 1917, Atty. Aurelio A. Torres wrote the Manila office stating that Herrer desired to
withdraw his application. The following day the local office replied stating that the policy had been
issued, and called attention to the notification of November 26, 1917. This letter was received by Mr.
Torres on the morning of December 21, 1917. Mr. Herrer died on December 20, 1917.

By: Elaine Marie G. Laceda

INSURANCE LAW

Issue: Whether the contract for a life annuity was perfected.


Held: No.
According to the provisional receipt, three things had to be accomplished by the insurance company
before there was a contract: (1) There had to be a medical examination of the applicant; (2) there had
to be approval of the application by the head office of the company; and (3) this approval had in some
way to be communicated by the company to the applicant.
Likewise, the second paragraph of Article 1262 of the Civil Code, the law applicable to the case, provides
that an acceptance made by letter shall not bind the person making the offer except from the time it
came to his knowledge.
The contract for a life annuity in the case at bar was not perfected because it has not been proved
satisfactorily that the acceptance of the application ever came to the knowledge of the applicant.
A letter will not be presumed to have been received by the addressee unless it is shown that it was
deposited in the post-office, properly addressed and stamped.
Eternal Gardens Memorial Park Corporation vs. Philippine American Life Insurance
Company
G.R. No. 166245 April 9, 2008
Philamlife entered into an agreement denominated as Creditor Group Life Policy with petitioner Eternal.
Under the policy, the clients of Eternal who purchased burial lots from it on installment basis would be
insured by Philamlife. The amount of insurance coverage depended upon the existing balance of the
purchased burial lots.
Eternal was required under the policy to submit to Philamlife a list of all new lot purchasers, together
with a copy of the application of each purchaser, and the amounts of the respective unpaid balances of
all insured lot purchasers. One of those included in the list as "new business" was a certain John
Chuang. His balance of payments was PhP 100,000. On August 2, 1984, Chuang died.
Eternal sent a letter to Philamlife, which served as an insurance claim for Chuangs death. In reply,
Philamlife required Eternal to submit the following documents relative to its insurance claim for Chuangs
death: (1) Certificate of Claimant (with form attached); (2) Assureds Certificate (with form attached);
(3) Application for Insurance accomplished and signed by the insured, Chuang, while still living; and (4)
Statement of Account showing the unpaid balance of Chuang before his death. Eternal transmitted the
required documents received by Philamlife on November 15, 1984.
After more than a year, Philamlife had not furnished Eternal with any reply to the latters insurance
claim. This prompted Eternal to demand from Philamlife the payment of the claim for PhP 100,000 on
April 25, 1986.
Philamlife denied Eternals insurance claim. It claimed that since no application had been submitted by
the Insured/Assured, prior to his death, for its approval but was submitted instead on November 15,
1984, after his death, Mr. John Uy Chuang was not covered under the Policy.
Consequently, Eternal filed a case for a sum of money against Philamlife. RTC decided in favor of Eternal
ordering PHILAMLIFE, to pay the sum of P100,000.00, representing the proceeds of the Policy of John
Uy Chuang, plus legal rate of interest, until fully paid.
The RTC found that Eternal submitted Chuangs application for insurance which he accomplished before
his death. It further ruled that due to Philamlifes inaction from the submission of the requirements of
the group insurance on December 29, 1982 to Chuangs death on August 2, 1984, as well as Philamlifes
acceptance of the premiums during the same period, Philamlife was deemed to have approved Chuangs
application. The RTC said that since the contract is a group life insurance, once proof of death is
submitted, payment must follow.

By: Elaine Marie G. Laceda

INSURANCE LAW

CA reversed RTC ruling. It ruled that the non-accomplishment of the submitted application form violated
Section 26 of the Insurance Code. Thus, the CA concluded, there being no application form, Chuang was
not covered by Philamlifes insurance.
Issue: Whether Philamlife assumed the risk of loss without approving the application
Held: Yes.
The fact of the matter is, the letter dated December 29, 1982, which Philamlife stamped as received,
states that the insurance forms for the attached list of burial lot buyers were attached to the letter. Such
stamp of receipt has the effect of acknowledging receipt of the letter together with the attachments.
Such receipt is an admission by Philamlife against its own interest. The burden of evidence has shifted to
Philamlife, which must prove that the letter did not contain Chuangs insurance application. However,
Philamlife failed to do so; thus, Philamlife is deemed to have received Chuangs insurance application.
Furthermore, an examination of the provision on Effective Date of Benefit under the policy would show
ambiguity between its two sentences. The first sentence appears to state that the insurance coverage of
the clients of Eternal already became effective upon contracting a loan with Eternal while the second
sentence appears to require Philamlife to approve the insurance contract before the same can become
effective.
It must be remembered that an insurance contract is a contract of adhesion which must be construed
liberally in favor of the insured and strictly against the insurer in order to safeguard the latters interest.
Clearly, the vague contractual provision must be construed in favor of the insured and in favor of the
effectivity of the insurance contract.
The seemingly conflicting provisions must be harmonized to mean that upon a partys purchase of a
memorial lot on installment from Eternal, an insurance contract covering the lot purchaser is created
and the same is effective, valid, and binding until terminated by Philamlife by disapproving the insurance
application. The second sentence of the policy on the Effective Date of Benefit is in the nature of a
resolutory condition which would lead to the cessation of the insurance contract.
The mere inaction of the insurer on the insurance application must not work to prejudice the insured; it
cannot be interpreted as a termination of the insurance contract. The termination of the insurance
contract by the insurer must be explicit and unambiguous.

5. Kinds of Insurance
B.

1.

INSURANCE DISTINGUISHED FROM OTHER CONTRACTS


Suretyship Art 2047, Civil Code; Sec 2(1), par. 2, I.C.

Art. 2047. By guaranty a person, called the guarantor, binds himself to the creditor to fulfill the
obligation of the principal debtor in case the latter should fail to do so.
If a person binds himself solidarily with the principal debtor, the provisions of Section 4, Chapter
3, Title I of this Book shall be observed. In such case the contract is called a suretyship. (1822a)
Section 2 (1), par. 2 -- A contract of suretyship shall be deemed to be an insurance contract, within
the meaning of this Code, only if made by a surety who or which, as such, is doing an insurance
business as hereinafter provided.

2.

Pre-Need Plans Sec 3.9, SRC

Section 3.9. Pre-Need Plans are contracts which provide for the performance of future services or the
payment of future monetary considerations at the time of actual need, for which planholders pay in cash

By: Elaine Marie G. Laceda

INSURANCE LAW

or installment at stated prices, with or without interest or insurance coverage and includes life, pension,
education, interment, and other plans which the Commission may from time to time approve.

3.

Variable Contracts Sec 232, I.C.

Sec. 232. (1) No insurance company authorized to transact business in the Philippines shall issue,
deliver, sell or use any variable contract in the Philippines, unless and until such company shall have
satisfied the Commissioner that its financial and general condition and its methods of operations,
including the issue and sale of variable contracts, are not and will not be hazardous to the public or to its
policy and contract owners. No foreign insurance company shall be authorized to issue, deliver or sell
any variable contract in the Philippines, unless it is likewise authorized to do so by the laws of its
domicile.
(2) The term "variable contract" shall mean any policy or contract on either a group or on an
individual basis issued by an insurance company providing for benefits or other contractual payments or
values thereunder to vary so as to reflect investment results of any segregated portfolio of investments
or of a designated separate account in which amounts received in connection with such contracts shall
have been placed and accounted for separately and apart from other investments and accounts. This
contract may also provide benefits or values incidental thereto payable in fixed or variable amounts, or
both. It shall not be deemed to be a "security" or "securities" as defined in The Securities Act, as
amended, or in the The Investment Company Act, as amended, nor subject to regulation under said
Acts.
(3) In determining the qualifications of a company requesting authority to issue, deliver, sell or
use variable contracts, the Commissioner shall always consider the following: (a) the history, financial
and general condition of the company: Provided, That such company, if a foreign company, must have
deposited with the Commissioner for the benefit and security of its variable contract owners in the
Philippines, securities satisfactory to the Commissioner consisting of bonds of the Government of the
Philippines or its instrumentalities with an actual market value of two million pesos; (b) the character,
responsibility and fitness of the officers and directors of the company; and (c) the law and regulation
under which the company is authorized in the state of domicile to issue such contracts.
(4) If after notice and hearing, the Commissioner shall find that the company is qualified to
issue, deliver, sell or use variable contracts in accordance with this Code and the regulations and rules
issued thereunder, the corresponding order of authorization shall be issued. Any decision or order
denying authority to issue, deliver, sell or use variable contracts shall clearly and distinctly state the
reasons and grounds on which it is based.
C.

1.

INSURANCE BUSINESS
Doing an Insurance Business Sec 2(2), I.C.

Sec. 2(2) The term "doing an insurance business" or "transacting an insurance business", within the
meaning of this Code, shall include:
(a) making or proposing to make, as insurer, any insurance contract;

(b)

making or proposing to make, as surety, any contract of suretyship as a vocation and not as
merely incidental to any other legitimate business or activity of the surety;
(c) doing any kind of business, including a reinsurance business, specifically recognized as
constituting the doing of an insurance business within the meaning of this Code;
(d) doing or proposing to do any business in substance equivalent to any of the foregoing in a
manner designed to evade the provisions of this Code.
In the application of the provisions of this Code the fact that no profit is derived from the making of
insurance contracts, agreements or transactions or that no separate or direct consideration is received
therefor, shall not be deemed conclusive to show that the making thereof does not constitute the doing
or transacting of an insurance business.

Phil. American Life Insurance Company vs. Ansaldo


G.R. No. 76452 July 26, 1994

By: Elaine Marie G. Laceda

INSURANCE LAW

Ramon M. Paterno, Jr. filed a letter-complaint to respondent Insurance Commissioner, alleging certain
problems encountered by agents, supervisors, managers and public consumers of Philamlife as a result
of certain practices by said company.
Respondent Commissioner requested petitioner Rodrigo de los Reyes, in his capacity as Philamlife's
president, to comment on respondent Paterno's letter. Petitioner De los Reyes suggested that private
respondent "submit some sort of a 'bill of particulars' to enable him to prepare an intelligent reply.
However, private respondent maintained that his letter-complaint was sufficient in form and substance,
and requested that a hearing thereon be conducted.
Private respondent later submitted a letter of specification to respondent Commissioner reiterating his
letter-complaint and praying that the provisions on charges and fees stated in the Contract of Agency
executed between Philamlife and its agents, as well as the implementing provisions as published in the
agents' handbook, agency bulletins and circulars, be declared as null and void. He also asked that the
amounts of such charges and fees already deducted and collected by Philamlife in connection therewith
be reimbursed to the agents, with interest at the prevailing rate reckoned from the date when they were
deducted.
Petitioner De los Reyes submitted an Answer stating inter alia that: (1) Private respondent's letter does
not contain any of the particular information which Philamlife was seeking from him and which he
promised to submit; and (2) That since the Commission's quasi-judicial power was being invoked with
regard to the complaint, private respondent must file a verified formal complaint before any further
proceedings.
On October 1, private respondent executed an affidavit, verifying his letters of April 17, 1986, and July
31, 1986.
Manuel Ortega, Philamlife's Senior Assistant Vice-President and Executive Assistant to the President,
asked that respondent Commission first rule on the questions of the jurisdiction of the Insurance
Commissioner over the subject matter of the letters-complaint and the legal standing of private
respondent.
Despite the foregoing, respondent Commissioner set the case for hearing and sent notice to the officers
of Philamlife. Ortega filed a MTQ the subpoena alleging that the Subpoena/Notice has no legal basis and
is premature and the Insurance Commission has no jurisdiction over the subject matter or nature of the
action and over the parties involved. MTQ denied.
Issue: Whether or not the resolution of the legality of the Contract of Agency falls within the jurisdiction
of the Insurance Commissioner.
Held: No.
A plain reading of Sec 414 and 415 of the Insurance Code show that the Insurance Commissioner has
the authority to regulate the business of insurance, which is defined as follows:
(a) making or proposing to make, as insurer, any insurance contract;
(b) making, or proposing to make, as surety, any contract of suretyship as a vocation and not as
merely incidental to any other legitimate business or activity of the surety;
(c) doing any kind of business, including a reinsurance business, specifically recognized as
constituting the doing of an insurance business within the meaning of this Code;
(d) doing or proposing to do any business in substance equivalent to any of the foregoing in a
manner designed to evade the provisions of this Code. (Insurance Code, Sec. 2[2])
Since the contract of agency entered into between Philamlife and its agents is not included within the
meaning of an insurance business, Section 2 of the Insurance Code cannot be invoked to give
jurisdiction over the same to the Insurance Commissioner.
Likewise, a reading of Section 416 of the Code shows that the quasi-judicial power of the Insurance
Commissioner is limited by law "to claims and complaints involving any loss, damage or liability for

By: Elaine Marie G. Laceda

INSURANCE LAW

which an insurer may be answerable under any kind of policy or contract of insurance, . . ." Hence, this
power does not cover the relationship affecting the insurance company and its agents but is limited to
adjudicating claims and complaints filed by the insured against the insurance company.
The Insurance Code does not have provisions governing the relations between insurance companies and
their agents. It follows that the Insurance Commissioner cannot, in the exercise of its quasi-judicial
powers, assume jurisdiction over controversies between the insurance companies and their agents.
Registered representatives who work on commission basis is governed by the Contract of Agency and
the provisions of the Civil Code on Agency, and disputes involving these agents are cognizable by the
regular courts.

2. Mutual Insurance Companies


White Gold Marine Services, Inc. vs. Pioneer Insurance and Surety Corporation
G.R. No. 154514 July 28, 2005
White Gold procured a protection and indemnity coverage for its vessels from Steamship Mutual through
Pioneer Insurance. Subsequently, White Gold was issued a Certificate of Entry and Acceptance. Pioneer
also issued receipts evidencing payments for the coverage. When White Gold failed to fully pay its
accounts, Steamship Mutual refused to renew the coverage.
Steamship Mutual thereafter filed a case against White Gold for collection of sum of money to recover
the latters unpaid balance. White Gold on the other hand, filed a complaint before the Insurance
Commission claiming that Steamship Mutual violated Sections 186 and 187 of the Insurance Code, while
Pioneer violated Sections 299, 300 and 301 in relation to Sections 302 and 303, thereof.
The Insurance Commission dismissed the complaint. It said that there was no need for Steamship
Mutual to secure a license because it was not engaged in the insurance business. It explained that
Steamship Mutual was a Protection and Indemnity Club (P & I Club). Likewise, Pioneer need not obtain
another license as insurance agent and/or a broker for Steamship Mutual because Steamship Mutual was
not engaged in the insurance business. Moreover, Pioneer was already licensed, hence, a separate
license solely as agent/broker of Steamship Mutual was already superfluous. CA affirmed.
Issues: Is Steamship Mutual, a P & I Club, engaged in the insurance business in the Philippines?
Held: Yes.
Section 2(2) of the Insurance Code enumerates what constitutes doing an insurance business or
transacting an insurance business. These are:
(a) making or proposing to make, as insurer, any insurance contract;
(b) making, or proposing to make, as surety, any contract of suretyship as a vocation and not as
merely incidental to any other legitimate business or activity of the surety;
(c) doing any kind of business, including a reinsurance business, specifically recognized as
constituting the doing of an insurance business within the meaning of this Code;
(d) doing or proposing to do any business in substance equivalent to any of the foregoing in a
manner designed to evade the provisions of this Code.
The same provision also provides, the fact that no profit is derived from the making of insurance
contracts, agreements or transactions, or that no separate or direct consideration is received therefor,
shall not preclude the existence of an insurance business.
The test to determine if a contract is an insurance contract or not, depends on the nature of the
promise, the act required to be performed, and the exact nature of the agreement in the light of the
occurrence, contingency, or circumstances under which the performance becomes requisite. It is not by
what it is called.

By: Elaine Marie G. Laceda

INSURANCE LAW

Basically, an insurance contract is a contract of indemnity. In it, one undertakes for a consideration to
indemnify another against loss, damage or liability arising from an unknown or contingent event.
A P & I Club is a form of insurance against third party liability, where the third party is anyone other
than the P & I Club and the members. By definition then, Steamship Mutual as a P & I Club is a mutual
insurance association engaged in the marine insurance business.
The records reveal Steamship Mutual is doing business in the country albeit without the requisite
certificate of authority mandated by Section 187 of the Insurance Code. It maintains a resident agent in
the Philippines to solicit insurance and to collect payments in its behalf. Thus, to continue doing
business here, Steamship Mutual or through its agent Pioneer, must secure a license from the Insurance
Commission.
Issue: Does Pioneer need a license as an insurance agent/broker for Steamship Mutual?
Held: Yes.
Although Pioneer is already licensed as an insurance company, it needs a separate license to act as
insurance agent for Steamship Mutual. Section 299 of the Insurance Code clearly states:
No person shall act as an insurance agent or as an insurance broker in the solicitation or
procurement of applications for insurance, or receive for services in obtaining insurance, any
commission or other compensation from any insurance company doing business in the
Philippines or any agent thereof, without first procuring a license so to act from the
Commissioner, which must be renewed annually on the first day of January, or within six
months thereafter.
D.

LAWS GOVERNING INSURANCE

1. Insurance Code
2. Civil Code Art 2011, Civil Code
Art. 2011. The contract of insurance is governed by special laws. Matters not expressly provided for in
such special laws shall be regulated by this Code. (n)

Insular Life Assurance Company, Ltd. vs. Ebrado


G.R. No. L-44059 October 28, 1977
Buenaventura Ebrado was issued by The Life Assurance Co., Ltd., Policy No. 009929 on a whole-life for
P5,882.00 with a, rider for Accidental Death for the same amount. He designated Carponia T. Ebrado as
the revocable beneficiary in his policy.
Buenaventura C. Ebrado died as a result of an accident when he was hit by a failing branch of a tree.
Thus, Carponia Ebrado filed claim with the Insular Life Assurance Co. which was contested by Pascuala
Ebrado who also filed claim for the proceeds of said policy. The latter asserts that she is the one entitled
to the insurance proceeds, not the common-law wife.
In doubt as to whom the insurance proceeds shall be paid, the insurer, The Insular Life Assurance Co.,
Ltd. commenced an action for Interpleader before the CFI.
CFI declared Carponia T. Ebrado disqualified from becoming beneficiary of the insured and directed the
payment of the insurance proceeds to the estate of the deceased insured.
Issue: Can a common-law wife named as beneficiary in the life insurance policy of a legally married man
claim the proceeds thereof in case of death of the latter?
Held: No.

By: Elaine Marie G. Laceda

10

INSURANCE LAW

In essence, a life insurance policy is no different from a civil donation insofar as the beneficiary is
concerned. Both are founded upon the same consideration: liberality. A beneficiary is like a donee,
because from the premiums of the policy which the insured pays out of liberality, the beneficiary will
receive the proceeds or profits of said insurance. As a consequence, the proscription in Article 739 of the
new Civil Code: Those made between persons who were guilty of adultery or concubinage at the time of
donation, should equally operate in life insurance contracts.
No criminal conviction for the offense is a condition precedent before the disabilities mentioned in Article
739 may effectuate. The guilt of the donee may be proved by preponderance of evidence in an action for
declaration of nullity of the donation.
The requisite proof of common-law relationship between the insured and the beneficiary has been
conveniently supplied by the stipulations between the parties in the pre-trial conference of the case.
These stipulations are nothing less than judicial admissions which, as a consequence, no longer require
proof and cannot be contradicted.

a.

Revocation of Irrevocable beneficiaries in terminated marriages Art 43(4),


50 and 64, Family Code

Art. 43(4). The innocent spouse may revoke the designation of the other spouse who acted in bad faith
as beneficiary in any insurance policy, even if such designation be stipulated as irrevocable; and
Art. 50. The effects provided for by paragraphs (2), (3), (4) and (5) of Article 43 and by Article 44 shall
also apply in the proper cases to marriages which are declared ab initio or annulled by final judgment
under Articles 40 and 45.
The final judgment in such cases shall provide for the liquidation, partition and distribution of
the properties of the spouses, the custody and support of the common children, and the delivery of third
presumptive legitimes, unless such matters had been adjudicated in previous judicial proceedings.
All creditors of the spouses as well as of the absolute community or the conjugal partnership
shall be notified of the proceedings for liquidation.
In the partition, the conjugal dwelling and the lot on which it is situated, shall be adjudicated in
accordance with the provisions of Articles 102 and 129.
Art. 64. After the finality of the decree of legal separation, the innocent spouse may revoke the
donations made by him or by her in favor of the offending spouse, as well as the designation of the
latter as beneficiary in any insurance policy, even if such designation be stipulated as irrevocable. The
revocation of the donations shall be recorded in the registries of property in the places where the
properties are located. Alienations, liens and encumbrances registered in good faith before the recording
of the complaint for revocation in the registries of property shall be respected. The revocation of or
change in the designation of the insurance beneficiary shall take effect upon written notification thereof
to the insured.
The action to revoke the donation under this Article must be brought within five years from the
time the decree of legal separation become final.

b.

Void donations Art 739, 2012, Civil Code

Art. 739. The following donations shall be void:


(1) Those made between persons who were guilty of adultery or concubinage at the time of the
donation;
(2) Those made between persons found guilty of the same criminal offense, in consideration
thereof;
(3) Those made to a public officer or his wife, descedants and ascendants, by reason of his
office.
In the case referred to in No. 1, the action for declaration of nullity may be brought by the
spouse of the donor or donee; and the guilt of the donor and donee may be proved by preponderance of
evidence in the same action. (n)

By: Elaine Marie G. Laceda

11

INSURANCE LAW

Art. 2012. Any person who is forbidden from receiving any donation under Article 739 cannot be named
beneficiary of a life insurance policy by the person who cannot make any donation to him, according to
said article. (n)

c.

Life annuity contracts Art 2021-2027, Civil Code

Art. 2021. The aleatory contract of life annuity binds the debtor to pay an annual pension or income
during the life of one or more determinate persons in consideration of a capital consisting of money or
other property, whose ownership is transferred to him at once with the burden of the income. (1802a)
Art. 2022. The annuity may be constituted upon the life of the person who gives the capital, upon that
of a third person, or upon the lives of various persons, all of whom must be living at the time the
annuity is established.
It may also be constituted in favor of the person or persons upon whose life or lives the contract
is entered into, or in favor of another or other persons. (1803)
Art. 2023. Life annuity shall be void if constituted upon the life of a person who was already dead at
the time the contract was entered into, or who was at that time suffering from an illness which caused
his death within twenty days following said date. (1804)
Art. 2024. The lack of payment of the income due does not authorize the recipient of the life annuity to
demand the reimbursement of the capital or to retake possession of the property alienated, unless there
is a stipulation to the contrary; he shall have only a right judicially to claim the payment of the income
in arrears and to require a security for the future income, unless there is a stipulation to the contrary.
(1805a)
Art. 2025. The income corresponding to the year in which the person enjoying it dies shall be paid in
proportion to the days during which he lived; if the income should be paid by installments in advance,
the whole amount of the installment which began to run during his life shall be paid. (1806)
Art. 2026. He who constitutes an annuity by gratuitous title upon his property, may provide at the time
the annuity is established that the same shall not be subject to execution or attachment on account of
the obligations of the recipient of the annuity. If the annuity was constituted in fraud of creditors, the
latter may ask for the execution or attachment of the property. (1807a)
Art. 2027. No annuity shall be claimed without first proving the existence of the person upon whose life
the annuity is constituted. (1808)

d.

Compulsory motor vehicle liability insurance Art 2186, Civil Code

Art. 2186. Every owner of a motor vehicle shall file with the proper government office a bond executed
by a government-controlled corporation or office, to answer for damages to third persons. The amount
of the bond and other terms shall be fixed by the competent public official. (n)

e.

Insurers right of subrogation Art 2207, Civil Code

Art. 2207. If the plaintiff's property has been insured, and he has received indemnity from the
insurance company for the injury or loss arising out of the wrong or breach of contract complained of,
the insurance company shall be subrogated to the rights of the insured against the wrongdoer or the
person who has violated the contract. If the amount paid by the insurance company does not fully cover
the injury or loss, the aggrieved party shall be entitled to recover the deficiency from the person causing
the loss or injury.
3. General Principles on Insurance
Constantino vs. Asia Life Insurance Company

By: Elaine Marie G. Laceda

12

INSURANCE LAW

G.R. No. L-1669 August 31, 1950


Case 1: In consideration of the sum of P176.04 as annual premium duly paid to it, the Asia Life
Insurance Company insured the life of Arcadio Constantino for a term of twenty years with the plaintiff
Paz Lopez de Constantino regularly appointed as beneficiary.
The policy contained, among others, this stipulation: All premium payments are due in advance and any
unpunctuality in making any such payment shall cause this policy to lapse unless and except as kept in
force by the Grace Period condition (31 days) or under Option 4 below.
After that first payment, no further premiums were paid. The insured died on September 22, 1944.
Case 2: On August 1, 1938, the defendant Asia Life Insurance Company issued its Policy No. 78145
(Joint Life 20-Year Endowment Participating with Accident Indemnity), covering the lives of the spouses
Tomas Ruiz and Agustina Peralta, for the sum of P3,000. The annual premium stipulated in the policy
was regularly paid covering the period up to January 31, 1942. No further payments were handed to the
insurer.
Because the insured had borrowed on the policy an amount of P234.00 in January, 1941, the cash
surrender value of the policy was sufficient to maintain the policy in force only up to September 7, 1942.
Tomas Ruiz died on February 16, 1945. The plaintiff Agustina Peraltas claim, as beneficiary, was refused
by defendant grounded on non-payment of the premiums.
The lower court absolved the defendant. Hence this appeal.
Issue: Whether the beneficiary in a life insurance policy may recover the amount thereof although the
insured died after repeatedly failing to pay the stipulated premiums, such failure having been caused by
the last war in the Pacific.
Held: No.
The insurance company, for a comparatively small consideration, undertakes to guarantee the insured
against loss or damage, upon the terms and conditions agreed upon, and upon no other, and when
called upon to pay, in case of loss, the insurer, therefore, may justly insists upon a fulfillment of these
terms. If the insured cannot bring himself within the conditions of the policy, he is not entitled for the
loss.
The terms of the policy constitute the measure of the insurer's liability, and in order to recover the
insured must show himself within those terms; and if it appears that the contract has been terminated
by a violation, on the part of the insured, of its conditions, then there can be no right of recovery. The
compliance of the insured with the terms of the contract is a condition precedent to the right of
recovery.
Promptness of payment is essential in the business of life insurance. All the calculations of the insurance
company are based on the hypothesis of prompt payments. They not only calculate on the receipt of the
premiums when due, but on compounding interest upon them. It is on this basis that they are enabled
to offer assurance at the favorable rates they do.
Non-payment at the day involves absolute forfeiture if such be the terms of the contract, as is the case
here. Courts cannot with safety vary the stipulation of the parties by introducing equities for the relief of
the insured against their own negligence.
It should be noted that the parties contracted not only for peacetime conditions but also for times of
war, because the policies contained provisions applicable expressly to wartime days. The logical
inference, therefore, is that the parties contemplated uninterrupted operation of the contract even if
armed conflict should ensue.
4. Special Laws

By: Elaine Marie G. Laceda

13

INSURANCE LAW

a. Revised Government Service Insurance Act of 1977 covers insurance of


government employees
b. Social Security Act of 1954 covers insurance of employees in private
employment
c. Property Insurance Law covers insurance of government property
d. RA No. 4898 as amended provides life, disability and accident insurance
coverage to barangay officials
e. EO No. 250
f. PDIC Charter insures deposits of all banks entitled to benefits of insurance

5.

Corporation Code Sec 185, I.C.

Sec. 185. Corporations formed or organized to save any person or persons or other corporations
harmless from loss, damage, or liability arising from any unknown or future or contingent event, or to
indemnify or to compensate any person or persons or other corporations for any such loss, damage, or
liability, or to guarantee the performance of or compliance with contractual obligations or the payment
of debt of others shall be known as "insurance corporations".
The provisions of the Corporation Law shall apply to all insurance corporations now or hereafter engaged
in business in the Philippines insofar as they do not conflict with the provisions of this chapter.
E.

INTERPRETATION OF INSURANCE CONTRACTS

1. Clear Provision Given Ordinary Meaning


Union Manufacturing Co., Inc. vs. Philippine Guaranty Co., Inc.
G.R. No. L-27932 October 30, 1972
The Union Mfg. obtained certain loans, overdrafts and other credit accommodations from the Republic
Bank in the total sum of P415,000.00 with interest at 9% per annum from said date. To secure the
payment thereof, Union Mfg. executed real and chattel mortgages on certain properties. As additional
condition of the mortgage contract, it undertook to secure insurance coverage over the mortgaged
properties for the same amount of P415,000.00.
For failure of Union Mfg. to secure insurance coverage on the mortgaged properties despite being
reminded of said requirement, the Republic Bank procured from the defendant, Philippine Guaranty Co.,
Inc. an insurance coverage on loss against fire for P500,000.00 over the subject properties.
Sometime on September 6, 1964, a fire occurred in the premises of the Union Mfg. Thus the company
filed its fire claim with the defendant Philippine Guaranty Co., Inc., thru its adjuster, H. H. Bayne
Adjustment Co., which was denied by said defendant on the following grounds: (1) violation of Policy
Condition No. 3 and/or the 'Other Insurance Clause' for representing that there were no other insurance
policies at the time of the issuance of said defendant's policy, and it appearing furthermore that while
the policy of the defendant was in full force and effect the company secured other fire insurance policies
without the written consent of the defendant endorsed on the policy; and (2) non-compliance with Policy
Condition No. 11 for having failed to submit the required documents and other proofs with respect to the
claim.
Held: SC affirmed.
Inasmuch as Union Mfg. has violated the condition of the policy to the effect that it did not reveal the
existence of other insurance policies over the same properties, as required by the warranty appearing on
the face of the policy issued by the defendant, the conclusion is inevitable that both the Republic Bank
and Union Manufacturing Co., Inc. cannot recover from the same policy of the defendant because the
same is null and void.

By: Elaine Marie G. Laceda

14

INSURANCE LAW

Ty vs. First National Surety & Assurance Co., Inc.


G.R. No. L-16138 April 29, 1961
The plaintiff herein Diosdado C. Ty, employed as operator mechanic foreman in the Broadway Cotton
Factory insured himself in 18 local insurance companies which issued to him personal accident policies,
upon payment of the premium of P8.12 for each policy. Plaintiff's beneficiary was his employer,
Broadway Cotton Factory, which paid the insurance premiums.
On December 24, 1953, a fire broke out which totally destroyed the Broadway Cotton Factory. Fighting
his way out of the factory, plaintiff was injured on the left hand by a heavy object. He was brought to
the MCU hospital, and after receiving first aid there, he went to the National Orthopedic Hospital for
treatment of his injuries which have caused temporary total disability of his left hand.
Plaintiff filed the corresponding notice of accident and notice of claim to recover indemnity. Defendants
rejected plaintiff's claim for indemnity for the reason that there being no severance of amputation of the
left hand, the disability suffered by him was not covered by his policy. Hence, plaintiff sued the
defendants.
MTC absolved the defendants from the complaints. Hence this appeal.
Held: SC affirmed.
The main contention of appellant is that in order that he may recover on the insurance policies issued
him for the loss of his left hand, it is not necessary that there should be an amputation thereof, but that
it is sufficient if the injuries prevent him from performing his work or labor necessary in the pursuance of
his occupation or business.
The agreement contained in the insurance policies is the law between the parties. As the terms of the
policies are clear, express and specific that only amputation of the left hand should be considered as a
loss thereof, an interpretation that would include the mere fracture or other temporary disability not
covered by the policies would certainly be unwarranted.

2. Ambiguous Provision Interpreted Against Insurer


Qua Chee Gan vs. Law Union & Rock Insurance Co., Ltd.
G.R. No. L-4611 December 17, 1955
Qua Chee Gan, a merchant of Albay, owned 4 warehouses or bodegas used for the storage of stocks of
copra and of hemp, baled and loose. They had been, with their contents, insured with the Law Union &
Rock Insurance Co., Ltd. since 1937, and the loss made payable to the PNB as mortgagee of the hemp
and crops, to the extent of its interest.
Fire of undetermined origin gutted and completely destroyed Bodegas Nos. 1, 2 and 4, with the
merchandise stored theren. The plaintiff submitted the corresponding fire claims however, the Insurance
Company resisted payment, claiming violation of warranties and conditions, filing of fraudulent claims,
and that the fire had been deliberately caused by the insured or by other persons in connivance with
him.
With counsel for the insurance company acting as private prosecutor, Que Chee Gan, with his brother,
Qua Chee Pao, and some employees of his, were indicted and tried for the crime of arson, it being
claimed that they had set fire to the destroyed warehouses to collect the insurance. They were,
however, acquitted by the trial court.
Thereafter, plaintiff instituted action seeking to recover the proceeds of the fire insurance policies. After
trial, the CFI rendered a decision in favor of the plaintiff.

By: Elaine Marie G. Laceda

15

INSURANCE LAW

Held: SC affirmed.
It is argued by the Insurance Company that since the bodegas insured had an external wall perimeter of
500 meters or 1,640 feet, the appellee should have 11 fire hydrants in the compound, and that he
actually had only 2, with a further pair nearby, belonging to the municipality of Tabaco.
The appellant is barred by waiver (or rather estoppel) to claim violation of the so-called fire hydrants
warranty, for the reason that knowing fully all that the number of hydrants demanded therein never
existed from the very beginning, the appellant neverthless issued the policies in question subject to such
warranty, and received the corresponding premiums.
Moreover, taking into account the well known rule that ambiguities or obscurities must be strictly
interpreted against the party that caused them, the "memo of warranty" invoked by appellant bars the
latter from questioning the existence of the appliances called for in the insured premises, since its initial
expression, "the undernoted appliances for the extinction of fire being kept on the premises insured
hereby, . . . it is hereby warranted . . .", admits of interpretation as an admission of the existence of
such appliances which appellant cannot now contradict, should the parol evidence rule apply.
As to maintenance of a trained fire brigade of 20 men, the record is preponderant that the same was
organized, and drilled, from time to give, although not maintained as a permanently separate unit,
which the warranty did not require. Anyway, it would be unreasonable to expect the insured to maintain
for his compound alone a fire fighting force that many municipalities in the Islands do not even possess.
Under the second assignment of error, appellant insurance company avers, that the insured violated the
"Hemp Warranty" provisions against the storage of gasoline, since appellee admitted that there were 36
cans of gasoline in the building designed as "Bodega No. 2" that was a separate structure not affected
by the fire.
It is well to note that gasoline is not specifically mentioned among the prohibited articles listed in the socalled "hemp warranty." The cause relied upon by the insurer speaks of "oils, animal and/or vegetable
and/or mineral and/or their liquid products having a flash point below 300o Fahrenheit", and is decidedly
ambiguous and uncertain; for in ordinary parlance, "Oils" mean "lubricants" and not gasoline or
kerosene. Thus, by reason of the exclusive control of the insurance company over the terms and
phraseology of the contract, the ambiguity must be held strictly against the insurer and liberally in favor
of the insured, especially to avoid forfeiture.
Furthermore, it is well settled that even though there are printed prohibitions against keeping certain
articles on the insured premises the policy will not be avoided by a violation of these prohibitions, if the
prohibited articles are necessary or in customary use in carrying on the trade or business conducted on
the premises.
The next two defenses pleaded by the insurer, that the insured connived at the loss and that the
fraudulently inflated the quantity of the insured stock in the burnt bodegas, are closely related to
each other. Both defenses are predicted on the assumption that the insured was in financial difficulties
and set the fire to defraud the insurance company, presumably in order to pay off the Philippine National
Bank, to which most of the insured hemp and copra was pledged. Both defenses are fatally undermined
by the established fact that, notwithstanding the insurer's refusal to pay the value of the policies the
extensive resources of the insured enabled him to pay off the National Bank in a short time; and if he
was able to do so, no motive appears for attempt to defraud the insurer.
While the acquittal of the insured in the arson case is not res judicata on the present civil action, the
insurer's evidence, to judge from the decision in the criminal case, is practically identical in both cases
and must lead to the same result, since the proof to establish the defense of connivance at the fire in
order to defraud the insurer "cannot be materially less convincing than that required in order to convict
the insured of the crime of arson".
Del Rosario vs. Equitable Insurance & Casualty Co., Inc.
G.R. No. L-16215 June 29, 1963

By: Elaine Marie G. Laceda

16

INSURANCE LAW

Equitable Insurance & Casualty Co., Inc. issued Personal Accident Policy No. 7136 on the life of
Francisco del Rosario, alias Paquito Bolero, binding itself to pay the sum of P1,000.00 to P3,000.00, as
indemnity in case of death of the insured.
On February 24, 1957, the insured Francisco del Rosario, alias Paquito Bolero, while on board the motor
launch "ISLAMA" together with 33 others, including his beneficiary in the Policy, Remedios Jayme, were
forced to jump off said launch on account of fire which broke out on said vessel, resulting in the death
by drowning, of the insured and beneficiary.
Simeon del Rosario, father of the insured, and as the sole heir, filed a claim for payment with defendant
company which paid the sum of P1,000.00, pursuant to Section 1 of Part I of the policy. Atty. Francisco,
wrote defendant company acknowledging receipt by his client (plaintiff herein), of the P1,000.00, but
informing said company that said amount was not the correct one. In a subsequent letter to the
insurance company, Atty. Francisco asked for P3,000.00 which the Company refused to pay. Hence, a
complaint for the recovery of the balance of P2,000.00 was instituted.
CFI ruled in favor of plaintiff and held that since the defendant has bound itself to pay P1000.00 to
P3,000.00 as indemnity for the death of the insured but the policy does not positively state any definite
amount that may be recovered in case of death by drowning, there is an ambiguity in this respect in the
policy, which ambiguity must be interpreted in favor of the insured and strictly against the insurer so as
to allow greater indemnity.
Held: SC Affirmed.
The "terms in an insurance policy, which are ambiguous, equivocal or uncertain . . . are to be construed
strictly against, the insurer, and liberally in favor of the insured so as to effect the dominant purpose of
indemnity or payment to the insured, especially where a forfeiture is involved," and the reason for this
rule is that the "insured usually has no voice in the selection or arrangement of the words employed and
that the language of the contract is selected with great care and deliberation by expert and legal
advisers employed by, and acting exclusively in the interest of, the insurance company".
Verendia vs. Court of Appeals
G.R. No. 75605 January 22, 1993
Fidelity and Surety Insurance issued Fire Insurance Policy No. F-18876 effective between June 23, 1980
and June 23, 1981 covering Rafael (Rex) Verendia's residential building. Designated as beneficiary was
the Monte de Piedad & Savings Bank.
Verendia also insured the same building with two other companies, namely, The Country Bankers
Insurance and The Development Insurance.
While the three fire insurance policies were in force, the insured property was completely destroyed by
fire on the early morning of December 28, 1980. Fidelity was accordingly informed of the loss and
despite demands, refused payment under its policy, thus prompting Verendia to file a complaint.
Answering the complaint, Fidelity, among other things, averred that the policy was avoided by reason of
over-insurance; that Verendia maliciously represented that the building at the time of the fire was
leased to a certain Roberto Garcia, when actually it was a Marcelo Garcia who was the lessee.
CFI rendered a decision ruling in favor of Fidelity. The trial court ruled that Paragraph 3 of the policy was
violated by Verendia in that the insured failed to inform Fidelity of his other insurance coverages with
Country Bankers Insurance and Development Insurance.
The CA reversed for the following reasons: (a) there was no misrepresentation concerning the lease for
the contract was signed by Marcelo Garcia in the name of Roberto Garcia; and (b) Paragraph 3 of the
policy contract requiring Verendia to give notice to Fidelity of other contracts of insurance was waived by
Fidelity as shown by its conduct in attempting to settle the claim of Verendia. MR denied.

By: Elaine Marie G. Laceda

17

INSURANCE LAW

Held: CFI reinstated.


Verendia failed to live by the terms of the policy, specifically Section 13 thereof which is expressed in
terms that are clear and unambiguous, that all benefits under the policy shall be forfeited "If the claim
be in any respect fraudulent, or if any false declaration be made or used in support thereof, or if any
fraudulent means or devises are used by the Insured or anyone acting in his behalf to obtain any benefit
under the policy". Verendia, having presented a false declaration to support his claim for benefits in the
form of a fraudulent lease contract, he forfeited all benefits therein by virtue of Section 13 of the policy
in the absence of proof that Fidelity waived such provision. Worse yet, by presenting a false lease
contract, Verendia, reprehensibly disregarded the principle that insurance contracts are uberrimae fidae
and demand the most abundant good faith.

3. Stipulations Cannot be Segregated


Gulf Resorts, Inc. vs. Philippine Charter Insurance Corp.
G.R. No. 156167 May 16, 2005
Plaintiff is the owner of the Plaza Resort insured originally with the American Home Assurance Company
(AHAC-AIU). In the first four insurance policies issued by AHAC-AIU the risk of loss from earthquake
shock was extended only to plaintiffs two swimming pools.
Plaintiff agreed to insure with defendant the properties covered by AHAC-AIU provided that the policy
wording and rates in said policy be copied in the policy to be issued by defendant.
On July 16, 1990 an earthquake struck Central Luzon and Northern Luzon and plaintiffs properties
covered by Policy No. 31944 issued by defendant, including the two swimming pools were damaged.
After the earthquake, petitioner advised respondent that it would be making a claim under its Insurance
Policy No. 31944 for damages on its properties. Respondent denied petitioners claim on the ground that
its insurance policy only afforded earthquake shock coverage to the two swimming pools of the resort.
Thus, petitioner filed a complaint praying for the payment for the losses sustained by the insured
properties.
RTC ruled in favor of respondent declaring after finding that only the 2 swimming pools had earthquake
shock coverage. CA affirmed.
Held: SC Affirmed.
It is basic that all the provisions of the insurance policy should be examined and interpreted in
consonance with each other. All its parts are reflective of the true intent of the parties. The policy cannot
be construed piecemeal. Certain stipulations cannot be segregated and then made to control; neither do
particular words or phrases necessarily determine its character. Petitioner cannot focus on the
earthquake shock endorsement to the exclusion of the other provisions. All the provisions and riders,
taken and interpreted together, indubitably show the intention of the parties to extend earthquake shock
coverage to the two swimming pools only.
An insurance contract exists where the following elements concur:
1. The insured has an insurable interest;
2. The insured is subject to a risk of loss by the happening of the designated peril;
3. The insurer assumes the risk;
4. Such assumption of risk is part of a general scheme to distribute actual losses among a large
group of persons bearing a similar risk; and
5. In consideration of the insurers promise, the insured pays a premium.
An insurance premium is the consideration paid an insurer for undertaking to indemnify the insured
against a specified peril. In the subject policy, no premium payments were made with regard to

By: Elaine Marie G. Laceda

18

INSURANCE LAW

earthquake shock coverage, except on the two swimming pools. There is no mention of any premium
payable for the other resort properties with regard to earthquake shock. This is consistent with the
history of petitioners previous insurance policies from AHAC-AIU.
There is no ambiguity in the terms of the contract and its riders. Petitioner cannot rely on the general
rule that insurance contracts are contracts of adhesion which should be liberally construed in favor of
the insured and strictly against the insurer company which usually prepares it.
Petitioner cannot claim it did not know the provisions of the policy. From the inception of the policy,
petitioner had required the respondent to copy verbatim the provisions and terms of its latest insurance
policy from AHAC-AIU. Respondent, in compliance with the condition set by the petitioner, copied AIU
Policy No. 206-4568061-9 in drafting its Insurance Policy No. 31944. It is true that there was variance in
some terms, specifically in the replacement cost endorsement, but the principal provisions of the policy
remained essentially similar to AHAC-AIUs policy.

4. Judicial Construction Cannot Alter Terms


Misamis Lumber Corporation vs. Capital Insurance and Surety Co., Inc.
G.R. No. L-21380 May 20, 1966
Misamis Lumber Corp, under its former name, Lanao Timber Mills, Inc., insured its Ford Falcon motor
car for the amount of P14,000 with Capital Insurance & Surety Company, Inc.
While the insurance policy was in force, the insured car, while traveling along in Aurora Boulevard in
front of the Pepsi-Cola plant in Quezon City, passed over a water hole which the driver did not see
because an oncoming car did not dim its light. The crankcase and flywheel housing of the car broke
when it hit a hollow block lying alongside the water hole. At the instance of the plaintiff-appellee, the car
was towed and repaired by Morosi Motors at its shop at 1906 Taft Avenue Extension at a total cost of
P302.27.
When the repairs on the car had already been made, the plaintiff-appellee made a report of the accident
to the defendant-appellant Capital Insurance & Surety Company. Since the defendant-appellant refused
to pay for the total cost of to wage and repairs, suit was filed.
The defendant-appellant admits liability in the amount of P150, but not for any excess thereof. However,
the CFI did not exonerate the said appellant for the excess because, according to it, the company's
absolution would render the insurance contract one-sided and that the said insurer had not shown that
the cost of repairs in the sum of P302.27 is unreasonable, excessive or padded, nor had it shown that it
could have undertaken the repairs itself at less expense.
Held: Defendant liable only to the extent of P150.00
The insurance policy stipulated in paragraph 4 that if the insured authorizes the repair the liability of the
insurer, per its sub-paragraph (a), is limited to P150.00. The literal meaning of this stipulation must
control, it being the actual contract, expressly and plainly provided for in the policy.
The option to undertake the repairs is accorded to the insurance company per paragraph 2. The said
company was deprived of the option because the insured took it upon itself to have the repairs made,
and only notified the insurer when the repairs were done. As a consequence, paragraph 4, which limits
the company's liability to P150.00, applies.
The insurance contract may be rather onerous ("one-sided", as the lower court put it), but that in itself
does not justify the abrogation of its express terms, terms which the insured accepted or adhered to and
which is the law between the contracting parties
Fortune Insurance and Surety Co., Inc. vs. CA
G.R. No. 115278 May 23, 1995

By: Elaine Marie G. Laceda

19

INSURANCE LAW

Producers Bank filed against petitioner Fortune Insurance and Surety Co., Inc. a complaint for recovery
of the sum of P725,000.00 under the theft or robbery insurance policy issued by Fortune. The sum was
allegedly lost during a robbery of Producer's armored vehicle while it was in transit to transfer the
money from its Pasay City Branch to its head office in Makati.
Fortune refused to pay as the loss is allegedly excluded from the coverage of the insurance policy which
provides that: The company shall not be liable under this policy for xxx any loss caused by any
dishonest, fraudulent or criminal act of the insured or any officer, employee, partner, director, trustee or
authorized representative of the Insured whether acting alone or in conjunction with others.
The plaintiff opposes the contention of defendant Fortune and contends that Atiga and Magalong are not
its "officer, employee, . . . trustee or authorized representative . . . at the time of the robbery.
RTC, finding that Magalong and Atiga were not employees or representatives of Producers, ordered the
defendant to pay plaintiff the net amount of P540,000.00 as liability under the policy.
According to the lower court, plaintiff may not be said to have selected and engaged Magalong and
Atiga, their services as armored car driver and as security guard having been merely offered by PRC
Management and by Unicorn Security and which latter firms assigned them to plaintiff. The wages and
salaries of both Magalong and Atiga are presumably paid by their respective firms, which alone wields
the power to dismiss them.
CA affirmed in toto the appealed decision. It agreed with the conclusion of the trial court that Magalong
and Atiga were neither employees nor authorized representatives of Producers.
The CA likewise ratiocinated that the language used by defendant-appellant in the policy is plain,
ordinary and simple. No other interpretation is necessary. The word "employee" must be taken to mean
in the ordinary sense.
Held: Fortune exempt from liability.
It should be noted that the insurance policy entered into by the parties is a theft or robbery insurance
policy which is a form of casualty insurance. Except with respect to compulsory motor vehicle liability
insurance, the Insurance Code contains no other provisions applicable to casualty insurance or to
robbery insurance in particular other than Sec. 174. These contracts are, therefore, governed by the
general provisions applicable to all types of insurance. Outside of these, the rights and obligations of the
parties must be determined by the terms of their contract, taking into consideration its purpose and
always in accordance with the general principles of insurance law.
It has been aptly observed that in burglary, robbery, and theft insurance, "the opportunity to defraud
the insurer the moral hazard is so great that insurers have found it necessary to fill up their
policies with countless restrictions, many designed to reduce this hazard. Seldom does the insurer
assume the risk of all losses due to the hazards insured against." Persons frequently excluded under
such provisions are those in the insured's service and employment. The purpose of the exception is to
guard against liability should the theft be committed by one having unrestricted access to the property.
In such cases, the terms specifying the excluded classes are to be given their meaning as understood in
common speech. The terms "service" and "employment" are generally associated with the idea of
selection, control, and compensation.
A contract of insurance is a contract of adhesion, thus any ambiguity therein should be resolved against
the insurer, or it should be construed liberally in favor of the insured and strictly against the insurer.
Conversely, if the terms of the contract are clear and unambiguous, there is no room for construction
and such terms cannot be enlarged or diminished by judicial construction.
An insurance contract is a contract of indemnity upon the terms and conditions specified therein. It is
settled that the terms of the policy constitute the measure of the insurer's liability. In the absence of
statutory prohibition to the contrary, insurance companies have the same rights as individuals to limit

By: Elaine Marie G. Laceda

20

INSURANCE LAW

their liability and to impose whatever conditions they deem best upon their obligations not inconsistent
with public policy.
Insofar as Fortune is concerned, it was its intention to exclude and exempt from protection and coverage
losses arising from dishonest, fraudulent, or criminal acts of persons granted or having unrestricted
access to Producers' money or payroll. When it used then the term "employee," it must have had in
mind any person who qualifies as such as generally and universally understood, or jurisprudentially
established in the light of the four standards in the determination of the employer-employee
relationship, or as statutorily declared even in a limited sense as in the case of Article 106 of the Labor
Code which considers the employees under a "labor-only" contract as employees of the party employing
them and not of the party who supplied them to the employer.
A "representative" is defined as one who represents or stands in the place of another; one who
represents others or another in a special capacity, as an agent, and is interchangeable with "agent."
Thus, even granting for the sake of argument that these contracts were not "labor-only" contracts, and
PRC Management Systems and Unicorn Security Services were truly independent contractors, we are
satisfied that Magalong and Atiga were, in respect of the transfer of Producer's money from its Pasay
City branch to its head office in Makati, its "authorized representatives" who served as such with its
teller Maribeth Alampay.

II.

PARTIES
A.

INSURED

1.

Definition the person to be indemnified; the person who applied for and to whom
an insurance policy is issued.

2.

Capacity Art 1390, Civil Code

Art. 1390. The following contracts are voidable or annullable, even though there may have been no
damage to the contracting parties:
(1) Those where one of the parties is incapable of giving consent to a contract;
(2) Those where the consent is vitiated by mistake, violence, intimidation, undue influence or
fraud.
These contracts are binding, unless they are annulled by a proper action in court. They are
susceptible of ratification. (n)
a. Married women Sec 3, par 2 and 4, I.C.; Art 73, Family Code
Sec. 3.

xxx
The consent of the husband is not necessary for the validity of an insurance policy taken out by
a married woman on her life or that of her children.
xxx
The married woman or the minor herein allowed to take out an insurance policy may exercise all
the rights and privileges of an owner under a policy.
Art. 73. Either spouse may exercise any legitimate profession, occupation, business or activity without
the consent of the other. The latter may object only on valid, serious, and moral grounds.
In case of disagreement, the court shall decide whether or not:
(1) The objection is proper; and
(2) Benefit has occurred to the family prior to the objection or thereafter. If the benefit accrued
prior to the objection, the resulting obligation shall be enforced against the separate property of the
spouse who has not obtained consent.
The foregoing provisions shall not prejudice the rights of creditors who acted in good faith.
(117a)
b. Minors Sec 3, par 3-5, I.C.; Art 38, Civil Code

By: Elaine Marie G. Laceda

21

INSURANCE LAW

Sec. 3.

xxx
Any minor of the age of eighteen years or more, may, notwithstanding such minority, contract
for life, health and accident insurance, with any insurance company duly authorized to do business in
the Philippines, provided the insurance is taken on his own life and the beneficiary appointed is the
minor's estate or the minor's father, mother, husband, wife, child, brother or sister.
The married woman or the minor herein allowed to take out an insurance policy may exercise all
the rights and privileges of an owner under a policy.
All rights, title and interest in the policy of insurance taken out by an original owner on the life
or health of a minor shall automatically vest in the minor upon the death of the original owner, unless
otherwise provided for in the policy.
Art. 38. Minority, insanity or imbecility, the state of being a deaf-mute, prodigality and civil interdiction
are mere restrictions on capacity to act, and do not exempt the incapacitated person from certain
obligations, as when the latter arise from his acts or from property relations, such as easements. (32a)
3. Disqualification: Public Enemy Sec 7, I.C.
Sec. 7. Anyone except a public enemy may be insured.
Filipinas Compaia De Seguros vs. Christern Huenefeld and Co., Inc.
G.R. No. L-2294 May 25, 1951
Christern Huenefeld, & Co., Inc., organized under and by virtue of the laws of the Philippines being
controlled by the German subjects, obtained from the petitioner Filipinas Cia. de Seguros, fire policy No.
29333. During the Japanese military occupation, the building and insured merchandise were burned. In
due time the respondent submitted to the petitioner its claim under the policy. The petitioner refused to
pay the claim on the ground that the policy in favor of the respondent had ceased to be in force on the
date the US declared war against Germany. The petitioner, however, in pursuance of the order of the
Director of Bureau of Financing, Philippine Executive Commission, paid to the respondent the sum of
P92,650.
Thus, petitioner filed with the CFI an action for the purpose of recovering from the respondent the sum
of P92,650. CFI dismissed the action.
The CA affirmed and overruled the contention of the petitioner that the respondent corporation became
an enemy when the US declared war against Germany, relying on English and American cases which
held that a corporation is a citizen of the country or state by and under the laws of which it was created
or organized. It rejected the theory that nationality of private corporation is determined by the character
or citizenship of its controlling stockholders.
Issue: WON the respondent corporation is entitled to indemnity pursuant to the fire insurance.
Held: No.
The Philippine Insurance Law (Act No. 2427, as amended,) in section 8, provides that "anyone except a
public enemy may be insured." It stands to reason that an insurance policy ceases to be allowable as
soon as an insured becomes a public enemy.
There is no question that majority of the stockholders of the respondent corporation were German
subjects. This being so, said respondent became an enemy corporation upon the outbreak of the war
between the US and Germany. Thus, having become an enemy corporation on December 10, 1941, the
insurance policy issued in its favor had ceased to be valid and enforcible, and since the insured goods
were burned after December 10, 1941, and during the war, the respondent was not entitled to any
indemnity under said policy from the petitioner. However, elementary rules of justice (in the absence of
specific provision in the Insurance Law) require that the premium paid by the respondent for the period
covered by its policy from December 11, 1941, should be returned by the petitioner.

By: Elaine Marie G. Laceda

22

INSURANCE LAW

4. Trustee or Agent Sec 54, I.C.


Sec. 54. When an insurance contract is executed with an agent or trustee as the insured, the fact that
his principal or beneficiary is the real party in interest may be indicated by describing the insured as
agent or trustee, or by other general words in the policy.
5. Partner Sec 55, I.C.
Sec. 55. To render an insurance effected by one partner or part-owner, applicable to the interest of his
co-partners or other part-owners, it is necessary that the terms of the policy should be such as are
applicable to the joint or common interest.
B.

INSURER

1. Definition the person who undertakes to indemnify another by a contract of


insurance; Sec 184, I.C.
Sec. 184. For purposes of this Code, the term "insurer" or "insurance company" shall include all
individuals, partnerships, associations, or corporations, including government-owned or controlled
corporations or entities, engaged as principals in the insurance business, excepting mutual benefit
associations. Unless the context otherwise requires, the terms shall also include professional reinsurers
defined in section two hundred eighty. "Domestic company" shall include companies formed, organized
or existing under the laws of the Philippines. "Foreign company" when used without limitation shall
include companies formed, organized, or existing under any laws other than those of the Philippines.
a. Insurance Corporations Sec 185, par 1, I.C.
Sec. 185. Corporations formed or organized to save any person or persons or other corporations
harmless from loss, damage, or liability arising from any unknown or future or contingent event, or to
indemnify or to compensate any person or persons or other corporations for any such loss, damage, or
liability, or to guarantee the performance of or compliance with contractual obligations or the payment
of debt of others shall be known as "insurance corporations".
The provisions of the Corporation Law shall apply to all insurance corporations now or hereafter
engaged in business in the Philippines insofar as they do not conflict with the provisions of this chapter.
b. Mutual Insurance Companies Sec 262, I.C.
Sec. 262. Any domestic stock life insurance company doing business in the Philippines may convert
itself into an incorporated mutual life insurer. To that end it may provide and carry out a plan for the
acquisition of the outstanding shares of its capital stock for the benefit of its policyholders, or any class
or classes of its policyholders, by complying with the requirements of this chapter.
c.

Professional Reinsurers Sec 280, I.C.

Sec. 280. Except as otherwise provided in this Code, no person, partnership, association or corporation
shall transact any business in the Philippines as a professional reinsurer until it shall have obtained a
certificate of authority for that purpose from the Commissioner upon the application therefor and
payment by such person, partnership, association or corporation of the fees hereinafter prescribed. As
used in this Code, the term "professional reinsurer" shall mean any person, partnership, association or
corporation that transacts solely and exclusively reinsurance business in the Philippines.
The Commissioner may refuse to issue a certificate of authority to any such person, partnership,
association or corporation if, in his judgment, such refusal will best promote public interest. No such
certificate of authority shall be granted to any such person, partnership, association or corporation
unless and until the Commissioner shall have satisfied himself by such examination as he may make and
such evidence as he may require that such person, partnership, association or corporation is qualified by
the laws of the Philippines to transact business therein as a professional reinsurer.

By: Elaine Marie G. Laceda

23

INSURANCE LAW

Before issuing such certificate of authority of the Commissioner must be satisfied that the name
of the applicant is not that of any other known company transacting insurance or reinsurance business
in the Philippines, or a name so similar as to be calculated to mislead the public.
Such certificate of authority shall expire on the last day of June of each year and shall be
renewed annually if such person, partnership, association, or corporation is continuing to comply with
provisions of this Code, or the circulars, instructions, rulings, or decisions of the Commissioner and such
other pertinent law, rules and regulations.
Every such person, partnership, association, or corporation receiving such certificate of authority
shall be subject to the provisions of this Code and other related laws, and to the jurisdiction and
supervision of the Commissioner.

d.

Mutual Benefit Associations (excluded from definition, but still subject to I.C.
regulation) Sec 390, I.C.

Sec. 390. Any society, association or corporation, without capital stock, formed or organized not for
profit but mainly for the purpose of paying sick benefits to members, or of furnishing financial support to
members while out of employment, or of paying to relatives of deceased members of fixed or any sum
of money, irrespective of whether such aim or purpose is carried out by means of fixed dues or
assessments collected regularly from the members, or of providing, by the issuance of certificates of
insurance, payment of its members of accident or life insurance benefits out of such fixed and regular
dues or assessments, but in no case shall include any society, association, or corporation with such
mutual benefit features and which shall be carried out purely from voluntary contributions collected not
regularly and or no fixed amount from whomsoever may contribute, shall be known as a mutual benefit
association within the intent of this Code.
Any society, association, or corporation principally organized as labor union shall be governed by
the Labor Code notwithstanding any mutual benefit feature provisions in its charter as incident to its
organization.
In no case shall a mutual benefit association be organized and authorized to transact business as a
charitable or benevolent organization, and whenever it has this feature as incident to its existence, the
corresponding charter provision shall be revised to conform with the provision of this section. Mutual
benefit association, already licensed to transact business as such on the date this Code becomes
effective, having charitable or benevolent feature shall abandon such incidental purpose upon effectivity
of this Code if they desire to continue operating as such mutual benefit associations.
2. Capitalization/ Certificate of Authority Secs 6, 186-187, I.C.
Sec. 6. Every person, partnership, association, or corporation duly authorized to transact insurance
business as elsewhere provided in this code, may be an insurer.
Sec. 186. No person, partnership, or association of persons shall transact any insurance business in the
Philippines except as agent of a person or corporation authorized to do the business of insurance in the
Philippines, unless possessed of the capital and assets required of an insurance corporation doing the
same kind of business in the Philippines and invested in the same manner; nor unless the Commissioner
shall have granted to him or them a certificate to the effect that he or they have complied with all the
provisions of law which an insurance corporation doing business in the Philippines is required to observe.
Every person, partnership, or association receiving any such certificate of authority shall be
subject to the insurance laws of the Philippines and to the jurisdiction and supervision of the
Commissioner in the same manner as if an insurance corporation authorized by the laws of the
Philippines to engage in the business of insurance specified in the certificate.
Sec. 187. No insurance company shall transact any insurance business in the Philippines until after it
shall have obtained a certificate of authority for that purpose from the Commissioner upon application
therefor and payment by the company concerned of the fees hereinafter prescribed.
The Commissioner may refuse to issue a certificate of authority to any insurance company if, in
his judgment, such refusal will best promote the interest of the people of this country. No such
certificate of authority shall be granted to any such company until the Commissioner shall have satisfied
himself by such examination as he may make and such evidence as he may require that such company
is qualified by the laws of the Philippines to transact business therein, that the grant of such authority

By: Elaine Marie G. Laceda

24

INSURANCE LAW

appears to be justified in the light of economic requirements, and that the direction and administration,
as well as the integrity and responsibility of the organizers and administrators, the financial organization
and the amount of capital, notwithstanding the provisions of section one hundred eighty-eight,
reasonably assure the safety of the interests of the policyholders and the public.
In order to maintain the quality of the management of the insurance companies and afford
better protection to policyholders and the public in general, any person of good moral character,
unquestioned integrity and recognized competence may be elected or appointed director or officer of
insurance companies. The Commissioner shall prescribe the qualifications of the executive officers and
other key officials of insurance companies for purposes of this section.
No person shall concurrently be a director and/or officer of an insurance company and an
adjustment company.
Incumbent directors and/or officers affected by the above provisions are hereby allowed to hold
on to their positions until the end of their terms or two years from the effectivity of this decree,
whichever is shorter.
Before issuing such certificate of authority, the Commissioner must be satisfied that the name of
the company is not that of any other known company transacting a similar business in the Philippines, or
a name so similar as to be calculated to mislead the public.
Such certificate of authority shall expire on the last day of June of each year and shall be
renewed annually if the company is continuing to comply with the provisions of this Code or the
circulars, instructions, rulings or decisions of the Commissioner. Every company receiving any such
certificates of authority shall be subject to the provisions of this Code and other related laws and to the
jurisdiction and supervision of the Commissioner.
No insurance company may be authorized to transact in the Philippines the business of life and
non-life insurance concurrently unless specifically authorized to do so: Provided, That the terms "life"
and "non-life" insurance shall be deemed to include health, accident and disability insurance.
No insurance company shall have equity in an adjustment company and neither shall an
adjustment company have an equity in an insurance company.
Insurance companies and adjustment companies presently affected by the above provision shall
have two years from the effectivity of this Decree within which to divest of their stockholdings.
3. Prohibited Acts Secs 361-362, I.C.
Sec. 361. No insurance company doing business in the Philippines or any agent thereof, no insurance
broker, and no employee or other representative of any such insurance company, agent, or broker, shall
make, procure or negotiate any contract of insurance or agreement as to policy contract, other than is
plainly expressed in the policy or other written contract issued or to be issued as evidence thereof, or
shall directly or indirectly, by giving or sharing a commission or in any manner whatsoever, pay or allow
or offer to pay or allow to the insured or to any employee of such insured, either as an inducement to
the making of such insurance or after such insurance has been effected, any rebate from the premium
which is specified in the policy, or any special favor or advantage in the dividends or other benefits to
accrue thereon, or shall give or offer to give any valuable consideration or inducement of any kind,
directly or indirectly, which is not specified in such policy or contract of insurance; nor shall any such
company, or any agent thereof, as to any policy or contract of insurance issued, make any
discrimination against any Filipino in the sense that he is given less advantageous rates, dividends or
other policy conditions or privileges than are accorded to other nationals because of his race.
Sec. 362. No insurance company doing business in the Philippines, and no officer, director, or agent
thereof, and no insurance broker or any other person, partnership or corporation shall issue or circulate
or cause or permit to be issued or circulated any literature, illustration, circular or statement of any sort
misrepresenting the terms of any policy issued by any insurance company of the benefits or advantages
promised thereby, or any misleading estimate of the dividends or share of surplus to be received
thereon, or shall use any name or title of any policy or class of policies misrepresenting the true nature
thereof; nor shall any such company or agent thereof, or any other person, partnership or corporation
make any misleading representation or incomplete comparison of policies to any person insured in such
company for the purpose of inducing or tending to induce such person to lapse, forfeit, or surrender his
said insurance.
C.

BENEFICIARY

By: Elaine Marie G. Laceda

25

INSURANCE LAW

1. Definition the person who receives a benefit or advantage, or who is entitled to the
benefit of the contract, i.e., the one to whom the insurance is payable or who is
entitled to the proceeds of the policy on the occurrence of the event designated.
2. To Whom Insurance Proceeds Payable Sec 53, I.C.
Sec. 53. The insurance proceeds shall be applied exclusively to the proper interest of the person in
whose name or for whose benefit it is made unless otherwise specified in the policy.

a.

Stipulation Pour Autrui Art 1311, Civil Code

Art. 1311. Contracts take effect only between the parties, their assigns and heirs, except in case where
the rights and obligations arising from the contract are not transmissible by their nature, or by
stipulation or by provision of law. The heir is not liable beyond the value of the property he received
from the decedent.
If a contract should contain some stipulation in favor of a third person, he may demand its
fulfillment provided he communicated his acceptance to the obligor before its revocation. A mere
incidental benefit or interest of a person is not sufficient. The contracting parties must have clearly and
deliberately conferred a favor upon a third person. (1257a)
Bonifacio Bros., Inc., et al. vs. Enrique Mora, et al.
G.R. No. L-20853 May 29, 1967
Enrique Mora, owner of Oldsmobile sedan model 1956, mortgaged the same to the H.S. Reyes, Inc.,
with the condition that the former would insure the automobile with the latter as beneficiary. Thus, the
automobile was insured with the State Bonding & Insurance Co., Inc. against loss of or damages to the
Motor Vehicle and its accessories and spare parts whilst thereon; (a) by accidental collision or
overturning or collision or overturning consequent upon mechanical breakdown or consequent upon
wear and tear, with the option of paying in cash the amount of the loss or damage or repair, reinstate,
or replace the Motor Vehicle or any part thereof or its accessories or spare parts.
During the effectivity of the insurance contract, the car met with an accident. Enrique Mora, without the
knowledge and consent of the H.S. Reyes, Inc., authorized the Bonifacio Bros. Inc. to furnish the labor
and materials, some of which were supplied by the Ayala Auto Parts Co. The insurance company then
drew a check in the amount of P2,002.73, as proceeds of the insurance policy, payable to the order of
Enrique Mora or H.S. Reyes,. Inc. In the meantime, the car was delivered to Enrique Mora without the
consent of the H.S. Reyes, Inc., and without payment to the Bonifacio Bros. Inc. and the Ayala Auto
Parts Co. of the cost of repairs and materials.
Upon the theory that the insurance proceeds should be paid directly to them, the Bonifacio Bros. Inc.
and the Ayala Auto Parts Co. filed a complaint against Enrique Mora and the State Bonding & Insurance
Co., Inc. for the collection of the sum of P2,002.73. The Municipal Court directed payment of the
proceeds of the motor insurance to H.S. Reyes, Inc. who a better right than the Bonifacio Bros., Inc. and
the Ayala Auto Parts Co. CFI affirmed.
Issue: WON the Bonifacio Bros. Inc. and the Ayala Auto Parts Co. have the right to claim the proceeds of
the insurance.
Held: No.
Appellants seek to recover the insurance proceeds, and for this purpose, they rely upon paragraph 4 of
the insurance contract document executed by and between the State Bonding & Insurance Company,
Inc. and Enrique Mora. The appellants are not mentioned in the contract as parties thereto nor is there
any clause or provision thereof from which we can infer that there is an obligation on the part of the
insurance company to pay the cost of repairs directly to them. It is fundamental that contracts take
effect only between the parties thereto, except in some specific instances provided by law where the
contract contains some stipulation in favor of a third person. Such stipulation is known as
stipulation pour autrui or a provision in favor of a third person. Under this doctrine, a third person is

By: Elaine Marie G. Laceda

26

INSURANCE LAW

allowed to avail himself of a benefit granted to him by the terms of the contract, provided that the
contracting parties have clearly and deliberately conferred a favor upon such person.
The question of whether a third person has an enforcible interest in a contract, must be settled by
determining whether the contracting parties intended to tender him such an interest by deliberately
inserting terms in their agreement with the avowed purpose of conferring a favor upon such third
person. In this connection, this Court has laid down the rule that the fairest test to determine whether
the interest of a third person in a contract is a stipulation pour autrui or merely an incidental interest, is
to rely upon the intention of the parties as disclosed by their contract. In the instant case the insurance
contract does not contain any words or clauses to disclose an intent to give any benefit to any repairmen
or materialmen in case of repair of the car in question. On the other hand, the "loss payable" clause of
the insurance policy stipulates that "Loss, if any, is payable to H.S. Reyes, Inc." indicating that it was
only the H.S. Reyes, Inc. which they intended to benefit.
As regards paragraph 4 of the insurance contract, a perusal thereof would show that instead of
establishing privity between the appellants and the insurance company, such stipulation merely
establishes the procedure that the insured has to follow in order to be entitled to indemnity for repair.
A policy of insurance is a distinct and independent contract between the insured and insurer, and third
persons have no right either in a court of equity, or in a court of law, to the proceeds of it, unless there
be some contract of trust, expressed or implied between the insured and third person.
The final contention of the appellants is that the right of the H.S. Reyes, Inc. to the insurance proceeds
arises only if there was loss and not where there is mere damage as in the instant case. Suffice it to say
that any attempt to draw a distinction between "loss" and "damage" is uncalled for, because the word
"loss" in insurance law embraces injury or damage.
Coquia vs. Fieldmen's Insurance Co., Inc.
G.R. No. L-23276 November 29, 1968
On December 1, 1961, appellant Fieldmen's Insurance Company, Inc. issued, in favor of the Manila
Yellow Taxicab Co., Inc. a common carrier accident insurance policy, covering the period from December
1, 1961 to December 1, 1962. Under the policy, the Insurer agreed indemnify the Insured in the event
of accident caused by or arising out of the use of Motor Vehicle against all sums which the Insured will
become legally liable to pay in respect of: Death or bodily injury to any fare-paying passenger including
the Driver, Conductor and/or Inspector who is riding in the Motor Vehicle insured at the time of accident
or injury.
While the policy was in force a taxicab of the Insured, driven by Carlito Coquia, met a vehicular accident
at Mangaldan, Pangasinan, in consequence of which Carlito died. The Insured filed therefor a claim for
P5,000.00 to which the Company replied with an offer to pay P2,000.00, by way of compromise. The
Insured rejected the same and made a counter-offer for P4,000.00, but the Company did not accept it.
Thus, the Insured and Carlito's parents filed a complaint against the Company to collect the proceeds of
the aforementioned policy. CFI rendered a decision sentencing the Company to pay to the plaintiffs the
sum of P4,000.00 and the costs.
Issue: WON the Coquias have a cause of action against the Company.
Held: Yes.
Although, in general, only parties to a contract may bring an action based thereon, this rule is subject to
exceptions, one of which is found in the second paragraph of Article 1311 of the Civil Code of the
Philippines, reading: If a contract should contain some stipulation in favor of a third person, he may
demand its fulfillment provided he communicated his acceptance to the obligor before its revocation. A
mere incidental benefit or interest of a person is not sufficient. The contracting parties must have clearly
and deliberately conferred a favor upon a third person.

By: Elaine Marie G. Laceda

27

INSURANCE LAW

Pursuant to the contract, the Company "will indemnify any authorized Driver who is driving the Motor
Vehicle" of the Insured and, in the event of death of said driver, the Company shall, likewise, "indemnify
his personal representatives." In fact, the Company "may, at its option, make indemnity
payable directly to the claimants or heirs of claimants ... it being the true intention of this Policy to
protect ... the liabilities of the Insured towards the passengers of the Motor Vehicle and the Public" in
other words, third parties.
Guingon vs. Del Monte
G.R. No. L-22042 August 17, 1967
Julio Aguilar entered into a contract with the Capital Insurance & Surety Co., Inc. insuring the operation
of his jeepneys against accidents with third-party liability. During the effectivity of such insurance policy
Iluminado del Monte, one of the drivers of the jeepneys operated by Aguilar, bumped with the jeepney
of Gervacio Guingon who had just alighted from another jeepney and as a consequence the latter died
some days thereafter. A corresponding information for homicide thru reckless imprudence was filed
against Iluminado del Monte, who pleaded guilty. A penalty of four months imprisonment was imposed
on him.
The heirs of Gervacio Guingon filed an action for damages praying that the sum of P82,771.80 be paid
to them jointly and severally by the defendants, driver Iluminado del Monte, owner and operator Julio
Aguilar, and the Capital Insurance & Surety Co., Inc. Capital Insurance & Surety Co., Inc. answered,
alleging that the plaintiff has no cause of action against it. CFI granted prayer.
Issue: WON the plaintiff could sue the Insurer jointly with the Insured.
Held: Yes.
The policy in the present case is one whereby the insurer agreed to indemnify the insured "against all
sums . . . which the Insured shall become legally liable to pay in respect of: a. death of or bodily injury
to any person . . . ." Clearly, therefore, it is one for indemnity against liability; from the fact then that
the insured is liable to the third person, such third person is entitled to sue the insurer.
The right of the person injured to sue the insurer of the party at fault (insured), depends on whether the
contract of insurance is intended to benefit third persons also or only the insured. And the test applied
has been this: Where the contract provides for indemnity against liability to third persons, then third
persons to whom the insured is liable, can sue the insurer. Where the contract is for indemnity against
actual loss or payment, then third persons cannot proceed against the insurer, the contract being solely
to reimburse the insured for liability actually discharged by him thru payment to third persons, said third
persons' recourse being thus limited to the insured alone.
The policy requires that suit and final judgment be first obtained against the insured; that only
"thereafter" can the person injured recover on the policy; it expressly disallows suing the insurer as a
co-defendant of the insured in a suit to determine the latter's liability. However, the "no action" clause in
the policy of insurance cannot prevail over the Rules of Court provision aimed at avoiding multiplicity of
suits.
b. Estate
Del Val vs. Del Val
G.R. No. L-9374 February 16, 1915
During the lifetime of Gregorio Nacianceno del Val, he took out insurance on his life for the sum of
P40,000 and made it payable to the defendant Andres del Val as sole beneficiary. After the death of the
insured, defendant collected the proceeds of the policy from which he paid the sum of P18,365.20 to
redeem certain real estate which the decedent had sold to third persons with a right to repurchase.
Plaintiffs filed a complaint praying for the partition of both real and personal, left by the deceased.
Plaintiffs contend that the amount of the insurance policy belonged to the estate of the deceased and

By: Elaine Marie G. Laceda

28

INSURANCE LAW

not to the defendant personally; and therefore, they are also entitled to a partition the P40,000 life
insurance. Defendant, on the other hand, asks that he be declared the owner of the real estate
redeemed by the payment of the P18,365.20 and the remaining P21,634.80 balance of the insurance
policy. CFI dismissed.
Issue: WON the proceeds of the insurance belong to the estate of the insured.
Held: No.
The contract of life insurance is a special contract and the destination of the proceeds thereof is
determined by special laws which deal exclusively with that subject. The Civil Code has no provisions
which relate directly and specifically to life-insurance contracts or to the destination of life insurance
proceeds. That subject is regulated exclusively by the Code of Commerce which provides for the terms
of the contract, the relations of the parties and the destination of the proceeds of the policy.
The proceeds of an insurance policy belong exclusively to the beneficiary and not to the estate of the
person whose life was insured, and that such proceeds are the separate and individual property of the
beneficiary, and not of the heirs of the person whose life was insured.
The proceeds of the life-insurance policy being the exclusive property of the defendant and he having
used a portion thereof in the repurchase of the real estate sold by the decedent prior to his death with
right to repurchase, and such repurchase having been made and the conveyance taken in the names of
all of the heirs instead of the defendant alone, plaintiffs claim that the property belongs to the heirs in
common and not to the defendant alone. However, it was not shown that the defendant acted as he did
with the intention that the other heirs should enjoy with him the ownership of the estate. Thus, the case
is remanded.
c.

Assignee

RCBC vs. Court of Appeals


G.R. No. 128833 April 20, 1998
GOYU applied for and was granted credit facilities and accommodations by RCBC in the amount of P117
million. As security for its credit facilities GOYU executed 2 REM and 2 CM in favor of RCBC. Under each
of these 4 mortgage contracts, GOYU committed itself to insure the mortgaged property with an
insurance company approved by RCBC, and subsequently, to endorse and deliver the insurance polices
to RCBC. Thus, GOYU obtained in its name a total of 10 insurance policies from Malayan Insurance
(MICO). In February 1992, Alchester Insurance Agency, Inc., the insurance agent where GOYU obtained
the Malayan insurance policies, issued 9 endorsements in favor of RCBC seemingly upon instructions of
GOYU.
On April 27, 1992, one of GOYU's factory buildings was gutted by fire. Consequently, GOYU submitted
its claim for indemnity on account of the loss insured against. MICO denied the claim on the ground that
the insurance policies were either attached pursuant to writs of attachments/garnishments issued by
various courts or that the insurance proceeds were also claimed by other creditors of GOYU alleging
better rights to the proceeds than the insured. Hence, GOYU filed a complaint for specific performance
and damages.
In an interlocutory order the RTC confirmed that GOYU's other creditors, namely, Urban Bank, Alfredo
Sebastian, and Philippine Trust Company obtained their respective writs of attachments from various
courts, covering an aggregate amount of P14,938,080.23, and ordered that the proceeds of the ten
insurance policies be deposited with the said court minus the aforementioned P14,938,080.23. However
after trial, RTC rendered judgment in favor of GOYU and ordered MICO to pay the plaintiff its fire loss
claims plus damages by way of interest for the duration of the delay. It also ordered RCBC to pay
plaintiff actual and compensatory damages in the amount of P2,000,000.00. CA affirmed with
modification with respect to the amount of damages.

By: Elaine Marie G. Laceda

29

INSURANCE LAW

Issue: WON RCBC, as mortgagee, has any right over the insurance policies taken by GOYU, the
mortgagor, in case of the occurrence of loss.
Held: Yes.
It is settled that a mortgagor and a mortgagee have separated and distinct insurable interests in the
same mortgaged property, such that each one of them may insure the same property for his own sole
benefit. There is no question that GOYU could insure the mortgaged property for its own exclusive
benefit. Although it appears that GOYU obtained the subject insurance policies naming itself as the sole
payee, 9 endorsement documents were prepared by Alchester in favor of RCBC. Had it not been for
GOYU, Alchester would not have known of GOYU's intention of obtaining insurance coverage in
compliance with its undertaking in the mortgage contracts with RCBC, and verily, Alchester would not
have endorsed the policies to RCBC had it not been so directed by GOYU. Thus, on equitable principles,
particularly on the ground of estoppel, the Court is constrained to rule in favor of mortgagor RCBC.
GOYU failed to seasonably repudiate the authority of the person or persons who prepared such
endorsements. If there had not been actually an implied ratification of said endorsements by virtue of
GOYU's inaction in this case, GOYU is at the very least estopped from assailing their operative effects.
To permit GOYU to capitalize on its non-confirmation of these endorsements while it continued to enjoy
the benefits of the credit facilities of RCBC which believed in good faith that there was due endorsement
pursuant to their mortgage contracts, is to countenance grave contravention of public policy, fair
dealing, good faith, and justice.
GOYU cannot seek relief under Section 53 of the Insurance Code which provides that the proceeds of
insurance shall exclusively apply to the interest of the person in whose name or for whose benefit it is
made. The peculiarity of the circumstances obtaining in the instant case presents a justification to take
exception to the strict application of said provision, it having been sufficiently established that it was the
intention of the parties to designate RCBC as the party for whose benefit the insurance policies were
taken out.
Section 53 of the Insurance Code ordains that the insurance proceeds of the endorsed policies shall be
applied exclusively to the proper interest of the person for whose benefit it was made. In this case, to
the extent of GOYU's obligation with RCBC, the interest of GOYU in the subject policies had been
transferred to RCBC effective as of the time of the endorsement. These policies may no longer be
attached by the other creditors of GOYU.
3. Designation of Beneficiary
a. No Designation
In re: Mario V. Chanliongco
A.M. No. 190 October 18, 1977
At the time of his death on July 12, 1976, Atty. Chanliongco was more than 63 years of age, with more
than 38 years of service in the government. His heirs filed with the GSIS a claim for the following
benefits: (1) retirement benefits; (2) money value of terminal leave; (3) life insurance; and (4) refund
of retirement premium. Acting on the claim, the GSIS released the life insurance proceeds; and the
refund of retirement premium.
Issue: To whom should the retirement benefits accrue on account of the failure to designate a
beneficiary?
Held: The retirement benefits shall accrue to the estate and will be distributed among the legal heirs in
accordance with intestate succession, as in the case of life insurance where no beneficiary is named in
the policy.
Vda. De Consuegra vs. GSIS
G.R. No. L-28093 January 30, 1971

By: Elaine Marie G. Laceda

30

INSURANCE LAW

Jose Consuegra, at the time of his death, was employed as a shop foreman of the office of the District
Engineer in the province of Surigao del Norte. In his lifetime, Consuegra contracted two marriages, the
first with Rosario Diaz, out of which marriage were born two children who both predeceased their father;
and the second, which was contracted in good faith while the first marriage was subsisting, with herein
petitioner Basilia Berdin, out of which marriage were born seven children.
When Consuegra died, the proceeds of his life insurance were paid by the GSIS to petitioner Basilia
Berdin and her children who were the beneficiaries named in the policy. However, Consuegra did not
designate any beneficiary who would receive the retirement insurance benefits due to him. Thus,
respondent Rosario Diaz, the widow by the first marriage, filed a claim with the GSIS asking that the
retirement insurance benefits be paid to her as the only legal heir of Consuegra, considering that the
deceased did not designate any beneficiary with respect to his retirement insurance benefits. Petitioner
Basilia Berdin and her children, likewise, filed a similar claim with the GSIS, asserting that being the
beneficiaries named in the life insurance policy of Consuegra, they are the only ones entitled to receive
the retirement insurance benefits due the deceased Consuegra.
Resolving the conflicting claims, the GSIS ruled that the legal heirs of the late Jose Consuegra were
Rosario Diaz, his widow by his first marriage who is entitled to one-half, or 8/16, of the retirement
insurance benefits, on the one hand; and Basilia Berdin, his widow by the second marriage and their
seven children, on the other hand, who are entitled to the remaining one-half, or 8/16, each of them to
receive an equal share of 1/16.
Dissatisfied with the foregoing ruling and apportionment made by the GSIS, Basilia Berdin and her
children filed petition for mandamus with preliminary injunction praying that they be declared the legal
heirs and exclusive beneficiaries of the retirement insurance of the late Jose Consuegra. The CFI held
that both parties are entitled to 1/2 of the estate.
Issue: Who is entitled to the retirement insurance benefits?
Held: Estate; 1/2 each
Upon entering the government service Consuegra became a compulsory member of the GSIS, being
automatically insured on his life, pursuant to the provisions of CA 186 which was in force at the time.
When Consuegra designated his beneficiaries in his life insurance he could not have intended those
beneficiaries of his life insurance as also the beneficiaries of his retirement insurance because the
provisions on retirement insurance under the GSIS came about only when CA 186 was amended by Rep.
Act 660 on June 16, 1951. Hence, it cannot be said that because herein appellants were designated
beneficiaries in Consuegra's life insurance they automatically became the beneficiaries also of his
retirement insurance.
Under sec. 11(b) of CA 186, as amended by RA 660, there is need for the employee to file an application
for retirement insurance benefits when he becomes a member of the GSIS, and he should state in his
application the beneficiary of his retirement insurance. Hence, the beneficiary named in the life
insurance does not automatically become the beneficiary in the retirement insurance unless the same
beneficiary in the life insurance is so designated in the application for retirement insurance.
Retirement insurance is primarily intended for the benefit of the employee to provide for his old age,
or incapacity, after rendering service in the government for a required number of years. If the employee
reaches the age of retirement, he gets the retirement benefits even to the exclusion of the beneficiary or
beneficiaries named in his application for retirement insurance. The beneficiary of the retirement
insurance can only claim the proceeds of the retirement insurance if the employee dies before
retirement. If the employee failed or overlooked to state the beneficiary of his retirement insurance, the
retirement benefits will accrue to his estate and will be given to his legal heirs in accordance with law, as
in the case of a life insurance if no beneficiary is named in the insurance policy.

b.

Invalid Designation Art 2012 in relation to Art 739, Civil Code

By: Elaine Marie G. Laceda

31

INSURANCE LAW

Art. 2012. Any person who is forbidden from receiving any donation under Article 739 cannot be
named beneficiary of a life insurance policy by the person who cannot make any donation to him,
according to said article. (n)
Art. 739. The following donations shall be void:
(1) Those made between persons who were guilty of adultery or concubinage at the time of the
donation;
(2) Those made between persons found guilty of the same criminal offense, in consideration
thereof;
(3) Those made to a public officer or his wife, descedants and ascendants, by reason of his
office.
In the case referred to in No. 1, the action for declaration of nullity may be brought by the
spouse of the donor or donee; and the guilt of the donor and donee may be proved by preponderance of
evidence in the same action. (n)
Insular Life Assurance Company, Ltd. vs. Ebrado
G.R. No. L-44059 October 28, 1977
Buenaventura Ebrado was issued by The Life Assurance Co., Ltd., Policy No. 009929 on a whole-life for
P5,882.00 with a, rider for Accidental Death for the same amount. He designated Carponia T. Ebrado as
the revocable beneficiary in his policy.
Buenaventura C. Ebrado died as a result of an accident when he was hit by a failing branch of a tree.
Thus, Carponia Ebrado filed claim with the Insular Life Assurance Co. which was contested by Pascuala
Ebrado who also filed claim for the proceeds of said policy. The latter asserts that she is the one entitled
to the insurance proceeds, not the common-law wife.
In doubt as to whom the insurance proceeds shall be paid, the insurer, The Insular Life Assurance Co.,
Ltd. commenced an action for Interpleader before the CFI.
CFI declared Carponia T. Ebrado disqualified from becoming beneficiary of the insured and directed the
payment of the insurance proceeds to the estate of the deceased insured.
Issue: Can a common-law wife named as beneficiary in the life insurance policy of a legally married man
claim the proceeds thereof in case of death of the latter?
Held: No.
In essence, a life insurance policy is no different from a civil donation insofar as the beneficiary is
concerned. Both are founded upon the same consideration: liberality. A beneficiary is like a donee,
because from the premiums of the policy which the insured pays out of liberality, the beneficiary will
receive the proceeds or profits of said insurance. As a consequence, the proscription in Article 739 of the
new Civil Code: Those made between persons who were guilty of adultery or concubinage at the time of
donation, should equally operate in life insurance contracts.
No criminal conviction for the offense is a condition precedent before the disabilities mentioned in Article
739 may effectuate. The guilt of the donee may be proved by preponderance of evidence in an action for
declaration of nullity of the donation.
The requisite proof of common-law relationship between the insured and the beneficiary has been
conveniently supplied by the stipulations between the parties in the pre-trial conference of the case.
These stipulations are nothing less than judicial admissions which, as a consequence, no longer require
proof and cannot be contradicted.
Southern Luzon Employees' Association vs. Golpeo
G.R. No. L-6114 October 30, 1954

By: Elaine Marie G. Laceda

32

INSURANCE LAW

Roman A. Concepcion was a member, until his death on December 13, 1950, of Southern Luzon
Employees' Association, one of the purposes of which is mutual aid of its members and their defendants
in case of death. In the form required by the association to be accomplished by its members, with
reference to the death benefit, Roman A. Concepcion listed as his beneficiaries Aquilina Maloles, Roman
M. Concepcion, Jr., Estela M. Concepcion, Rolando M. Concepcion and Robin M. Concepcion.
After the death of Roman A. Concepcion, 3 sets of claimants presented themselves: Juanita Golpeo,
legal wife and her children; the named beneficiaries by the deceased; and Elsie Hicban, another
common law wife and her child. Thus, plaintiff association was accordingly constrained to institute an
action for interpleader against the conflicting claimants as defendants. After hearing, the CFI rendered a
decision, declaring the defendants Aquilina Maloles and her children the sole beneficiaries of the sum of
P2,505.00, and ordering the plaintiff to deliver said amount to them.
Issue: WON the designation of beneficiary is void.
Held: Partially void.
Article 2012 of the NCC provides that "Any person who is forbidden from receiving any donation under
article 739 cannot be named beneficiary of a life insurance policy." Inasmuch as, according to article 739
of the NCC, a donation is valid when made "between persons who are guilty or adultery or concubinage
at the time of the donation," it is alleged that the defendant-appellee Aquilina Maloles, cannot be named
a beneficiary. However, the argument would certainly not apply to the children of Aquilina likewise
named beneficiaries by the deceased Roman A. Concepcion. As a matter of a fact the new Civil Code
recognized certain successional rights of illegitimate children.
SSS vs. Davac
G.R. No. L-21642 July 30, 1966
The late Petronilo Davac, a former employee of Lianga Bay Logging Co., Inc. became a member of the
SSS on September 1, 1957. In SSS form E-1 which he accomplished and filed with the SSS, he
designated respondent Candelaria Davac as his beneficiary and indicated his relationship to her as that
of his "wife". Upon his death on April 5, 1959 respondents Candelaria Davac and Lourdes Tuplano (both
of whom was married to deceased) filed their claims for death benefit with the SSS. Due to their
conflicting claims, SSS filed a petition for interpleader. The SSS later issued a resolution declaring
respondent Candelaria Davac as the person entitled to receive the death benefits payable for the death
of Petronilo Davac.
Issue: WON the SSC acted correctly in declaring respondent Candelaria Davac as the person entitled to
receive the death benefits in question.
Held: Yes.
Under Section 13, RA No. 1161, as amended by RA No. 1792, in force at the time Petronilo Davac's
death, the beneficiary "as recorded" by the employee's employer is the one entitled to the death
benefits.
Without deciding whether the naming of a beneficiary of the benefits accruing from membership in the
SSS is a donation, or that it creates a situation analogous to the relation of an insured and the
beneficiary under a life insurance policy, it is enough, for the purpose of the instant case, to state that
the disqualification mentioned in Article 739 is not applicable to herein appellee Candelaria Davac
because she was not guilty of concubinage, there being no proof that she had knowledge of the previous
marriage of her husband Petronilo.
Moreover, the benefit receivable under the Act is in the nature of a special privilege or an arrangement
secured by the law, pursuant to the policy of the State to provide social security to the workingmen. The
amounts that may thus be received cannot be considered as property earned by the member during his
lifetime. His contribution to the fund, it may be noted, constitutes only an insignificant portion thereof.

By: Elaine Marie G. Laceda

33

INSURANCE LAW

If there is a named beneficiary and the designation is not invalid (as it is not so in this case), it is not
the heirs of the employee who are entitled to receive the benefits (unless they are the designated
beneficiaries themselves). It is only when there is no designated beneficiaries or when the designation is
void, that the laws of succession are applicable.
c.

Revocable Designation Sec 11, I.C.

Sec. 11. The insured shall have the right to change the beneficiary he designated in the policy, unless
he has expressly waived this right in said policy.
Gercio vs. Sun Life Assurance of Canada, et al.
G.R. No. 23703 September 28, 1925
On January 29, 1910, the Sun Life Assurance Co. of Canada issued insurance policy No. 161481 on the
life of Hilario Gercio. The policy was what is known as a twenty-year endowment policy. By its terms,
the insurance company agreed to insure the life of Hilario Gercio for the sum of P2,000, to be paid him
on February 1, 1930, or if the insured should die before said date, then to his wife, Mrs. Andrea Zialcita,
should she survive him; otherwise to the executors, administrators, or assigns of the insured. The policy
did not include any provision reserving to the insured the right to change the beneficiary.
On September 4, 1920, a decree of divorce was issued in civil case no. 17955, which had the effect of
completely dissolving the bonds of matrimony contracted by Hilario Gercio and Andrea Zialcita. Hilario
Gercio formally notified the Sun Life Assurance Co. of Canada that he had revoked his donation in favor
of Andrea Zialcita, and that he had designated in her stead his present wife, Adela Garcia de Gercio, as
the beneficiary of the policy. The insurance company refused.
A petition for mandamus was filed to compel the defendant Sun Life Assurance Co. of Canada to change
the beneficiary in the policy. The trial court ruled in favor of the plaintiff and ordered the defendant
company to eliminate from the insurance policy the name of Andrea Zialcita as beneficiary and to
substitute therefor such name as the plaintiff might furnish to the defendant for that purpose.
Issue: Whether the insured may change the beneficiary (the former wife) and to name instead his actual
wife, where the insured and the beneficiary have been divorced and where the policy of insurance does
not expressly reserve to the insured the right to change the beneficiary.
Held: No.
A person who procures a policy upon his own life, payable to a designated beneficiary, although he pays
the premiums himself, and keeps the policy in his exclusive possession, has no power to change the
beneficiary, unless the policy itself, or the charter of the insurance company, so provides. The
beneficiary has a vested right in the policy, of which she cannot be deprived without her consent.
As to the effect produced by the divorce, the Philippine Divorce Law, Act No. 2710, merely provides in
section 9 that the decree of divorce shall dissolve the community property as soon as such decree
becomes final. Unlike the statutes of a few jurisdictions, there is no provision in the Philippine Law
permitting the beneficiary in a policy for the benefit of the wife of the husband to be changed after a
divorce. It must follow, therefore, in the absence of a statute to the contrary, that if a policy is taken out
upon a husband's life the wife is named as beneficiary therein, a subsequent divorce does not destroy
her rights under the policy.
d. Irrevocable Designation
Nario vs. The Philippine American Life Insurance Company
G.R. No. L-22796 June 26, 1967
Mrs. Alejandra Santos-Mario was issued by the Philippine American Life Insurance Co., a life insurance
policy under a 20-year endowment plan, with her husband, Delfin Nario, and their unemancipated minor
son, Ernesto Nario, as her irrevocable beneficiaries. About the middle of June, 1963, Mrs. Nario applied

By: Elaine Marie G. Laceda

34

INSURANCE LAW

for a loan with the Insurance Company, which loan she, as policy-holder, has been entitled to avail of
under one of the provisions of said policy, for the purpose of using the proceeds thereof for the school
expenses of her minor son. Said application bore the written signature and consent of Delfin Nario as
one of the irrevocable beneficiaries of the policy; and as the father-guardian of said minor son and
irrevocable beneficiary, Ernesto Nario. The Insurance Company denied said application, manifesting to
the policy holder that the written consent for the minor son must not only be given by his father as legal
guardian but it must also be authorized by the court in a competent guardianship proceeding.
After the denial of said policy loan application, Mrs. Nario signified her decision to surrender her policy to
the Insurance Company, which she was also entitled to avail of under one of the provisions of the same
policy, and demanded its cash value which then amounted to P520.00. This was likewise denied on the
same ground as the denial of the loan application. Thus, Mrs. Nario and her husband brought suit
against the Philippine American Life Insurance Co to compel them to grant their policy loan application
and/or to accept the surrender of said policy in exchange for its cash value. CFI sustained the view of
the Insurance Company.
Issue: WON consent of the minor beneficiary may be made by the guardian only by proper court order.
Held: Yes.
The vested interest or right of the beneficiaries in the policy should be measured on its full face value
and not on its cash surrender value, for in case of death of the insured, said beneficiaries are paid on
the basis of its face value and in case the insured should discontinue paying premiums, the beneficiaries
may continue paying it and are entitled to automatic extended term or paid-up insurance options, etc.
and that said vested right under the policy cannot be divisible at any given time.
Under Article 320 of the Civil Code, the father, or in his absence the mother, is the legal administrator of
the property pertaining to the child under parental authority. However, if the property is worth more
than P2,000, the father or mother shall give a bond subject to the approval of the CFI.
It appearing that the minor beneficiary's vested interest or right on the policy exceeds P2,000; that
plaintiffs did not file any guardianship bond to be approved by the court; and as later implemented in
the abovequoted Section 7, Rule 93 of the Revised Rules of Court, plaintiffs should have, but, had not,
filed a formal application or petition for guardianship, plaintiffs-parents cannot possibly exercise the
powers vested on them, as legal administrators of their child's property, under article 320 of the Civil
Code. As there was no such petition and bond, the consent given by the father-guardian, for and in
behalf of the minor son, without prior court authorization, to the policy loan application and the
surrender of said policy, was insufficient and ineffective, and defendant-appellee was justified in
disapproving the proposed transactions in question.
It must be noted that the proposed transactions in question (policy loan and surrender of policy)
constitute acts of disposition or alienation of property rights and not merely of management or
administration because they involve the incurring or termination of contractual obligations. Hence,t
result would be the same even if we regarded the interest of the ward to be worth less than P2,000.
While the father or mother would in such event be exempt from the duty of filing a bond, and securing
judicial appointment, still the parent's authority over the estate of the ward as a legal-guardian would
not extend to acts of encumbrance or disposition, as distinguished from acts of management or
administration.
The Philippine American Insurance Company vs. Pineda
G.R. No. L-54216 July 19, 1989
On January 15, 1968, private respondent Dimayuga procured an ordinary life insurance policy from the
petitioner company and designated his wife and children as irrevocable beneficiaries of said policy. On
February 22, 1980 private respondent filed a petition with the CFI to amend the designation of the
beneficiaries in his life policy from irrevocable to revocable. CFI judge granted the petition.

By: Elaine Marie G. Laceda

35

INSURANCE LAW

Issue: WON the designation of the irrevocable beneficiaries could be changed or amended without the
consent of all the irrevocable beneficiaries.
Held: No.
Under the Insurance Act, the beneficiary designated in a life insurance contract cannot be changed
without the consent of the beneficiary because he has a vested interest in the policy.
Similarly, the alleged acquiescence of the 6 children beneficiaries of the policy (the beneficiary-wife
predeceased the insured) cannot be considered an effective ratification to the change of the beneficiaries
from irrevocable to revocable. Indubitable is the fact that all the 6 children named as beneficiaries were
minors at the time, for which reason, they could not validly give their consent. Neither could they act
through their father insured since their interests are quite divergent from one another.
e. Exceptions to Irrevocable Designation Arts 43(4), 50, 64, Family Code
Art. 43 (4). The innocent spouse may revoke the designation of the other spouse who acted in bad
faith as beneficiary in any insurance policy, even if such designation be stipulated as irrevocable;
Art. 50. The effects provided for by paragraphs (2), (3), (4) and (5) of Article 43 and by Article 44 shall
also apply in the proper cases to marriages which are declared ab initio or annulled by final judgment
under Articles 40 and 45.
The final judgment in such cases shall provide for the liquidation, partition and distribution of
the properties of the spouses, the custody and support of the common children, and the delivery of third
presumptive legitimes, unless such matters had been adjudicated in previous judicial proceedings.
All creditors of the spouses as well as of the absolute community or the conjugal partnership
shall be notified of the proceedings for liquidation.
In the partition, the conjugal dwelling and the lot on which it is situated, shall be adjudicated in
accordance with the provisions of Articles 102 and 129.
Art. 64. After the finality of the decree of legal separation, the innocent spouse may revoke the
donations made by him or by her in favor of the offending spouse, as well as the designation of the
latter as beneficiary in any insurance policy, even if such designation be stipulated as irrevocable. The
revocation of the donations shall be recorded in the registries of property in the places where the
properties are located. Alienations, liens and encumbrances registered in good faith before the recording
of the complaint for revocation in the registries of property shall be respected. The revocation of or
change in the designation of the insurance beneficiary shall take effect upon written notification thereof
to the insured.
The action to revoke the donation under this Article must be brought within five years from the
time the decree of legal separation become final. (107a)
4. Forfeiture Sec 12, I.C.
Sec. 12. The interest of a beneficiary in a life insurance policy shall be forfeited when the beneficiary is
the principal, accomplice, or accessory in willfully bringing about the death of the insured; in which
event, the nearest relative of the insured shall receive the proceeds of said insurance if not otherwise
disqualified.
5. Insured Outlives Policy Sec 180, I.C.
Sec. 180. An insurance upon life may be made payable on the death of the person, or on his surviving
a specified period, or otherwise contingently on the continuance or cessation of life.
Every contract or pledge for the payment of endowments or annuities shall be considered a life
insurance contract for purpose of this Code.
In the absence of a judicial guardian, the father, or in the latter's absence or incapacity, the
mother, or any minor, who is an insured or a beneficiary under a contract of life, health or accident
insurance, may exercise, in behalf of said minor, any right under the policy, without necessity of court
authority or the giving of a bond, where the interest of the minor in the particular act involved does not
exceed twenty thousand pesos. Such right may include, but shall not be limited to, obtaining a policy

By: Elaine Marie G. Laceda

36

INSURANCE LAW

loan, surrendering the policy, receiving the proceeds of the policy, and giving the minor's consent to any
transaction on the policy.
Villanueva vs. Oro
G.R. No. L-2227 August 31, 1948
The West Coast Life Insurance Company issued two policies of insurance on the life of Esperanza J.
Villanueva. In both policies the insurer agreed to pay the insured, if living, on April 1, 1943, or the
beneficiary Bartolome Villanueva, father of the insured, immediately upon receipt of due proof of the
prior death of the insured, Esperanza J. Villanueva, during the continuance of this policy, with right on
the part of the insured to change the beneficiary. After the death of the named beneficiary Mariano J.
Villanueva, a brother of the insured, was substituted as beneficiary.
Esperanza survived the insurance period without, however, collecting the insurance proceeds. Hence,
adverse claims for said proceeds were presented by the estate of Esperanza J. Villanueva on the one
hand and by Mariano J. Villanueva. The CFI held the estate of the insured as entitled to the insurance
proceeds, to the exclusion of the beneficiary, Mariano J. Villanueva.
Issue: WON the estate of the insured is entitled to the insurance proceeds.
Held: Yes.
Under the policies, the insurer obligated itself to pay the insurance proceeds (1) to the insured if the
latter lived on the dates of maturity or (2) to the beneficiary if the insured died during the continuance
of the policies. The first contingency of course excludes the second, and vice versa. In other words, as
the insured Esperanza J. Villanueva was still alive on April 1, 1943, the proceeds are payable exclusively
to her estate unless she had before her death otherwise assigned the matured policies.
If a policy of insurance provides that the proceeds shall be payable to the assured, if he lives to a certain
date, and, in case of his death before that date, then they shall be payable to the beneficiary
designated, the interest of the beneficiary is a contingent one, and the benefit of the policy will only
inure to such beneficiary in case the assured dies before the end of the period designated in the policy.
D.

INSURANCE AGENT AND INSURANCE BROKER

1. Insurance Agent Sec 300, I.C.


Sec. 300. Any person who for compensation solicits or obtains insurance on behalf of any insurance
company or transmits for a person other than himself an application for a policy or contract of insurance
to or from such company or offers or assumes to act in the negotiating of such insurance shall be an
insurance agent within the intent of this section and shall thereby become liable to all the duties,
requirements, liabilities and penalties to which an insurance agent is subject.
Aisporna vs. Court of Appeals
G.R. No. L-39419 April 12, 1982
Petitioner Aisporna was charged with violation of Section 189 of the Insurance Act for acting as agent in
the solicitation or procurement of an application for insurance, by actively participating with her licensed
husband in the issuance of a Personal Accident Policy of one Eugenio S. Isidro for and in behalf of Perla
Compaa de Seguros, without having first secured a certificate of authority to act as such agent from
the office of the Insurance Commissioner. City Court found petitioner guilty. CA affirmed.
Issue: WON petitioner is guilty of violating Sec. 189 of the Insurance Act.
Held: No.
The definition of an insurance agent as found in the second paragraph of Section 189 is intended to
define the word "agent" mentioned in the first and second paragraphs of the aforesaid section -- Any

By: Elaine Marie G. Laceda

37

INSURANCE LAW

person who for compensation ... shall be an insurance agent within the intent of this section. Therefore,
considering that the definition of an insurance agent as found in the second paragraph is also applicable
to the agent mentioned in the first paragraph, to receive a compensation by the agent is an essential
element for a violation of the first paragraph of the aforesaid section. The appellate court has
established ultimately that the petitioner-accused did not receive any compensation for the issuance of
the insurance policy of Eugenio Isidro. Thus, there was no violation of Sec. 189 of the Insurance Act.
2. Insurance Broker Sec 301, I.C.
Sec. 301. Any person who for any compensation, commission or other thing of value acts or aids in any
manner in soliciting, negotiating or procuring the making of any insurance contract or in placing risk or
taking out insurance, on behalf of an insured other than himself, shall be an insurance broker within the
intent of this Code, and shall thereby become liable to all the duties, requirements, liabilities and
penalties to which an insurance broker is subject.
Philippine Health-Care Providers, Inc. vs. Estrada
G.R. No. 171052 January 28, 2008
Maxicare, a domestic corporation engaged in selling health insurance plans, engaged the services of
Carmela Estrada who was doing business under the name of Cara Health Services to promote and sell
the prepaid group practice health care delivery program called MAXICARE Plan. Maxicare formally
appointed Estrada as its "General Agent," evidenced by a letter-agreement dated February 16, 1991.
The letter agreement provided for plaintiff-appellees Estradas compensation in the form of commission
equivalent to 15 to 18% from individual, family, group accounts; 2.5 to 10% on tailored fit plans; and
10% on standard plans of commissionable amount on corporate accounts from all membership dues
collected and remitted.
Maxicare alleged that it followed a "franchising system" in dealing with its agents whereby an agent had
to first secure permission from Maxicare to list a prospective company as client. Estrada alleged that it
did apply with Maxicare for the MERALCO account and other accounts, and in fact, its franchise to solicit
corporate accounts, MERALCO account included, was renewed on February 11, 1991.
Plaintiff-appellee Estrada submitted proposals and made representations to the officers of MERALCO
regarding the MAXICARE Plan but when MERALCO decided to subscribe to the MAXICARE Plan, Maxicare
directly negotiated with MERALCO regarding the terms and conditions of the agreement and left plaintiffappellee Estrada out of the discussions on the terms and conditions. MERALCO eventually subscribed to
the MAXICARE Plan and signed a Service Agreement directly with Maxicare for medical coverage of its
qualified members.
On March 24, 1992, plaintiff-appellee Estrada, through counsel, demanded from Maxicare that it be paid
commissions for the MERALCO account and 9 other accounts. In reply, Maxicare, through counsel,
denied Estradas claims for commission for the MERALCO and other accounts because Maxicare directly
negotiated with MERALCO and the other accounts, and that no agent was given the go signal to
intervene in the negotiations for the terms and conditions and the signing of the service agreement with
MERALCO and the other accounts. Thus, Estrada filed a complaint against Maxicare and its officers with
the RTC.
After trial, the RTC found Maxicare liable for breach of contract and ordered it to pay Estrada actual
damages in the amount equivalent to 10% of P20,169,335.00, representing her commission for the total
premiums paid by Meralco to Maxicare from the year 1991 to 1996, plus legal interest computed from
the filing of the complaint on March 18, 1993, and attorneys fees in the amount of P100,000.00. CA
affirmed in toto. In ruling for Estrada, both the trial and appellate courts held that Estrada was the
"efficient procuring cause" in the execution of the service agreement between Meralco and Maxicare.
Issue: Whether Estrada is entitled to commissions for the 2 consecutive renewals of the service
agreement effective on December 1, 1992 and December 1, 1995.
Held: Yes.

By: Elaine Marie G. Laceda

38

INSURANCE LAW

Estrada is entitled to commissions for the premiums paid under the service agreement between Meralco
and Maxicare from 1991 to 1996. Maxicare successfully landed the Meralco account for the sale of
healthcare plans only by virtue of Estradas involvement and participation in the negotiations. At the
very least, Estrada penetrated the Meralco market, initially closed to Maxicare, and laid the groundwork
for a business relationship. The only reason Estrada was not able to participate in the collection and
remittance of premium dues to Maxicare was because she was prevented from doing so by the acts of
Maxicare, its officers, and employees.
The term "procuring cause" in describing a brokers activity, refers to a cause originating a series of
events which, without break in their continuity, result in the accomplishment of the prime objective of
the employment of the brokerproducing a purchaser ready, willing and able to buy on the owners
terms. To be regarded as the "procuring cause" of a sale as to be entitled to a commission, a brokers
efforts must have been the foundation on which the negotiations resulting in a sale began. Verily,
Estrada was instrumental in the sale of the Maxicare health plans to Meralco. Without her intervention,
no sale could have been consummated.
3. Authority to Receive Payment/ Effect of Payment Sec 306, I.C.
Sec. 306. The premium, or any portion thereof, which an insurance agent or insurance broker collects
from an insured and which is to be paid to an insurance company because of the assumption of liability
through the issuance of policies or contracts of insurance, shall be held by the agent or broker in a
fiduciary capacity and shall not be misappropriated or converted to his own use or illegally withheld by
the agent or broker.
Any insurance company which delivers to an insurance agent or insurance broker a policy or
contract of insurance shall be deemed to have authorized such agent or broker to receive on its behalf
payment of any premium which is due on such policy or contract of insurance at the time of its issuance
or delivery or which becomes due thereon.
Malayan Insurance Co., Inc. vs. Arnaldo
G.R. No. L-67835 October 12, 1987
Petitioner issued to private respondent, Coronacion Pinca, Fire Insurance Policy No. F-001-17212 on her
property for the amount of P14,000.00 effective July 22, 1981, until July 22, 1982. However, on October
15, 1981, MICO allegedly cancelled the policy for non-payment of the premium and sent the
corresponding notice to Pinca. Payment of the premium was then later received by Domingo Adora,
agent of MICO who remitted this payment to MICO, together with other payments on January 15, 1982.
On January 18, 1982, Pinca's property was completely burned. On February 5, 1982, Pinca's payment
was returned by MICO to Adora on the ground that her policy had been cancelled earlier. But Adora
refused to accept it. In due time, Pinca made the requisite demands for payment, which MICO rejected.
She then went to the Insurance Commission which sustained her claim for compensation for her burned
property.
Issue: WON Adora had the authority to receive payment.
Held: Yes.
MICO's acknowledgment of Adora as its agent defeats its contention that he was not authorized to
receive the premium payment on its behalf. It is clearly provided in Section 306 of the Insurance Code
that: Any insurance company which delivers to an insurance agant or insurance broker a policy or
contract of insurance shall be demmed to have authorized such agent or broker to receive on its behalf
payment of any premium which is due on such policy or contract of insurance at the time of its issuance
or delivery or which becomes due thereon.
It is also a well-known principle under the law of agency that: Payment to an agent having authority to
receive or collect payment is equivalent to payment to the principal himself; such payment is complete
when the money delivered is into the agent's hands and is a discharge of the indebtedness owing to the
principal.

By: Elaine Marie G. Laceda

39

INSURANCE LAW

Issue: WON the policy was subsisting at the time of the fire.
Held: Yes.
MICO's claims it cancelled the policy in question on October 15, 1981, for non-payment of premium. To
support this assertion, it presented one of its employees, who testified that "the original of the
endorsement and credit memo" presumably meaning the alleged cancellation "were sent the
assured by mail through our mailing section" However, there is no proof that the notice, assuming it
complied with the other requisites for cancellation, was actually mailed to and received by Pinca. All
MICO's offers to show that the cancellation was communicated to the insured is its employee's testimony
that the said cancellation was sent "by mail through our mailing section." without more.
It stands to reason that if Pinca had really received the said notice, she would not have made payment
on the original policy on December 24, 1981. Instead, she would have asked for a new insurance,
effective on that date and until one year later, and so taken advantage of the extended period. The
Court finds that if she did pay on that date, it was because she honestly believed that the policy issued
on June 7, 1981, was still in effect and she was willing to make her payment retroact to July 22, 1981,
its stipulated commencement date. After all, the premium invoice issued to Pinca at the time of the
delivery of the policy on June 7, 1981 was stamped "Payment Received" of the amount of P930.60 on
"12-24-81" by Domingo Adora. This is important because it suggests an understanding between MICO
and the insured that such payment could be made later, as agent Adora had assured Pinca.
South Sea Surety and Insurance Co., Inc. vs. Court of Appeals
G.R. No. 102253 June 2, 1995
On 16 January 1984, plaintiff Valenzuela Hardwood and Industrial Supply, Inc. entered into an
agreement with the defendant Seven Brothers whereby the latter undertook to load on board its vessel
M/V Seven Ambassador the former's lauan round logs numbering 940 at the port of Maconacon, Isabela
for shipment to Manila. The logs were insured by plaintiff against loss and/or, damage with defendant
South Sea Surety and Insurance Co., Inc. for P2M. On 24 January 1984, the plaintiff gave the check in
payment of the premium on the insurance policy to Mr. Victorio Chua. In the meantime, the said vessel
M/V Seven Ambassador sank on 25 January 1984 resulting in the loss of the plaintiffs insured logs.
On 30 January 1984, a check for P5,625.00 to cover payment of the premium and documentary stamps
due on the policy was tendered to the insurer but was not accepted. Instead, the South Sea Surety and
Insurance Co., Inc. cancelled the insurance policy it issued as of the date of inception for non-payment
of the premium. Thus, insurer denied liability under the policy when plaintiff demanded for payment
thereof. A formal claim with defendant Seven Brothers Shipping Corporation was likewise denied.
Plaintiff then filed a complaint for the recovery of the value of lost logs and freight charges from Seven
Brothers Shipping Corporation or, to the extent of its alleged insurance cover, from South Sea Surety
and insurance Company. RTC decided for plaintiff. CA affirmed but absolved the shipping entity from
liability on the basis of a stipulation in the charter party that the ship owner would be exempted from
liability in case of loss.
Issue: WON Victorio Chua acted as an agent of the surety company or of the insured when he received
the check for insurance premiums.
Held: Yes.
Undoubtedly, the payment of the premium is a condition precedent to, and essential for, the
efficaciousness of the contract. The only two statutorily provided exceptions are (a) in case the
insurance coverage relates to life or industrial life (health) insurance when a grace period applies and
(b) when the insurer makes a written acknowledgment of the receipt of premium, this acknowledgment
being declared by law to be then conclusive evidence of the premium payment.

By: Elaine Marie G. Laceda

40

INSURANCE LAW

Section 306 of the Insurance Code provides as follows: Any insurance company which delivers to an
insurance agent or insurance broker a policy or contract of insurance shall be deemed to have
authorized such agent or broker to receive on its behalf payment of any premium which is due on such
policy of contract of insurance at the time of its issuance or delivery or which becomes due thereon.
On cross-examination in behalf of South Sea Surety and Insurance Co., Inc. Mr. Chua testified that the
marine cargo insurance policy for the plaintiff's logs was delivered to him, to be delivered to the plaintiff.
Thus, when the appellant South Sea Surety and Insurance Co., Inc. delivered to Mr. Chua the marine
cargo insurance policy for the plaintiffs logs, he is deemed to have been authorized by the South Sea
Surety and Insurance Co., Inc. to receive the premium which is due on its behalf. Therefore, when the
insured logs were lost, the insured had already paid the premium to an agent of the South Sea Surety
and Insurance Co., Inc., which is consequently liable to pay the insurance proceeds under the policy it
issued to the insured.
III.

INSURABLE INTEREST
A.

1.

CONCEPT
Definition in general, a person has an insurable interest in the subject matter
insured where he has such a relation or connection with, or concern in, such subject
matter that he derive pecuniary benefit or advantage from its preservation or will
suffer pecuniary loss or damage from its destruction, termination or injury by the
happening of the event insured against.

2. Necessity of Insurable Interest; Consequence of Lack of Secs 3, 4, 18 and 25, I.C.


Sec. 3. Any contingent or unknown event, whether past or future, which may damnify a person having
an insurable interest, or create a liability against him, may be insured against, subject to the provisions
of this chapter.
The consent of the husband is not necessary for the validity of an insurance policy taken out by
a married woman on her life or that of her children.
Any minor of the age of eighteen years or more, may, notwithstanding such minority, contract
for life, health and accident insurance, with any insurance company duly authorized to do business in
the Philippines, provided the insurance is taken on his own life and the beneficiary appointed is the
minor's estate or the minor's father, mother, husband, wife, child, brother or sister.
The married woman or the minor herein allowed to take out an insurance policy may exercise all
the rights and privileges of an owner under a policy.
All rights, title and interest in the policy of insurance taken out by an original owner on the life
or health of a minor shall automatically vest in the minor upon the death of the original owner, unless
otherwise provided for in the policy.
Sec. 4. The preceding section does not authorize an insurance for or against the drawing of any lottery,
or for or against any chance or ticket in a lottery drawing a prize.
Sec. 18. No contract or policy of insurance on property shall be enforceable except for the benefit of
some person having an insurable interest in the property insured.
Sec. 25. Every stipulation in a policy of insurance for the payment of loss whether the person insured
has or has not any interest in the property insured, or that the policy shall be received as proof of such
interest, and every policy executed by way of gaming or wagering, is void.
B.

IN LIFE AND HEALTH INSURANCE

1. Who Has Insurable Interest Sec 10, I.C.


Sec. 10. Every person has an insurable interest in the life and health:
(a) Of himself, of his spouse and of his children;

By: Elaine Marie G. Laceda

41

INSURANCE LAW

(b)
(c)
(d)

Of any person on whom he depends wholly or in part for education or support, or in whom
he has a pecuniary interest;
Of any person under a legal obligation to him for the payment of money, or respecting
property or services, of which death or illness might delay or prevent the performance; and
Of any person upon whose life any estate or interest vested in him depends.
a. Blood relationship Sec 10(a), I.C.

PhilamCare Health Systems, Inc. vs. Court of Appeals


G.R. No. 125678 March 18, 2002
Ernani Trinos, deceased husband of respondent Julita Trinos, applied for a health care coverage with
petitioner Philamcare Health Systems, Inc. In the standard application form, he stated that he was
never treated for high blood pressure, heart trouble, diabetes, cancer, liver disease, asthma or peptic
ulcer.
Under the agreement, respondents husband was entitled to avail of hospitalization benefits, whether
ordinary or emergency, listed therein. He was also entitled to avail of "out-patient benefits" such as
annual physical examinations, preventive health care and other out-patient services.
During the period of his coverage, Ernani suffered a heart attack and was confined at the Manila Medical
Center (MMC) for one month. While her husband was in the hospital, respondent tried to claim the
benefits under the health care agreement. However, petitioner denied her claim saying that the Health
Care Agreement was void. According to petitioner, there was a concealment regarding Ernanis medical
history. Doctors at the MMC allegedly discovered at the time of Ernanis confinement that he was
hypertensive, diabetic and asthmatic, contrary to his answer in the application form.
After Ernanis death, Julita Trinos filed an action for damages where she asked for reimbursement of her
expenses plus moral damages and attorneys fees.
RTC rendered judgment in favor of the plaintiff ordering defendants to pay and reimburse the medical
and hospital coverage of the late Ernani Trinos, and to pay moral and exemplary damages plus
attorneys fees and cost of suit. CA affirmed.
Held: SC Affirmed.
An insurance contract exists where the following elements concur:
1. The insured has an insurable interest;
2. The insured is subject to a risk of loss by the happening of the designated peril;
3. The insurer assumes the risk;
4. Such assumption of risk is part of a general scheme to distribute actual losses among a large
group of persons bearing a similar risk; and
5. In consideration of the insurers promise, the insured pays a premium.
Section 10 provides that: Every person has an insurable interest in the life and health
1. of himself, of his spouse and of his children;
2. of any person on whom he depends wholly or in part for education or support, or in whom he
has a pecuniary interest;
3. of any person under a legal obligation to him for the payment of money, respecting property or
service, of which death or illness might delay or prevent the performance; and
4. of any person upon whose life any estate or interest vested in him depends.
In the case at bar, the insurable interest of respondents husband in obtaining the health care
agreement was his own health. The health care agreement was in the nature of non-life insurance,
which is primarily a contract of indemnity. Once the member incurs hospital, medical or any other
expense arising from sickness, injury or other stipulated contingent, the health care provider must pay
for the same to the extent agreed upon under the contract.

By: Elaine Marie G. Laceda

42

INSURANCE LAW

Having assumed a responsibility under the agreement, petitioner is bound to answer the same to the
extent agreed upon. In the end, the liability of the health care provider attaches once the member is
hospitalized for the disease or injury covered by the agreement or whenever he avails of the covered
benefits which he has prepaid.
Under Section 27 of the Insurance Code, "a concealment entitles the injured party to rescind a contract
of insurance." However, the right to rescind should be exercised previous to the commencement of an
action on the contract. In this case, no rescission was made. Besides, the cancellation of health care
agreements as in insurance policies require the concurrence of certain conditions, none of which was
fulfilled in this case.
1. Prior notice of cancellation to insured;

2.
3.
4.

Notice must be based on the occurrence after effective date of the policy of one or more of the
grounds mentioned;
Must be in writing, mailed or delivered to the insured at the address shown in the policy;
Must state the grounds relied upon provided in Section 64 of the Insurance Code and upon
request of insured, to furnish facts on which cancellation is based.

The defendant Philamcare Health Systems Inc. had twelve months from the date of issuance of the
Agreement within which to contest the membership of the patient if he had previous ailment of asthma,
and six months from the issuance of the agreement if the patient was sick of diabetes or hypertension.
The periods having expired, the defense of concealment or misrepresentation no longer lie.
Lastly, petitioner alleges that respondent was not the legal wife of the deceased member considering
that at the time of their marriage, the deceased was previously married to another woman who was still
alive. However, the health care agreement is in the nature of a contract of indemnity. Hence, payment
should be made to the party who incurred the expenses.
Gercio vs. Sun Life Assurance of Canada, et al.
G.R. No. 23703 September 28, 1925
On January 29, 1910, the Sun Life Assurance Co. of Canada issued insurance policy No. 161481 on the
life of Hilario Gercio. The policy was what is known as a twenty-year endowment policy. By its terms,
the insurance company agreed to insure the life of Hilario Gercio for the sum of P2,000, to be paid him
on February 1, 1930, or if the insured should die before said date, then to his wife, Mrs. Andrea Zialcita,
should she survive him; otherwise to the executors, administrators, or assigns of the insured. The policy
did not include any provision reserving to the insured the right to change the beneficiary.
On September 4, 1920, a decree of divorce was issued in civil case no. 17955, which had the effect of
completely dissolving the bonds of matrimony contracted by Hilario Gercio and Andrea Zialcita. Hilario
Gercio formally notified the Sun Life Assurance Co. of Canada that he had revoked his donation in favor
of Andrea Zialcita, and that he had designated in her stead his present wife, Adela Garcia de Gercio, as
the beneficiary of the policy. The insurance company refused.
A petition for mandamus was filed to compel the defendant Sun Life Assurance Co. of Canada to change
the beneficiary in the policy. The trial court ruled in favor of the plaintiff and ordered the defendant
company to eliminate from the insurance policy the name of Andrea Zialcita as beneficiary and to
substitute therefor such name as the plaintiff might furnish to the defendant for that purpose.
Issue: Whether the insured may change the beneficiary (the former wife) and to name instead his actual
wife, where the insured and the beneficiary have been divorced and where the policy of insurance does
not expressly reserve to the insured the right to change the beneficiary.
Held: No.
A person who procures a policy upon his own life, payable to a designated beneficiary, although he pays
the premiums himself, and keeps the policy in his exclusive possession, has no power to change the

By: Elaine Marie G. Laceda

43

INSURANCE LAW

beneficiary, unless the policy itself, or the charter of the insurance company, so provides. The
beneficiary has a vested right in the policy, of which she cannot be deprived without her consent.
As to the effect produced by the divorce, the Philippine Divorce Law, Act No. 2710, merely provides in
section 9 that the decree of divorce shall dissolve the community property as soon as such decree
becomes final. Unlike the statutes of a few jurisdictions, there is no provision in the Philippine Law
permitting the beneficiary in a policy for the benefit of the wife of the husband to be changed after a
divorce. It must follow, therefore, in the absence of a statute to the contrary, that if a policy is taken out
upon a husband's life the wife is named as beneficiary therein, a subsequent divorce does not destroy
her rights under the policy.
b. Education or support Sec 10(b), I.C.; Art 195, Family Code
Art. 195. Subject to the provisions of the succeeding articles, the following are obliged to support each
other to the whole extent set forth in the preceding article:
(1)
The spouses;

(2)
(3)

Legitimate ascendants and descendants;


Parents and their legitimate children and the legitimate and illegitimate children of the

latter;

(4)

Parents and their illegitimate children and the legitimate and illegitimate children of the
latter; and
(5)
Legitimate brothers and sisters, whether of full or half-blood (291a)

c.

Creditor Sec 10(c), I.C.

Great Pacific Life Assurance Corp. vs. Court of Appeals


G.R. No. 113899 October 13, 1999
A contract of group life insurance was executed between petitioner Grepalife and DBP with Grepalife
agreeing to insure the lives of eligible housing loan mortgagors of DBP. Dr. Wilfredo Leuterio, a
physician and a housing debtor of DBP applied for membership in the group life insurance plan. In an
application form, Dr. Leuterio answered in the negative when asked if he ever had, or consulted, a
physician for a heart condition, high blood pressure, cancer, diabetes, lung; kidney or stomach disorder
or any other physical impairment. Thereafter, Grepalife issued Certificate No. B-18558, as insurance
coverage of Dr. Leuterio, to the extent of his DBP mortgage indebtedness amounting to P86,200.00.
On August 6, 1984, Dr. Leuterio died due to "massive cerebral hemorrhage." Consequently, DBP
submitted a death claim to Grepalife. Grepalife denied the claim alleging that Dr. Leuterio was not
physically healthy when he applied for an insurance coverage on November 15, 1983. Grepalife insisted
that Dr. Leuterio did not disclose he had been suffering from hypertension, which caused his death.
Allegedly, such non-disclosure constituted concealment that justified the denial of the claim.
Consequently, the widow of the late Dr. Leuterio, respondent Medarda V. Leuterio, filed a complaint with
the RTC of Misamis Oriental, against Grepalife for "Specific Performance with Damages."
RTC rendered a decision in favor of respondent widow and against Grepalife. CA sustained the trial
court's decision.
Held: SC affirmed.
The rationale of a group insurance policy of mortgagors, otherwise known as the "mortgage redemption
insurance," is a device for the protection of both the mortgagee and the mortgagor. On the part of the
mortgagee, it has to enter into such form of contract so that in the event of the unexpected demise of
the mortgagor during the subsistence of the mortgage contract, the proceeds from such insurance will
be applied to the payment of the mortgage debt, thereby relieving the heirs of the mortgagor from
paying the obligation. In a similar vein, ample protection is given to the mortgagor under such a concept
so that in the event of death; the mortgage obligation will be extinguished by the application of the
insurance proceeds to the mortgage indebtedness. Consequently, where the mortgagor pays the

By: Elaine Marie G. Laceda

44

INSURANCE LAW

insurance premium under the group insurance policy, making the loss payable to the mortgagee, the
insurance is on the mortgagor's interest, and the mortgagor continues to be a party to the contract. In
this type of policy insurance, the mortgagee is simply an appointee of the insurance fund, such losspayable clause does not make the mortgagee a party to the contract.
The insured private respondent did not cede to the mortgagee all his rights or interests in the insurance,
the policy stating that: "In the event of the debtor's death before his indebtedness with the Creditor
[DBP] shall have been fully paid, an amount to pay the outstanding indebtedness shall first be paid to
the creditor and the balance of sum assured, if there is any, shall then be paid to the beneficiary/ies
designated by the debtor." Since a policy of insurance upon life or health may pass by transfer, will or
succession to any person, whether he has an insurable interest or not, and such person may recover
whatever the insured might have recovered, the widow of the decedent Dr. Leuterio may file the suit
against the insurer, Grepalife.
Petitioner contends that Dr. Leuterio failed to disclose that he had hypertension, which might have
caused his death. Concealment exists where the assured had knowledge of a fact material to the risk,
and honesty, good faith, and fair dealing requires that he should communicate it to the assured, but he
designedly and intentionally withholds the same. The fraudulent intent on the part of the insured must
be established to entitle the insurer to rescind the contract. Misrepresentation as a defense of the insurer
to avoid liability is an affirmative defense and the duty to establish such defense by satisfactory and
convincing evidence rests upon the insurer. In the case at bar, the petitioner failed to clearly and
satisfactorily establish its defense, and is therefore liable to pay the proceeds of the insurance.
A life insurance policy is a valued policy. Unless the interest of a person insured is susceptible of exact
pecuniary measurement, the measure of indemnity under a policy of insurance upon life or health is the
sum fixed in the policy. The policy states that upon receipt of due proof of the Debtor's death during the
terms of this insurance, a death benefit in the amount of P86,200.00 shall be paid. However, on account
of the foreclosure made by DBP, the insurance proceeds shall inure to the benefit of the heirs of the
deceased person or his beneficiaries.
d. Pecuniary Interest Sec 10(d), I.C.
El Oriente Fabrica de Tabacos, Inc. vs. Posadas
G.R. No. 34774 September 21, 1931
On March 18, 1925, plaintiff, in order to protect itself against the loss that it might suffer by reason of
the death of its manager, A. Velhagen, who had had more than 35 years of experience in the
manufacture of cigars in the Philippines, and whose death would be a serious loss to the plaintiff,
procured from the Manufacturers Life Insurance Co., of Toronto, Canada, thru its local agent E.E. Elser,
an insurance policy on the life of the said A. Velhagen for the sum of $50,000, designating itself as the
sole beneficiary of the policy. The plaintiff charged as expenses of its business all the premiums paid and
deducted the same from its gross incomes as reported in its annual ITR, which deductions were allowed
by the CIR upon a showing made by the plaintiff that such premiums were legitimate expenses of its
business.
Upon the death of A. Velhagen in the year 1929, the plaintiff received all the proceeds of the life
insurance policy, together with the interests and the dividends accruing thereon, aggregating
P104,957.88. Thereafter, the CIR assessed and levied the sum of P3,148.74 as income tax on the
proceeds of the insurance policy which plaintiff paid under protest. CIR denied protest.
Issue: Whether the proceeds of insurance taken by a corporation on the life of an important official to
indemnify it against loss in case of his death, are taxable as income under the Philippine Income Tax
Law.
Held: No.
We do not believe that when the plaintiff received P104,957.88 from the insurance on the life of its
manager, it thereby realized a net profit in this amount. It is true that the Income Tax Law, in

By: Elaine Marie G. Laceda

45

INSURANCE LAW

exempting individual beneficiaries, speaks of the proceeds of life insurance policies as income, but this is
a very slight indication of legislative intention. In reality, what the plaintiff received was in the nature of
an indemnity for the loss which it actually suffered because of the death of its manager.
2. When Insurable Interest Must Exist Sec 19(c), I.C.
Sec. 19. An interest in property insured must exist when the insurance takes effect, and when the loss
occurs, but not exist in the meantime; and interest in the life or health of a person insured must exist
when the insurance takes effect, but need not exist thereafter or when the loss occurs.

3.

Transfer, Will or Succession Upon Death of Insured Sec 181, I.C.

Sec. 181. A policy of insurance upon life or health may pass by transfer, will or succession to any
person, whether he has an insurable interest or not, and such person may recover upon it whatever the
insured might have recovered.
C.

IN PROPERTY INSURANCE

1. Who Has Insurable Interest Secs 13 and 14, I.C.


Sec. 13. Every interest in property, whether real or personal, or any relation thereto, or liability in
respect thereof, of such nature that a contemplated peril might directly damnify the insured, is an
insurable interest.
Sec. 14. An insurable interest in property may consist in:
(a) An existing interest;
(b) An inchoate interest founded on an existing interest; or
(c) An expectancy, coupled with an existing interest in that out of which the expectancy arises.
a. Existing Interest Sec 14(a), I.C.
Traders Insurance & Surety Co. vs. Golangco
(1954)
Juan Golangco is the owner of No. 34 Plaza Sta. Cruz, Manila, Philippines, District 4, Block No. 47,
leased by Lianco. On April 7, 1949, a fire insurance policy was issued by respondent insurance company.
When the property was destroyed by fire and the claim for the proceeds of the insurance was made,
respondent insurance company refused.
Issue: Whether Golangco had any insurable interest therein when the policy was issued and when the
fire occurred.
Held: Yes.
All insurance covered under said policy, includes the 'rent or other subject matter of insurance in respect
of or in connection with any building or any property contained in any building'.
Filipino Merchants Insurance Co., Inc. vs. Court of Appeals
G.R. No. 85141 November 28, 1989
In December 1976, plaintiff insured a shipment of fishmeal with defendant-petitioner insurance company
under cargo Policy No. M-2678 for the sum of P267,653.59 for the goods described as 600 metric tons
of fishmeal in new gunny bags of 90 kilos each from Bangkok, Thailand to Manila against all risks under
warehouse to warehouse terms. The fishmeal in 666 new gunny bags were unloaded from the ship on
December 11, 1976 at Manila unto the arrastre contractor E. Razon, Inc. and defendant's surveyor
ascertained and certified that in such discharge 105 bags were in bad order condition as jointly surveyed
by the ship's agent and the arrastre contractor. The cargo was also surveyed by the arrastre contractor
before delivery of the cargo to the consignee and the condition of the cargo on such delivery was

By: Elaine Marie G. Laceda

46

INSURANCE LAW

reflected in E. Razon's Bad Order Certificate No. 14859, 14863 and 14869 covering a total of 227 bags
in bad order condition. Based on said computation the plaintiff made a formal claim against the
defendant Filipino Merchants Insurance Company for P51,568.62. A formal claim statement was also
presented by the plaintiff against the vessel dated December 21, 1976, but the defendant Filipino
Merchants Insurance Company refused to pay the claim. Consequently, the plaintiff brought an action to
recover from the defendant insurance company the amount of P51,568.62 representing damages to said
shipment.
The trial court rendered judgment in favor of plaintiff ordering the defendants to pay the plaintiff the
sum of P51,568.62 with interest at legal rate from the date of the filing of the complaint with right of
reimbursement from the third party defendant Compagnie Maritime Des Chargeurs Reunis and third
party defendant E. Razon, Inc. CA affirmed with modification.
Issue: WON private respondent/ plaintiff have an insurable interest over the cargo.
Held: Yes.
An "all risks policy" should be read literally as meaning all risks whatsoever and covering all losses by an
accidental cause of any kind. The terms "accident" and "accidental", as used in insurance contracts,
have not acquired any technical meaning. They are construed by the courts in their ordinary and
common acceptance. Thus, the terms have been taken to mean that which happens by chance or
fortuitously, without intention and design, and which is unexpected, unusual and unforeseen. An
accident is an event that takes place without one's foresight or expectation; an event that proceeds from
an unknown cause, or is an unusual effect of a known cause and, therefore, not expected.
A marine insurance policy providing that the insurance was to be "against all risks" must be construed
as creating a special insurance and extending to other risks than are usually contemplated, and covers
all losses except such as arise from the fraud of the insured. The burden of the insured, therefore, is to
prove merely that the goods he transported have been lost, destroyed or deteriorated. Thereafter, the
burden is shifted to the insurer to prove that the loss was due to excepted perils. To impose on the
insured the burden of proving the precise cause of the loss or damage would be inconsistent with the
broad protective purpose of "all risks" insurance.
In the present case, there being no showing that the loss was caused by any of the excepted perils, i.e.
delay or the inherent vice or nature of the subject matter insured, the insurer is liable under the policy.
There is no evidence presented to show that the condition of the gunny bags in which the fishmeal was
packed was such that they could not hold their contents in the course of the necessary transit, much
less any evidence that the bags of cargo had burst as the result of the weakness of the bags
themselves.
Insurable interest in property may consist in (a) an existing interest; (b) an inchoate interest founded on
an existing interest; or (c) an expectancy, coupled with an existing interest in that out of which the
expectancy arises. Herein private respondent, as vendee/consignee of the goods in transit has such
existing interest therein as may be the subject of a valid contract of insurance. His interest over the
goods is based on the perfected contract of sale. The perfected contract of sale between him and the
shipper of the goods operates to vest in him an equitable title even before delivery or before be
performed the conditions of the sale. The perfected contract of sale even without delivery vests in the
vendee an equitable title, an existing interest over the goods sufficient to be the subject of insurance.
Gaisano Cagayan, Inc. vs. Insurance Company of North America
G.R. No. 147839 June 8, 2006
IMC, the maker of Wrangler Blue Jeans, and Levi Strauss (Phils.) Inc., the local distributor of products
bearing trademarks owned by Levi Strauss & Co., separately obtained from respondent fire insurance
policies with book debt endorsements. The insurance policies provide for coverage on "book debts in
connection with ready-made clothing materials which have been sold or delivered to various customers
and dealers of the Insured anywhere in the Philippines." The policies defined book debts as the "unpaid

By: Elaine Marie G. Laceda

47

INSURANCE LAW

account still appearing in the Book of Account of the Insured 45 days after the time of the loss covered
under this Policy."
Petitioner is a customer and dealer of the products of IMC and LSPI. On February 25, 1991, the Gaisano
Superstore Complex in Cagayan de Oro City, owned by petitioner, was consumed by fire. Included in the
items lost or destroyed in the fire were stocks of ready-made clothing materials sold and delivered by
IMC and LSPI. Thus, respondent filed a complaint for damages against petitioner as subrogee of IMC and
LSPI after payment to the latter of their claims under their respective fire insurance policies with book
debt endorsements.
Petitioner contends that it could not be held liable because the property covered by the insurance
policies were destroyed due to fortuities event or force majeure; that respondent's right of subrogation
has no basis inasmuch as there was no breach of contract committed by it since the loss was due to fire
which it could not prevent or foresee; that IMC and LSPI never communicated to it that they insured
their properties; that it never consented to paying the claim of the insured.
RTC dismissed the case and held that the fire was purely accidental; that the cause of the fire was not
attributable to the negligence of the petitioner; that it has not been established that petitioner is the
debtor of IMC and LSPI; that since the sales invoices state that "it is further agreed that merely for
purpose of securing the payment of purchase price, the above-described merchandise remains the
property of the vendor until the purchase price is fully paid", IMC and LSPI retained ownership of the
delivered goods and must bear the loss.
CA rendered its decision setting aside the decision of the RTC. The CA held that the sales invoices are
proofs of sale; that loss of the goods in the fire must be borne by petitioner since the proviso contained
in the sales invoices is an exception under Article 1504 (1) of the Civil Code, to the general rule that if
the thing is lost by a fortuitous event, the risk is borne by the owner of the thing at the time the loss
under the principle of res perit domino; that petitioner's obligation to IMC and LSPI is not the delivery of
the lost goods but the payment of its unpaid account and as such the obligation to pay is not
extinguished, even if the fire is considered a fortuitous event; that by subrogation, the insurer has the
right to go against petitioner; that, being a fire insurance with book debt endorsements, what was
insured was the vendor's interest as a creditor.
Issue: WON the fire insurance policy on book debts covers the unpaid accounts of IMC and LSPI since
such insurance applies to loss of the ready-made clothing materials sold and delivered to petitioner.
Held: Yes.
The questioned insurance policies provide coverage for "book debts in connection with ready-made
clothing materials which have been sold or delivered to various customers and dealers of the Insured
anywhere in the Philippines."; and defined book debts as the "unpaid account still appearing in the Book
of Account of the Insured 45 days after the time of the loss covered under this Policy." Nowhere is it
provided in the questioned insurance policies that the subject of the insurance is the goods sold and
delivered to the customers and dealers of the insured. Thus, what were insured against were the
accounts of IMC and LSPI with petitioner which remained unpaid 45 days after the loss through fire, and
not the loss or destruction of the goods delivered.
IMC and LSPI did not lose complete interest over the goods. They have an insurable interest until full
payment of the value of the delivered goods. Unlike the civil law concept of res perit domino, where
ownership is the basis for consideration of who bears the risk of loss, in property insurance, one's
interest is not determined by concept of title, but whether insured has substantial economic interest in
the property.
An insurable interest in property does not necessarily imply a property interest in, or a lien upon, or
possession of, the subject matter of the insurance, and neither the title nor a beneficial interest is
requisite to the existence of such an interest, it is sufficient that the insured is so situated with reference
to the property that he would be liable to loss should it be injured or destroyed by the peril against
which it is insured. Anyone has an insurable interest in property who derives a benefit from its existence
or would suffer loss from its destruction. Indeed, a vendor or seller retains an insurable interest in the

By: Elaine Marie G. Laceda

48

INSURANCE LAW

property sold so long as he has any interest therein, in other words, so long as he would suffer by its
destruction, as where he has a vendor's lien. In this case, the insurable interest of IMC and LSPI pertain
to the unpaid accounts appearing in their Books of Account 45 days after the time of the loss covered by
the policies.
It must be stressed that the insurance in this case is not for loss of goods by fire but for petitioner's
accounts with IMC and LSPI that remained unpaid 45 days after the fire. Accordingly, petitioner's
obligation is for the payment of money. As correctly stated by the CA, where the obligation consists in
the payment of money, the failure of the debtor to make the payment even by reason of a fortuitous
event shall not relieve him of his liability. The rationale for this is that the rule that an obligor should be
held exempt from liability when the loss occurs thru a fortuitous event only holds true when the
obligation consists in the delivery of a determinate thing and there is no stipulation holding him liable
even in case of fortuitous event. It does not apply when the obligation is pecuniary in nature.
Respondent's action against petitioner is squarely sanctioned by Article 2207 of the Civil Code which
provides: If the plaintiff's property has been insured, and he has received indemnity from the insurance
company for the injury or loss arising out of the wrong or breach of contract complained of, the
insurance company shall be subrogated to the rights of the insured against the wrongdoer or the person
who has violated the contract.
Ong Ling Sing vs. FEB Leasing & Finance Corporation (2007)

Lampano vs. Jose


G.R. No. L-9401 March 30, 1915
The defendant, Mariano R. Barretto, constructed a house for the other defendant, Placida A. Jose ,who
subsequently sold the house to the plaintiff, Antonina Lampano, for the sum of P6,000. On March 22,
1913, the house was destroyed by fire. At the time of the fire Antonina Lampano still owed Placida A.
Jose the sum of P2,000, evidenced by a promissory note, and Placida A. Jose still owed Mariano R.
Barretto on the cost of the construction the sum of P2,000.
After the completion of the house and sometime before it was destroyed, Mariano R. Barretto took out
an insurance policy upon it in his own name, with the consent of Placida A. Jose, for the sum of P4,000.
After its destruction, he collected P3,600 from the insurance company, having paid in premiums the sum
of P301.50.
The plaintiff alleged in her complaint that there was a verbal agreement between her and Placida A.
Jose, at the time of the purchase and sale of the house, to the effect that the latter agreed to deliver to
her the insurance policy on the building, therefore neither Placida A. Jose nor Mariano R. Barretto has
any right to the insurance or to the money received therefrom. The alleged agreement was however
denied by Placida A. Jose.
Judgment was entered against Barretto and in favor of Placida A. Jose for the sum of P1,298.50, being
the difference between the amount collected by Barretto on the insurance and the amount yet due him
for the construction of the house, including the premiums paid. Judgment was also entered in favor of
the defendant, Placida A. Jose, against the plaintiff for the sum of P2,000, being the balance of the
purchase price of the house.
Issue: WON the plaintiff has any right to recover from Barretto any portion of the insurance money.
Held: No.
If Barretto had an insurable interest in the house, he could insure this interest for his sole protection.
The policy was in the name of Barretto alone. It was, therefore, a personal contract between him and
the company and not a contract which ran with the property. According to this personal contract the
insurance policy was payable to the insured without regard to the nature and extent of his interest in the
property, provided that he had, as we have said, an insurable interest at the time of the making of the

By: Elaine Marie G. Laceda

49

INSURANCE LAW

contract, and also at the time of the fire. Where different persons have different interests in the same
property, the insurance taken by one in his own right and in his own interest does not in any way insure
to the benefit of another.
In the case at bar Barretto assumed the responsibility for the insurance. The premiums, as we have
indicated, were paid by him without any agreement or right to recoup the amount paid therefor should
no loss result to the property. It would not, therefore, be in accordance with t he law and his contractual
obligations to compel him to account for the insurance money, or any par thereof, to the plaintiff, who
assumed no risk whatever.
That Barretto had an insurable interest in the house, we think there can be no question. He construed
the building, furnishing all the materials and supplies, and insured it after it had been completed.
b. Inchoate Interest Sec 14(b), I.C.
c.

Expectancy Secs 14(c) and 16, I.C.

Sec. 16. A mere contingent or expectant interest in anything, not founded on an actual right to the
thing, nor upon any valid contract for it, is not insurable.
d. Mortgagor Secs 8 and 9, I.C.; Art 2127, Civil Code
Sec. 8. Unless the policy otherwise provides, where a mortgagor of property effects insurance in his
own name providing that the loss shall be payable to the mortgagee, or assigns a policy of insurance to
a mortgagee, the insurance is deemed to be upon the interest of the mortgagor, who does not cease to
be a party to the original contract, and any act of his, prior to the loss, which would otherwise avoid the
insurance, will have the same effect, although the property is in the hands of the mortgagee, but any
act which, under the contract of insurance, is to be performed by the mortgagor, may be performed by
the mortgagee therein named, with the same effect as if it had been performed by the mortgagor.
Sec. 9. If an insurer assents to the transfer of an insurance from a mortgagor to a mortgagee, and, at
the time of his assent, imposes further obligation on the assignee, making a new contract with him, the
act of the mortgagor cannot affect the rights of said assignee.
Art. 2127. The mortgage extends to the natural accessions, to the improvements, growing fruits, and
the rents or income not yet received when the obligation becomes due, and to the amount of the
indemnity granted or owing to the proprietor from the insurers of the property mortgaged, or in virtue of
expropriation for public use, with the declarations, amplifications and limitations established by law,
whether the estate remains in the possession of the mortgagor, or it passes into the hands of a third
person. (1877)
San Miguel Brewery vs. Law Union and Rock Insurance Co.
G.R. No. L-14300 January 19, 1920
On January 12, 1916, D. P. Dunn, the owner of the subject property, mortgaged the same to the San
Miguel Brewery to secure a debt of P10,000. In the contract of mortgage Dunn agreed to keep the
property insured at his expense to the full amount of its value in companies to be selected by the
Brewery Company and authorized the latter in case of loss to receive the proceeds of the insurance and
to retain such part as might be necessary to cover the mortgage debt. At the same time, in order more
conveniently to accomplish the end in view, Dunn authorized and requested the Brewery Company to
effect said insurance itself. Accordingly on the same date Antonio Brias, general manager of the
Brewery, made a verbal application to the Law Union and Rock Insurance Company for insurance to the
extent of P15,000 upon said property. In reply to a question of the company's agent as to whether the
Brewery was the owner of the property, he stated that the company was interested only as a
mortgagee. No information was asked as to who was the owner of the property, and no information
upon this point was given. In the month of March of the year 1917 Dunn sold the insured property to
the defendant Henry Harding, but no assignment of the insurance, or of the insurance policies, was at
any time made to him.

By: Elaine Marie G. Laceda

50

INSURANCE LAW

When the property was destroyed by fire, San Miguel Brewery filed for a complaint for recovery from
Union and Rock Insurance Company Ltd., and the "Filipinas" Compania de Seguros, for the sum of
P7,500 each. Harding claimed for himself the right to recover the difference between the plaintiff's
mortgage credit and the face value of the policies. The two insurance companies admitted their liability
to the San Miguel Brewery to the extent of its mortgage credit, but denied liability to Harding on the
ground that under the contracts of insurance the liability of the insurance companies was limited to the
insurable interest of the plaintiff therein.
Soon after the action was begun the insurance companies effected a settlement with the San Miguel
Brewery by paying the full amount of the credit claimed by it, with the result that the litigation as
between the original plaintiff and the two insurance companies came to an end, leaving the action to be
prosecuted to final judgment by Harding with respect to the balance claimed to be due to him upon the
policies. The trial judge came to the conclusion that Harding had no right of action whatever against the
companies and absolved them from liability.
Issue: WON the insurance companies are liable for payment to Harding.
Held: No.
Both policies were issued in the name of the San Miguel Brewery as the assured, and contained no
reference to any other interest in the property. Thus, Henry Harding does not have a cause of action
against the insurance companies. He is not a party to the contracts of insurance and cannot directly
maintain an action thereon. His claim is merely of an equitable and subsidiary nature and must be made
effective, if at all, through the San Miguel Brewery in whose name the contracts are written. Now the
Brewery, as mortgagee of the insured property, undoubtedly had an insurable interest therein; but it
could not, in any event, recover upon these policies an amount in excess of its mortgage credit. This is
clear from the express provisions of Sec 16 and Sec 50 of the Insurance Act -- "the measure of an
insurable interest in property is the extent to which the insured might be damnified by loss or injury
thereof" (sec. 16); and "the insurance shall be applied exclusively to the proper interest of the person in
whose name it is made unless otherwise specified in the policy" (sec. 50).
The policies of insurance might have been so framed as to have been "payable to the San Miguel
Brewery, mortgagee, as its interest may appear, remainder to whomsoever, during the continuance of
the risk, may become the owner of the interest insured." (Sec 54, Act No. 2427.) Such a clause would
have proved an intention to insure the entire interest in the property, not merely the insurable interest
of the San Miguel Brewery, and would have shown exactly to whom the money, in case of loss, should
be paid. But the policies are not so written.
If during the negotiations which resulted in the writing of the insurance policy it had been agreed
between the contracting parties that the insurance should be so written as to protect not only the
interest of the mortgagee but also the residuary interest of the owner, and the policies had been, by
inadvertence, ignorance, or mistake written in the form in which they were issued, a court would have
the power to reform the contracts and give effect to them in the sense in which the parties intended to
be bound. But in order to justify this, it must be made clearly to appear that the minds of the
contracting parties did actually meet in agreement and that they labored under some mutual error or
mistake in respect to the expression of their purpose. The proof must be of the most satisfactory
character, and it must clearly appear that the contract failed to express the real agreement between the
parties. In the case now before us the proof is entirely insufficient to authorize the reformation.
Saura Import & Export Co., Inc. vs. Philippine International Surety Co., Inc.
G.R. No. L-15184 May 31, 1963
Saura Import & Export Co Inc., mortgaged to the PNB, a parcel of land with a building of strong
materials to secure the payment of the total mortgaged debt to P37,000.00. The mortgage contact
required that the mortgagor shall insure the mortgaged property at all times against fire and earthquake
for an amount and with such company satisfactory to the Mortgagee, indorsing to the latter the
corresponding policies. Pursuant to the requirement, Saura insured the building and its contents with the

By: Elaine Marie G. Laceda

51

INSURANCE LAW

Philippine International Surety, an insurance firm acceptable to mortgagee Bank, for P29,000.00 against
fire for the period of one year from October 2, 1954. As required therefor, the insurance policy was
endorsed to the mortgagee PNB.
Barely 13 days after the issuance of the fire insurance policy, the insurer cancelled the same, effective
as of the date of issue. Notice of the cancellation was given to appellee bank in writing, sent by
Registered Mail and personally addressed to the Branch Manager, and was received by the Bank on
November 8, 1954. On April 6, 1955, the building and its contents, worth P40,685.69 were burned.
Upon the presentation of notice of loss with the PNB, Saura learned for the first time that the policy had
previously been cancelled by the insurer. Upon refusal of the insurer to pay the amount of the
insurance, Civil Case No. 26847 was filed with the Manila CFI against the Insurer, and the PNB was later
included as party defendant, after it had refused to prosecute the case jointly with Saura Import &
Export Co., Inc. CFI dismissed complaint.
Issue: WON the insurer effectively cancelled the insurance policy.
Held: No.
Fire insurance policies and other contracts of insurance upon property, in addition to the common
provision for cancellation of the policy upon request of the insured, generally provide for cancellation by
the insurer by notice to the insured for a prescribed period, which is usually 5 days, and the return of
the unearned portion of the premium paid by the insured. The purpose of provisions or stipulations for
notice to the insured, is to prevent the cancellation of the policy, without allowing the insured ample
opportunity to negotiate for other insurance in its stead. Actual personal notice to the insured is
essential to a cancellation under a provision for cancellation by notice. The actual receipt by the insured
of a notice of cancellation is universally recognized as a condition precedent to a cancellation of the
policy by the insurer, and consequently a letter containing notice of cancellation which is mailed by the
insurer but not received by the insured, is ineffective as cancellation.
However, the policy in question does not provide for the notice, its form or period. The Insurance Law
does not likewise provide for such notice. This being the case, it devolves upon the Court to apply the
generally accepted principles of insurance, regarding cancellation of the insurance policy by the insurer.
Actual notice of cancellation in a clear and unequivocal manner, preferably in writing, in view of the
importance of an insurance contract, should be given by the insurer to the insured, so that the latter
might be given an opportunity to obtain other insurance for his own protection. The notice should be
personal to the insured and not to and/or through any unauthorized person by the policy. In the case at
bar, the defendant insurance company, must have realized the paramount importance of sending a
notice of cancellation, when it sent the notice of cancellation of the policy to the defendant bank (as
mortgagee), but not to the insured with which it had direct dealing. It was the primary duty of the
insurance company to notify the insured, but it did not. Notice to the bank is not effective notice.
Palileo vs. Cosio
G.R. No. L-7667 November 28, 1955
On December 18, 1951, plaintiff Palileo obtained from defendant Cosio a loan in the sum of P12,000
subject to the following conditions: (a) that plaintiff shall pay to defendant an interest in the amount of
P250 a month; (b) that defendant shall deduct from the loan certain obligations of plaintiff to third
persons amounting to P4,550, plus the sum of P250 as interest for the first month; and (c) that after
making the above deductions, defendant shall deliver to plaintiff only the balance of the loan of P12,000.
To secure the payment of the aforesaid loan, defendant required plaintiff to sign a document known as
"Conditional Sale of Residential Building", purporting to convey to defendant, with right to repurchase, a
two-story building of strong materials belonging to plaintiff. This document did not express the true
intention of the parties which was merely to place said property as security for the payment of the loan.
After the execution of the aforesaid document, defendant insured the building against fire with the
Associated Insurance & Surety Co., Inc. for the sum of P15,000, the insurance policy having been issued
in the name of defendant. The building was partly destroyed by fire and, after proper demand,
defendant collected from the insurance company an indemnity of P13,107.00. Plaintiff demanded from

By: Elaine Marie G. Laceda

52

INSURANCE LAW

defendant that she be credited with the necessary amount to pay her obligation out of the insurance
proceeds but defendant refused to do so.
After hearing on the complaint filed by plaintiff, the trial court held the transaction as an equitable
mortgage and ordered defendant to credit the sum of P13,107 received by the defendant from the
Associated Insurance & surety Co., Inc. to the payment of plaintiff's obligation in the sum of P12,000.00,
thus considering the agreement extinguished.
Issue: WON the obligation of plaintiff is fully compensated by the insurance amount notwithstanding the
fact that it was not proven that the insurance was taken for the benefit of the mortgagor.
Held: No.
The ruling runs counter to the rule governing an insurance taken by a mortgagee independently of the
mortgagor. The rule is that "where a mortgagee, independently of the mortgagor, insures the
mortgaged property in his own name and for his own interest, he is entitled to the insurance proceeds in
case of loss, but in such case, he is not allowed to retain his claim against the mortgagor, but is passed
by subrogation to the insurer to the extent of the money paid."
Great Pacific Life Assurance Corp. vs. Court of Appeals (1999), supra
e. Carrier or Depositary Sec 15, I.C.
Sec. 15. A carrier or depository of any kind has an insurable interest in a thing held by him as such, to
the extent of his liability but not to exceed the value thereof.
Lopez vs. Del Rosario and Quiogue
G.R. No. L-19189 November 27, 1922
Mrs. Del Rosario, engaged in the business of a warehouse keeper, and stored copra and other
merchandise, was the owner of a bonded warehouse situated in the City of Manila. Among the persons
who had copra deposited in the Del Rosario warehouse was Froilan Lopez, the holder of 14 warehouse
receipts in his own name, and the name of Elias T. Zamora, with a declared value of P107,990.40. The
warehouse receipts provided for insurance at the rate of 1% per month on the declared value. It is
admitted that insurance was paid by Lopez on May 18, 1920, but not thereafter.
Mrs. Del Rosario secured insurance on the warehouse and its contents with the National Insurance Co.,
Inc., the Commercial Union Insurance Company, the Alliance Insurance Company, the South British
Insurance Co., Ltd., and the British Traders Insurance Co., Ltd., in the amount of P404,800. All the
policies were in the name of Sra. Benita Quiogue de V. del Rosario, with the exception of one of the
National Insurance Company, Inc., for P40,000, in favor of the Compaia Coprera de Tayabas.
On June 6, 1920 the warehouse of Mrs. Del Rosario and its contents were destroyed by fire. The
warehouse was a total loss, while of the copra stored therein, only an amount equal to P49,985 was
salvaged. Ineffectual attempts by Mrs. Del Rosario to effect a compromise with Lopez were made. Lopez
stubbornly contended that he should receive not a centavo less than P88,595.43.
After trial on the complaint, the trial court awarded to the plaintiff the sum of 88,495.21 with legal
interest.
Issue: WON plaintiff is entitled to a proportional share of the proceeds of the insurance policies taken
out by defendant despite non-payment by the plaintiff of the insurance agreement.
Held: Yes.
The law is that a policy effected by bailee and covering by its terms his own property and property held
in trust; inures, in the event of a loss, equally and proportionately to the benefit of all the owners of the
property insured. Even if one secured insurance covering his own goods and goods stored with him, and

By: Elaine Marie G. Laceda

53

INSURANCE LAW

even if the owner of the stored goods did not request or know of the insurance, and did not ratify it
before the payment of the loss, yet it has been held by a reputable court that the warehouseman is
liable to the owner of such stored goods for his share.
Plaintiff's rights to the insurance money have not been forfeited by failure to pay the insurance provided
for in the warehouse receipts. A preponderance of the proof does not demonstrate that the plaintiff ever
ordered the cancellation of his insurance with the defendant. Nor is it shown that the plaintiff ever
refused to pay the insurance when the bills were presented to him, and that notice of an intention to
cancel the insurance was ever given the plaintiff.
2. Measure of Insurable Interest Secs 17 and 14(a), I.C.
Sec. 17. The measure of an insurable interest in property is the extent to which the insured might be
damnified by loss or injury thereof.
Sec. 14. (a) An existing interest;
San Miguel Brewery vs. Law Union and Rock Insurance Co. (1920), supra
Ong Ling Sing vs. FEB Leasing & Finance Corporation (2007), supra
3. Effect of Lack of Insurable Interest Sec 18, I.C.
Sec. 18. No contract or policy of insurance on property shall be enforceable except for the benefit of
some person having an insurable interest in the property insured.
Sharuff & Co. vs. Baloise Fire Insurance Co.
G.R. No. 44119 March 30, 1937
Salomon Sharruf and Elias Eskenazi doing business under the firm name of Sharruf & Co. were issued by
Baloise Fire Insurance Co. insurance policies in the total amount of P25,000 covering various
merchandise. Defendant Springfield Insurance Co. likewise issued an additional policy in the sum of
P15,000 in favor of said firm Sharruf & Co., raising the total amount of the insurance on said
merchandise to P40,000. The name of Sharruf & Co. was later substituted with that of Sharruf &
Eskenazi.
On September 22, 1933, fire broke out in the building which housed the insured merchandise. After the
fire had been extinguished, earthen pots were found, all of which smelled of petroleum. In the burned
building the plaintiffs kept petroleum used for cleaning the floor.
Issue: WON Sharruf & Eskenazi had insurable interest.
Held: Yes.
While it is true that at the beginning the plaintiffs had been doing business under the firm, name of
"Sharruf & Co.", insuring their business in said name, and upon executing the contract of partnership
they changed the title thereof to "Sharruf & Eskenazi," the membership of the partnership in question
remained unchanged. It does not appear that in changing the title of the partnership they had the
intention of defrauding the herein defendant insurance-companies. Therefore the responsibility of said
defendants to the plaintiffs by virtue of the respective insurance policies has not been altered.
The fact that the liquid found by the detectives in the earthen jars smelled of petroleum does not
constitute conclusive evidence that they had been used as containers for petroleum to burn the house.
Said smell could have very well come from the strips of China wood of which boxes from abroad are
made, the resin of which smells of petroleum, or from the rags found therein which might have been
used to clean the floor by saturating them with petroleum. There being petroleum for cleaning the floor
in the building, it is not strange that when the house caught fire the petroleum also caught fire, the
flames floating on the water coming out from under the door from the pumps. There is neither direct nor

By: Elaine Marie G. Laceda

54

INSURANCE LAW

strong circumstantial evidence that the plaintiffs personally or through their agents placed petroleum in
the building in order to burn it, because it was locked on the outside and nobody was staying therein.
As it cannot be assumed that the petroleum might have burned by itself, it is probable that the fire
might have originated from the electric wiring.
However, the plaintiffs, upon whom devolve the legal obligation to prove the existence, at the time of
the fire, of the articles and merchandise for the destruction of which they claim indemnity from the
defendant companies, have not complied with their duty because they have failed to prove by a
preponderance of evidence that when the fire took place there were in the burnt building articles and
merchandise in the total amount of the insurance policies or that the textiles and other damaged and
undamaged goods found in the building after the fire were worth P40,000.
Garcia vs. Hong Kong Fire & Marine Insurance Co.
G.R. No. 20341 September 1, 1923
Plaintiff Domingo Garcia, a merchant and owner of a bazaar known as "Las Novedades", entered into a
contract with the defendant whereby it insured his merchandise in the sum of P15,000 at a premium of
P300 per annum, however, the defendant issued its fire insurance policy in favor of the plaintiff, not on
the merchandise in the building, but on the building which contained the merchandise. It appears that
the policy was in the English language, of which the plaintiff Garcia is ignorant. Hence, when he received
it he noticed that the amount P15,000 was correct, and never personally made a further investigation.
Subsequently, Garcia executed a mortgage to the PNB on the merchandise insured by the defendant,
and with the consent of the defendant, the plaintiff endorsed the policy to the Bank. On February 6,
1920, and while the policy was in force and effect, a fire took place which destroyed the merchandise in
the building of the value of P20,000, together with the building itself; that demand was made upon the
defendant for the payment of P15,000, as provided for in the policy, however payment was refused.
After trial, the lower court rendered judgment for the plaintiff.
Held: SC affirmed.
Cha vs. Court of Appeals
G.R. No. 124520 August 18, 1997
Petitioner-spouses Nilo Cha and Stella Uy-Cha, as lessees, entered into a lease contract with private
respondent CKS Development Corporation. The lease contract provides: The LESSEE shall not insure
against fire the chattels, merchandise, textiles, goods and effects placed at any stall or store or space in
the leased premises without first obtaining the written consent and approval of the LESSOR. If the
LESSEE obtain(s) the insurance thereof without the consent of the LESSOR then the policy is deemed
assigned and transferred to the LESSOR for its own benefit. Notwithstanding said stipulation, the Cha
spouses insured against loss by fire the merchandise inside the leased premises for P500,000.00 with
the United Insurance Co., Inc. without the written consent of CKS. On the day that the lease contract
was to expire, fire broke out inside the leased premises. When CKS learned of the insurance procured by
the Cha spouses without its consent, it demanded that the proceeds of the insurance contract be paid
directly to CKS, based on its lease contract with the Cha spouses. United refused to pay CKS. Hence, the
latter filed a complaint against the Cha spouses and United. RTC ruled in favor of CKS. CA affirmed.
Issue: WON the paragraph 18 of the lease contract entered into between CKS and the Cha spouses is
valid insofar as it provides that any fire insurance policy obtained by the lessee (Cha spouses) over their
merchandise inside the leased premises is deemed assigned or transferred to the lessor (CKS) if said
policy is obtained without the prior written consent of the latter.
Held: No.
A non-life insurance policy such as the fire insurance policy taken by petitioner-spouses over their
merchandise is primarily a contract of indemnity. Insurable interest in the property insured must exist at
the time the insurance takes effect and at the time the loss occurs. The basis of such requirement of
insurable interest in property insured is based on sound public policy: to prevent a person from taking

By: Elaine Marie G. Laceda

55

INSURANCE LAW

out an insurance policy on property upon which he has no insurable interest and collecting the proceeds
of said policy in case of loss of the property. In such a case, the contract of insurance is a mere wager
which is void under Section 25 of the Insurance Code.
In the present case, it cannot be denied that CKS has no insurable interest in the goods and
merchandise inside the leased premises under the provisions of Section 17 of the Insurance Code.
Therefore, respondent CKS cannot be validly a beneficiary of the fire insurance policy taken by the
petitioner-spouses over their merchandise. This insurable interest over said merchandise remains with
the insured, the Cha spouses. The automatic assignment of the policy to CKS under the provision of the
lease contract previously quoted is void for being contrary to law and/or public policy.
4. When Insurable Interest Must Exist Sec 19, I.C.
Sec. 19. An interest in property insured must exist when the insurance takes effect, and when the loss
occurs, but not exist in the meantime; and interest in the life or health of a person insured must exist
when the insurance takes effect, but need not exist thereafter or when the loss occurs.
Tai Tong Chuache & Co. vs. Insurance Commission
G.R. No. L-55397 February 29, 1988
On April 19, 1975, Azucena Palomo obtained a loan from Tai Tong Chuache Inc. using the land and
building as security. The mortgagee further insured its interest with Travellers Multi-Indemnity
Corporation for P100,000.00. On June 11, 1975, Pedro Palomo secured a Fire Insurance Policy covering
the building for P50,000.00 with respondent Zenith Insurance Corporation. Another Fire Insurance Policy
was procured from respondent Philippine British Assurance Company, covering the same building for
P50,000.00 and the contents thereof for P70,000.00. On July 31, 1975, the building and the contents
were totally razed by fire.
Zenith Insurance, Phil. British Assurance and S.S.S. Accredited Group of Insurers, paid their
corresponding shares of the loss. Demand was made from respondent Travellers Multi-Indemnity for its
share in the loss but the same was refused. Hence, complainants demanded from the other 3
respondents the balance of each share in the loss based on the computation of the Adjustment
Standards Report excluding Travellers Multi-Indemnity in the amount of P30,894.31 but the same was
refused, hence, this action.
Philippine British Assurance and Zenith Insurance Corporation both denied liability on the ground that
the claim of the complainants had already been waived, extinguished or paid. SSS Accredited Group of
Insurers on the other hand paid the claim in full. Travellers Insurance, on its part, admitted the issuance
of the insurance policy but alleged that it was secured by the mortgage creditor for the purpose of
protecting his mortgage credit and that the policy was issued in the name of Azucena Palomo, only to
indicate that she owns the insured premises.
Tai Tong Chuache & Co. filed a complaint in intervention to which Travellers Insurance answered that
intervenor is not entitled to indemnity under its Fire Insurance Policy for lack of insurable interest before
the loss of the insured premises because the spouses Pedro and Azucena Palomo, had already paid in
full their mortgage indebtedness to the intervenor.
The Insurance Commission dismissed spouses Palomos' complaint on the ground that the insurance
policy subject of the complaint was taken out by Tai Tong Chuache & Company for its own interest only
as mortgagee of the insured property and thus complainant as mortgagors of the insured property have
no right of action against herein respondent. Complaint in intervention was likewise dismissed.
Issue: WON petitioner lack insurable interest before the occurrence of the peril insured against because
of the payment of the mortgage by the mortgagor.
Held: No.

By: Elaine Marie G. Laceda

56

INSURANCE LAW

Respondent insurance company did not assail the validity of the insurance policy taken out by petitioner
over the mortgaged property. Neither did it deny that the said property was totally razed by fire within
the period covered by the insurance. However, respondent advanced an affirmative defense of lack of
insurable interest on the part of the petitioner that before the occurrence of the peril insured against the
Palomos had already paid their credit due the petitioner. Respondent having admitted the material
allegations in the complaint, has the burden of proof to show that petitioner has no insurable interest
over the insured property at the time the contingency took place. Upon that point, there is a failure of
proof. Respondent exerted no effort to present any evidence to substantiate its claim.
The respondent insurance company having issued a policy in favor of herein petitioner which policy was
of legal force and effect at the time of the fire, it is bound by its terms and conditions. Upon its failure to
prove the allegation of lack of insurable interest on the part of the petitioner, respondent insurance
company is and must be held liable.
5. Effect of Change of Interest in Thing Insured
a. General rule: suspended Sec 20, I.C.
Sec. 20. Except in the cases specified in the next four sections, and in the cases of life, accident, and
health insurance, a change of interest in any part of a thing insured unaccompanied by a corresponding
change in interest in the insurance, suspends the insurance to an equivalent extent, until the interest in
the thing and the interest in the insurance are vested in the same person.
San Miguel Brewery vs. Law Union and Rock Insurance Co. (1920), supra
Bachrach vs. British American Assurance Co.
G.R. No. L-5715 December 20, 1910
E. M. Bachrach was issued a fire insurance policy by defendant insurance company. On the July 13,
1908, the plaintiff commenced an action against the defendant to recover the sum of P9,841.50, the
amount due, deducting the salvage, upon the fire insurance policy issued by the defendant. The
defendant alleged that it was released from all obligations whatever under said policy because the
plaintiff transferred his interest in and to the property covered by the policy to H. W. Peabody & Co. to
secure certain indebtedness due and owing to said company, and also that the plaintiff had transferred
his interest in certain of the goods covered by the said policy to one Macke, to secure certain obligations
assumed by the said Macke for and on behalf of the insured without the consent of the insurer as
required by the policy. TC found the defendant liable to the plaintiff.
Issue: WON the execution of the said chattel mortgage, without the knowledge and consent of the
insurance company and without receiving the sanction of said company, annulled the said policy of
insurance.
Held: No.
It is claimed that the execution of a chattel mortgage on the insured property violated what is known as
the "alienation clause". However it has been held in a line of cases that the interest in property insured
does not pass by the mere execution of a chattel mortgage and that while a chattel mortgage is a
conditional sale, there is no alienation within the meaning of the insurance law until the mortgage
acquires a right to take possession by default under the terms of the mortgage. No such right is claimed
to have accrued in the case at bar, and the alienation clause is therefore inapplicable.
Furthermore, upon reading the policy of insurance issued by the defendant to the plaintiff, it will be
noted that there is no provision in said policy prohibiting the plaintiff from placing a mortgage upon the
property insured.
b. Exception
i.

Life, accident and health insurance Sec 20, I.C.

By: Elaine Marie G. Laceda

57

INSURANCE LAW

Sec. 20. Except in the cases specified in the next four sections, and in the cases of life, accident, and
health insurance, a change of interest in any part of a thing insured unaccompanied by a corresponding
change in interest in the insurance, suspends the insurance to an equivalent extent, until the interest in
the thing and the interest in the insurance are vested in the same person.
ii.

Change of interest after occurrence of loss Sec 21, I.C.

Sec. 21. A change in interest in a thing insured, after the occurrence of an injury which results in a loss,
does not affect the right of the insured to indemnity for the loss.
iii.

Change of interest in things separately insured Sec 22, I.C.

Sec. 22. A change of interest in one or more several distinct things, separately insured by one policy,
does not avoid the insurance as to the others.
iv.

Transfer of interest by will or succession upon death of insured Sec 23,


I.C.

Sec. 23. A change on interest, by will or succession, on the death of the insured, does not avoid an
insurance; and his interest in the insurance passes to the person taking his interest in the thing insured.
v.

Transfer of interest by one partners, joint owners, or common owners


jointly insured, to the others Sec 24, I.C.

Sec. 24. A transfer of interest by one of several partners, joint owners, or owners in common, who are
jointly insured, to the others, does not avoid an insurance even though it has been agreed that the
insurance shall cease upon an alienation of the thing insured.
vi.

Insurance policy framed to insure to benefit of whomsoever becomes


owner of thing insured Sec 57, I.C.

Sec. 57. A policy may be so framed that it will inure to the benefit of whomsoever, during the
continuance of the risk, may become the owner of the interest insured.
6. Effect of Transfer of Thing Insured Sec 58, I.C.
Sec. 58. The mere transfer of a thing insured does not transfer the policy, but suspends it until the
same person becomes the owner of both the policy and the thing insured.
San Miguel Brewery vs. Law Union and Rock Insurance Co. (1920), supra
IV.

PREMIUM
A.

PAYMENT REQUIRED

1. Binding Effect of Payment Secs 77 and 306, par. 2, I.C.


Sec. 77. An insurer is entitled to payment of the premium as soon as the thing insured is exposed to
the peril insured against. Notwithstanding any agreement to the contrary, no policy or contract of
insurance issued by an insurance company is valid and binding unless and until the premium thereof has
been paid, except in the case of a life or an industrial life policy whenever the grace period provision
applies.
Sec. 306. x x x
Any insurance company which delivers to an insurance agent or insurance broker a policy or
contract of insurance shall be deemed to have authorized such agent or broker to receive on its behalf

By: Elaine Marie G. Laceda

58

INSURANCE LAW

payment of any premium which is due on such policy or contract of insurance at the time of its issuance
or delivery or which becomes due thereon.
South Sea Surety and Insurance Co., Inc. vs. Court of Appeals
G.R. No. 102253 June 2, 1995
On 16 January 1984, plaintiff Valenzuela Hardwood and Industrial Supply, Inc. entered into an
agreement with the defendant Seven Brothers whereby the latter undertook to load on board its vessel
M/V Seven Ambassador the former's lauan round logs numbering 940 at the port of Maconacon, Isabela
for shipment to Manila. The logs were insured by plaintiff against loss and/or, damage with defendant
South Sea Surety and Insurance Co., Inc. for P2M. On 24 January 1984, the plaintiff gave the check in
payment of the premium on the insurance policy to Mr. Victorio Chua. In the meantime, the said vessel
M/V Seven Ambassador sank on 25 January 1984 resulting in the loss of the plaintiffs insured logs.
On 30 January 1984, a check for P5,625.00 to cover payment of the premium and documentary stamps
due on the policy was tendered to the insurer but was not accepted. Instead, the South Sea Surety and
Insurance Co., Inc. cancelled the insurance policy it issued as of the date of inception for non-payment
of the premium. Thus, denied liability under the policy when plaintiff demanded for payment thereof. A
formal claim with defendant Seven Brothers Shipping Corporation was likewise denied.
Plaintiff then filed a complaint for the recovery of the value of lost logs and freight charges from Seven
Brothers Shipping Corporation or, to the extent of its alleged insurance cover, from South Sea Surety
and insurance Company. RTC decided for plaintiff. CA affirmed but absolved the shipping entity from
liability on the basis of a stipulation in the charter party that the ship owner would be exempted from
liability in case of loss.
Issue: WON Victorio Chua acted as an agent of the surety company or of the insured when he received
the check for insurance premiums.
Held: Yes.
Undoubtedly, the payment of the premium is a condition precedent to, and essential for, the
efficaciousness of the contract. The only two statutorily provided exceptions are (a) in case the
insurance coverage relates to life or industrial life (health) insurance when a grace period applies and
(b) when the insurer makes a written acknowledgment of the receipt of premium, this acknowledgment
being declared by law to be then conclusive evidence of the premium payment.
Section 306 of the Insurance Code provides as follows: Any insurance company which delivers to an
insurance agent or insurance broker a policy or contract of insurance shall be deemed to have
authorized such agent or broker to receive on its behalf payment of any premium which is due on such
policy of contract of insurance at the time of its issuance or delivery or which becomes due thereon.
On cross-examination in behalf of South Sea Surety and Insurance Co., Inc. Mr. Chua testified that the
marine cargo insurance policy for the plaintiff's logs was delivered to him, to be delivered to the plaintiff.
Thus, when the appellant South Sea Surety and Insurance Co., Inc. delivered to Mr. Chua the marine
cargo insurance policy for the plaintiffs logs, he is deemed to have been authorized by the South Sea
Surety and Insurance Co., Inc. to receive the premium which is due on its behalf. Therefore, when the
insured logs were lost, the insured had already paid the premium to an agent of the South Sea Surety
and Insurance Co., Inc., which is consequently liable to pay the insurance proceeds under the policy it
issued to the insured.
Areola vs. Court of Appeals
G.R. No. 95641 September 22, 1994
Petitioner-insured, Santos Areola, a lawyer from Dagupan City, procured from Prudential a personal
accident insurance policy covering a period of one-year from November 28, 1984. On December 17,
1984, respondent insurance company issued collector's provisional which has to be confirmed by an
official receipt within 7 days. However, on June 29, 1985, 7 months after the issuance of the policy,

By: Elaine Marie G. Laceda

59

INSURANCE LAW

respondent insurance company unilaterally cancelled the same since company records revealed that
petitioner-insured failed to pay his premiums. But on August 3, 1985, respondent insurance company
offered to reinstate same policy it had previously cancelled and even proposed to extend its lifetime to
December 17, 1985, upon a finding that the cancellation was erroneous and that the premiums were
paid in full by petitioner-insured but were not remitted by Teofilo M. Malapit, respondent insurance
company's branch manager.
An action for damages due to breach of contract was instituted by petitioner-insured. Respondent
insurance company admitted that the cancellation of petitioner-insured's policy was due to the failure of
Malapit to turn over the premiums collected, for which reason no official receipt was issued to him.
However, it argued that, by acknowledging the inconvenience caused on petitioner-insured and after
taking steps to rectify its omission by reinstating the cancelled policy prior to the filing of the complaint,
respondent insurance company had complied with its obligation under the contract. Hence, it concluded
that petitioner-insured no longer has a cause of action against it. It insists that it cannot be held liable
for damages arising from breach of contract, having demonstrated fully well its fulfillment of its
obligation.
RTC held that there was breach of contract on the part of respondent insurance company, entitling
petitioner-insured to an award of the damages prayed for. It declared that respondent insurance
company acted in bad faith in unilaterally canceling subject insurance policy, having done so only after
seven months from the time that it had taken force and effect and despite the fact of full payment of
premiums and other charges on the issued insurance policy.
CA reversed and held that insurance company was not motivated by negligence, malice or bad faith in
cancelling subject policy. Rather, the cancellation of the insurance policy was based on what the existing
records showed, i.e., absence of an official receipt issued to petitioner-insured confirming payment of
premiums.
Issue: WON the erroneous act of canceling subject insurance policy entitle petitioner-insured to payment
of damages.
Held: Yes.
Malapit's fraudulent act of misappropriating the premiums paid by petitioner-insured is beyond doubt
directly imputable to respondent insurance company. A corporation, such as respondent insurance
company, acts solely thru its employees. The latters' acts are considered as its own for which it can be
held to account.
Malapit was the manager of its Baguio branch. It is beyond doubt that he represented its interest and
acted in its behalf. His act of receiving the premiums collected is well within the province of his
authority. Thus, his receipt of said premiums is receipt by private respondent insurance company who,
by provision of law, particularly under Article 1910 of the Civil Code, is bound by the acts of its agent.
The fact that private respondent insurance company was itself defrauded due to the anomalies that took
place in its Baguio branch office, such as the non-accrual of said premiums to its account, does not free
the same from its obligation to petitioner Areola.
A contract of insurance creates reciprocal obligations for both insurer and insured. Reciprocal obligations
are those which arise from the same cause and in which each party is both a debtor and a creditor of
the other, such that the obligation of one is dependent upon the obligation of the other. Under the
circumstances of instant case, the relationship as creditor and debtor between the parties arose from a
common cause: i.e., by reason of their agreement to enter into a contract of insurance under whose
terms, respondent insurance company promised to extend protection to petitioner-insured against the
risk insured for a consideration in the form of premiums to be paid by the latter. Under the law
governing reciprocal obligations, particularly the second paragraph of Article 1191, the injured party,
petitioner-insured in this case, is given a choice between fulfillment or rescission of the obligation in case
one of the obligors, such as respondent insurance company, fails to comply with what is incumbent upon
him. However, said article entitles the injured party to payment of damages, regardless of whether he
demands fulfillment or rescission of the obligation. Untenable then is reinstatement insurance company's

By: Elaine Marie G. Laceda

60

INSURANCE LAW

argument, namely, that reinstatement being equivalent to fulfillment of its obligation, divests petitionerinsured of a rightful claim for payment of damages. Such a claim finds no support in our laws on
obligations and contracts.
2. Effect of Non-Payment Sec 64(a), I.C.
Sec. 64. No policy of insurance other than life shall be cancelled by the insurer except upon prior notice
thereof to the insured, and no notice of cancellation shall be effective unless it is based on the
occurrence, after the effective date of the policy, of one or more of the following:
(a) non-payment of premium;
National Leather Co., Inc. vs. United States Life Insurance Co.
G.R. No. L-2668 September 30, 1950
On April 14, 1939, plaintiff insured with the defendant the life of Pedro Alejandrino for P5,000 under a
10-year term non-participating policy in consideration of the payment to the defendant, in advance, of
the sum of $23.11 as quarterly premium beginning April 14, 1939, and the payment of a like sum every
quarter, thereafter, also in advance, for a period of 10 years or until the prior death of said Pedro
Alejandrino. The stipulated quarterly premiums on the policy were regularly paid up to October 14,
1941, when the last quarterly premium payment was made, covering the period from that date to
January 14, 1942. Thereafter no more premiums were paid, it appearing that, because defendant was
an American corporation, its branch office in Manila was closed when that city was occupied by the
Japanese forces on January 2, 1942, and it had remained closed during the entire period of enemy
occupation.
Pedro Alejandrino died on September 23, 1943, that is, beyond the period covered by the premiums
paid by the insured. But just the same, after the liberation of Manila in 1945, plaintiff, as the beneficiary
named in the policy, made a claim for the proceeds thereof and tendered a check for P323.54, the total
unpaid premiums from January 14, 1942, to October 14, 1943. Defendant, however, rejected the claim
and returned the check on the ground that the policy had ceased to be in force as of January 14, 1942,
for non-payment of the stipulated premiums.
Issue: WON a life insurance policy lapses pursuant to its terms because of non payment of the stipulated
premium when such non-payment occurred at a time when the insurer and the assured were separated
by the lines of war.
Held: Yes.
In determining the effect of non-payment of premiums occasioned by war, we follow the United States
rule which declares that the contract is not merely suspended, but is abrogated by reason of nonpayment of premiums, since the time of the payments is peculiarly of the essence of the contract. The
doctrine of the revival of contracts suspended during the war is one based on considerations of equity
and justice, and cannot be invoked to revive a contract which it would be unjust or inequitable to revive.
It should be noted that the parties contracted not only for peacetime conditions but also for times of
war, because the policies contained provisions applicable expressly to wartime days. The logical
inference, therefore, is that the parties contemplated uninterrupted operation of the contract even if
armed conflict should ensue.
The annual premium due is not a debt. It is not an obligation upon which the insurer can maintain an
action against insured; nor is its settlement governed by the strict rule controlling payment of debts.
The fact that it is payable annually or semi-annually, or at any other stipulated time, does not of itself
constitute a promise to pay, either express or implied. In case of non-payment, the policy is forfeited,
except so far as the forfeiture may be saved by agreement, by waiver, estoppel, or by statute. The
payment of the premium is entirely optional, while a debt may be enforced at law, and the fact that the
premium is agreed to be paid is without force, in the absence of an unqualified and absolute agreement
to pay a specified sum at some certain time. In the ordinary policy there is no promise to pay, but it is
optional with the insured whether he will continue the policy or forfeit it.

By: Elaine Marie G. Laceda

61

INSURANCE LAW

Sales de Gonzaga vs. Crown Life Insurance


G.R. No. L-4197 March 20, 1952
On September 26, 1939 the Crown Life Insurance Co. issued to Ramon Gonzaga a 20-year endowment
policy for P15,000. The insured paid in due time the agreed yearly premium, which was P591.00, for
three consecutive years, the last payment having been effected on September 6, 1941. On account of
the outbreak of war, no premiums were paid after that date, although the policy was continued in force
up to June 12, 1943, under its automatic premium loan clause.
Ramon Gonzaga died on June 27, 1945 from an accident. Unsuccessful in her attempt to collect the
amount of the policy, his widow and the beneficiary named in the policy commenced suit against
defendant company who set up the defense that the policy had lapsed by non-payment of the stipulated
premiums of the stipulated dates. It alleged that despite the outbreak of war, it had its offices open in
the city of Manila during the Japanese occupation in the Philippines. The plaintiff asserts however that it
was the defendant's duty to notify her husbands of its postal address during the war, and that its failure
to do so excused deliquency in the payment of the premiums. TC ruled against the plaintiff.
Issue: WON non-payment of premiums by reason of war puts an end to the contract.
Held: Yes.
The insured had no right to demand that the defendant maintain an office during the war, and the
defendant was not obligated to do so. Had the defendant not opened any office at all during the
occupation and stopped receiving premiums absolutely, the plaintiff's position would not have been any
better or worse for the closing and suspension of the defendant's business. Had the plaintiff's husband
actually tendered his premiums and the defendant's employees rejected them, he could not have
insisted on the payment as a matter of right. Stated otherwise, the defendant's opening of an interim
office partook of the nature of the privilege to the policy holders to keep their policies operative rather
than a duty to them under the contract.
Valenzuela vs. Court of Appeals
G.R. No. 83122 October 19, 1990
Petitioner Valenzuela is a General Agent of private respondent Philamgen since 1965. As such, he was
authorized to solicit and sell in behalf of Philamgen all kinds of non-life insurance, and in consideration of
services rendered was entitled to receive the full agent's commission of 32.5% from Philamgen under
the scheduled commission rates. From 1973 to 1975, Valenzuela solicited marine insurance from one of
his clients, the Delta Motors, Inc. in the amount of P4.4 Million from which he was entitled to a
commission of 32%. However, Valenzuela did not receive his full commission which amounted to P1.6
Million. Likewise, during the period 1976 to 1978, premium payments amounting to P1,946,886.00 were
paid directly to Philamgen and Valenzuela's commission to which he is entitled amounted to
P632,737.00. In 1977, Philamgen started to become interested in and expressed its intent to share in
the commission due Valenzuela on a 50-50 basis. Valenzuela refused. Again, on February 8, 1978
Philamgen and its President, Bienvenido M. Aragon insisted on the sharing of the commission with
Valenzuela. This was followed by another sharing proposal dated June 1, 1978. But Valenzuela firmly
objected to the proposals.
Because of the refusal of Valenzuela, Philamgen and its officers took drastic action against Valenzuela.
They: (a) reversed the commission due him by not crediting in his account the commission earned from
the Delta Motors, Inc. insurance; (b) placed agency transactions on a cash and carry basis; (c)
threatened the cancellation of policies issued by his agency; and (d) started to leak out news that
Valenzuela has a substantial account with Philamgen. All of these acts resulted in the decline of his
business as insurance agent. On December 27, 1978, Philamgen terminated the General Agency
Agreement of Valenzuela. Thus, petitioner sought relief by filing the complaint against the private
respondents. TC ruled in favor of Valenzuela and ordered his reinstatement with payment of his
commissions and damages. CA modified judgment by removing award of damages.

By: Elaine Marie G. Laceda

62

INSURANCE LAW

Issue: WON Philamgen and/or its officers can be held liable for damages due to the termination of the
General Agency Agreement it entered into with the petitioners.
Held: Yes.
The principal cause of the termination of Valenzuela as General Agent of Philamgen arose from his
refusal to share his Delta commission. The records sustain the conclusions of the trial court on the
apparent bad faith of the private respondents in terminating the General Agency Agreement of
petitioners.

By: Elaine Marie G. Laceda

63

INSURANCE LAW

It is also evident from the records that the agency involving petitioner and private respondent is one
"coupled with an interest," and, therefore, should not be freely revocable at the unilateral will of the
latter. The private respondents by the simple expedient of terminating the General Agency Agreement
appropriated the entire insurance business of Valenzuela. With the termination of the General Agency
Agreement, Valenzuela would no longer be entitled to commission on the renewal of insurance policies
of clients sourced from his agency. Worse, despite the termination of the agency, Philamgen continued
to hold Valenzuela jointly and severally liable with the insured for unpaid premiums. Under these
circumstances, it is clear that Valenzuela had an interest in the continuation of the agency when it was
unceremoniously terminated not only because of the commissions he should continue to receive from
the insurance business he has solicited and procured but also for the fact that by the very acts of the
respondents, he was made liable to Philamgen in the event the insured fail to pay the premiums due.
They are estopped by their own positive averments and claims for damages. Therefore, the respondents
cannot state that the agency relationship between Valenzuela and Philamgen is not coupled with
interest.
A wrongfully discharged agent has a right of action for damages. The principal may not defeat the
agent's right to indemnification by a termination of the contract of agency. Where the principal
terminates or repudiates the agent's employment in violation of the contract of employment and without
cause ... the agent is entitled to receive either the amount of net losses caused and gains prevented by
the breach, or the reasonable value of the services rendered. Thus, the agent is entitled to prospective
profits which he would have made except for such wrongful termination provided that such profits are
not conjectural, or speculative but are capable of determination upon some fairly reliable basis.
The pivotal factor rendering Philamgen and the other private respondents liable in damages is that the
termination by them of the General Agency Agreement was tainted with bad faith. Hence, if a principal
acts in bad faith and with abuse of right in terminating the agency, then he is liable in damages.
Issue: WON the petitioner is liable to Philamgen for the unpaid and uncollected.
Held: No.
Under Section 77 of the Insurance Code, the remedy for the non-payment of premiums is to put an end
to and render the insurance policy not binding. Since admittedly the premiums have not been paid, the
policies issued have lapsed. The insurance coverage did not go into effect or did not continue and the
obligation of Philamgen as insurer ceased. Hence, for Philamgen which had no more liability under the
lapsed and inexistent policies to demand, much less sue Valenzuela for the unpaid premiums would be
the height of injustice and unfair dealing. In this instance, with the lapsing of the policies through the
nonpayment of premiums by the insured there were no more insurance contracts to speak of.
3. Statutory Exceptions
a. Grace period Sec 77, I.C.
Sec. 77. An insurer is entitled to payment of the premium as soon as the thing insured is exposed to
the peril insured against. Notwithstanding any agreement to the contrary, no policy or contract of
insurance issued by an insurance company is valid and binding unless and until the premium thereof has
been paid, except in the case of a life or an industrial life policy whenever the grace period provision
applies.
i.

For individual life or endowment insurance Sec 227(a), I.C.

Sec. 227. In the case of individual life or endowment insurance, the policy shall contain in substance
the following conditions:
(a) A provision that the policyholder is entitled to a grace period either of thirty days or of one
month within which the payment of any premium after the first may be made, subject at the option of
the insurer to an interest charge not in excess of six per centum per annum for the number of days of
grace elapsing before the payment of the premium, during which period of grace the policy shall
continue in full force, but in case the policy becomes a claim during the said period of grace before the

By: Elaine Marie G. Laceda

64

INSURANCE LAW

overdue premium is paid, the amount of such premium with interest may de deducted from the amount
payable under the policy in settlement;
ii.

For group life insurance Sec 228(a), I.C.

Sec. 228. No policy of group life insurance shall be issued and delivered in the Philippines unless it
contains in substance the following provisions, or provisions which in the opinion of the Commissioner
are more favorable to the persons insured, or at least as favorable to the persons insured and more
favorable to the policy-holders:
(a) A provision that the policyholder is entitled to a grace period of either thirty days or of one
month for the payment of any premium due after the first, during which grace period the death benefit
coverage shall continue in force, unless the policyholder shall have given the insurer written notice of
discontinuance in advance of the date of discontinuance and in accordance with the terms of the policy.
The policy may provide that the policyholder shall be liable for the payment of a pro rata premium for
the time the policy is in force during such grace period;
iii.

For industrial life insurance Sec 230(a), I.C.

Sec. 230. In the case of industrial life insurance, the policy shall contain in substance the following
provisions:
(a) A provision that the insured is entitled to a grace period of four weeks within which the
payment of any premium after the first may be made, except that where premiums are payable
monthly, the period of grace shall be either one month or thirty days; and that during the period of
grace, the policy shall continue in full force, but if during such grace period the policy becomes a claim,
then any overdue and unpaid premiums may be deducted from any amount payable under the policy in
settlement;
b. Acknowledgement receipt Sec 78, I.C.
Sec. 78. An acknowledgment in a policy or contract of insurance or the receipt of premium is conclusive
evidence of its payment, so far as to make the policy binding, notwithstanding any stipulation therein
that it shall not be binding until the premium is actually paid.
American Home Assurance Company vs. Chua
G.R. No. 130421 June 28, 1999
Sometime in 1990, respondent obtained from petitioner a fire insurance covering the stock-in-trade of
his business, Moonlight Enterprises. Upon expiration of the policy or on 5 April 1990, respondent issued
a PCI Bank Check to petitioner's agent, James Uy, as payment for the renewal of the policy. The check
was drawn against a Manila bank and deposited in petitioner's bank account in Cagayan de Oro City with
the corresponding official receipt issued on April 10. Subsequently, a new insurance policy was issued,
whereby petitioner undertook to indemnify respondent for any damage or loss arising from fire up to
P200,000 for the period 25 March 1990 to 25 March 1991.
On 6 April 1990 Moonlight Enterprises was completely razed by fire. Respondent filed an insurance claim
with petitioner but petitioner refused to honor the claim notwithstanding several demands by
respondent, thus, the latter filed an action against petitioner before the trial court. Petitioner claimed
there was no existing insurance contract when the fire occurred since respondent did not pay the
premium.
RTC ruled in favor of respondent. It found that respondent paid by way of check a day before the fire
occurred. The check, which was deposited in petitioner's bank account, was even acknowledged in the
renewal certificate issued by petitioner's agent. CA affirmed in toto.
Petitioner emphasizes that when the fire occurred on 6 April 1990 the insurance contract was not yet
subsisting pursuant to Article 1249 of the Civil Code, which recognizes that a check can only effect
payment once it has been cashed. Although respondent testified that he gave the check on 5 April to a
certain James Uy, the check, drawn against a Manila bank and deposited in a Cagayan de Oro City bank,

By: Elaine Marie G. Laceda

65

INSURANCE LAW

could not have been cleared by 6 April, the date of the fire. In fact, the official receipt issued for
respondent's check payment was dated 10 April 1990, four days after the fire occurred.
Issue: Whether there was a valid payment of premium, considering that respondent's check was cashed
after the occurrence of the fire.
Held: Yes.
The general rule in insurance laws is that unless the premium is paid the insurance policy is not valid
and binding. The only exceptions are life and industrial life insurance. Whether payment was indeed
made is a question of fact which is best determined by the trial court. The trial court found, as affirmed
by the Court of Appeals, that there was a valid check payment by respondent to petitioner. We see no
reason to depart from this ruling.
According to the trial court the renewal certificate issued to respondent contained the acknowledgment
that premium had been paid. It is not disputed that the check drawn by respondent in favor of petitioner
and delivered to its agent was honored when presented and petitioner forthwith issued its official receipt
to respondent on 10 April 1990. Section 306 of the Insurance Code provides that any insurance
company which delivers a policy or contract of insurance to an insurance agent or insurance broker shall
be deemed to have authorized such agent or broker to receive on its behalf payment of any premium
which is due on such policy or contract of insurance at the time of its issuance or delivery or which
becomes due thereon. In the instant case, the best evidence of such authority is the fact that petitioner
accepted the check and issued the official receipt for the payment. It is, as well, bound by its agent's
acknowledgment of receipt of payment.
Sec. 78 of the Insurance Code explicitly provides: An acknowledgment in a policy or contract of
insurance of the receipt of premium is conclusive evidence of its payment, so far as to make the policy
binding, notwithstanding any stipulation therein that it shall not be binding until the premium is actually
paid. This Section establishes a legal fiction of payment and should be interpreted as an exception to
Section 77 which provides that no policy or contract of insurance issued by an insurance company is
valid and binding unless and until the premium thereof has been paid.
Issue: WON there was a violation of the policy that entitles the insurer to avoid the policy.
Held: No.
Ordinarily, where the insurance policy specifies as a condition the disclosure of existing co-insurers, nondisclosure thereof is a violation that entitles the insurer to avoid the policy. This condition is common in
fire insurance policies and is known as the "other insurance clause." The purpose for the inclusion of this
clause is to prevent an increase in the moral hazard.
Citing Section 29 of the Insurance Code, the trial court reasoned that respondent's failure to disclose
was not intentional and fraudulent. The application of Section 29 is misplaced. Section 29 concerns
concealment which is intentional.
Furthermore, to constitute a violation the other existing insurance contracts must be upon the same
subject matter and with the same interest and risk. Indeed, respondent acquired several co-insurers and
he failed to disclose this information to petitioner. Nonetheless, petitioner is estopped from must
invoking this argument. The trial court cited the testimony of petitioner's loss adjuster who admitted
previous knowledge of the co-insurers. The loss adjuster, being an employee of petitioner, is deemed a
representative of the latter whose awareness of the other insurance contracts binds petitioner.
c.

Acceptance by obligee of surety bond Sec 177, I.C.

Sec. 177. The surety is entitled to payment of the premium as soon as the contract of suretyship or
bond is perfected and delivered to the obligor. No contract of suretyship or bonding shall be valid and
binding unless and until the premium therefor has been paid, except where the obligee has accepted the
bond, in which case the bond becomes valid and enforceable irrespective of whether or not the premium
has been paid by the obligor to the surety: Provided, That if the contract of suretyship or bond is not

By: Elaine Marie G. Laceda

66

INSURANCE LAW

accepted by, or filed with the obligee, the surety shall collect only reasonable amount, not exceeding
fifty per centum of the premium due thereon as service fee plus the cost of stamps or other taxes
imposed for the issuance of the contract or bond: Provided, however, That if the non-acceptance of the
bond be due to the fault or negligence of the surety, no such service fee, stamps or taxes shall be
collected.
In the case of a continuing bond, the obligor shall pay the subsequent annual premium as it falls
due until the contract of suretyship is cancelled by the obligee or by the Commissioner or by a court of
competent jurisdiction, as the case may be.
Philippine Pryce Assurance Corp. vs. Court of Appeals
G.R. No. 107062 February 21, 1994
Gegroco, Inc. filed with the RTC a collection case against petitioner Philippine Pryce on account of two
surety bonds issued by petitioner in behalf of its principal Sagum General Merchandise for P500,000.00
and 1,000,000.00. In its answer, petitioner admitted having executed the said bonds, but denied liability
because allegedly (1) the checks which were to pay for the premiums bounced and were dishonored
hence there is no contract to speak of between petitioner and its supposed principal; and (2) that the
bonds were merely to guarantee payment of its principal's obligation, thus, excussion is necessary. TC
ruled for plaintiff. CA affirmed.
Issue: WON petitioner is liable as a surety.
Held: Yes.
Sec 177 of the Insurance Code provides: The surety is entitled to payment of the premium as soon as
the contract of suretyship or bond is perfected and delivered to the obligor. No contract of suretyship or
bonding shall be valid and binding unless and until the premium therefor has been paid, except where
the obligee has accepted the bond, in which case the bond becomes valid and enforceable irrespective
of whether or not the premium has been paid by the obligor to the surety. This provision outrightly
negates petitioner's first defense.
Furthermore, petitioner, in its answer, admitted to have issued the bonds subject matter of the original
action. Petitioner's defense that it did not have authority to issue a Surety Bond when it did is an
admission of fraud committed against respondent. No person can claim benefit from the wrong he
himself committed. A representation made is rendered conclusive upon the person making it and cannot
be denied or disproved as against the person relying thereon.
d. Rules on cover notes (if premium cannot yet be computed) Sec 52, I.C.
Sec. 52. Cover notes may be issued to bind insurance temporarily pending the issuance of the policy.
Within sixty days after the issue of the cover note, a policy shall be issued in lieu thereof, including
within its terms the identical insurance bound under the cover note and the premium therefor.
Cover notes may be extended or renewed beyond such sixty days with the written approval of
the Commissioner if he determines that such extension is not contrary to and is not for the purpose of
violating any provisions of this Code. The Commissioner may promulgate rules and regulations
governing such extensions for the purpose of preventing such violations and may by such rules and
regulations dispense with the requirement of written approval by him in the case of extension in
compliance with such rules and regulations.
Pacific Timber vs. Court of Appeals
G.R. No. L-38613 February 25, 1982
On March 19, l963, the plaintiff secured temporary insurance from the defendant for its exportation of
1,250,000 board feet of Philippine Lauan and Apitong logs to be shipped from the Diapitan Bay, Quezon
Province to Okinawa and Tokyo, Japan. The defendant issued on said date Cover Note No. 1010,
insuring the said cargo of the plaintiff. After the issuance of Cover Note No. 1010, but before the
issuance of the marine policies on April 2, 1963, some of the logs intended to be exported were lost
during loading operations in the Diapitan Bay.

By: Elaine Marie G. Laceda

67

INSURANCE LAW

The plaintiff subsequently submitted a 'Claim Statement demanding payment of the loss under Policies
Nos. 53 HO 1032 and 53 HO 1033, in the total amount of P19,286.79. The claim was denied on the
ground that the said loss may be considered as covered under Cover Note No. 1010 which had become
'null and void by virtue of the issuance of Marine Policies. Defendant later alleged that the cover note is
null and void for lack of valuable consideration. In a complaint filed by plaintiff, CFI ordered payment by
defendant. CA reversed. Hence the petition.
Issue: WON plaintiff cannot claim on the Cover Note for lack of consideration.
Held: No.
The fact that no separate premium was paid on the Cover Note before the loss insured against occurred,
does not militate against the validity of petitioner's contention, for no such premium could have been
paid, since by the nature of the Cover Note, it did not contain, as all Cover Notes do not contain
particulars of the shipment that would serve as basis for the computation of the premiums. As a logical
consequence, no separate premiums are intended or required to be paid on a Cover Note.
At any rate, it is not disputed that petitioner paid in full all the premiums as called for by the statement
issued by private respondent after the issuance of the two regular marine insurance policies, thereby
leaving no account unpaid by petitioner due on the insurance coverage, which must be deemed to
include the Cover Note. If the Note is to be treated as a separate policy instead of integrating it to the
regular policies subsequently issued, the purpose and function of the Cover Note would be set at naught
or rendered meaningless, for it is in a real sense a contract, not a mere application for insurance which
is a mere offer.
It may be true that the marine insurance policies issued were for logs no longer including those which
had been lost during loading operations. This had to be so because the risk insured against is not for
loss during operations anymore, but for loss during transit, the logs having already been safely placed
aboard. This would make no difference, however, insofar as the liability on the cover note is concerned,
for the number or volume of logs lost can be determined independently as in fact it had been so
ascertained at the instance of private respondent itself when it sent its own adjuster to investigate and
assess the loss, after the issuance of the marine insurance policies.
e. Non-lapse of individual life or endowment insurance
i.

Automatic policy loan Sec 227(g), I.C.

Sec. 227. In the case of individual life or endowment insurance, the policy shall contain in substance
the following conditions:
(g) A provision that at anytime after a cash surrender value is available under the policy and
while the policy is in force, the company will advance, on proper assignment or pledge of the policy and
on sole security thereof, a sum equal to, or at the option of the owner of the policy, less than the cash
surrender value on the policy, at a specified rate of interest, not more than the maximum allowed by
law, to be determined by the company from time to time, but not more often than once a year, subject
to the approval of the Commissioner; and that the company will deduct from such loan value any
existing indebtedness on the policy and any unpaid balance of the premium for the current policy year,
and may collect interest in advance on the loan to the end of the current policy year, which provision
may further provide that such loan may be deferred for not exceeding six months after the application
therefor is made;
ii.

Application of dividend Sec 227(e), I.C.

Sec. 227. In the case of individual life or endowment insurance, the policy shall contain in substance
the following conditions:
(e) If the policy is participating, a provision that the company shall periodically ascertain and
apportion any divisible surplus accruing on the policy under conditions specified therein;
iii.

Reinstatement Clause Sec 227(j), I.C.

By: Elaine Marie G. Laceda

68

INSURANCE LAW

Sec. 227. In the case of individual life or endowment insurance, the policy shall contain in substance
the following conditions:
(j) A provision that the policyholder shall be entitled to have the policy reinstated at any time
within three years from the date of default of premium payment unless the cash surrender value has
been duly paid, or the extension period has expired, upon production of evidence of insurability
satisfactory to the company and upon payment of all overdue premiums and any indebtedness to the
company upon said policy, with interest rate not exceeding that which would have been applicable to
said premiums and indebtedness in the policy years prior to reinstatement.
4. Jurisprudential Exceptions
a. Estoppel and credit extension
Philippine Phoenix Surety & Insurance Co. vs. Woodworks
G.R. No. L-25317 August 6, 1979
On July 21, 1960, upon defendant Woodworks, Inc.s application, plaintiff Philippine Phoenix Surety &
Insurance Company issued in its favor Fire Insurance Policy No. 9749 for P500,000.00 whereby plaintiff
insured defendant's building, machinery and equipment for a term of one year from July 21, 1960 to
July 21, 1961 against loss by fire. Defendant did not pay the premium stipulated in the Policy when it
was issued nor at any time thereafter. Before the expiration of the one-year term, plaintiff notified
defendant of the cancellation of the Policy allegedly upon request of defendant and credited defendant
with the amount of P3,110.25 for the unexpired period of 94 days, but claimed the balance of P7,483.11
representing the premium from July 21, 1960 to 18th April 1961. Defendant disclaimed any liability
contending, in essence, that it need not pay premium "because the Insurer did not stand liable for any
indemnity during the period the premiums were not paid."
An action for recovery of unpaid premium was instituted by plaintiff against defendant Woodworks, Inc.
Defendant controverted basically on the theory that its failure "to pay the premium after the issuance of
the policy put an end to the insurance contract and rendered the policy unenforceable." CFI ruled in
favor of plaintiff and ordering defendant to pay the earned premium.
Issue: WON a premium not paid is a debt enforceable by action of the insurer.
Held: No.
Insurance is "a contract whereby one undertakes for a consideration to indemnify another against loss,
damage or liability arising from an unknown or contingent event." The consideration is the "premium".
"The premium must be paid at the time and in the way and manner specified in the policy and, if not so
paid, the policy will lapse and be forfeited by its own terms."
The Policy provides for pre-payment of premium. Accordingly; "when the policy is tendered the insured
must pay the premium unless credit is given or there is a waiver, or some agreement obviating the
necessity for prepayment." To constitute an extension of credit there must be a clear and express
agreement therefor." However, we fail to find any clear agreement that a credit extension was accorded
defendant. And even if it were to be presumed that plaintiff had extended credit from the circumstances
of the unconditional delivery of the Policy without prepayment of the premium, yet it is obvious that
defendant had not accepted the insurer's offer to extend credit, which is essential for the validity of such
agreement. An acceptance of an offer to allow credit, if one was made, is as essential to make a valid
agreement for credit, to change a conditional delivery of an insurance policy to an unconditional
delivery, as it is to make any other contract. Such an acceptance could not be merely a mental act or
state of mind, but would require a promise to pay made known in some manner to defendant.
Since the premium had not been paid, the policy must be deemed to have lapsed. The non-payment of
premiums does not merely suspend but put, an end to an insurance contract, since the time of the
payment is peculiarly of the essence of the contract.

By: Elaine Marie G. Laceda

69

INSURANCE LAW

Capital Insurance and Surety Co. vs. Plastic Era Co., Inc.
G.R. No. L-22375 July 18, 1975
On December 17, 1960, petitioner Capital Insurance delivered to the respondent Plastic Era Mfg. its
open Fire Policy No. 22760 wherein the former undertook to insure the latter's building, equipments, raw
materials, products and accessories. The policy expressly provides that if the property insured would be
destroyed or damaged by fire after the payment of the premiums, at anytime between December 15,
1960 and 1pm December 15, 1961, the insurance company shall make good all such loss or damage in
an amount not exceeding P100,000. When the policy was delivered, Plastic Era failed to pay the
corresponding insurance premium. However, through its duly authorized representative, it executed an
acknowledgment receipt of the policy with a promise to pay the premium 30 days after effectivity date.
On January 8, 1961, in partial payment of the insurance premium, Plastic Era delivered to Capital
Insurance, a check for the amount of P1,000 postdated January 16, 1961 payable to the order of the
latter and drawn against the Bank of America. The records show that as of January 19, 1961 Plastic Era
had a balance of P1,193.41 with the Bank of America. However, Capital Insurance tried to deposit the
check only on February 20, 1961 and the same was dishonored by the bank for lack of funds.
Meanwhile, on January 18, 1961 or two days after the insurance premium became due, the property
insured by Plastic Era was destroyed by fire. In due time, the latter notified Capital Insurance of the loss
of the insured property by fire and accordingly filed its claim for indemnity. According to the records the
same property has been insured by Plastic Era with the Philamgen Insurance Company for P200,000.
Capital Insurance refused payment for the reason that, among others, Plastic Era failed to pay the
insurance premium. Hence, Plastic Era filed its complaint against Capital Insurance for the recovery. TC
ruled for plaintiff. CA affirmed.
Issue: WON a contract of insurance has been duly perfected between the petitioner, Capital Insurance,
and respondent Plastic Era.
Held: Yes.
In clear and unequivocal terms the insurance policy provides that it is only upon payment of the
premiums by Plastic Era that Capital Insurance agrees to insure the properties of the former against loss
or damage in an amount not exceeding P100,000.
It appears on record that on the day the insurance policy was delivered, Plastic Era did not pay the
Capital Insurance, but instead executed an acknowledgment receipt of Policy No. 22760. In said receipt
Plastic Era promised to pay the premium within 30 days from the effectivity date of the policy on
December 17, 1960 and Capital Insurance accepted it.
Pursuant to Article 1249 of the New Civil Code: The delivery of promissory notes payable to order, or
bills of exchange or other mercantile documents shall produce the effect of payment only when they
have been cashed, or when through the fault of the creditor they have been impaired. Under this
provision the mere delivery of a bill of exchange in payment of a debt does not immediately effect
payment. It simply suspends the action arising from the original obligation in satisfaction of which it was
delivered, until payment is accomplished either actually or presumptively. Tender of draft or check in
order to effect payment that would extinguish the debtor's liability should be actually cashed. If the
delivery of the check of Plastic Era to Capital Insurance were to be viewed in the light of the foregoing,
no payment of the premium had been effected, for it is only when the check is cashed that it is said to
effect payment.
Significantly, however, the Capital Insurance accepted the promise of Plastic Era to pay the insurance
premium within 30 days from the effective date of policy. By so doing, it has implicitly agreed to modify
the tenor of the insurance policy and in effect, waived the provision therein that it would only pay for the
loss or damage in case the same occurs after the payment of the premium. Considering that the
insurance policy is silent as to the mode of payment, Capital Insurance is deemed to have accepted the
promissory note in payment of the premium. This rendered the policy immediately operative on the date
it was delivered. In the absence of express agreement or stipulation to that effect in the policy, the non-

By: Elaine Marie G. Laceda

70

INSURANCE LAW

payment at maturity of a note given for and accepted as premium on a policy does not operate to forfeit
the rights of the insured even though the note is given for an initial premium, nor does the fact that the
collection of the note had been enjoined by the insured in any way affect the policy.
By accepting its promise to pay the insurance premium within 30 days from the effectivity date of the
policy, Capital Insurance had in effect extended credit to Plastic Era. The payment of the premium on
the insurance policy therefore became an independent obligation the non-fulfillment of which would
entitle Capital Insurance to recover. It could just deduct the premium due and unpaid upon the
satisfaction of the loss under the policy. It did not have the right to cancel the policy for nonpayment of
the premium except by putting Plastic Era in default and giving it personal notice to that effect.
Arce vs. Capital Insurance and Surety Co.
G.R. No. L-28501 September 30, 1982
INSURED Pedro Arce insured his residential house in Tondo, Manila with the Capital Insurance and
Surety Co., Inc. (COMPANY). On November 27, 1965, the COMPANY sent to the INSURED Renewal
Certificate No. 47302 to cover the period December 5, 1965 to December 5, 1966. The COMPANY also
requested payment of the corresponding premium in the amount of P38.10. Anticipating that the
premium could not be paid on time, the INSURED, thru his wife, promised to pay it on January 4, 1966.
The COMPANY accepted the promise but the premium was not paid on January 4, 1966. On January 8,
1966, the house of the INSURED was totally destroyed by fire. Thereafter, INSURED's wife presented a
claim for indemnity to the COMPANY but was told that no indemnity was due because the premium on
the policy was not paid. Nonetheless the COMPANY tendered a check for P300.00 as financial aid which
was received by the INSURED's daughter, Evelina R. Arce. The voucher for the check which Evelina
signed stated that it was "in full settlement (ex gratia) of the fire loss claim." The INSURED cashed the
check but then sued the COMPANY on the policy. TC held that since the COMPANY could have demanded
payment of the premium, mutuality of obligation requires that it should also be liable on its policy.
Issue: WON the policy became effective upon the COMPANYs acceptance of the promise to pay.
Held: No.
It is obvious from both the Insurance Act, as amended, and the stipulation of the parties that time is of
the essence in respect of the payment of the insurance premium so that if it is not paid the contract
does not take effect unless there is still another stipulation to the contrary. In the instant case, the
INSURED was given a grace period to pay the premium but the period having expired with no payment
made, he cannot insist that the COMPANY is nonetheless obligated to him.
Malayan Insurance Co., Inc. vs. Arnaldo
G.R. No. L-67835 October 12, 1987
Petitioner issued to private respondent, Coronacion Pinca, Fire Insurance Policy No. F-001-17212 on her
property for the amount of P14,000.00 effective July 22, 1981, until July 22, 1982. Howver, on October
15, 1981, MICO allegedly cancelled the policy for non-payment, of the premium and sent the
corresponding notice to Pinca. Payment of the premium was then later received by Domingo Adora,
agent of MICO who remitted this payment to MICO, together with other payments on January 15, 1982.
On January 18, 1982, Pinca's property was completely burned. On February 5, 1982, Pinca's payment
was returned by MICO to Adora on the ground that her policy had been cancelled earlier. But Adora
refused to accept it. In due time, Pinca made the requisite demands for payment, which MICO rejected.
She then went to the Insurance Commission which sustained her claim for compensation for her burned
property.
Issue: WON Adora had the authority to receive payment.
Held: Yes.
MICO's acknowledgment of Adora as its agent defeats its contention that he was not authorized to
receive the premium payment on its behalf. It is clearly provided in Section 306 of the Insurance Code

By: Elaine Marie G. Laceda

71

INSURANCE LAW

that: Any insurance company which delivers to an insurance agant or insurance broker a policy or
contract of insurance shall be demmed to have authorized such agent or broker to receive on its behalf
payment of any premium which is due on such policy or contract of insurance at the time of its issuance
or delivery or which becomes due thereon.
It is also a well-known principle under the law of agency that: Payment to an agent having authority to
receive or collect payment is equivalent to payment to the principal himself; such payment is complete
when the money delivered is into the agent's hands and is a discharge of the indebtedness owing to the
principal.
Issue: WON the policy was subsisting at the time of the fire.
Held: Yes.
MICO's claims it cancelled the policy in question on October 15, 1981, for non-payment of premium. To
support this assertion, it presented one of its employees, who testified that "the original of the
endorsement and credit memo" presumably meaning the alleged cancellation "were sent the
assured by mail through our mailing section" However, there is no proof that the notice, assuming it
complied with the other requisites for cancellation, was actually mailed to and received by Pinca. All
MICO's offers to show that the cancellation was communicated to the insured is its employee's testimony
that the said cancellation was sent "by mail through our mailing section." without more.
It stands to reason that if Pinca had really received the said notice, she would not have made payment
on the original policy on December 24, 1981. Instead, she would have asked for a new insurance,
effective on that date and until one year later, and so taken advantage of the extended period. The
Court finds that if she did pay on that date, it was because she honestly believed that the policy issued
on June 7, 1981, was still in effect and she was willing to make her payment retroact to July 22, 1981,
its stipulated commencement date. After all, the premium invoice issued to Pinca at the time of the
delivery of the policy on June 7, 1981 was stamped "Payment Received" of the amount of P930.60 on
"12-24-81" by Domingo Adora. This is important because it suggests an understanding between MICO
and the insured that such payment could be made later, as agent Adora had assured Pinca.
UCPB General Insurance Co. vs. Masagana Telemart, Inc.
G.R. No. 137172 April 4, 2001
Respondent obtained from Petitioner 5 insurance policies on its properties in Pasay City, effective for one
year from May 22, 1991. On June 13, 1992, plaintiffs properties were razed by fire. A month later,
plaintiff tendered, and defendant accepted, 5 Equitable Bank MCs as renewal premium payments for
which an OR was issued by defendant. On July 14, 1992, Masagana made its formal demand for
indemnification for the burned insured properties. But defendant returned the 5 MCs stating that it was
rejecting Masagana's claim on the following grounds: (1) the policies expired on May 22, 1992 and were
not renewed for another term; and (2) the properties covered by the said policies were burned in a fire
that took place before tender of premium payment for the renewal of the policy.
Issue: Whether the fire insurance policies issued by petitioner to the respondent covering the period
from May 22, 1991 to May 22, 1992 had been extended or renewed by an implied credit arrangement
though actual payment of premium was tendered on a later date and after the occurrence of the (fire)
risk insured against.
Held: Yes.
Section 77 of the Insurance Code of 1978 provides: An insurer is entitled to payment of the premium as
soon as the thing insured is exposed to the peril insured against. Notwithstanding any agreement to the
contrary, no policy or contract of insurance issued by an insurance company is valid and binding unless
and until the premium thereof has been paid, except in the case of a life or an industrial life policy
whenever the grace period provision applies. It can be seen at once that Section 77 does not restate the
portion of Section 72 of the Insurance Act expressly permitting an agreement to extend the period to
pay the premium. Nonetheless, there are exceptions to the provision. The first exception is provided by

By: Elaine Marie G. Laceda

72

INSURANCE LAW

Section 77 itself, and that is, in case of a life or industrial life policy whenever the grace period provision
applies. The second is that covered by Section 78 of the Insurance Code, which provides: Any
acknowledgment in a policy or contract of insurance of the receipt of premium is conclusive evidence of
its payment, so far as to make the policy binding, notwithstanding any stipulation therein that it shall
not be binding until premium is actually paid. And the third exception was that laid down in Makati
Tuscany Condominium Corporation vs. Court of Appeals, wherein SC ruled that Section 77 may not
apply if the parties have agreed to the payment in installments of the premium and partial payment has
been made at the time of loss. But Tuscany has provided a fourth exception to Section 77, namely, that
the insurer may grant credit extension for the payment of the premium. This simply means that if the
insurer has granted the insured a credit term for the payment of the premium and loss occurs before the
expiration of the term, recovery on the policy should be allowed even though the premium is paid after
the loss but within the credit term.
Both the CA and the TC found that sufficient proof exists that Respondent, which had procured insurance
coverage from Petitioner for a number of years, had been granted a 60 to 90-day credit term for the
renewal of the policies.
b. Installments and partial payment
Philippine Phoenix Surety & Insurance Co. vs. Woodworks (1979), supra

Tibay vs. Court of Appeals


G.R. No. 119655 May 24, 1996
On 22 January 1987 private respondent Fortune Life and General Insurance Co., Inc. issued Fire
Insurance Policy No. 136171 in favor of Violeta R. Tibay and/or Nicolas Roraldo on their two-storey
residential building together with all their personal effects therein. Of the total premium of P2,983.50,
petitioner Violeta Tibay only paid P600.00 thus leaving a considerable balance unpaid. On 8 March 1987
the insured building was completely destroyed by fire. Two days later Violeta Tibay paid the balance of
the premium and filed a claim on the fire insurance policy. FORTUNE denied the claim. Hence, Violeta
and the other petitioners sued FORTUNE for damages in the amount of P600,000.00 representing the
total coverage of the fire insurance policy. TC ruled for petitioners and adjudged FORTUNE liable for the
total value of the insured building and personal properties. CA reversed the court a quo by declaring
FORTUNE not liable to plaintiff but ordering defendant to return to the former the premium paid.
Issue: May a fire insurance policy be valid, binding and enforceable upon mere partial payment of
premium?
Held: No.
Insurance is a contract whereby one undertakes for a consideration to indemnify another against loss,
damage or liability arising from an unknown or contingent event. The consideration is the premium,
which must be paid at the time and in the way and manner specified in the policy, and if not so paid, the
policy will lapse and be forfeited by its own terms.
The Policy provides for payment of premium in full. Accordingly, where the premium has only been
partially paid and the balance paid only after the peril insured against has occurred, the insurance
contract did not take effect and the insured cannot collect at all on the policy. This is fully supported by
Sec. 77 of the Insurance Code. It would have been altogether different were it not so stipulated. Ergo,
petitioners had absolute freedom of choice whether or not to be insured by FORTUNE under the terms of
its policy and they freely opted to adhere thereto.
it is elemental law that the payment of premium is requisite to keep the policy of insurance in force. If
the premium is not paid in the manner prescribed in the policy as intended by the parties the policy is
ineffective. Partial payment even when accepted as a partial payment will not keep the policy alive even
for such fractional part of the year as the part payment bears to the whole payment.

By: Elaine Marie G. Laceda

73

INSURANCE LAW

Makati Tuscany vs. Court of Appeals


G.R. No. 95546 November 6, 1992
Sometime in early 1982, private respondent American Home Assurance Co. (AHAC) issued in favor of
petitioner Makati Tuscany Condominium Corporation Insurance Policy No. AH-CPP-9210452 on the
latter's building and premises, for a period of 1yr beginning March 1, 1982. The premium was paid in 4
installments, all of which were accepted by private respondent. The policy was renewed for another year
under the same payment terms. For the 3rd year, petitioner made 2 installment payments, both
accepted by private respondent. But thereafter, petitioner refused to pay the balance. Consequently,
private respondent filed an action to recover the unpaid balance.
Petitioner admitted the issuance of Insurance Policy but explained that it discontinued the payment of
premiums because the policy did not contain a credit clause in its favor. Petitioner further claimed that
the policy was never binding and valid, and no risk attached to the policy. TC dismissed the complaint.
CA modified judgment by ordering payment of the balance of the premium.
Issue: Whether payment by installment of the premiums due on an insurance policy invalidated the
contract of insurance.
Held: No.
We hold that the subject policies are valid even if the premiums were paid on installments. The records
clearly show that petitioner and private respondent intended subject insurance policies to be binding and
effective notwithstanding the staggered payment of the premiums. The initial insurance contract entered
into in 1982 was renewed in 1983, then in 1984. In those 3 years, the insurer accepted all the
installment payments. Such acceptance of payments speaks loudly of the insurer's intention to honor
the policies it issued to petitioner. Certainly, basic principles of equity and fairness would not allow the
insurer to continue collecting and accepting the premiums, although paid on installments, and later deny
liability on the lame excuse that the premiums were not prepared in full.
While the import of Section 77 is that prepayment of premiums is strictly required as a condition to the
validity of the contract, We are not prepared to rule that the request to make installment payments duly
approved by the insurer, would prevent the entire contract of insurance from going into effect despite
payment and acceptance of the initial premium or first installment. Section 78 of the Insurance Code in
effect allows waiver by the insurer of the condition of prepayment by making an acknowledgment in the
insurance policy of receipt of premium as conclusive evidence of payment so far as to make the policy
binding despite the fact that premium is actually unpaid. Section 77 merely precludes the parties from
stipulating that the policy is valid even if premiums are not paid, but does not expressly prohibit an
agreement granting credit extension, and such an agreement is not contrary to morals, good customs,
public order or public policy
B.

RETURN OF PREMIUM

1. Grounds
a. No exposed to insured against Secs 79(a) and 80, I.C.
Sec. 79. A person insured is entitled to a return of premium, as follows:
(a) To the whole premium if no part of his interest in the thing insured be exposed to any of the
perils insured against;
Sec. 80. If a peril insured against has existed, and the insurer has been liable for any period, however
short, the insured is not entitled to return of premiums, so far as that particular risk is concerned.
Makati Tuscany vs. Court of Appeals (1992), supra
b. Time policy surrendered before expiration Sec 79(b), I.C.

By: Elaine Marie G. Laceda

74

INSURANCE LAW

Sec. 79. A person insured is entitled to a return of premium, as follows:


(b) Where the insurance is made for a definite period of time and the insured surrenders his
policy, to such portion of the premium as corresponds with the unexpired time, at a pro rata rate, unless
a short period rate has been agreed upon and appears on the face of the policy, after deducting from
the whole premium any claim for loss or damage under the policy which has previously accrued;
Provided, That no holder of a life insurance policy may avail himself of the privileges of this paragraph
without sufficient cause as otherwise provided by law.
c.

Voidable policy Sec 81, I.C.

Sec. 81. A person insured is entitled to return of the premium when the contract is voidable, on account
of fraud or misrepresentation of the insurer, or of his agent, or on account of facts, the existence of
which the insured was ignorant without his fault; or when by any default of the insured other than actual
fraud, the insurer never incurred any liability under the policy.

d.

When by default of insured (other than actual fraud), insurer never incurred
liability under the policy Sec 81, I.C., supra

Great Pacific Life Insurance Corp. vs. Court of Appeals


G.R. No. L-57308 April 23, 1990
Private respondent Teodoro Cortez, upon the solicitation of Margarita Siega an underwriter for the
petitioner Great Pacific Insurance Corporation, applied for a 20-year endowment policy for P30,000. His
application, with the requisite medical examination, was accepted and approved by the company. The
effective date indicated on the face of the policy in question was December 25, 1972. Mrs. Siega assured
private respondent that the first premium may be paid within the grace period of 30 days from date of
delivery of the policy. The first premium of P1,416.60 was paid by him in 3 installments, the last being
paid on February 21, 1973 or within the grace period. In a letter dated June 1, 1973, defendant advised
plaintiff that the Policy was not in force. To make it enforceable and operative, plaintiff was asked to
remit the balance of P1,015.60 to complete his initial annual premium due December 15, 1972. Cortez'
reaction to the company's act was to immediately inform it that he was cancelling the policy and he
demanded the return of his premium plus damages.
A complaint for damages was filed with the CFI praying for the refund of the insurance premium. TC
ruled in favor of the Cortez and ordered refund of the premium paid. CA affirmed with modification with
respect to the damages.
Issue: WON Cortez is entitled to a refund of his premium.
Held: Yes.
When the petitioner advised private respondent on June 1, 1973, four months after he had paid the first
premium, that his policy had never been in force, and that he must pay another premium and undergo
another medical examination to make the policy effective, the petitioner committed a serious breach of
the contract of insurance. Petitioner should have informed Cortez of the deadline for paying the first
premium before or at least upon delivery of the policy to him, so he could have complied with what was
needful and would not have been misled into believing that his life and his family were protected by the
policy, when actually they were not. And, if the premium paid by Cortez was unacceptable for being late,
it was the company's duty to return it. By accepting his premiums without giving him the corresponding
protection, the company acted in bad faith.
Sections 79, 81 and 82 of P.D. 612 of the Insurance Code of 1978 provide when the insured is entitled
to the return of premium paid.
SECTION 79. A person insured is entitled to a return of premium, as follows:
(a) To the whole premium, if no part of his interest in the thing insured be exposed to any
of the perils insured against.

By: Elaine Marie G. Laceda

75

INSURANCE LAW

(b)

Where the insure is made for a definite period of time and the insured surrenders his
policy, to such portion of the premium as corresponds with the unexpired time, at a pro
rata rate, unless a short period rate has been agreed upon and appears on the face of
the policy, after deducting from the whole premium any claim for loss or damage under
the policy which has previously accrued: Provided, That no holder of a life insurance
policy may avail himself of the privileges of this paragraph without sufficient causes as
otherwise provided by law.

SECTION 81. A person insured is entitled to a return of the premium when the contract is
voidable on account of the fraud or misrepresentation of the insurer or of his agent or on
account of facts the existence of which the insured was ignorant without his fault; or when, by
any default of the insured other than actual fraud, the insurer never incurred any liability under
the policy.
SECTION 82. In case of an over-insurance by several insurers, the insured is entitled to a
ratable return of the premium, proportioned to the amount by which the aggregate sum insured
in all the policies exceeds the insurable value of the thing at risk.
Since his policy was in fact inoperative or ineffectual from the beginning, the company was never at risk,
hence, it is not entitled to keep the premium.
e. Over-insurance by several insurers Sec 82, I.C.
Sec. 82. In case of an over-insurance by several insurers, the insured is entitled to a ratable return of
the premium, proportioned to the amount by which the aggregate sum insured in all the policies
exceeds the insurable value of the thing at risk.
2. Rebate of Premium - Sec 361, I.C.
Sec. 361. No insurance company doing business in the Philippines or any agent thereof, no insurance
broker, and no employee or other representative of any such insurance company, agent, or broker, shall
make, procure or negotiate any contract of insurance or agreement as to policy contract, other than is
plainly expressed in the policy or other written contract issued or to be issued as evidence thereof, or
shall directly or indirectly, by giving or sharing a commission or in any manner whatsoever, pay or allow
or offer to pay or allow to the insured or to any employee of such insured, either as an inducement to
the making of such insurance or after such insurance has been effected, any rebate from the premium
which is specified in the policy, or any special favor or advantage in the dividends or other benefits to
accrue thereon, or shall give or offer to give any valuable consideration or inducement of any kind,
directly or indirectly, which is not specified in such policy or contract of insurance; nor shall any such
company, or any agent thereof, as to any policy or contract of insurance issued, make any
discrimination against any Filipino in the sense that he is given less advantageous rates, dividends or
other policy conditions or privileges than are accorded to other nationals because of his race.
V.

THE POLICY
A.

POLICY OF INSURANCE

1. Definition; Contract of Insurance Sec 49, I.C.


Sec. 49. The written instrument in which a contract of insurance is set forth, is called a policy of
insurance.
Enriquez vs. Sun Life Assurance Company of Canada
G.R. No. L-15895 November 29, 1920
On September 24, 1917, Joaquin Herrer applied with the Sun Life Assurance Company of Canada for a
life annuity and paid the sum of P6,000.

By: Elaine Marie G. Laceda

76

INSURANCE LAW

The application was immediately forwarded to the head office of the company at Montreal, Canada. On
November 26, 1917, the head office gave notice of acceptance by cable to Manila. On December 4,
1917, the policy was issued at Montreal.
On December 18, 1917, Atty. Aurelio A. Torres wrote the Manila office stating that Herrer desired to
withdraw his application. The following day the local office replied stating that the policy had been
issued, and called attention to the notification of November 26, 1917. This letter was received by Mr.
Torres on the morning of December 21, 1917. Mr. Herrer died on December 20, 1917.
Issue: Whether the contract for a life annuity was perfected.
Held: No.
According to the provisional receipt, three things had to be accomplished by the insurance company
before there was a contract: (1) There had to be a medical examination of the applicant; (2) there had
to be approval of the application by the head office of the company; and (3) this approval had in some
way to be communicated by the company to the applicant.
Likewise, the second paragraph of Article 1262 of the Civil Code, the law applicable to the case, provides
that an acceptance made by letter shall not bind the person making the offer except from the time it
came to his knowledge.
The contract for a life annuity in the case at bar was not perfected because it has not been proved
satisfactorily that the acceptance of the application ever came to the knowledge of the applicant.
A letter will not be presumed to have been received by the addressee unless it is shown that it was
deposited in the post-office, properly addressed and stamped.
Perez vs. Court of Appeals
G.R. No. 112329 January 28, 2000
Primitivo B. Perez had been insured with the BF Lifeman Insurance Corporation since 1980. Sometime in
October 1987, an agent of the insurance corporation, Rodolfo Lalog, visited Perez and convinced him to
apply for additional insurance coverage of P50,000 to avail of the ongoing promotional discount of
P400.00 if the premium were paid annually. Perez accomplished an application form for the additional
insurance coverage of P50,000.00. On the same day, petitioner Virginia A. Perez, Primitivo's wife, paid
P2,075.00 to Lalog. The receipt issued by Lalog indicated the amount received was a "deposit." Lalog
lost the application form accomplished by Perez and so on October 28, 1987, he asked the latter to fill
up another application form. Lalog then forwarded the application to the office of BF Lifeman Insurance
Corporation at Gumaca, Quezon which office was supposed to forward the papers to the Manila office.
On November 25, 1987, Perez died in an accident. At the time of his death, his application papers for
the additional insurance of P50,000.00 were still with the Gumaca office. It was only on November 27,
1987 that said papers were received in Manila. Without knowing that Perez died on November 25, 1987,
BF Lifeman Insurance Corporation approved the application and issued the corresponding policy for the
P50,000.00 on December 2, 1987.
Petitioner Virginia Perez went to Manila to claim the benefits under the insurance policies of the
deceased. She was paid P40,000 under the first insurance policy for P20,000 but the insurance company
refused to pay the claim under the additional policy coverage of P50,000. The insurance company
maintained that the insurance for P50,000.00 had not been perfected at the time of the death of
Primitivo Perez. Consequently, the insurance company refunded the amount of P2,075.00 which Virginia
Perez had paid.
TC ruled in favor of Perez ordering the company to pay the insurance proceeds. CA reversed holding that
at the time the policy was issued, Primitivo was already dead.

By: Elaine Marie G. Laceda

77

INSURANCE LAW

Issue: WON the insurance contract had been perfected.


Held: No.
A contract of insurance, like all other contracts, must be assented to by both parties, either in person or
through their agents and so long as an application for insurance has not been either accepted or
rejected, it is merely a proposal or an offer to make a contract.
When Primitivo filed an application for insurance, paid P2,075.00 and submitted the results of his
medical examination, his application was subject to the acceptance of private respondent BF Lifeman
Insurance Corporation. The perfection of the contract of insurance between the deceased and
respondent corporation was further conditioned upon compliance with the following requisites stated in
the application form: there shall be no contract of insurance unless and until a policy is issued on this
application and that the said policy shall not take effect until the premium has been paid and the policy
delivered to and accepted in good health.
The condition imposed by the corporation that the policy must have been delivered to and accepted by
the applicant while he is in good health can hardly be considered as a potestative or facultative
condition. On the contrary, the health of the applicant at the time of the delivery of the policy is beyond
the control or will of the insurance company. Rather, the condition is a suspensive one whereby the
acquisition of rights depends upon the happening of an event which constitutes the condition. In this
case, the suspensive condition was the policy must have been delivered and accepted by the applicant
while he is in good health. There was non-fulfillment of the condition, however, inasmuch as the
applicant was already dead at the time the policy was issued. Hence, the non-fulfillment of the condition
resulted in the non-perfection of the contract.
An application is a mere offer which requires the overt act of the insurer for it to ripen into a contract.
Delay in acting on the application does not constitute acceptance even though the insured has forwarded
his first premium with his application.
Lucero vda. de Singayen vs. Insular Life Assurance Co.
G.R. No. 41702 September 4, 1935
Arturo Sindayen, employed as a linotype operator in the Bureau of Printing in Manila, made a written
application on December 26, 1932, to the defendant Insular Life Assurance Co., Ltd., through its agent,
Cristobal Mendoza, for a policy of insurance on his life in the sum of P1,000 for which he paid P15 cash
as part of the first premium. It was agreed with the agent that the policy, when and if issued, should be
delivered to his aunt, Felicidad Estrada in Camiling, Tarlac, with whom Sindayen left the sum of P25.06
to complete the payment of the first annual premium of P40.06. On January 11, 1933, the company
accepted the risk and issued the policy dated back to December 1, 1932, and mailed the same to its
agent, Cristobal Mendoza, in Camiling, Tarlac, for delivery to the insured. On January 15, Sindayens
physician found that he was suffering from acute nephritis and uremia, and on January 19, 1939 he
died.
The policy which the company issued and mailed in Manila on January 11, 1933, was received by its
agent in Camiling, Tarlac, on January 16, 1933. On January 18, 1933, the agent, in accordance with his
agreement with the insured, delivered the policy to Felicidad Estrada upon her payment of the balance
of the first year's annual premium. The agent asked Felicidad Estrada if her nephew was in good health
and she replied that she believed so because she had no information that he was sick and he thereupon
delivered to her the policy.
On January 20, 1933, the agent learned of the death of Arturo Sindayen and called on Felicidad Estrada
and asked her to return the policy. On February 4, 1933, the company obtained from the beneficiary,
the widow of Arturo Sindayen, her signature to a legal document entitled "ACCORD, SATISFACTION AND
RELEASE" in consideration of the sum of P40.06 paid to her by a check of the company. The said check
for P40.06 was never cashed but returned to the company. Thereupon an action was brought to enforce
payment of the policy.

By: Elaine Marie G. Laceda

78

INSURANCE LAW

Issue: WON the policy was perfected.


Held: Yes.
Mendoza was duly licensed by the Insurance Commissioner to act as the agent of the defendant
insurance company. Mendoza was not regarded by the company as a mere conduit or automaton for the
performance of the physical act of placing the policy in the hands of the insured. If Mendoza were only
an automaton then the legally effective delivery of the policy and the consummation of the contract
occurred when the company expressed its will to release the policy by mailing it to its agent, namely, on
January 11, 1933. The legal result would be the same as if the company had mailed the policy on
January 11, 1933, to the insured directly using the post-office as its conduit for delivery at which time
the insured was in good health performing his regular duties in the Bureau of Printing.
However, the record shows that Mendoza had the authority, given him by the company, to withhold the
delivery of the policy to the insured "until the first premium has been paid and the policy has been
delivered to and accepted by the insured while I am in good health". Mendoza's decision that the
condition had been met by the insured and that it was proper to make delivery of the policy to him is
just as binding on the company as if the decision had been made by its board of directors.
Mendoza neither waived nor pretended to waive any right or requirement of the company. In fact, his
inquiry as to the state of health of the insured discloses that he was endeavoring to assure himself that
this requirement of the company had been satisfied. In doing so, he acted within the authority conferred
on him by his agency and his acts within that authority bind the company. The company, therefore,
having decided that all the conditions precedent to the taking effect of the policy had been complied with
and having accepted the premium and delivered the policy thereafter to the insured, the company is
now estopped to assert that it never intended that the policy should take effect.
Tang vs. Court of Appeals
G.R. No. L-48563 May 25, 1979
On September 25, 1965, Lee See Guat, a widow, 61 years old, and an illiterate who spoke only Chinese,
applied for an insurance on her life for P60,000 with the respondent Company. The application consisted
of two parts, both in the English language. The second part of her application dealt with her state of
health and because her answers indicated that she was healthy, the Company issued her Policy No.
0690397, effective October 23, 1965, with her nephew Vicente E. Tang, herein Petitioner, as her
beneficiary. On November 15, 1965, Lee See Guat again applied for an additional insurance on her life
for P40,000. Considering that her first application had just been approved, no further medical
examination was made.
On April 20, 1966, Lee See Guat died of lung cancer. Thereafter, the beneficiary of the two policies,
Vicente E. Tang claimed for their face value in the amount of P100,000 which the insurance company
refused to pay on the ground that the insured was guilty of concealment and misrepresentation at the
time she applied for the two policies. CFI dismissed complaint on the ground of concealment. CA
affirmed.
Issue: WON there was concealment and misrepresentation which avoids the policy.
Held: Yes.
Art. 1332 of the Civil Code provides: When one of the parties is unable to read, or if the contract is in a
language not understood by him, and mistake or fraud is alleged, the person enforcing the contract
must show that the terms thereof have been fully explained to the former. Under this article, the
obligation to show that the terms of the contract had been fully explained to the party who is unable to
read or understand the language of the contract, when fraud or mistake is alleged, devolves on the
party seeking to enforce it. Here the insurance company is not seeking to enforce the contracts; on the
contrary, it is seeking to avoid their performance. Accordingly, respondent company was under no
obligation to prove that the terms of the insurance contracts were fully explained to the other party.

By: Elaine Marie G. Laceda

79

INSURANCE LAW

2. Form Sec 50, I.C.


Sec. 50. The policy shall be in printed form which may contain blank spaces; and any word, phrase,
clause, mark, sign, symbol, signature, number, or word necessary to complete the contract of insurance
shall be written on the blank spaces provided therein.
Any rider, clause, warranty or endorsement purporting to be part of the contract of insurance
and which is pasted or attached to said policy is not binding on the insured, unless the descriptive title
or name of the rider, clause, warranty or endorsement is also mentioned and written on the blank
spaces provided in the policy.
Unless applied for by the insured or owner, any rider, clause, warranty or endorsement issued
after the original policy shall be countersigned by the insured or owner, which countersignature shall be
taken as his agreement to the contents of such rider, clause, warranty or endorsement.
Group insurance and group annuity policies, however, may be typewritten and need not be in
printed form.
3. Necessity of Approval of Form Sec 226, I.C.
Sec. 226. No policy, certificate or contract of insurance shall be issued or delivered within the
Philippines unless in the form previously approved by the Commissioner, and no application form shall
be used with, and no rider, clause, warranty or endorsement shall be attached to, printed or stamped
upon such policy, certificate or contract unless the form of such application, rider, clause, warranty or
endorsement has been approved by the Commissioner.
4. Basic Provisions Sec 51, I.C.
Sec. 51. A
(a)
(b)
(c)
(d)
(e)
(f)
(g)

policy of insurance must specify:


The parties between whom the contract is made;
The amount to be insured except in the cases of open or running policies;
The premium, or if the insurance is of a character where the exact premium is only
determinable upon the termination of the contract, a statement of the basis and rates upon
which the final premium is to be determined;
The property or life insured;
The interest of the insured in property insured, if he is not the absolute owner thereof;
The risks insured against; and
The period during which the insurance is to continue.
a.
b.
c.
d.
e.
f.
g.

Parties to the contract;


Amount of insurance (except in open or running policies);
Premium
Property or life insured;
Interest of insured in property insured if he is not the owner;
Risk insured against; and
Period of insurance

5. Additional Matters Secs 227, 228, 230, I.C.


Sec. 227. In the case of individual life or endowment insurance, the policy shall contain in substance
the following conditions:
(a) A provision that the policyholder is entitled to a grace period either of thirty days or of one
month within which the payment of any premium after the first may be made, subject at the option of
the insurer to an interest charge not in excess of six per centum per annum for the number of days of
grace elapsing before the payment of the premium, during which period of grace the policy shall
continue in full force, but in case the policy becomes a claim during the said period of grace before the
overdue premium is paid, the amount of such premium with interest may de deducted from the amount
payable under the policy in settlement;
(b) A provision that the policy shall be incontestable after it shall have been in force during the
lifetime of the insured for a period of two years from its date of issue as shown in the policy, or date of
approval of last reinstatement, except for non-payment of premium and except for violation of the
conditions of the policy relating to military or naval service in time of war;

By: Elaine Marie G. Laceda

80

INSURANCE LAW

(c) A provision that the policy shall constitute the entire contract between the parties, but if the
company desires to make the application a part of the contract it may do so provided a copy of such
application shall be indorsed upon or attached to the policy when issued, and in such case the policy
shall contain a provision that the policy and the application therefor shall constitute the entire contract
between the parties;
(d) A provision that if the age of the insured is considered in determining the premium and the
benefits accruing under the policy, and the age of the insured has been misstated, the amount payable
under the policy shall be such as the premium would have purchased at the correct age;
(e) If the policy is participating, a provision that the company shall periodically ascertain and
apportion any divisible surplus accruing on the policy under conditions specified therein;
(f) A provision specifying the options to which the policyholder is entitled to in the event of
default in a premium payment after three full annual premiums shall have been paid. Such option shall
consist of:
(1) A cash surrender value payable upon surrender of the policy which shall not be less
than the reserve on the policy, the basis of which shall be indicated, for the then current policy
year and any dividend additions thereto, reduced by a surrender charge which shall not be more
than one-fifth of the entire reserve or two and one-half per centum of the amount insured and
any dividend additions thereto;
(2) One or more paid-up benefits on a plan or plans specified in the policy of such value
as may be purchased by the cash surrender value;
(g) A provision that at anytime after a cash surrender value is available under the policy and
while the policy is in force, the company will advance, on proper assignment or pledge of the policy and
on sole security thereof, a sum equal to, or at the option of the owner of the policy, less than the cash
surrender value on the policy, at a specified rate of interest, not more than the maximum allowed by
law, to be determined by the company from time to time, but not more often than once a year, subject
to the approval of the Commissioner; and that the company will deduct from such loan value any
existing indebtedness on the policy and any unpaid balance of the premium for the current policy year,
and may collect interest in advance on the loan to the end of the current policy year, which provision
may further provide that such loan may be deferred for not exceeding six months after the application
therefor is made;
(h) A table showing in figures cash surrender values and paid-up options available under the
policy each year upon default in premium payments, during at least twenty years of the policy beginning
with the year in which the values and options first become available, together with a provision that in
the event of the failure of the policyholder to elect one of the said options within the time specified in
the policy, one of said options shall automatically take effect and no policyholder shall ever forfeit his
right to same by reason of his failure to so elect;
(i) In case the proceeds of a policy are payable in installments or as an annuity, a table showing
the minimum amounts of the installments or annuity payments;
(j) A provision that the policyholder shall be entitled to have the policy reinstated at any time
within three years from the date of default of premium payment unless the cash surrender value has
been duly paid, or the extension period has expired, upon production of evidence of insurability
satisfactory to the company and upon payment of all overdue premiums and any indebtedness to the
company upon said policy, with interest rate not exceeding that which would have been applicable to
said premiums and indebtedness in the policy years prior to reinstatement.
Any of the foregoing provisions or portions thereof not applicable to single premium or term
policies shall to that extent not be incorporated therein; and any such policy may be issued and
delivered in the Philippines which in the opinion of the Commissioner contains provisions on any one or
more of the foregoing requirements more favorable to the policyholder than hereinbefore required.
This section shall not apply to policies of group life or industrial life insurance.
Sec. 228. No policy of group life insurance shall be issued and delivered in the Philippines unless it
contains in substance the following provisions, or provisions which in the opinion of the Commissioner
are more favorable to the persons insured, or at least as favorable to the persons insured and more
favorable to the policy-holders:
(a) A provision that the policyholder is entitled to a grace period of either thirty days or of one
month for the payment of any premium due after the first, during which grace period the death benefit
coverage shall continue in force, unless the policyholder shall have given the insurer written notice of

By: Elaine Marie G. Laceda

81

INSURANCE LAW

discontinuance in advance of the date of discontinuance and in accordance with the terms of the policy.
The policy may provide that the policyholder shall be liable for the payment of a pro rata premium for
the time the policy is in force during such grace period;
(b) A provision that the validity of the policy shall not be contested, except for non-payment of
premiums after it has been in force for two years from its date of issue; and that no statement made by
any insured under the policy relating to his insurability shall be used in contesting the validity of the
insurance with respect to which such statement was made after such insurance has been in force prior
to the contest for a period of two years during such person's lifetime nor unless contained in written
instrument signed by him;
(c) A provision that a copy of the application, if any, of the policyholder shall be attached to the
policy when issued, that all statements made by the policyholder or by persons insured shall be deemed
representations and not warranties, and that no statement made by any insured shall be used in any
contest unless a copy of the instrument containing the statement is or has been furnished to such
person or to his beneficiary;
(d) A provision setting forth the conditions, if any, under which the insurer reserves the right to
require a person eligible for insurance to furnish evidence of individual insurability satisfactory to the
insurer as a condition to part or all of his coverage;
(e) A provision specifying an equitable adjustment of premiums or of benefits or of both to be
made in the event that the age of a person insured has been misstated, such provision to contain a clear
statement of the method of adjustment to be used;
(f) A provision that any sum becoming due by reason of death of the person insured shall be
payable to the beneficiary designated by the insured, subject to the provisions of the policy in the event
that there is no designated beneficiary, as to all or any part of such sum, living at the death of the
insured, and subject to any right reserved by the insurer in the policy and set forth in the certificate to
pay at its option a part of such sum not exceeding five hundred pesos to any person appearing to the
insurer to be equitably entitled thereto by reason of having incurred funeral or other expenses incident
to the last illness or death of the person insured;
(g) A provision that the insurer will issue to the policyholder for delivery to each person insured
an individual certificate setting forth a statement as to the insurance protection to which he is entitled,
to whom the insurance benefits are payable, and the rights set forth in paragraphs (h), (i) and (j)
following;
(h) A provision that if the insurance, or any portion of it, on a person covered under the policy
ceases because of termination of employment or of membership in the class or classes eligible for
coverage under the policy, such person shall be entitled to have issued to him by the insurer, without
evidence of insurability, an individual policy of life insurance without disability or other supplementary
benefits, provided application for the individual policy and payment of the first premium to the insurer
shall be made within thirty days after such termination and provided further that:
(1) the individual policy shall be on any one of the forms, except term insurance, then
customarily issued by the insurer at the age and for an amount not in excess of the coverage
under the group policy; and
(2) the premium on the individual policy shall be at the insurer's then customary rate
applicable to the form and amount of the individual policy, to the class of risk to which such
person then belongs, and to his age attained on the effective date of the individual policy.
(i) A provision that if the group policy terminates or is amended so as to terminate the insurance
of any class of insured persons, every person insured thereunder at the date of such termination whose
insurance terminates and who has been so insured for five years prior to such termination date shall be
entitled to have issued to him by the insurer an individual policy of life insurance subject to the same
limitations as set forth in paragraph (h), except that the group policy may provide that the amount of
such individual policy shall not exceed the smaller of (a) the amount of the person's life insurance
protection ceasing less the amount of any life insurance for what he is or becomes eligible under any
group policy issued or reinstated by the same or another reinsurer within thirty days after such
termination, and (b) two thousand pesos;
(j) A provision that if a person insured under the group policy dies during the thirty-day period
within which he would have been entitled to an individual policy issued to him in accordance with (h)
and (i) above and before such individual policy shall have become effective, the amount of life insurance
which he would have been entitled to have issued to him as an individual policy shall be payable as a
claim under the group policy whether or not application for the individual policy or the payment of the
first premium has been made;

By: Elaine Marie G. Laceda

82

INSURANCE LAW

(k) In the case of a policy issued to a creditor to insure debtors of such creditor, a provision that
the insurer will furnish to the policyholder for delivery to each debtor insured under the policy a form
which will contain a statement that the life of the debtor is insured under the policy and that any death
benefit paid thereunder by reason of his death shall be applied to reduce or extinguish indebtedness.
The provisions of paragraphs (f) to (j) shall not apply to policies issued to a creditor to insure his
debtors. If a group life policy is on a plan of insurance other than term, it shall contain a non-forfeiture
provision or provisions which in the opinion of the Commissioner is or are equitable to the insured or the
policyholder: Provided, That nothing herein contained shall be so construed as to require group life
policies to contain the same non-forfeiture provisions as are required of individual life policies.
Sec. 230. In the case of industrial life insurance, the policy shall contain in substance the following
provisions:
(a) A provision that the insured is entitled to a grace period of four weeks within which the
payment of any premium after the first may be made, except that where premiums are payable
monthly, the period of grace shall be either one month or thirty days; and that during the period of
grace, the policy shall continue in full force, but if during such grace period the policy becomes a claim,
then any overdue and unpaid premiums may be deducted from any amount payable under the policy in
settlement;
(b) A provision that the policy shall be incontestable after it has been in force during the lifetime
of the insured for a specified period, not more than two years from its date of issue, except for nonpayment of premiums and except for violation of the conditions of the policy relating to naval or military
service, or services auxiliary thereto, and except as to provisions relating to benefits in the event of
disability as defined in the policy, and those granting additional insurance specifically against death by
accident or by accidental means, or to additional insurance against loss of, or loss of use of, specific
members of the body;
(c) A provision that the policy shall constitute the entire contract between the parties, or if a
copy of the application is endorsed upon and attached to the policy when issued, a provision that the
policy and the application therefor shall constitute the entire contract between the parties, and in the
latter case, a provision that all statements made by the insured shall, in the absence of fraud, be
deemed representations and not warranties;
(d) A provision that if the age of the person insured, or the age of any person, considered in
determining the premium, or the benefits accruing under the policy, has been misstated, any amount
payable or benefit accruing under the policy shall be such as the premium paid would have purchased at
the correct age;
(e) A provision that if the policy is a participating policy, the company shall periodically ascertain
and apportion any divisible surplus accruing on the policy under the conditions specified therein;
(f) A provision that in the event of default in premium payments after three full years' premiums
have been paid, the policy shall be converted into a stipulated form of insurance, and that in the event
of default in premium payments after five full years' premiums have been paid, a specified cash
surrender value shall be available, in lieu of the stipulated form of insurance, at the option of the
policyholder. The net value of such stipulated form of insurance and the amount of such cash value shall
not be less than the reserve on the policy and dividend additions thereto, if any, at the end of the last
completed policy year for which premiums shall have been paid (the policy to specify the mortality table,
rate of interest and method of valuation adopted to compute such reserve), exclusive of any reserve on
disability benefits and accidental death benefits, less an amount not to exceed two and one-half per
centum of the maximum amount insured by the policy and dividend additions thereto, if any, at the end
of the last completed policy year for which premiums shall have been paid (the policy to specify the
mortality table, rate of interest and method of valuation adopted to compute such reserve), exclusive of
any reserve on disability benefits and accidental death benefits, less an amount not to exceed two and
one-half per centum of the maximum amount insured by the policy and dividend additions thereto, if
any, when the issue age is under ten years, and less an amount not to exceed two and one-half per
centum of the current amount insured by the policy and dividend additions thereto, if any, if the issue
age is ten years or older, and less any existing indebtedness to the company on or secured by the
policy;
(g) A provision that the policy may be surrendered to the company at its home office within a
period of not less than sixty days after the due date of a premium in default for the specified cash value,
provided that the insurer may defer payment for not more than six months after the application therefor
is made;

By: Elaine Marie G. Laceda

83

INSURANCE LAW

(h) A table that shows in figures the non-forfeiture benefits available under the policy every year
upon default in payment of premiums during at least the first twenty years of the policy, such table to
begin with the year in which such values become available, and a provision that the company will furnish
upon request an extension of such table beyond the year shown in the policy;
(i) A provision that specifies which one of the stipulated forms of insurance provided for under
the provision of paragraph (f) of this section shall take effect in the event of the insured's failure, within
sixty days from the due date of the premium in default, to notify the insurer in writing as to which one
of such forms he has selected;
(j) A provision that the policy may be reinstated at any time within two years from the due date
of the premium in default unless the cash surrender value has been paid or the period of extended term
insurance expired, upon production of evidence of insurability satisfactory to the company and payment
of arrears of premiums with interest at a rate not exceeding six per centum per annum payable
annually;
(k) A provision that when a policy shall become a claim by death of the insured, settlement shall
be made upon receipt of due proof of death, or not later than two months after receipt of such proof;
(l) A title on the face and on the back of the policy correctly describing its form;
(m) A space on the front or the back of the policy for the name of the beneficiary designated by
the insured with a reservation of the insured's right to designate or change the beneficiary after the
issuance of the policy. The policy may also provide that no designation or change of beneficiary shall be
binding on the insurer until endorsed on the policy by the insurer, and that the insurer may refuse to
endorse the name of any proposed beneficiary who does not appear to the insurer to have an insurable
interest in the life of the insured. Such policy may also contain a provision that if the beneficiary
designated in the policy does not surrender the policy with due proof of death within the period stated in
the policy, which shall not be less than thirty days after the death of the insured, or if the beneficiary is
the estate of the insured, or is a minor, or dies before the insured, or is not legally competent to give
valid release, then the insurer may make any payment thereunder to the executor or administrator of
the insured, or to any of the insured's relatives by blood or legal adoption or connections by marriage or
to any person appearing to the insurer to be equitably entitled thereto by reason of having incurred
expense for the maintenance, medical attention or burial of the insured; and
(n) A provision that when an industrial life insurance policy is issued providing for accidental or
health benefits, or both, in addition to life insurance, the foregoing provisions shall apply only to the life
insurance portion of the policy.
Any of the foregoing provisions or portions thereof not applicable to non-participating or term
policies shall to that extent not be incorporated therein. The foregoing provisions shall not apply to
policies issued or granted pursuant to the non-forfeiture provisions prescribed in provisions of
paragraphs (f) and (i) of this section, nor shall provisions of paragraphs (f), (g), (h), and (i) hereof be
required in term insurance of twenty years or less but such term policies shall specify the mortality
table, rate of interest, and method of computing reserves.
6. Designation of Beneficiaries Secs 54-57, I.C.
Sec. 54. When an insurance contract is executed with an agent or trustee as the insured, the fact that
his principal or beneficiary is the real party in interest may be indicated by describing the insured as
agent or trustee, or by other general words in the policy.
Sec. 55. To render an insurance effected by one partner or part-owner, applicable to the interest of his
co-partners or other part-owners, it is necessary that the terms of the policy should be such as are
applicable to the joint or common interest.
Sec. 56. When the description of the insured in a policy is so general that it may comprehend any
person or any class of persons, only he who can show that it was intended to include him can claim the
benefit of the policy.
Sec. 57. A policy may be so framed that it will inure to the benefit of whomsoever, during the
continuance of the risk, may become the owner of the interest insured.
7. Rider, Clause, Warranty or Endorsement Sec 50, I.C.

By: Elaine Marie G. Laceda

84

INSURANCE LAW

Sec. 50. The policy shall be in printed form which may contain blank spaces; and any word, phrase,
clause, mark, sign, symbol, signature, number, or word necessary to complete the contract of insurance
shall be written on the blank spaces provided therein.
Any rider, clause, warranty or endorsement purporting to be part of the contract of insurance
and which is pasted or attached to said policy is not binding on the insured, unless the descriptive title
or name of the rider, clause, warranty or endorsement is also mentioned and written on the blank
spaces provided in the policy.
Unless applied for by the insured or owner, any rider, clause, warranty or endorsement issued
after the original policy shall be countersigned by the insured or owner, which countersignature shall be
taken as his agreement to the contents of such rider, clause, warranty or endorsement.
Group insurance and group annuity policies, however, may be typewritten and need not be in
printed form.
Commissioner of Internal Revenue vs. Lincoln Philippine Life Insurance Co.
G.R. No. 119176 March 19, 2002
In the years prior to 1984, private respondent Lincoln Philippine Life Insurance Co., Inc. issued a special
kind of life insurance policy known as the "Junior Estate Builder Policy," the distinguishing feature of
which is a clause providing for an automatic increase in the amount of life insurance coverage upon
attainment of a certain age by the insured without the need of issuing a new policy. The clause was to
take effect in the year 1984. DST due on the policy were paid by petitioner only on the initial sum
assured. Thus, petitioner issued deficiency DST assessment for the year 1984 in the amount of
P464,898.75, corresponding to the amount of automatic increase of the sum assured on the policy
issued by respondent. CTA found no valid basis for the deficiency tax assessment. CA affirmed.
Issue: WON the "automatic increase clause" is separate and distinct from the main agreement and
involves another transaction.
Held: No.
The subject insurance policy at the time it was issued contained an "automatic increase clause."
Although the clause was to take effect only in 1984, it was written into the policy at the time of its
issuance. The distinctive feature of the "junior estate builder policy" called the "automatic increase
clause" already formed part and parcel of the insurance contract, hence, there was no need for an
execution of a separate agreement for the increase in the coverage that took effect in 1984 when the
assured reached a certain age.
Here, although the automatic increase in the amount of life insurance coverage was to take effect later
on, the date of its effectivity, as well as the amount of the increase, was already definite at the time of
the issuance of the policy. Thus, the amount insured by the policy at the time of its issuance necessarily
included the additional sum covered by the automatic increase clause because it was already
determinable at the time the transaction was entered into and formed part of the policy.
The "automatic increase clause" in the policy is in the nature of a conditional obligation under Article
1181, by which the increase of the insurance coverage shall depend upon the happening of the event
which constitutes the obligation. In the instant case, the additional insurance that took effect in 1984
was an obligation subject to a suspensive obligation, but still a part of the insurance sold to which
private respondent was liable for the payment of the documentary stamp tax.
8. Interpretation and proof Art 1377, Civil Code
B.

COVER NOTES

1. Binding Effect Sec 52, I.C.

By: Elaine Marie G. Laceda

85

INSURANCE LAW

Sec. 52. Cover notes may be issued to bind insurance temporarily pending the issuance of the policy.
Within sixty days after the issue of the cover note, a policy shall be issued in lieu thereof, including
within its terms the identical insurance bound under the cover note and the premium therefor.
Cover notes may be extended or renewed beyond such sixty days with the written approval of
the Commissioner if he determines that such extension is not contrary to and is not for the purpose of
violating any provisions of this Code. The Commissioner may promulgate rules and regulations
governing such extensions for the purpose of preventing such violations and may by such rules and
regulations dispense with the requirement of written approval by him in the case of extension in
compliance with such rules and regulations.
De Lim vs. Sun Life Assurance Co. of Canada
G.R. No. L-15774 November 29, 1920
On July 6, 1917, Luis Lim made application to the Sun Life Assurance Company of Canada for a policy of
insurance on his life in the sum of P5,000. In his application Lim designated his wife, Pilar C. de Lim, the
plaintiff herein, as the beneficiary. The first premium of P433 was paid by Lim, and upon such payment
the company issued what was called a "provisional policy" (for four months only from the date of
application). Luis Lim died on August 23, 1917, after the issuance of the provisional policy but before
approval of the application by the home office of the insurance company. The instant action is brought
by the beneficiary, Pilar C. de Lim, to recover from the Sun Life Assurance Company of Canada the sum
of P5,000, the amount named in the provisional policy.
Issue: WON the contract was consummated.
Held: No.
The policy for four months is expressly made subjected to the affirmative condition that "the company
shall confirm this agreement by issuing a policy on said application when the same shall be submitted to
the head office in Montreal." Should the company not issue such a policy, then this agreement shall be
null and void ab initio, and the company shall be held not to have been on the risk.
It is a primary rule that a contract of insurance, like other contracts, must be assented to by both
parties either in person or by their agents. So long as an application for insurance has not been either
accepted or rejected, it is merely an offer or proposal to make a contract. The contract, to be binding
from the date of the application, must have been a completed contract, one that leaves nothing to be
done, nothing to be completed, nothing to be passed upon, or determined, before it shall take effect.
There can be no contract of insurance unless the minds of the parties have met in agreement. Our view
is, that a contract of insurance was not here consummated by the parties.
Where an agreement is made between the applicant and the agent whether by signing an application
containing such condition, or otherwise, that no liability shall attach until the principal approves the risk
and a receipt is given by the agent, such acceptance is merely conditional, and it subordinated to the act
of the company in approving or rejecting; so in life insurance a "binding slip" or "binding receipt" does
not insure of itself.
Great Pacific Life Insurance Corp. vs. Court of Appeals
G.R. No. L-31845 April 30, 1979
On March 14, 1957, private respondent Ngo Hing filed an application with Pacific Life for a 20-year
endownment policy in the amount of P50,000.00 on the life of his 1yr old daughter Helen Go. Upon the
payment of the insurance premuim, the binding deposit receipt was issued to private respondent Ngo
Hing. Pacific Life disapproved the insurance application on the ground that the 20-year endowment plan
is not available for minors below 7yrs old, but Pacific Life can consider the same under the Juvenile
Triple Action Plan, and advised that if the offer is acceptable, the Juvenile Non-Medical Declaration be
sent to the company. The non-acceptance of the insurance plan by Pacific Life was allegedly not
communicated by petitioner Mondragon, Branch Manager of the Pacific Life in Cebu City. Instead, on
May 6, 1957, Mondragon wrote back Pacific Life again strongly recommending the approval of the 20-

By: Elaine Marie G. Laceda

86

INSURANCE LAW

year endowment insurance plan to children, pointing out that since 1954 the customers, especially the
Chinese, were asking for such coverage.
On May 28, 1957 Helen Go died of influenza with complication of bronchopneumonia. Thereupon, private
respondent sought the payment of the proceeds of the insurance, but having failed in his effort, he filed
the action for recovery. CFI ordered payment. CA affirmed.
Issue: Whether the binding deposit receipt constituted a temporary contract of the life insurance in
question.
Held: No.
The binding deposit receipt is intended to be merely a provisional or temporary insurance contract and
only upon compliance of the following conditions: (1) that the company shall be satisfied that the
applicant was insurable on standard rates; (2) that if the company does not accept the application and
offers to issue a policy for a different plan, the insurance contract shall not be binding until the applicant
accepts the policy offered; otherwise, the deposit shall be refunded; and (3) that if the applicant is not
insurable according to the standard rates, and the company disapproves the application, the insurance
applied for shall not be in force at any time, and the premium paid shall be returned to the applicant.
Clearly implied from the aforesaid conditions is that the binding deposit receipt in question is merely an
acknowledgment, on behalf of the company, that the latter's branch office had received from the
applicant the insurance premium and had accepted the application subject for processing by the
insurance company; and that the latter will either approve or reject the same on the basis of whether or
not the applicant is "insurable on standard rates." Since petitioner Pacific Life disapproved the insurance
application of respondent Ngo Hing, the binding deposit receipt in question had never become in force at
any time.
Issue: WON there was concealment.
Held: Yes.
Private respondent had deliberately concealed the state of health and physical condition of his daughter
Helen Go. When private respondent supplied the required essential data for the insurance application
form, he was fully aware that his one-year old daughter is typically a mongoloid child. Such a congenital
physical defect could never be ensconced nor disguised. Nonetheless, private respondent, in apparent
bad faith, withheld the fact material to the risk to be assumed by the insurance company. As an
insurance agent of Pacific Life, he ought to know, as he surely must have known his duty and
responsibility to such a material fact.
2. Separate Premium Not Required
Pacific Timber vs. Court of Appeals
G.R. No. L-38613 February 25, 1982
On March 19, 1963, the plaintiff secured temporary insurance from the defendant for its exportation of
1,250,000 board feet of Philippine Lauan and Apitong logs to be shipped from the Diapitan Bay, Quezon
Province to Okinawa and Tokyo, Japan. The defendant issued on said date Cover Note No. 1010,
insuring the said cargo of the plaintiff. On March 29, 1963, after the issuance of Cover Note No. 1010,
but before the issuance of the marine policies on April 2, 1963, some of the logs intended to be exported
were lost during loading operations in the Diapitan Bay.
In a letter dated April 4, 1963, the plaintiff informed the defendant about the loss of 'appropriately 32
pieces of log's during loading of the 'SS Woodlock'. Although dated April 4, 1963, the letter was received
in the office of the defendant only on April 15, 1963, as shown by the stamp impression appearing on
the left bottom corner of said letter. The plaintiff subsequently submitted a 'Claim Statement demanding
payment of the loss under Policies Nos. 53 HO 1032 and 53 HO 1033, in the total amount of
P19,286.79. The claim was denied on the ground that the said loss may be considered as covered under

By: Elaine Marie G. Laceda

87

INSURANCE LAW

Cover Note No. 1010 which had become 'null and void by virtue of the issuance of Marine Policies.
Defendant later alleged that the cover note is null and void for lack of valuable consideration. In a
complaint filed by plaintiff, CFI ordered payment by defendant. CA reversed. Hence the petition.
Issue: WON plaintiff cannot claim on the Cover Note for lack of consideration.
Held: No.
The fact that no separate premium was paid on the Cover Note before the loss insured against occurred,
does not militate against the validity of petitioner's contention, for no such premium could have been
paid, since by the nature of the Cover Note, it did not contain, as all Cover Notes do not contain
particulars of the shipment that would serve as basis for the computation of the premiums. As a logical
consequence, no separate premiums are intended or required to be paid on a Cover Note.
At any rate, it is not disputed that petitioner paid in full all the premiums as called for by the statement
issued by private respondent after the issuance of the two regular marine insurance policies, thereby
leaving no account unpaid by petitioner due on the insurance coverage, which must be deemed to
include the Cover Note. If the Note is to be treated as a separate policy instead of integrating it to the
regular policies subsequently issued, the purpose and function of the Cover Note would be set at naught
or rendered meaningless, for it is in a real sense a contract, not a mere application for insurance which
is a mere offer.
It may be true that the marine insurance policies issued were for logs no longer including those which
had been lost during loading operations. This had to be so because the risk insured against is not for
loss during operations anymore, but for loss during transit, the logs having already been safely placed
aboard. This would make no difference, however, insofar as the liability on the cover note is concerned,
for the number or volume of logs lost can be determined independently as in fact it had been so
ascertained at the instance of private respondent itself when it sent its own adjuster to investigate and
assess the loss, after the issuance of the marine insurance policies.
Issue: WON insurer was released from liability under the cover note due to unreasonable delay in giving
notice of loss despite failure of the insurer to promptly and specifically object to the claim on the ground
of delay.
Held: No.
The defense of delay as raised by private respondent in resisting the claim cannot be sustained. The law
requires this ground of delay to be promptly and specifically asserted when a claim on the insurance
agreement is made. The undisputed facts show that instead of invoking the ground of delay in objecting
to petitioner's claim of recovery on the cover note, it took steps clearly indicative that this particular
ground for objection to the claim was never in its mind. The nature of this specific ground for resisting a
claim places the insurer on duty to inquire when the loss took place, so that it could determine whether
delay would be a valid ground upon which to object to a claim against it.
As already stated earlier, private respondent's reaction upon receipt of the notice of loss, which was on
April 15, 1963, was to set in motion from July 1963 what would be necessary to determine the cause
and extent of the loss, with a view to the payment thereof on the insurance agreement. Thus it sent its
adjuster to investigate and assess the loss in July, 1963. The adjuster submitted his report on August
23, 1963 and its computation of respondent's liability on September 14, 1963. From April 1963 to July,
1963, enough time was available for private respondent to determine if petitioner was guilty of delay in
communicating the loss to respondent company. In the proceedings that took place later in the Office of
the Insurance Commissioner, private respondent should then have raised this ground of delay to avoid
liability. It did not do so. It must be because it did not find any delay, as this Court fails to find a real
and substantial sign thereof. But even on the assumption that there was delay, this Court is satisfied
and convinced that as expressly provided by law, waiver can successfully be raised against private
respondent.
C.

KINDS OF PROPERTY INSURANCE POLICY

By: Elaine Marie G. Laceda

88

INSURANCE LAW

1. Classification Sec 59, I.C.


Sec. 59. A policy is either open, valued or running.
2. Open Policy Secs 60, 161, 171, I.C.
Sec. 60. An open policy is one in which the value of the thing insured is not agreed upon, but is left to
be ascertained in case of loss.
Sec. 161. In estimating a loss under an open policy of marine insurance the following rules are to be
observed:
(a) The value of a ship is its value at the beginning of the risk, including all articles or charges
which add to its permanent value or which are necessary to prepare it for the voyage insured;
(b) The value of the cargo is its actual cost to the insured, when laden on board, or where the
cost cannot be ascertained, its market value at the time and place of lading, adding the charges incurred
in purchasing and placing it on board, but without reference to any loss incurred in raising money for its
purchase, or to any drawback on its exportation, or to the fluctuation of the market at the port of
destination, or to expenses incurred on the way or on arrival;
(c) The value of freightage is the gross freightage, exclusive of primage, without reference to
the cost of earning it; and
(d) The cost of insurance is in each case to be added to the value thus estimated.
Sec. 171. If there is no valuation in the policy, the measure of indemnity in an insurance against fire is
the expense it would be to the insured at the time of the commencement of the fire to replace the thing
lost or injured in the condition in which at the time of the injury; but if there is a valuation in a policy of
fire insurance, the effect shall be the same as in a policy of marine insurance.
Development Insurance Corp. vs. IAC
G.R. No. 71360 July 16, 1986
A fire occurred in the building of the private respondent and it sued for recovery of damages from the
petitioner on the basis of an insurance contract between them. A judgment of default was subsequently
rendered on the strength of the evidence submitted ex parte by the private respondent, which was
allowed full recovery of its claimed damages. IAC affirmed.
Issue: Amount of Indemnity.
Held: P508,867
Policy RY/F-082 is an open policy and is subject to the express condition that: In the event of loss,
whether total or partial, it is understood that the amount of the loss shall be subject to appraisal and the
liability of the company, if established, shall be limited to the actual loss, subject to the applicable
terms, conditions, warranties and clauses of this Policy, and in no case shall exceed the amount of the
policy. This means that the actual loss, as determined, will represent the total indemnity due the
insured from the insurer except only that the total indemnity shall not exceed the face value of the
policy.
The petitioner argues that since at the time of the fire the building insured was worth P5,800,000.00,
the private respondent should be considered its own insurer for the difference between that amount and
the face value of the policy and should share pro rata in the loss sustained. Accordingly, the private
respondent is entitled to an indemnity of only P67,629.31, the rest of the loss to be shouldered by it
alone. However, there is no evidence on record that the building was worth P5,800,000.00 at the time of
the loss; only the petitioner says so and it does not back up its self-serving estimate with any
independent corroboration. On the contrary, the building was insured at P2,500,000.00, and this must
be considered, by agreement of the insurer and the insured, the actual value of the property insured on
the day the fire occurred. This valuation becomes even more believable if it is remembered that at the
time the building was burned it was still under construction and not yet completed.

By: Elaine Marie G. Laceda

89

INSURANCE LAW

3. Valued Policy Secs 61, 156, 171, I.C.


Sec. 61. A valued policy is one which expresses on its face an agreement that the thing insured shall be
valued at a specific sum.
Sec. 156. A valuation in a policy of marine insurance in conclusive between the parties thereto in the
adjustment of either a partial or total loss, if the insured has some interest at risk, and there is no fraud
on his part; except that when a thing has been hypothecated by bottomry or respondentia, before its
insurance, and without the knowledge of the person actually procuring the insurance, he may show the
real value. But a valuation fraudulent in fact, entitles the insurer to rescind the contract.
Sec. 171. If there is no valuation in the policy, the measure of indemnity in an insurance against fire is
the expense it would be to the insured at the time of the commencement of the fire to replace the thing
lost or injured in the condition in which at the time of the injury; but if there is a valuation in a policy of
fire insurance, the effect shall be the same as in a policy of marine insurance.
4. Running Policy Sec 62, I.C.
Sec. 62. A running policy is one which contemplates successive insurances, and which provides that the
object of the policy may be from time to time defined, especially as to the subjects of insurance, by
additional statements or indorsements.
D.

CANCELLATION, RENEWAL, REFORMATION

1. Cancellation
a. By insurer Sec 64, I.C.
Sec. 64. No policy of insurance other than life shall be cancelled by the insurer except upon prior notice
thereof to the insured, and no notice of cancellation shall be effective unless it is based on the
occurrence, after the effective date of the policy, of one or more of the following:
(a) non-payment of premium;
(b) conviction of a crime arising out of acts increasing the hazard insured against;
(c) discovery of fraud or material misrepresentation;
(d) discovery of willful or reckless acts or omissions increasing the hazard insured against;
(e) physical changes in the property insured which result in the property becoming
uninsurable; or
(f) a determination by the Commissioner that the continuation of the policy would violate or
would place the insurer in violation of this Code.
PhilamCare Health Systems, Inc. vs. Court of Appeals
G.R. No. 125678 March 18, 2002
Ernani Trinos, deceased husband of respondent Julita Trinos, applied for a health care coverage with
petitioner Philamcare Health Systems, Inc. In the standard application form, he stated that he was
never treated for high blood pressure, heart trouble, diabetes, cancer, liver disease, asthma or peptic
ulcer.
Under the agreement, respondents husband was entitled to avail of hospitalization benefits, whether
ordinary or emergency, listed therein. He was also entitled to avail of "out-patient benefits" such as
annual physical examinations, preventive health care and other out-patient services.
During the period of his coverage, Ernani suffered a heart attack and was confined at the Manila Medical
Center (MMC) for one month. While her husband was in the hospital, respondent tried to claim the
benefits under the health care agreement. However, petitioner denied her claim saying that the Health
Care Agreement was void. According to petitioner, there was a concealment regarding Ernanis medical
history. Doctors at the MMC allegedly discovered at the time of Ernanis confinement that he was
hypertensive, diabetic and asthmatic, contrary to his answer in the application form.

By: Elaine Marie G. Laceda

90

INSURANCE LAW

After Ernanis death, Julita Trinos filed an action for damages where she asked for reimbursement of her
expenses plus moral damages and attorneys fees.
RTC rendered judgment in favor of the plaintiff ordering defendants to pay and reimburse the medical
and hospital coverage of the late Ernani Trinos, and to pay moral and exemplary damages plus
attorneys fees and cost of suit. CA affirmed.
Held: SC Affirmed.
An insurance contract exists where the following elements concur:
1. The insured has an insurable interest;
2. The insured is subject to a risk of loss by the happening of the designated peril;
3. The insurer assumes the risk;
4. Such assumption of risk is part of a general scheme to distribute actual losses among a large
group of persons bearing a similar risk; and
5. In consideration of the insurers promise, the insured pays a premium.
Section 10 provides that: Every person has an insurable interest in the life and health
1. of himself, of his spouse and of his children;
2. of any person on whom he depends wholly or in part for education or support, or in whom he
has a pecuniary interest;
3. of any person under a legal obligation to him for the payment of money, respecting property or
service, of which death or illness might delay or prevent the performance; and
4. of any person upon whose life any estate or interest vested in him depends.
In the case at bar, the insurable interest of respondents husband in obtaining the health care
agreement was his own health. The health care agreement was in the nature of non-life insurance,
which is primarily a contract of indemnity. Once the member incurs hospital, medical or any other
expense arising from sickness, injury or other stipulated contingent, the health care provider must pay
for the same to the extent agreed upon under the contract.
Having assumed a responsibility under the agreement, petitioner is bound to answer the same to the
extent agreed upon. In the end, the liability of the health care provider attaches once the member is
hospitalized for the disease or injury covered by the agreement or whenever he avails of the covered
benefits which he has prepaid.
Under Section 27 of the Insurance Code, "a concealment entitles the injured party to rescind a contract
of insurance." However, the right to rescind should be exercised previous to the commencement of an
action on the contract. In this case, no rescission was made. Besides, the cancellation of health care
agreements as in insurance policies require the concurrence of certain conditions, none of which was
fulfilled in this case.
1. Prior notice of cancellation to insured;
2. Notice must be based on the occurrence after effective date of the policy of one or more of the
grounds mentioned;
3. Must be in writing, mailed or delivered to the insured at the address shown in the policy;
4. Must state the grounds relied upon provided in Section 64 of the Insurance Code and upon
request of insured, to furnish facts on which cancellation is based.
The defendant Philamcare Health Systems Inc. had twelve months from the date of issuance of the
Agreement within which to contest the membership of the patient if he had previous ailment of asthma,
and six months from the issuance of the agreement if the patient was sick of diabetes or hypertension.
The periods having expired, the defense of concealment or misrepresentation no longer lie.
Lastly, petitioner alleges that respondent was not the legal wife of the deceased member considering
that at the time of their marriage, the deceased was previously married to another woman who was still
alive. However, the health care agreement is in the nature of a contract of indemnity. Hence, payment
should be made to the party who incurred the expenses.

By: Elaine Marie G. Laceda

91

INSURANCE LAW

i.
ii.
iii.
iv.
v.
vi.

Non-payment of premiums
Conviction of crime increasing hazard insured against
Discovery of fraud or material misrepresentation
Discovery of acts or omissions increasing hazard insured against
Physical changes in property insured making it uninsurable
Insurance Commissioner determines policy or insured may violate I.C.

b. By insured Sec 79(b), I.C.


Sec. 79. A person insured is entitled to a return of premium, as follows:
(b) Where the insurance is made for a definite period of time and the insured surrenders his
policy, to such portion of the premium as corresponds with the unexpired time, at a pro rata rate, unless
a short period rate has been agreed upon and appears on the face of the policy, after deducting from
the whole premium any claim for loss or damage under the policy which has previously accrued;
Provided, That no holder of a life insurance policy may avail himself of the privileges of this paragraph
without sufficient cause as otherwise provided by law.
c.

For compulsory motor vehicle liability insurance Secs 380-381, I.C.

Sec. 380. No cancellation of the policy shall be valid unless written notice thereof is given to the land
transportation operator or owner of the vehicle and to the Land Transportation Commission at least
fifteen days prior to the intended effective date thereof. Upon receipt of such notice, the Land
Transportation Commission, unless it receives evidence of a new valid insurance or guaranty in cash or
surety bond as prescribed in this chapter, or an endorsement of revival of the cancelled one, shall order
the immediate confiscation of the plates of the motor vehicle covered by such cancelled policy. The
same may be re-issued only upon presentation of a new insurance policy or that a guaranty in cash or
surety band has been made or posted with the Commissioner and which meets the requirements of this
chapter, or an endorsement or revival of the cancelled one. (As amended by Presidential Decree No.
1455).
Sec. 381. If the cancellation of the policy or surety bond is contemplated by the land transportation
operator or owner of the vehicle, he shall, before the policy or surety bond ceases to be effective, secure
a similar policy of insurance or surety bond to replace the policy or surety bond to be cancelled or make
a cash deposit in sufficient amount with the Commissioner and without any gap, file the required
documentation with the Land Transportation Commission, and notify the insurance company concerned
of the cancellation of its policy or surety bond. (As amended by Presidential Decree No. 1455).
d. Notice Sec 65, I.C.
Sec. 65. All notices of cancellation mentioned in the preceding section shall be in writing, mailed or
delivered to the named insured at the address shown in the policy, and shall state (a) which of the
grounds set forth in section sixty-four is relied upon and (b) that, upon written request of the named
insured, the insurer will furnish the facts on which the cancellation is based.
Paulino vs. Capital Insurance
G.R. No. L-11728 May 15, 1959
On February 8, 1952, the plaintiff accepted a fire insurance policy issued by the defendant; that on April
30, 1952, the plaintiff wrote the defendant requesting cancellation of the policy, which the latter
received on May 10, 1952; that the plaintiff did not return the policy or demanded for the return of the
proportionate premium; that neither did the defendant offer to return the premium; that the property
covered by the policy was destroyed by fire on August 16, 1952. The defendant refused to make
payment on plaintiff's claim, on the ground that the policy was cancelled as of May 10, 1952. Plaintiff
contends in this appeal that her letter, dated April 30, 1952, was a mere request or offer to cancel the
Policy and did not terminate the same since it was not accompanied by the surrender of the policy for
cancellation.
Issue: WON the policy was cancelled.

By: Elaine Marie G. Laceda

92

INSURANCE LAW

Held: Yes.
Paragraph 10 of the policy provides: This insurance may be terminated at any time at the request of the
Insured, in which case the Company will retain the customary short period rate for the time the policy
has been in force. This insurance may also at any time be terminated at the option of the Company, on
notice to that effect being given to the Insured, in which case the Company shall be liable to repay on
demand a ratable proportion of the premium for the expired term from the date of cancelment.
Pursuant to this stipulation, the contract in question could be terminated, "at any time", upon the
unilateral act of either party. Whichever party exercised the "option", did not need the approval, consent
or concurrence of the other thereto. That consent was given at the time of the making of the contract.
Upon these facts, the insured was not entitled to collect the amount of the policy, because the
unconditional return thereof upon request of the company implied "a waiver of his right to treat the
policy as in full force and effect until the company paid or tendered to him the unearned premium."
Saura Import & Export Co., Inc. vs. Philippine International Surety Co., Inc.
G.R. No. L-15184 May 31, 1963
Saura Import & Export Co Inc., mortgaged to the PNB, a parcel of land with a building of strong
materials to secure the payment of the total mortgaged debt to P37,000.00. The mortgage contact
required that the mortgagor shall insure the mortgaged property at all times against fire and earthquake
for an amount and with such company satisfactory to the Mortgagee, indorsing to the latter the
corresponding policies. Pursuant to the requirement, Saura insured the building and its contents with the
Philippine International Surety, an insurance firm acceptable to mortgagee Bank, for P29,000.00 against
fire for the period of one year from October 2, 1954. As required therefor, the insurance policy was
endorsed to the mortgagee PNB.
Barely 13 days after the issuance of the fire insurance policy, the insurer cancelled the same, effective
as of the date of issue. Notice of the cancellation was given to appellee bank in writing, sent by
Registered Mail and personally addressed to the Branch Manager, and was received by the Bank on
November 8, 1954. On April 6, 1955, the building and its contents, worth P40,685.69 were burned.
Upon the presentation of notice of loss with the PNB, Saura learned for the first time that the policy had
previously been cancelled by the insurer. Upon refusal of the insurer to pay the amount of the
insurance, Civil Case No. 26847 was filed with the Manila CFI against the Insurer, and the PNB was later
included as party defendant, after it had refused to prosecute the case jointly with Saura Import &
Export Co., Inc. CFI dismissed complaint.
Issue: WON the insurer effectively cancelled the insurance policy.
Held: No.
Fire insurance policies and other contracts of insurance upon property, in addition to the common
provision for cancellation of the policy upon request of the insured, generally provide for cancellation by
the insurer by notice to the insured for a prescribed period, which is usually 5 days, and the return of
the unearned portion of the premium paid by the insured. The purpose of provisions or stipulations for
notice to the insured, is to prevent the cancellation of the policy, without allowing the insured ample
opportunity to negotiate for other insurance in its stead. Actual personal notice to the insured is
essential to a cancellation under a provision for cancellation by notice. The actual receipt by the insured
of a notice of cancellation is universally recognized as a condition precedent to a cancellation of the
policy by the insurer, and consequently a letter containing notice of cancellation which is mailed by the
insurer but not received by the insured, is ineffective as cancellation.
However, the policy in question does not provide for the notice, its form or period. The Insurance Law
does not likewise provide for such notice. This being the case, it devolves upon the Court to apply the
generally accepted principles of insurance, regarding cancellation of the insurance policy by the insurer.
Actual notice of cancellation in a clear and unequivocal manner, preferably in writing, in view of the
importance of an insurance contract, should be given by the insurer to the insured, so that the latter

By: Elaine Marie G. Laceda

93

INSURANCE LAW

might be given an opportunity to obtain other insurance for his own protection. The notice should be
personal to the insured and not to and/or through any unauthorized person by the policy. In the case at
bar, the defendant insurance company, must have realized the paramount importance of sending a
notice of cancellation, when it sent the notice of cancellation of the policy to the defendant bank (as
mortgagee), but not to the insured with which it had direct dealing. It was the primary duty of the
insurance company to notify the insured, but it did not. Notice to the bank is not effective notice.
Malayan Insurance Co., Inc. vs. Arnaldo
G.R. No. L-67835 October 12, 1987
Petitioner issued to private respondent, Coronacion Pinca, Fire Insurance Policy No. F-001-17212 on her
property for the amount of P14,000.00 effective July 22, 1981, until July 22, 1982. Howver, on October
15, 1981, MICO allegedly cancelled the policy for non-payment, of the premium and sent the
corresponding notice to Pinca. Payment of the premium was then later received by Domingo Adora,
agent of MICO who remitted this payment to MICO, together with other payments on January 15, 1982.
On January 18, 1982, Pinca's property was completely burned. On February 5, 1982, Pinca's payment
was returned by MICO to Adora on the ground that her policy had been cancelled earlier. But Adora
refused to accept it. In due time, Pinca made the requisite demands for payment, which MICO rejected.
She then went to the Insurance Commission which sustained her claim for compensation for her burned
property.
Issue: WON Adora had the authority to receive payment.
Held: Yes.
MICO's acknowledgment of Adora as its agent defeats its contention that he was not authorized to
receive the premium payment on its behalf. It is clearly provided in Section 306 of the Insurance Code
that: Any insurance company which delivers to an insurance agant or insurance broker a policy or
contract of insurance shall be demmed to have authorized such agent or broker to receive on its behalf
payment of any premium which is due on such policy or contract of insurance at the time of its issuance
or delivery or which becomes due thereon.
It is also a well-known principle under the law of agency that: Payment to an agent having authority to
receive or collect payment is equivalent to payment to the principal himself; such payment is complete
when the money delivered is into the agent's hands and is a discharge of the indebtedness owing to the
principal.
Issue: WON the policy was subsisting at the time of the fire.
Held: Yes.
MICO's claims it cancelled the policy in question on October 15, 1981, for non-payment of premium. To
support this assertion, it presented one of its employees, who testified that "the original of the
endorsement and credit memo" presumably meaning the alleged cancellation "were sent the
assured by mail through our mailing section" However, there is no proof that the notice, assuming it
complied with the other requisites for cancellation, was actually mailed to and received by Pinca. All
MICO's offers to show that the cancellation was communicated to the insured is its employee's testimony
that the said cancellation was sent "by mail through our mailing section." without more.
It stands to reason that if Pinca had really received the said notice, she would not have made payment
on the original policy on December 24, 1981. Instead, she would have asked for a new insurance,
effective on that date and until one year later, and so taken advantage of the extended period. The
Court finds that if she did pay on that date, it was because she honestly believed that the policy issued
on June 7, 1981, was still in effect and she was willing to make her payment retroact to July 22, 1981,
its stipulated commencement date. After all, the premium invoice issued to Pinca at the time of the
delivery of the policy on June 7, 1981 was stamped "Payment Received" of the amount of P930.60 on
"12-24-81" by Domingo Adora. This is important because it suggests an understanding between MICO
and the insured that such payment could be made later, as agent Adora had assured Pinca.

By: Elaine Marie G. Laceda

94

INSURANCE LAW

2. Renewal Sec 66, I.C.


Sec. 66. In case of insurance other than life, unless the insurer at least forty-five days in advance of the
end of the policy period mails or delivers to the named insured at the address shown in the policy notice
of its intention not to renew the policy or to condition its renewal upon reduction of limits or elimination
of coverages, the named insured shall be entitled to renew the policy upon payment of the premium due
on the effective date of the renewal. Any policy written for a term of less than one year shall be
considered as if written for a term of one year. Any policy written for a term longer than one year or
any policy with no fixed expiration date shall be considered as if written for successive policy periods or
terms of one year.
3. Reformation
San Miguel Brewery vs. Law Union and Rock Insurance Co.
G.R. No. L-14300 January 19, 1920
On January 12, 1916, D. P. Dunn, the owner of the subject property, mortgaged the same to the San
Miguel Brewery to secure a debt of P10,000. In the contract of mortgage Dunn agreed to keep the
property insured at his expense to the full amount of its value in companies to be selected by the
Brewery Company and authorized the latter in case of loss to receive the proceeds of the insurance and
to retain such part as might be necessary to cover the mortgage debt. At the same time, in order more
conveniently to accomplish the end in view, Dunn authorized and requested the Brewery Company to
effect said insurance itself. Accordingly on the same date Antonio Brias, general manager of the
Brewery, made a verbal application to the Law Union and Rock Insurance Company for insurance to the
extent of P15,000 upon said property. In reply to a question of the company's agent as to whether the
Brewery was the owner of the property, he stated that the company was interested only as a
mortgagee. No information was asked as to who was the owner of the property, and no information
upon this point was given. In the month of March of the year 1917 Dunn sold the insured property to
the defendant Henry Harding, but no assignment of the insurance, or of the insurance policies, was at
any time made to him.
When the property was destroyed by fire, San Miguel Brewery filed for a complaint for recovery from
Union and Rock Insurance Company Ltd., and the "Filipinas" Compania de Seguros, for the sum of
P7,500 each. Harding claimed for himself the right to recover the difference between the plaintiff's
mortgage credit and the face value of the policies. The two insurance companies admitted their liability
to the San Miguel Brewery to the extent of its mortgage credit, but denied liability to Harding on the
ground that under the contracts of insurance the liability of the insurance companies was limited to the
insurable interest of the plaintiff therein.
Soon after the action was begun the insurance companies effected a settlement with the San Miguel
Brewery by paying the full amount of the credit claimed by it, with the result that the litigation as
between the original plaintiff and the two insurance companies came to an end, leaving the action to be
prosecuted to final judgment by Harding with respect to the balance claimed to be due to him upon the
policies. The trial judge came to the conclusion that Harding had no right of action whatever against the
companies and absolved them from liability.
Issue: WON the insurance companies are liable for payment to Harding.
Held: No.
Both policies were issued in the name of the San Miguel Brewery as the assured, and contained no
reference to any other interest in the property. Thus, Henry Harding does not have a cause of action
against the insurance companies. He is not a party to the contracts of insurance and cannot directly
maintain an action thereon. His claim is merely of an equitable and subsidiary nature and must be made
effective, if at all, through the San Miguel Brewery in whose name the contracts are written. Now the
Brewery, as mortgagee of the insured property, undoubtedly had an insurable interest therein; but it
could not, in any event, recover upon these policies an amount in excess of its mortgage credit. This is

By: Elaine Marie G. Laceda

95

INSURANCE LAW

clear from the express provisions of Sec 16 and Sec 50 of the Insurance Act -- "the measure of an
insurable interest in property is the extent to which the insured might be damnified by loss or injury
thereof" (sec. 16); and "the insurance shall be applied exclusively to the proper interest of the person in
whose name it is made unless otherwise specified in the policy" (sec. 50).
The policies of insurance might have been so framed as to have been "payable to the San Miguel
Brewery, mortgagee, as its interest may appear, remainder to whomsoever, during the continuance of
the risk, may become the owner of the interest insured." (Sec 54, Act No. 2427.) Such a clause would
have proved an intention to insure the entire interest in the property, not merely the insurable interest
of the San Miguel Brewery, and would have shown exactly to whom the money, in case of loss, should
be paid. But the policies are not so written.
If during the negotiations which resulted in the writing of the insurance policy it had been agreed
between the contracting parties that the insurance should be so written as to protect not only the
interest of the mortgagee but also the residuary interest of the owner, and the policies had been, by
inadvertence, ignorance, or mistake written in the form in which they were issued, a court would have
the power to reform the contracts and give effect to them in the sense in which the parties intended to
be bound. But in order to justify this, it must be made clearly to appear that the minds of the
contracting parties did actually meet in agreement and that they labored under some mutual error or
mistake in respect to the expression of their purpose. The proof must be of the most satisfactory
character, and it must clearly appear that the contract failed to express the real agreement between the
parties. In the case now before us the proof is entirely insufficient to authorize the reformation.
VI.

ASCERTAINING AND CONTROLLING RISKS


A.

1.

CONCEALMENT
Definition Sec. 26, I.C

Sec. 26. A neglect to communicate that which a party knows and ought to communicate, is called a
concealment.

2.

What Must Be Communicated; Requisites Sec. 28, I.C.;

Sec. 28. Each party to a contract of insurance must communicated to the other, in good faith, all facts
within his knowledge which are material to the contract and as to which he makes no warranty, and
which the other has not the means of ascertaining.
a. Facts within his knowledge
b. Facts material to the contract Secs. 31 and 107, I.C.
Sec. 31. Materiality is to be determined not by the event, but solely by the probable and reasonable
influence of the facts upon the party to whom the communication is due, in forming his estimate of the
disadvantages of the proposed contract, or in making his inquiries.
Sec. 107. In marine insurance each party is bound to communicate, in addition to what is required by
section twenty-eight, all the information which he possesses, material to the risk, except such as is
mentioned in Section thirty, and to state the exact and whole truth in relation to all matters that he
represents, or upon inquiry discloses or assumes to disclose.
Saturnino vs. Philippine American Life Ins. Co.
G.R. No. L-16163 February 28, 1963
The policy subject of the case is one for 20-year endowment non-medical insurance obtained by
Estefania A. Saturnino. This kind of policy dispenses with the medical examination of the applicant
usually required in ordinary life policies. However, detailed information is called for in the application
concerning the applicant's health and medical history. Notwithstanding the fact of her operation

By: Elaine Marie G. Laceda

96

INSURANCE LAW

Estefania did not make a disclosure thereof in her application for insurance. On the contrary, she stated
therein that she did not have, nor had she ever had, among other ailments listed in the application,
cancer or other tumors; that she had not consulted any physician, undergone any operation or suffered
any injury within the preceding five years; and that she had never been treated for nor did she ever
have any illness or disease peculiar to her sex, particularly of the breast, ovaries, uterus, and menstrual
disorders. The written application in this case was submitted by Saturnino to appellee on November 16,
1957, witnessed by appellee's agent Edward A. Santos. The policy was issued on the same day, upon
payment of the first year's premium of P339.25. On September 19, 1958 Saturnino died of pneumonia,
secondary to influenza. Appellants here, who are her surviving husband and minor child, respectively,
demanded payment of the face value of the policy. The claim was rejected and this suit was
subsequently instituted.
Issue: WON the insured made such false representations of material facts as to avoid the policy.
Held: Yes.
There can be no dispute that the information given by Estefania in her application for insurance was
false, namely, that she had never had cancer or tumors, or consulted any physician or undergone any
operation within the preceding period of five years. The information was likewise material. Materiality is
to be determined not by the event, but solely by the probable and reasonable influence of the facts upon
the party to whom the communication is due, in forming his estimate of the proposed contract, or in
making his inquiries. The waiver of medical examination renders even more material the information
required of the applicant concerning previous condition of health and diseases suffered, for such
information necessarily constitutes an important factor which the insurer takes into consideration in
deciding whether to issue the policy or not. It is logical to assume that if appellee had been properly
apprised of the insured's medical history she would at least have been made to undergo medical
examination in order to determine her insurability. No negligence can be imputed to appellee in not
asking her to submit to a medical examination as appellants maintain. It was precisely because the
insured had given herself a clean bill of health that appellee no longer considered an actual medical
checkup necessary.
In this jurisdiction a concealment, whether intentional or unintentional, entitles the insurer to rescind
the contract of insurance, concealment being defined as "negligence to communicate that which a party
knows and ought to communicate" The basis of the rule vitiating the contract in cases of concealment is
that it misleads or deceives the insurer into accepting the risk, or accepting it at the rate of premium
agreed upon. The insurer, relying upon the belief that the assured will disclose every material fact within
his actual or presumed knowledge, is misled into a belief that the circumstance withheld does not exist,
and he is thereby induced to estimate the risk upon a false basis that it does not exist.
Sunlife Assurance Company of Canada vs. Court of Appeals
G.R. No. 105135 June 22, 1995
On April 15, 1986, Robert John B. Bacani procured a life insurance contract valued at P100,000.00, with
double indemnity in case of accidental death. The designated beneficiary was his mother, respondent
Bernarda Bacani. On June 26, 1987, the insured died in a plane crash. Respondent Bernarda Bacani filed
a claim with petitioner, seeking the benefits of the insurance policy taken by her son. Petitioner
conducted an investigation and its findings prompted it to reject the claim. Petitioner discovered that,
contrary to what was stated, two weeks prior to his application for insurance, the insured was examined
and confined at the Lung Center of the Philippines, where he was diagnosed for renal failure. During his
confinement, the deceased was subjected to urinalysis, ultra-sonography and hematology tests.
As a result of the denial of the claim, respondent Bacani filed an action for specific performance against
petitioner. Upon petitioners instance TC rendered a summary judgment ruling in favor of respondent
Bacani. The TC concluded that the facts concealed by the insured were made in good faith and under a
belief that they need not be disclosed. Moreover, it held that the health history of the insured was
immaterial since the insurance policy was "non-medical". CA affirmed ruling further that petitioner
cannot avoid its obligation by claiming concealment because the cause of death was unrelated to the
facts concealed by the insured.

By: Elaine Marie G. Laceda

97

INSURANCE LAW

Issue: WON the health history of the insured is immaterial.


Held: No.
Section 26 of The Insurance Code is explicit in requiring a party to a contract of insurance to
communicate to the other, in good faith, all facts within his knowledge which are material to the contract
and as to which he makes no warranty, and which the other has no means of ascertaining. Materiality is
to be determined not by the event, but solely by the probable and reasonable influence of the facts upon
the party to whom communication is due, in forming his estimate of the disadvantages of the proposed
contract or in making his inquiries.
The information which the insured failed to disclose were material and relevant to the approval and
issuance of the insurance policy. The matters concealed would have definitely affected petitioner's action
on his application, either by approving it with the corresponding adjustment for a higher premium or
rejecting the same. Moreover, a disclosure may have warranted a medical examination of the insured by
petitioner in order for it to reasonably assess the risk involved in accepting the application.
Anent the finding that the facts concealed had no bearing to the cause of death of the insured, it is well
settled that the insured need not die of the disease he had failed to disclose to the insurer. It is
sufficient that his non-disclosure misled the insurer in forming his estimates of the risks of the proposed
insurance policy or in making inquiries.
c.

Other party has no means of ascertaining such facts; and

d. He makes no warranty as to such facts; exception Sec. 29, I.C.


Sec. 29. An intentional and fraudulent omission, on the part of one insured, to communicate
information of matters proving or tending to prove the falsity of a warranty, entitles the insurer to
rescind.
3. What Need Not Be Communicated

a.

Facts other party knows Sec. 30(a), I.C.

Sec. 30. Neither party to a contract of insurance is bound to communicate information of the matters
following, except in answer to the inquiries of the other:
(a) Those which the other knows;
(b) Those which, in the exercise of ordinary care, the other ought to know, and of which the
former has no reason to suppose him ignorant;
(c) Those of which the other waives communication;
(d) Those which prove or tend to prove the existence of a risk excluded by a warranty, and
which are not otherwise material; and
(e) Those which relate to a risk excepted from the policy and which are not otherwise material.
Insular Life Assurance vs. Feliciano
G.R. No. L-47593 September 13, 1941
Two insurance policies to the aggregate amount of P25,000 were issued to Evaristo Feliciano upon the
solicitation of one of Insular Lifes agents. Feliciano died on September 29, 1935. The defendant
company refused to pay on the ground that the policies were fraudulently obtained, the insured having
given false answers and statements in the application as well as in the medical report.
The lower court found that at the time Feliciano filed his application and at the time he was subjected to
physical examination by the medical examiner of the herein petitioner, he was already suffering from
tuberculosis. However, Feliciano was made to sign the application and the examiner's report in blank,
with the blank spaces afterwards filled in by the agent and the medical examiner, who made it appear

By: Elaine Marie G. Laceda

98

INSURANCE LAW

therein that Feliciano was a fit subject for insurance. Thus, it ruled for the plaintiff holding that neither
the insured nor any member of his family concealed the real state of health of the insured.
Issue: WON an insurer has the right to avoid a policy where its agent knowingly and intentionally wrote
down the answers in the application differing from those made by the insured.
Held: No.
In the present case, the agent knew all the time the true state of health of the insured. The insurer's
medical examiner approved the application knowing full well that the applicant was sick. The situation is
one in which one of two innocent parties must bear a loss for his reliance upon a third person. In this
case, it was the insurer who gave the agent authority to deal with the applicant. It was the one who
selected the agent, thus implying that the insured could put his trust on him. It was the one who drafted
and accepted the policy and consummated the contract. It seems reasonable that as between the two of
them, the one who employed and gave character to the third person as its agent should be the one to
bear the loss.
The weight of authority is that if an agent of the insurer, after obtaining from an applicant for insurance
a correct and truthful answer to interrogatories contained in the application for insurance, without
knowledge of the applicant fills in false answers, either fraudulently or otherwise, the insurer cannot
assert the falsity of such answers as a defense to liability on the policy, and this is true generally without
regard to the subject matter of the answers or the nature of the agent's duties or limitations on his
authority, at least if not brought to the attention of the applicant.
Insular Life Assurance Co. vs. Feliciano
G.R. No. L-47593 December 29, 1943
Evaristo Feliciano, who died on September 29, 1935, was suffering with advanced pulmonary
tuberculosis when he signed his applications for insurance with the petitioner on October 12, 1934.
Nevertheless the question contained in the application "Have you ever suffered from any ailment or
disease of the lungs, pleurisy, pneumonia or asthma?" appears to have been answered , "No". The
false answer was written by the Company's soliciting agent Romulo M. David, in collusion with the
medical examiner Dr. Gregorio Valdez, for the purpose of securing the Company's approval of the
application so that the policy to be issued thereon might be credited to said agent in connection with the
inter-provincial contest which the Company was then holding among its soliciting agents to boost the
sales of its policies.
Issue: WON an insurer has the right to avoid a policy where its agent knowingly and intentionally wrote
down the answers in the application differing from those made by the insured.
Held: Yes.
When Evaristo Feliciano, the applicant for insurance, signed the application in blank and authorized the
soliciting agent and/or medical examiner of the Company to write the answers for him, he made them
his own agents for that purpose, and he was responsible for their acts in that connection. If they falsified
the answers for him, he could not evade the responsibility for he falsification. He was not supposed to
sign the application in blank. He knew that the answers to the questions therein contained would be "the
basis of the policy," and for that every reason he was required with his signature to vouch for truth
thereof.
Moreover, from the facts of the case we cannot escape the conclusion that the insured acted in
connivance with the soliciting agent and the medical examiner of the Company in accepting the policies
in question. Above the signature of the applicant is the printed statement or representation: " . . . I am
a proper subject for life insurance." In another sheet of the same application and above another
signature of the applicant was also printed this statement: "That the said policy shall not take effect
until he first premium has been paid and the policy as been delivered to and accepted by me, while I am
in good health." When the applicant signed the application he was "having difficulty in breathing, . . .
with a very high fever." He had gone three times to the Santol Sanatorium and had X-ray pictures taken

By: Elaine Marie G. Laceda

99

INSURANCE LAW

of his lungs. He therefore knew that he was not "a proper subject for life insurance." When he accepted
the policy, he knew that he was not in good health. Nevertheless, he not only accepted the first policy of
P20,000 but then and there applied for and later accepted another policy of P5,000.
b. Facts other party ought to know Secs. 30(b) and 32, I.C.
Sec. 32. Each party to a contract of insurance is bound to know all the general causes which are open
to his inquiry, equally with that of the other, and which may affect the political or material perils
contemplated; and all general usages of trade.
c.

Facts of which other party waives communication Secs. 30(c) and 33, I.C.

Sec. 33. The right to information of material facts may be waived, either by the terms of the insurance
or by neglect to make inquiry as to such facts, where they are distinctly implied in other facts of which
information is communicated.
Ng Gan Zee vs. Asian Crusader Life Assurance Corp.
G.R. No. L-30685 May 30, 1983
On May 12, 1962, Kwong Nam applied for a 20-year endowment insurance on his life for the sum of
P20,000.00, with his wife, appellee Ng Gan Zee as beneficiary. On the same date, appellant, upon
receipt of the required premium from the insured, approved the application and issued the
corresponding policy. On December 6, 1963, Kwong Nam died of cancer of the liver with metastasis. Ng
Gan Zee presented a claim in due form to appellant for payment of the face value of the policy.
Appellant denied the claim on the ground that the insured was guilty of misrepresentation when he
answered "No" to the following question appearing in the application for life insurance- Has any life
insurance company ever refused your application for insurance or for reinstatement of a lapsed policy or
offered you a policy different from that applied for?
Appellant further maintains that when the insured was examined in connection with his application for
life insurance, he gave the appellant's medical examiner false and misleading information as to his
ailment and previous operation.
Issue: WON appellant, because of insured's aforesaid representation, misled or deceived into entering
the contract or in accepting the risk at the rate of premium agreed upon.
Held: No.
Concealment exists where the assured had knowledge of a fact material to the risk, and honesty, good
faith, and fair dealing requires that he should communicate it to the assurer, but he designedly and
intentionally withholds the same.
As to the first issue, there is no evidence that the Insular Life Assurance Co., Ltd. ever refused any
application of Kwong Nam for insurance. Neither is there any evidence that any other insurance
company has refused any application of Kwong Nam for insurance. Nonetheless, assuming that the
answer given by the insured was false, Sec. 27 of the Insurance Law nevertheless requires that
fraudulent intent on the part of the insured be established to entitle the insurer to rescind the contract.
And as correctly observed by the lower court, "misrepresentation as a defense of the insurer to avoid
liability is an 'affirmative' defense. The duty to establish such a defense by satisfactory and convincing
evidence rests upon the defendant. The evidence before the Court does not clearly and satisfactorily
establish that defense."
As to the second issue, it bears emphasis that Kwong Nam had informed the appellant's medical
examiner that the tumor for which he was operated on was "associated with ulcer of the stomach." In
the absence of evidence that the insured had sufficient medical knowledge as to enable him to
distinguish between "peptic ulcer" and "a tumor", his statement that said tumor was "associated with
ulcer of the stomach," should be construed as an expression made in good faith of his belief as to the
nature of his ailment and operation. Indeed, such statement must be presumed to have been made by

By: Elaine Marie G. Laceda

100

INSURANCE LAW

him without knowledge of its incorrectness and without any deliberate intent on his part to mislead the
appellant.
Where, upon the face of the application, a question appears to be not answered at all or to be
imperfectly answered, and the insurers issue a policy without any further inquiry, they waive the
imperfection of the answer and render the omission to answer more fully immaterial. As aptly noted by
the lower court, "if the ailment and operation of Kwong Nam had such an important bearing on the
question of whether the defendant would undertake the insurance or not, the court cannot understand
why the defendant or its medical examiner did not make any further inquiries on such matters before
acting on the application for insurance. The fact of the matter is that the defendant was too eager to
accept the application and receive the insured's premium. It would be inequitable now to allow the
defendant to avoid liability under the circumstances."
d. Facts which prove or tend to prove existence of risk excluded by a warranty,
and which are not otherwise material Sec. 30(d), I.C.
e. Facts relating to a risk excepted from the policy and which are not otherwise
material Sec. 30(e), I.C.
f.

Nature and amount of interest Secs. 34 and 51(e), I.C.

Sec. 34. Information of the nature or amount of the interest of one insured need not be communicated
unless in answer to an inquiry, except as prescribed by section fifty-one.
Sec. 51. A policy of insurance must specify:
(e) The interest of the insured in property insured, if he is not the absolute owner thereof;
g. Opinion or judgment Secs. 35 and 108, I.C.
Sec. 35. Neither party to a contract of insurance is bound to communicate, even upon inquiry,
information of his own judgment upon the matters in question.
Sec. 108. In marine insurance, information of the belief or expectation of a third person, in reference to
a material fact, is material.
4. Effect Secs. 27 and 29, I.C.
Sec. 27. A concealment whether intentional or unintentional entitles the injured party to rescind a
contract of insurance.
Sec. 29. An intentional and fraudulent omission, on the part of one insured, to communicate
information of matters proving or tending to prove the falsity of a warranty, entitles the insurer to
rescind.
Argente vs. West Coast Life Insurance Co.
G.R. No. L-24899 March 19, 1928
On February 9, 1925, Bernardo Argente and Vicenta de Ocampo signed an application for joint
insurance. Both applications, with the exception of the names and the signatures of the applicants, were
written by Jose Geronimo del Rosario, an agent for the West Coast Life Insurance Co. with the
information furnished by the applicants. The spouses were examined by Dr. Cesareo Sta. Ana, a medical
examiner for the insurer. The result of such examination was recorded in the Medical Examiner's Report,
and with the exception of the signature of Bernardo Argente, was in the hand-writing of Dr. Sta. Ana.
But the information or answers to the questions contained on the face of the Medical Examiner's Report
were furnished by the applicants.
A temporary policy for P15,000 was issued to Bernardo Argente and his wife as of May 15, but it was not
delivered to Bernardo Argente until July 2, 1925, when the first quarterly premium on the policy was

By: Elaine Marie G. Laceda

101

INSURANCE LAW

paid. In view of the fact that more than thirty days had elapsed since the applicants were examined by
the company's physician, each of them was required to file a certificate of health before the policy was
delivered to them.
On November 18, 1925, Vicenta de Ocampo died of cerebral apoplexy. Thereafter Bernardo Argente
presented a claim. However, following investigation conducted by the insurer, it was apparently
disclosed that the answers given by the insured in their medical examinations with regard to their health
and previous illness and medical attendance were untrue. For that reason, the West Coast Life Insurance
Co. refused to pay the claim.
The plaintiff, Bernardo Argente, while readily conceding most of the facts claimed, yet alleges that both
he and his wife revealed to the company's physician Dr. Sta. Ana, all the facts concerning the previous
illnesses and medical attendance, but that Dr. Sta. Ana, presumably acting in collusion, with the
insurance agent, Jose Geronimo del Rosario, failed to record them in the medical reports.
TC ruled for the insurer. The court found from the evidence that the representations made by Bernardo
Argente and his wife in their applications to the defendant for life insurance were false with respect to
their state of health during the period of five years preceding the date of such applications, and that
they knew the representations made by them in their applications were false. Further there appears no
motive whatever on the part of Dr. Sta. Ana to falsify the Medical Examiner's Reports and thereby not
only jeopardize his career as a physician, but also gravely implicate himself criminally.
Issue: WON the concealment would avoid the policy.
Held: Yes.
One ground for the rescission of a contract of insurance under the Insurance Act is "a concealment,"
which in section 25 is defined as "A neglect to communicate that which a party knows and ought to
communicate." In an action on a life insurance policy where the evidence conclusively shows that the
answers to questions concerning diseases were untrue, the truth or falsity of the answers become the
determining factor. In the policy which was procured by fraudulent representations, the contract of
insurance apparently set forth therein was never legally existent. It can fairly be assumed that had the
true facts been disclosed by the assured, the insurance would never have been granted.
The basis of the rule vitiating the contract in case of concealment is that it misleads or deceives the
insurer into accepting the risk, or accepting it at the rate of premium agreed upon. The insurer, relying
upon the belief that the assured will disclose every material within his actual or presumed knowledge, is
misled into a belief that the circumstance withheld does not exist, and he is thereby induced to estimate
the risk upon a false basis that it does not exist. It therefore follows that the assurer in assuming a risk
is entitled to know every material fact of which the assured has exclusive or peculiar knowledge, as well
as all material facts which directly tend to increase the hazard or risk which are known by the assured,
or which ought to be or are presumed to be known by him. And a concealment of such facts vitiates the
policy. "It does not seem to be necessary . . . that the . . . suppression of the truth should have been
willful." If it were but an inadvertent omission, yet if it were material to the risk and such as the plaintiff
should have known to be so, it would render the policy void.
Yu Pang Cheng vs. Court of Appeals
G.R. No. L-12465 May 29, 1959
On September 8, 1950, Yu Pang Eng was issued Policy No. 812858. Upon his death on February 27,
1951 caused by "infiltrating medullary carcinoma, Grade 4, advanced cardiac and of lesser curvature,
stomach metastases spleen", the beneficiary of the insured, demanded from the defendant the payment
of the proceeds of the insurance policy. The claim was denied on the ground of concealment.
It appears that the insured entered the Chinese General Hospital for medical treatment on January 29,
1950 having stayed there up to February 11, 1950. Upon entering the hospital, he complained of
dizziness, anemia, abdominal pains and tarry stools, and in the evening of his admission he had several
abdominal pains and his discharges were with black tarry stools and felt dizzy and weak. The history of

By: Elaine Marie G. Laceda

102

INSURANCE LAW

his illness shows that the same "started a year ago as frequent dizziness." An X-Ray picture of his
stomach was taken and the diagnosis made of him by his doctors showed that his illness was "peptic
ulcer, bleeding."
Issue: WON the insured is guilty of concealment of some facts material to the risk insured against which
has the effect of avoiding the policy.
Held: Yes.
Our Insurance law provides that "A neglect to communicate that which a party knows and ought to
communicate, is called concealment". Whether intentional or unintentional, the concealment entitles the
insurer to rescind the contract of insurance. Our law even requires the insured to communicate to the
insurer all facts within his knowledge which are material to the contract and which the other party has
not the means of ascertaining, and the materiality is to be determined not by the event but solely by the
probable and reasonable influence of the facts upon the party to whom the communication is due.
It should be noted that the insured's confinement in the Chinese General Hospital took place from
January 29, 1950 to February 11, 1950, whereas his application for insurance wherein he stated his
answer to the questions propounded to him by the examining physician of defendant was submitted to
defendant on September 5, 1950. It is apparent that when the insured gave his answers regarding his
previous ailment, particularly with regard to "Gastritis, Ulcer of the Stomach or any disease of that
organ" and "Vertigo, Dizziness, Fainting-spells or Unconsciousness", he concealed the ailment of which
he was treated in the Chinese General Hospital which precisely has direct connection with the subject of
the questions propounded. The negative answers given by the insured regarding his previous ailment, or
his concealment of the fact that he was hospitalized and treated for sometime of peptic ulcer and had
suffered from "dizziness, anemia, abdominal pains and tarry stools", deprived defendant of the
opportunity to make the necessary inquiry as to the nature of his past illness so that as it may form its
estimate relative to the approval of his application. Had defendant been given such opportunity,
considering the previous illness of the insured as disclosed by the record of the Chinese General
Hospital, defendant would probably had never consented to the issuance of the policy in question.
Saturnino v. Philippine American Life Ins. Co., supra
Great Pacific Life Assurance Corp. vs. Court of Appeals
G.R. No. 113899 October 13, 1999
A contract of group life insurance was executed between petitioner Grepalife and DBP with Grepalife
agreeing to insure the lives of eligible housing loan mortgagors of DBP. Dr. Wilfredo Leuterio, a
physician and a housing debtor of DBP applied for membership in the group life insurance plan. In an
application form, Dr. Leuterio answered in the negative when asked if he ever had, or consulted, a
physician for a heart condition, high blood pressure, cancer, diabetes, lung; kidney or stomach disorder
or any other physical impairment. Thereafter, Grepalife issued Certificate No. B-18558, as insurance
coverage of Dr. Leuterio, to the extent of his DBP mortgage indebtedness amounting to P86,200.00.
On August 6, 1984, Dr. Leuterio died due to "massive cerebral hemorrhage." Consequently, DBP
submitted a death claim to Grepalife. Grepalife denied the claim alleging that Dr. Leuterio was not
physically healthy when he applied for an insurance coverage on November 15, 1983. Grepalife insisted
that Dr. Leuterio did not disclose he had been suffering from hypertension, which caused his death.
Allegedly, such non-disclosure constituted concealment that justified the denial of the claim.
Consequently, the widow of the late Dr. Leuterio, respondent Medarda V. Leuterio, filed a complaint with
the RTC of Misamis Oriental, against Grepalife for "Specific Performance with Damages."
RTC rendered a decision in favor of respondent widow and against Grepalife. CA sustained the trial
court's decision.
Held: SC affirmed.

By: Elaine Marie G. Laceda

103

INSURANCE LAW

The rationale of a group insurance policy of mortgagors, otherwise known as the "mortgage redemption
insurance," is a device for the protection of both the mortgagee and the mortgagor. On the part of the
mortgagee, it has to enter into such form of contract so that in the event of the unexpected demise of
the mortgagor during the subsistence of the mortgage contract, the proceeds from such insurance will
be applied to the payment of the mortgage debt, thereby relieving the heirs of the mortgagor from
paying the obligation. In a similar vein, ample protection is given to the mortgagor under such a concept
so that in the event of death; the mortgage obligation will be extinguished by the application of the
insurance proceeds to the mortgage indebtedness. Consequently, where the mortgagor pays the
insurance premium under the group insurance policy, making the loss payable to the mortgagee, the
insurance is on the mortgagor's interest, and the mortgagor continues to be a party to the contract. In
this type of policy insurance, the mortgagee is simply an appointee of the insurance fund, such losspayable clause does not make the mortgagee a party to the contract.
The insured private respondent did not cede to the mortgagee all his rights or interests in the insurance,
the policy stating that: "In the event of the debtor's death before his indebtedness with the Creditor
[DBP] shall have been fully paid, an amount to pay the outstanding indebtedness shall first be paid to
the creditor and the balance of sum assured, if there is any, shall then be paid to the beneficiary/ies
designated by the debtor." Since a policy of insurance upon life or health may pass by transfer, will or
succession to any person, whether he has an insurable interest or not, and such person may recover
whatever the insured might have recovered, the widow of the decedent Dr. Leuterio may file the suit
against the insurer, Grepalife.
Petitioner contends that Dr. Leuterio failed to disclose that he had hypertension, which might have
caused his death. Concealment exists where the assured had knowledge of a fact material to the risk,
and honesty, good faith, and fair dealing requires that he should communicate it to the assured, but he
designedly and intentionally withholds the same. The fraudulent intent on the part of the insured must
be established to entitle the insurer to rescind the contract. Misrepresentation as a defense of the insurer
to avoid liability is an affirmative defense and the duty to establish such defense by satisfactory and
convincing evidence rests upon the insurer. In the case at bar, the petitioner failed to clearly and
satisfactorily establish its defense, and is therefore liable to pay the proceeds of the insurance.
A life insurance policy is a valued policy. Unless the interest of a person insured is susceptible of exact
pecuniary measurement, the measure of indemnity under a policy of insurance upon life or health is the
sum fixed in the policy. The policy states that upon receipt of due proof of the Debtor's death during the
terms of this insurance, a death benefit in the amount of P86,200.00 shall be paid. However, on account
of the foreclosure made by DBP, the insurance proceeds shall inure to the benefit of the heirs of the
deceased person or his beneficiaries.
General Insurance & Surety Corp. vs. Ng Hua
G.R. No. L-14373 January 30, 1960
On April 15, 1952, the defendant insurer issued its insurance Policy No. 471, insuring against fire, for
one year, the stock in trade of the Central Pomade Factory owned by Ng Hua. The next day, the Pomade
factory building burned, resulting in destruction by fire of the insured properties. Ng Hua claimed
indemnity from the insurer. Nevertheless, the defendant insurer refused to pay for various reasons,
namely (a) action was not filed in time; (b) violation of warranty; (c) submission of fraudulent claim;
and (f) failure to pay the premium.
The aforesaid Policy No. 471 contains this stipulation on the back thereof: The insured shall give notice
to the company of any insurance or insurances already affected, or which may subsequently be effected,
covering any of the property hereby insured, and unless such notice be given and the particulars of such
insurance or insurances be stated in or endorsed on this Policy by or on behalf of the Company before
the occurrence of any loss or damage, all benefits under the policy shall be forfeited.
Issue: WON there was co-insurance.
Held: Yes.

By: Elaine Marie G. Laceda

104

INSURANCE LAW

Co-insurance exists when a condition of the policy requires the insured to bear ratable proportion of the
loss when the value of the insured property exceeds the face value of the policy. But that is one kind of
co-insurance. It is not the only situation where co-insurance exists. Other insurers of the same property
against the same hazard are sometimes referred as co-insurers and the ensuing combination as coinsurance. And considering the terms of the policy which required the insured to declare other
insurances, the statement in question must be deemed to be a statement (warranty) binding on both
insurer and insured, that there were no other insurance on the property. The annotation must be
deemed to be a warranty that the property was not insured by any other policy. Violation thereof
entitles the insurer to rescind. Such misrepresentation is fatal.
Vda de Canilang vs. Court of Appeals
G.R. No. 92492 June 17, 1993
On August 4, 1982, after being diagnosed of having "acute bronchitis" the day before, Jaime Canilang
applied for a "non-medical" insurance policy with respondent Great Pacific Life Assurance Company
naming his wife, Thelma Canilang, as his beneficiary. Thus he was issued ordinary life insurance, with
the face value of P19,700, effective as of 9 August 1982. On 5 August 1983, Jaime Canilang died of
"congestive heart failure," "anemia," and "chronic anemia." Petitioner, widow and beneficiary of the
insured, filed a claim with Great Pacific which the insurer denied upon the ground that the insured had
concealed material information from it.
Petitioner then filed a complaint against Great Pacific with the Insurance Commission for recovery of the
insurance proceeds. During the hearing, petitioner testified that she was not aware of any serious illness
suffered by her late husband and that, as far as she knew, her husband had died because of a kidney
disorder. Great Pacific for its part presented Dr. Esperanza Quismorio, a physician and a medical
underwriter working for Great Pacific. She testified that the deceased's insurance application had been
approved on the basis of his medical declaration. She explained that as a rule, medical examinations are
required only in cases where the applicant has indicated in his application for insurance coverage that he
has previously undergone medical consultation and hospitalization.
Insurance Commissioner ruled in favor of petitioner after holding that: (a) the ailment of Jaime Canilang
was not so serious that, even if it had been disclosed, it would not have affected Great Pacific's decision
to insure him; (b) Great Pacific had waived its right to inquire into the health condition of the applicant
by the issuance of the policy despite the lack of answers to "some of the pertinent questions" in the
insurance application; (c) there was no intentional concealment on the part of the insured Jaime
Canilang as he had thought that he was merely suffering from a minor ailment and simple cold; and (d)
BP Blg. 847 which voids an insurance contract, whether or not concealment was intentionally made, was
not applicable to Canilang's case as that law became effective only on 1 June 1985. CA reversed.
Issue: WON Jaime Canilang "intentionally" made material concealment in stating his state of health.
Held: Yes.
The information which Jaime Canilang failed to disclose was material to the ability of Great Pacific to
estimate the probable risk he presented as a subject of life insurance. Had Canilang disclosed his visits
to his doctor, the diagnosis made and medicines prescribed by such doctor, in the insurance application,
it may be reasonably assumed that Great Pacific would have made further inquiries and would have
probably refused to issue a non-medical insurance policy or, at the very least, required a higher
premium for the same coverage. The materiality of the information withheld by Great Pacific did not
depend upon the state of mind of Jaime Canilang.
The waiver of medical examination [in a non-medical insurance contract] renders even more material
the information required of the applicant concerning previous condition of health and diseases suffered,
for such information necessarily constitutes an important factor which the insurer takes into
consideration in deciding whether to issue the policy or not.
A concealment whether intentional or unintentional entitles the injured party to rescind a contract of
insurance. The restoration in 1985 by B.P. Blg. 874 of the phrase "whether intentional or unintentional"

By: Elaine Marie G. Laceda

105

INSURANCE LAW

merely underscored the fact that all throughout (from 1914 to 1985), the statute did not require proof
that concealment must be "intentional" in order to authorize rescission by the injured party.
In any case, in the case at bar, the nature of the facts not conveyed to the insurer was such that the
failure to communicate must have been intentional rather than merely inadvertent. For Jaime Canilang
could not have been unaware that his heart beat would at times rise to high and alarming levels and
that he had consulted a doctor twice in the two (2) months before applying for non-medical insurance.
Indeed, the last medical consultation took place just the day before the insurance application was filed.
In all probability, Jaime Canilang went to visit his doctor precisely because of the discomfort and concern
brought about by his experiencing "sinus tachycardia."
Sunlife Assurance Company of Canada v. Court of Appeals, supra
B.

REPRESENTATION

1. Definition and Characteristics


a. Form Sec. 36, I.C.
Sec. 36. A representation may be oral or written.
b. When Made Sec. 37, I.C.
Sec. 37. A representation may be made at the time of, or before, issuance of the policy.
i.

at time of issuance of policy

ii.

before issuance of policy Secs. 41 and 42, I.C.

Sec. 41. A representation may be altered or withdrawn before the insurance is effected, but not
afterwards.
Sec. 42. A representation must be presumed to refer to the date on which the contract goes into effect.
c.

Interpretation Sec. 38, I.C.

Sec. 38. The language of a representation is to be interpreted by the same rules as the language of
contracts in general.
d. Promissory representation; statement of belief or expectation Sec. 39, I.C.
Sec. 39. A representation as to the future is to be deemed a promise, unless it appears that it was
merely a statement of belief or expectation.
e. Modification of warranty Sec. 40, I.C.
Sec. 40. A representation cannot qualify an express provision in a contract of insurance, but it may
qualify an implied warranty.

2.

Falsity Secs. 43 and 44, I.C.

Sec. 43. When a person insured has no personal knowledge of a fact, he may nevertheless repeat
information which he has upon the subject, and which he believes to be true, with the explanation that
he does so on the information of others; or he may submit the information, in its whole extent, to the
insurer; and in neither case is he responsible for its truth, unless it proceeds from an agent of the
insured, whose duty it is to give the information.

By: Elaine Marie G. Laceda

106

INSURANCE LAW

Sec. 44. A representation is to be deemed false when the facts fail to correspond with its assertions or
stipulations.
Harding vs. Commercial Union Assurance Company
G.R. No. L-12707 August 10, 1918
On February 16, 1916, the plaintiff Mrs. Henry E. Harding, the owner of a Studebaker automobile, in
consideration of the payment to the defendant of the premium of P150, was issued an insurance policy
on said automobile (with estimated present value of P3,000) by the defendant through its duly
authorized agent, Smith, Bell & Company. On March 24, 1916, said automobile was totally destroyed by
fire. Thereafter, within the period mentioned in the said policy of insurance, the plaintiff furnished the
defendant the proofs of her said loss and interest, and otherwise performed all the conditions of said
policy on her part, but the defendant has not paid said loss nor any part thereof despite due demand.
TC decided that there was no proof of fraud on the part of plaintiff in her statement of the value of the
automobile, or with respect to its ownership; that she had an insurable interest therein; and that
defendant, having agreed to the estimated value, P3,000, and having insured the automobile for that
amount, upon the basis of which the premium was paid, is bound by it and must pay the loss in
accordance with the stipulated insured value.
Issue: WON the insured is guilty of fraud with regard to the estimated value of the automobile.
Held: No.
It has not been shown by the evidence that the statement regarding the cost of the automobile was
false on the contrary it shows that the automobile had in fact cost more than the amount mentioned.
The court below found, and the evidence shows, that the automobile was bought by plaintiff's husband a
few weeks before the issuance of the policy in question for the sum of P2,800, and that between that
time and the issuance of the policy some P900 was spent upon it in repairs and repainting. The witness
Server, an expert automobile mechanic, testified that the automobile was practically as good as new at
the time the insurance was effected.
Furthermore, the court below found and the evidence shows, without dispute, that the proposal upon
which the policy in question was issued was made out by defendant's agent by whom the insurance was
solicited, and that appellee simply signed the same. It also appears that an examiner employed by the
defendant made an inspection of the automobile before the acceptance of the risk, and that the sum
after this examination. The trial court found that Mrs. Harding, in fixing the value of the automobile at
P3,000, acted upon information given her by her husband and by Mr. Server, the manager of the Luneta
Garage the agent of the defendant corporation in the solicitation of the insurance. Mrs. Harding did not
state of her own knowledge that the automobile originally cost P3,000, or that its value at the time of
the insurance was P3,000. She merely repeated the information which had been given her by her
husband, and at the same time disclosed to defendant's agent the source of her information. There is no
evidence to sustain the contention that this communication was made in bad faith. Thus, the defendant,
upon the information given by plaintiff, and after an inspection of the automobile by its examiner,
having agreed that it was worth P3,000, is bound by this valuation in the absence of fraud on the part of
the insured.

3.

Materiality Sec. 46 in rel. to Sec. 31, I.C.

Sec. 46. The materiality of a representation is determined by the same rules as the materiality of a
concealment.
Sec. 31. Materiality is to be determined not by the event, but solely by the probable and reasonable
influence of the facts upon the party to whom the communication is due, in forming his estimate of the
disadvantages of the proposed contract, or in making his inquiries.
4. Effect of False and Material Representation Sec. 45, I.C.

By: Elaine Marie G. Laceda

107

INSURANCE LAW

Sec. 45. If a representation is false in a material point, whether affirmative or promissory, the injured
party is entitled to rescind the contract from the time when the representation becomes false. The right
to rescind granted by this Code to the insurer is waived by the acceptance of premium payments despite
knowledge of the ground for rescission.
Saturnino v. Philippine American Life Ins. Co., supra
Musgi vs. West Coast Life Insurance Co.
G.R. No. L-41794 August 30, 1935
Arsenio T. Garcia was insured by the defendant company in the sum of P5,000 and P10,000 effective as
of July 25, 1931 and October 20, 1931 respectively. In each of the said applications the following
question was asked: "1. What physician or practitioner or any other person not named above have you
consulted or been treated by, and for what illness, or ailment? " In both applications, the insured
answered "None ". These answers of the insured as well as his other statements contained in his
applications were one of the causes or considerations for the issuance of the policies, and they so
positively appear therein. However, after the death of the insured and as a result of the demand made
by the beneficiaries upon the defendant to pay the value of the policies, the latter discovered that the
aforementioned answers were false and fraudulent, because the truth was that the insured, before
answering and signing the applications and before the issuance of the policies, had been treated in the
General Hospital for different ailments.
The defendant contended at the outset that the two policies did not create any valid obligation because
they were fraudulently obtained by the insured. But the appealed decision holds that the health of the
insured before the acceptance of his applications and the issuance of the policies could neither be
discussed nor questioned by the defendant, because the insured was examined by three physicians of
the company and all of them unanimously certified that he was in good health and that he could be
properly insured.
Issue: WON the two answers given by the insured in his applications are false, and if they were the
cause, or one of the causes, which induced the defendant to issue the policies.
Held: Yes.
The insured knew that he had suffered from a number of ailments, including incipient pulmonary
tuberculosis, before subscribing the applications, yet he concealed them and omitted the hospital where
he was confined as well as the name of the lady physician who treated him. That this concealment and
the false statements constituted fraud, is likewise clear, because the defendant by reason thereof
accepted the risk which it would otherwise have flatly refused.
Article 1261 of the Civil Code provides that there is no contract unless there should be, in addition to
consent and a definite object, a consideration for the obligation established. And article 1276 provides
that the statement of a false consideration shall render the contract void. The two answers being one of
the considerations of the policies, and it appearing that they are false and fraudulent, it is evident that
the insurance contracts were null and void and did not give rise to any right to recover their value or
amount.
Edillon vs. Manila Bankers Life Insurance Corp.
G.R. No. L-34200 September 30, 1982
Sometime in April 1969, Carmen O, Lapuz applied with respondent insurance corporation for insurance
coverage against accident and injuries. She filled up the blank application form given to her and filed the
same with the respondent insurance corporation. In the said application form, she gave the date of her
birth as July 11, 1904. Upon the filing of said application and the payment of the premium on the policy
applied for, the respondent insurance corporation issued to Carmen O. Lapuz its Certificate of Insurance
No. 128866. On May 31, 1969 Carmen died in a vehicular accident in the North Diversion Road.
petitioner Regina L. Edillon, a sister of the insured and who was the named beneficiary in the policy,
filed her claim for the proceeds of the insurance.

By: Elaine Marie G. Laceda

108

INSURANCE LAW

Insurer denied claim relying on a provision contained in the Certificate of Insurance, excluding its
liability to pay claims under the policy in behalf of "persons who are over the age of 60 years". It is
pointed out that the insured being over 60 years of age when she applied for the insurance coverage,
the policy was null and void, and no risk on the part of the respondent insurance corporation had arisen
therefrom.
TC dismissed.
Issue: WON the acceptance by the insurance corporation of the premium and the issuance of the
corresponding certificate of insurance should be deemed a waiver of the exclusionary condition of
coverage stated in the said certificate of insurance.
Held: Yes.
The age of the insured Carmen 0. Lapuz was not concealed to the insurance company. Her application
for insurance coverage which was on a printed form furnished by private respondent and which
contained very few items of information clearly indicated her age of the time of filing the same to be
almost 65 years of age. Despite such information which could hardly be overlooked in the application
form, considering its prominence thereon and its materiality to the coverage applied for, the respondent
insurance corporation received her payment of premium and issued the corresponding certificate of
insurance without question. The accident which resulted in the death of the insured, a risk covered by
the policy, occurred 45 DAYS after the insurance coverage was applied for. There was sufficient time for
the private respondent to process the application and to notice that the applicant was over 60 years of
age and thereby cancel the policy on that ground if it was minded to do so. If the private respondent
failed to act, it is either because it was willing to waive such disqualification; or, through the negligence
or incompetence of its employees for which it has only itself to blame, it simply overlooked such fact.
Under the circumstances, the insurance corporation is already deemed in estoppel.
Gonzalez Lao vs. Yek Tong Lin Fire & Marine Insurance Co., Ltd.
G.R. No. L-33131 December 13, 1930
This is an action to recover from the defendant the Yek Tong Lin Fire & Marine Insurance Co., Ltd., the
amount of two insurance policies totaling P100,000 upon leaf tobacco belonging to the plaintiff, which
was damaged by the fire that destroyed the building on Soler Street No. 188, where said tobacco was
stored. TC ordered payment.
Defendants contend that the TC erred in not declaring that in as much as the plaintiff failed to notify the
defendant corporation in writing, of other insurance policies obtained by him, he has violated article 3 of
the conditions of the policies in question, thereby rendering these policies null and void. Article 3 of the
conditions of the policies in question prescribes: Any insurance in force upon all or part of the things
insured must be declared in writing by the insured and he should cause the company to insert or
mention it in the policy, and without such requisite said policy will be regarded as null and void, and the
assured deprived of all rights of indemnity in case of loss.
Issue: WON the violation of the warranties rendered the policy void.
Held: No.
That the defendant had knowledge of the existence of other policies obtained by the plaintiff from other
insurance companies is specifically shown by the defendant's answer wherein it alleges, by way of
special defense, the fact that there exist other policies issued by the companies mentioned therein. If,
with the knowledge of existence of other insurances which the defendant deemed violations of the
contract, it has preferred to continue the policy, its action amounts to a waiver of the annulment of the
contract
Tan Chay Heng vs. West Coast Life Insurance Co.
G.R. No. L-27541 November 21, 1927

By: Elaine Marie G. Laceda

109

INSURANCE LAW

In April 1925, the defendant insurer accepted and approved a life insurance policy on the life of Tan
Ceang for the sum of P10,000 in which the plaintiff was the sole beneficiary. The defendant agreed to
pay the plaintiff as beneficiary the amount of the policy upon the receipt of the proofs of the death of
the insured while the policy was in force. On May 10, 1925 Tan Ceang died. Thus June 1925, plaintiff
submitted the proofs of the death of Tan Ceang with a claim for the payment of the policy but the
defendant refused to pay claiming, by special defense, that the policy was obtained thru fraud and
deceit as follows
It was falsely represented to the defendant that the said Tan Ceang was single and was a
merchant, and that the plaintiff Tan Chai Heng, the beneficiary, was his nephew, whereas in
truth and in fact the said Tan Ceang was not single but was legally married to Marcelina Patalita
with whom he had several children; and that he was not a merchant but was a mere employee
of another Chinaman by the name of Tan Quina from whom he received only a meager salary,
and that the present plaintiff was not a nephew of the said Tan Ceang.
Dr. V. S. Locsin, in his capacity as medical examiner for the defendant insurance company,
prepared and falsified the necessary medical certificate, in which it was made to appear, among
other things, that the said Tan Ceang had never used morphine, cocaine or any other drug; that
he was then in good health and had never consulted any physician; that he had never spit
blood; and that there was no sign of either present or past disease of his lungs; whereas in truth
and in fact, as the plaintiff and his said coconspirators well knew, the said Tan Ceang was
addicted to morphine, cocaine, and opium and had been convicted and imprisoned therefor, and
was then, and for about three year prior thereto had been suffering from pulmonary
tuberculosis.
To this special defense, the plaintiff, claiming that it was a cross-complaint, filed a general demurrer
upon the ground that it does not state facts sufficient to constitute a cause of defense. TC sustained
plaintiffs demurrer and after trial rendered judgment for the plaintiff for P10,000.
Issue: WON the answer with special defense is a cross-complaint asking for the rescission of the
insurance contract.
Held: No. SC Reversed.
In the instant case, it will be noted that even in its prayer, the defendant does not seek to have the
alleged insurance contract rescinded. It denies that it ever made any contract of insurance on the life of
Tan Ceang or that any such a contract ever existed, and that is the question which it seeks to have
litigated by its special defense. In the very nature of things, if the defendant never made or entered into
the contract in question, there is no contract to rescind, and, hence, section 47 upon which the lower
based its decision in sustaining the demurrer does not apply. As stated, an action to rescind a contract is
founded upon and presupposes the existence of the contract which is sought to be rescinded. If all of the
material matters set forth and alleged in the defendant's special plea are true, there was no valid
contract of insurance, for the simple reason that the minds of the parties never met and never agreed
upon the terms and conditions of the contract. We are clearly of the opinion that, if such matters are
known to exist by a preponderance of the evidence, they would constitute a valid defense to plaintiff's
cause of action. Upon the question as to whether or not they or are not true, we do not at this time have
or express any opinion, but we are clear that section 47 does not apply to the allegations made in the
answer, and that the trial court erred in sustaining the demurrer.
Qua Chee Gan vs. Law Union & Rock Insurance Co., Ltd.
G.R. No. L-4611 December 17, 1955
Qua Chee Gan, a merchant of Albay, owned 4 warehouses or bodegas used for the storage of stocks of
copra and of hemp, baled and loose. They had been, with their contents, insured with the Law Union &
Rock Insurance Co., Ltd. since 1937, and the loss made payable to the PNB as mortgagee of the hemp
and crops, to the extent of its interest.

By: Elaine Marie G. Laceda

110

INSURANCE LAW

Fire of undetermined origin gutted and completely destroyed Bodegas Nos. 1, 2 and 4, with the
merchandise stored theren. The plaintiff submitted the corresponding fire claims however, the Insurance
Company resisted payment, claiming violation of warranties and conditions, filing of fraudulent claims,
and that the fire had been deliberately caused by the insured or by other persons in connivance with
him.
With counsel for the insurance company acting as private prosecutor, Que Chee Gan, with his brother,
Qua Chee Pao, and some employees of his, were indicted and tried for the crime of arson, it being
claimed that they had set fire to the destroyed warehouses to collect the insurance. They were,
however, acquitted by the trial court.
Thereafter, plaintiff instituted action seeking to recover the proceeds of the fire insurance policies. After
trial, the CFI rendered a decision in favor of the plaintiff.
Held: SC affirmed.
It is argued by the Insurance Company that since the bodegas insured had an external wall perimeter of
500 meters or 1,640 feet, the appellee should have 11 fire hydrants in the compound, and that he
actually had only 2, with a further pair nearby, belonging to the municipality of Tabaco.
The appellant is barred by waiver (or rather estoppel) to claim violation of the so-called fire hydrants
warranty, for the reason that knowing fully all that the number of hydrants demanded therein never
existed from the very beginning, the appellant neverthless issued the policies in question subject to such
warranty, and received the corresponding premiums.
Moreover, taking into account the well known rule that ambiguities or obscurities must be strictly
interpreted against the party that caused them, the "memo of warranty" invoked by appellant bars the
latter from questioning the existence of the appliances called for in the insured premises, since its initial
expression, "the undernoted appliances for the extinction of fire being kept on the premises insured
hereby, . . . it is hereby warranted . . .", admits of interpretation as an admission of the existence of
such appliances which appellant cannot now contradict, should the parol evidence rule apply.
As to maintenance of a trained fire brigade of 20 men, the record is preponderant that the same was
organized, and drilled, from time to give, although not maintained as a permanently separate unit,
which the warranty did not require. Anyway, it would be unreasonable to expect the insured to maintain
for his compound alone a fire fighting force that many municipalities in the Islands do not even possess.
Under the second assignment of error, appellant insurance company avers, that the insured violated the
"Hemp Warranty" provisions against the storage of gasoline, since appellee admitted that there were 36
cans of gasoline in the building designed as "Bodega No. 2" that was a separate structure not affected
by the fire.
It is well to note that gasoline is not specifically mentioned among the prohibited articles listed in the socalled "hemp warranty." The cause relied upon by the insurer speaks of "oils, animal and/or vegetable
and/or mineral and/or their liquid products having a flash point below 300o Fahrenheit", and is decidedly
ambiguous and uncertain; for in ordinary parlance, "Oils" mean "lubricants" and not gasoline or
kerosene. Thus, by reason of the exclusive control of the insurance company over the terms and
phraseology of the contract, the ambiguity must be held strictly against the insurer and liberally in favor
of the insured, especially to avoid forfeiture.
Furthermore, it is well settled that even though there are printed prohibitions against keeping certain
articles on the insured premises the policy will not be avoided by a violation of these prohibitions, if the
prohibited articles are necessary or in customary use in carrying on the trade or business conducted on
the premises.
The next two defenses pleaded by the insurer, that the insured connived at the loss and that the
fraudulently inflated the quantity of the insured stock in the burnt bodegas, are closely related to
each other. Both defenses are predicted on the assumption that the insured was in financial difficulties
and set the fire to defraud the insurance company, presumably in order to pay off the Philippine National

By: Elaine Marie G. Laceda

111

INSURANCE LAW

Bank, to which most of the insured hemp and copra was pledged. Both defenses are fatally undermined
by the established fact that, notwithstanding the insurer's refusal to pay the value of the policies the
extensive resources of the insured enabled him to pay off the National Bank in a short time; and if he
was able to do so, no motive appears for attempt to defraud the insurer.
While the acquittal of the insured in the arson case is not res judicata on the present civil action, the
insurer's evidence, to judge from the decision in the criminal case, is practically identical in both cases
and must lead to the same result, since the proof to establish the defense of connivance at the fire in
order to defraud the insurer "cannot be materially less convincing than that required in order to convict
the insured of the crime of arson".
C.

WARRANTIES

1. Definition and Characteristics


a. Kinds
i.

Express or Implied Sec 67, I.C.

Sec. 67. A warranty is either expressed or implied.


ii.

Affirmative or Promissory

b. To what warranties relate Sec 68, I.C.


Sec. 68. A warranty may relate to the past, the present, the future, or to any or all of these.
c.

Form Sec 69, I.C.

Sec. 69. No particular form of words is necessary to create a warranty.

2.

Express Warranties Sec. 70, I.C.

Sec. 70. Without prejudice to section fifty-one, every express warranty, made at or before the
execution of a policy, must be contained in the policy itself, or in another instrument signed by the
insured and referred to in the policy as making a part of it.
Ang Giok Chip vs. Springfield Fire & Marine Insurance Co.
G.R. No. L-33637 December 31, 1931
Ang Giok Chip doing business under the name and style of Hua Bee Kong Si was formerly the owner of a
warehouse. The contents of the warehouse were insured with the three insurance companies for the
total sum of P60,000. One insurance policy, in the amount of P10,000, was taken out with the
Springfield Fire & Marine Insurance Company. The warehouse was destroyed by fire on January 11,
1928, while the policy issued by the latter company was in force. By virtue of the policy, plaintiff sought
the proceeds from defendant insurance company.
Held:
Section 65 of the Insurance Act and its counterpart, section 265 of the Civil Code of California, will bear
analysis as tested by reason and authority. The law says that every express warranty must be
"contained in the policy itself." The word "contained," according to the dictionaries, means "included,"
inclosed," "embraced," "comprehended," etc. When, therefore, the courts speak of a rider attached to
the policy, and thus "embodied" therein, or of a warranty "incorporated" in the policy, it is believed that
the phrase "contained in the policy itself" must necessarily include such rider and warranty. As to the
alternative relating to "another instrument," "instrument" as here used could not mean a mere slip of
paper like a rider, but something akin to the policy itself, which in section 48 of the Insurance Act is
defined as "The written instrument, in which a contract of insurance is set forth." In California, every

By: Elaine Marie G. Laceda

112

INSURANCE LAW

paper writing is not necessarily an "instrument" within the statutory meaning of the term. The word
"instrument has a well defined definition in California, and as used in the Codes invariably means some
written paper or instrument signed and delivered by one person to another, transferring the title to, or
giving a lien, on property, or giving a right to debt or duty.
In other words, the rider, warranty F, is contained in the policy itself, because by the contract of
insurance agreed to by the parties it is made to form a part of the same, but is not another instrument
signed by the insured and referred to in the policy as forming a part of it.
It is admitted that the policy before us was accepted by the plaintiff. The receipt of this policy by the
insured without objection binds both the acceptor and the insured to the terms thereof. The insured may
not thereafter be heard to say that he did not read the policy or know its terms, since it is his duty to
read his policy and it will be assumed that he did so.
3. Affirmative Warranties Sec. 71, I.C.
Sec. 71. A statement in a policy of matter relating to the person or thing insured, or to the risk, as a
fact, is an express warranty thereof.
4. Promissory Warranties Secs. 72 and 73, I.C.
Sec. 72. A statement in a policy which imparts that it is intended to do or not to do a thing which
materially affects the risk, is a warranty that such act or omission shall take place.
Sec. 73. When, before the time arrives for the performance of a warranty relating to the future, a loss
insured against happens, or performance becomes unlawful at the place of the contract, or impossible,
the omission to fulfill the warranty does not avoid the policy.
5. Effect of Breach

a.

Breach of material warranty or provision Sec. 74, I.C.

Sec. 74. The violation of a material warranty, or other material provision of a policy, on the part of
either party thereto, entitles the other to rescind.
Young vs. Midland Textile Insurance Co.
G.R. No. L-9370 March 31, 1915
K.S. Young had a candy and fruit store on the Escolta, Manila, and occupied a building at 321 Calle
Claveria, as a residence and bodega. The Midland Textile Insurance Co. in consideration of the payment
of a premium of P60, entered into a contract of insurance with Young by the terms of which the
company, upon certain conditions, promised to pay Young the sum of P3,000 in case said residence and
bodega and contents should be destroyed by fire. One of the conditions of the contract is: "Warranty B
It is hereby declared and agreed that during the pendency of this policy no hazardous goods be stored
or kept for sale, and no hazardous trade or process be carried on, in the building to which this insurance
applies, or in any building connected therewith."
Young placed in the residence and bodega three boxes filled with fireworks. The fireworks had been
given to Young by the former owner of the Luneta Candy Store. He intended to use them in the
celebration of the Chinese New Year. However, the authorities of the city of Manila had prohibited the
use of fireworks on said occasion, so Young then placed them in the bodega where they remained from
the 4th or 5th of February, 1913 until after the fire of March 18, 1913.
Both parties agree that the fireworks come within the phrase "hazardous goods," mentioned in
"Warranty B" of the policy; that the fireworks were found in a part of the building not destroyed by the
fire and that they in no way contributed to the fire, or to the loss that resulted.
The lower court rendered a judgment in favor of Young for the sum of P2,708.78, and costs.

By: Elaine Marie G. Laceda

113

INSURANCE LAW

Issue: WON the placing of the fireworks in the building insured, they being "hazardous goods," was a
violation of the terms of the contract of insurance and especially of "Warranty B."
Held: Yes.
It is admitted by both parties that the fireworks are hazardous goods. The defendant alleged that they
were "stored." The plaintiff contends that under all the facts and circumstances of the case, they were
not stored in said building, and that the placing of them in the building was not a violation of the terms
of the contract.
Whether a particular article is "stored" or not must, in some degree, depend upon the intention of the
parties. Nearly all of the cases cited by the lower court are cases where the article was being put to
some reasonable and actual use, which might easily have been permitted by the terms of the policy, and
within the intention of the parties, and excepted from the operation of the warranty, like the present.
(1) Where merchants have had or kept the "hazardous" articles in small quantities, and for actual daily
use, for sale, such as gasoline, gunpowder, etc.; (2) Where such articles have been brought on the
premises for actual use thereon, and in small quantities, such as oil, paints, etc; and (3) Where such
articles or goods were used for lighting purposes, and in small quantities.
In the present case no claim is made that the "hazardous goods" were placed in the bodega for present
or daily use. It is admitted that they were placed in the bodega "for future use," or for future
consumption, or for safe keeping. It seems clear to us that the "hazardous goods" in question were
"stored" in the bodega, as that word is generally defined. That being true, suppose the defendant had
made an examination of the premises, even in the absence of a fire, and had found the "hazardous
goods" there, would it not have been justified in declaring the policy null and of no effect by reason of a
violation of its terms? If it might, then may it not repudiate its liability, even after the fire? If the
"warranty" is a term of the contract, will not its violation cause a breach and justify noncompliance or
repudiation?
Contracts of insurance are contracts of indemnity, upon the terms and conditions specified therein.
Parties have a right to impose such reasonable conditions at the time of the making of the contract as
they deem wise and necessary. The rate of premium is measured by the character of the risk assumed.
The insurer, for a comparatively small consideration, undertakes to guarantee the insured against loss
or damage, upon the terms and conditions agreed upon, and upon no other. When the insurer is called
upon to pay, in case of loss, he may justly insist upon a fulfillment of the terms of the contract. If the
insured, cannot bring himself within the terms and conditions of the contract, he is not entitled to
recover for any loss suffered. The terms of the contract constitute the measure of the insurer's liability.
If the contract has been terminated, by a violation of its terms on the part of the insured, there can be
no recovery. Compliance with the terms of the contract is a condition precedent to the right of recovery.
Young argues that since the "storing" of the fireworks on the premises did not contribute in any way to
the damage occasioned by the fire, he should be permitted to recover. That argument, however, is
beside the question, if the "storing" was a violation of the terms of the contract. The violation of the
terms of the contract, by virtue of the provisions of the policy itself, terminated, at the election of either
party, the contractual relations.
Young paid a premium based upon the risk at the time the policy was issued. Certainly, the placing of
the firecrackers in the building insured increased the risk. Young had not paid a premium based upon
the increased risk, neither had the defendant issued a policy upon the theory of a different risk. He was
enjoying, if his contention may be allowed, the benefits of an insurance policy upon one risk, whereas,
as a matter of fact, it was issued upon an entirely different risk. The defendant had neither been paid
nor had issued a policy to cover the increased risk. An increase of risk which is substantial and which is
continued for a considerable period of time, is a direct and certain injury to the insurer, and changes the
basis upon which the contract of insurance rests.
Qua Chee Gan vs. Law Union & Rock Insurance Co., Ltd.

By: Elaine Marie G. Laceda

114

INSURANCE LAW

G.R. No. L-4611 December 17, 1955


Qua Chee Gan, a merchant of Albay, owned 4 warehouses or bodegas used for the storage of stocks of
copra and of hemp, baled and loose. They had been, with their contents, insured with the Law Union &
Rock Insurance Co., Ltd. since 1937, and the loss made payable to the PNB as mortgagee of the hemp
and crops, to the extent of its interest.
Fire of undetermined origin gutted and completely destroyed Bodegas Nos. 1, 2 and 4, with the
merchandise stored theren. The plaintiff submitted the corresponding fire claims however, the Insurance
Company resisted payment, claiming violation of warranties and conditions, filing of fraudulent claims,
and that the fire had been deliberately caused by the insured or by other persons in connivance with
him.
With counsel for the insurance company acting as private prosecutor, Que Chee Gan, with his brother,
Qua Chee Pao, and some employees of his, were indicted and tried for the crime of arson, it being
claimed that they had set fire to the destroyed warehouses to collect the insurance. They were,
however, acquitted by the trial court.
Thereafter, plaintiff instituted action seeking to recover the proceeds of the fire insurance policies. After
trial, the CFI rendered a decision in favor of the plaintiff.
Held: SC affirmed.
It is argued by the Insurance Company that since the bodegas insured had an external wall perimeter of
500 meters or 1,640 feet, the appellee should have 11 fire hydrants in the compound, and that he
actually had only 2, with a further pair nearby, belonging to the municipality of Tabaco.
The appellant is barred by waiver (or rather estoppel) to claim violation of the so-called fire hydrants
warranty, for the reason that knowing fully all that the number of hydrants demanded therein never
existed from the very beginning, the appellant neverthless issued the policies in question subject to such
warranty, and received the corresponding premiums.
Moreover, taking into account the well known rule that ambiguities or obscurities must be strictly
interpreted against the party that caused them, the "memo of warranty" invoked by appellant bars the
latter from questioning the existence of the appliances called for in the insured premises, since its initial
expression, "the undernoted appliances for the extinction of fire being kept on the premises insured
hereby, . . . it is hereby warranted . . .", admits of interpretation as an admission of the existence of
such appliances which appellant cannot now contradict, should the parol evidence rule apply.
As to maintenance of a trained fire brigade of 20 men, the record is preponderant that the same was
organized, and drilled, from time to give, although not maintained as a permanently separate unit,
which the warranty did not require. Anyway, it would be unreasonable to expect the insured to maintain
for his compound alone a fire fighting force that many municipalities in the Islands do not even possess.
Under the second assignment of error, appellant insurance company avers, that the insured violated the
"Hemp Warranty" provisions against the storage of gasoline, since appellee admitted that there were 36
cans of gasoline in the building designed as "Bodega No. 2" that was a separate structure not affected
by the fire.
It is well to note that gasoline is not specifically mentioned among the prohibited articles listed in the socalled "hemp warranty." The cause relied upon by the insurer speaks of "oils, animal and/or vegetable
and/or mineral and/or their liquid products having a flash point below 300o Fahrenheit", and is decidedly
ambiguous and uncertain; for in ordinary parlance, "Oils" mean "lubricants" and not gasoline or
kerosene. Thus, by reason of the exclusive control of the insurance company over the terms and
phraseology of the contract, the ambiguity must be held strictly against the insurer and liberally in favor
of the insured, especially to avoid forfeiture.

By: Elaine Marie G. Laceda

115

INSURANCE LAW

Furthermore, it is well settled that even though there are printed prohibitions against keeping certain
articles on the insured premises the policy will not be avoided by a violation of these prohibitions, if the
prohibited articles are necessary or in customary use in carrying on the trade or business conducted on
the premises.
The next two defenses pleaded by the insurer, that the insured connived at the loss and that the
fraudulently inflated the quantity of the insured stock in the burnt bodegas, are closely related to
each other. Both defenses are predicted on the assumption that the insured was in financial difficulties
and set the fire to defraud the insurance company, presumably in order to pay off the Philippine National
Bank, to which most of the insured hemp and copra was pledged. Both defenses are fatally undermined
by the established fact that, notwithstanding the insurer's refusal to pay the value of the policies the
extensive resources of the insured enabled him to pay off the National Bank in a short time; and if he
was able to do so, no motive appears for attempt to defraud the insurer.
While the acquittal of the insured in the arson case is not res judicata on the present civil action, the
insurer's evidence, to judge from the decision in the criminal case, is practically identical in both cases
and must lead to the same result, since the proof to establish the defense of connivance at the fire in
order to defraud the insurer "cannot be materially less convincing than that required in order to convict
the insured of the crime of arson".

b.

Breach of immaterial provision Sec. 75, I.C.

Sec. 75. A policy may declare that a violation of specified provisions thereof shall avoid it, otherwise the
breach of an immaterial provision does not avoid the policy.
General Insurance & Surety Corp. vs. Ng Hua
G.R. No. L-14373 January 30, 1960
On April 15, 1952, the defendant insurer issued its insurance Policy No. 471, insuring against fire, for
one year, the stock in trade of the Central Pomade Factory owned by Ng Hua. The next day, the Pomade
factory building burned, resulting in destruction by fire of the insured properties. Ng Hua claimed
indemnity from the insurer. Nevertheless, the defendant insurer refused to pay for various reasons,
namely (a) action was not filed in time; (b) violation of warranty; (c) submission of fraudulent claim;
and (f) failure to pay the premium.
The aforesaid Policy No. 471 contains this stipulation on the back thereof: The insured shall give notice
to the company of any insurance or insurances already affected, or which may subsequently be effected,
covering any of the property hereby insured, and unless such notice be given and the particulars of such
insurance or insurances be stated in or endorsed on this Policy by or on behalf of the Company before
the occurrence of any loss or damage, all benefits under the policy shall be forfeited.
Issue: WON there was co-insurance.
Held: Yes.
Co-insurance exists when a condition of the policy requires the insured to bear ratable proportion of the
loss when the value of the insured property exceeds the face value of the policy. But that is one kind of
co-insurance. It is not the only situation where co-insurance exists. Other insurers of the same property
against the same hazard are sometimes referred as co-insurers and the ensuing combination as coinsurance. And considering the terms of the policy which required the insured to declare other
insurances, the statement in question must be deemed to be a statement (warranty) binding on both
insurer and insured, that there were no other insurance on the property. The annotation must be
deemed to be a warranty that the property was not insured by any other policy. Violation thereof
entitles the insurer to rescind. Such misrepresentation is fatal.
c.

Breach of warranty without fraud Sec. 76, I.C.

By: Elaine Marie G. Laceda

116

INSURANCE LAW

Sec. 76. A breach of warranty without fraud merely exonerates an insurer from the time that it occurs,
or where it is broken in its inception, prevents the policy from attaching to the risk.
D.

OTHER DEVICES

1. Conditions
2. Exceptions, Exclusions, or Exemptions
E.

INCONTESTABLE CLAUSE

1. Rescission of Insurance Contract (for Life and Non-Life Insurance) Sec. 48, par. 1,
I.C.
Sec. 48. Whenever a right to rescind a contract of insurance is given to the insurer by any provision of
this chapter, such right must be exercised previous to the commencement of an action on the contract.
a. Time to rescind
Tan Chay Heng vs. West Coast Life Insurance Co.
G.R. No. L-27541 November 21, 1927
In April 1925, the defendant insurer accepted and approved a life insurance policy on the life of Tan
Ceang for the sum of P10,000 in which the plaintiff was the sole beneficiary. The defendant agreed to
pay the plaintiff as beneficiary the amount of the policy upon the receipt of the proofs of the death of
the insured while the policy was in force. On May 10, 1925 Tan Ceang died. Thus June 1925, plaintiff
submitted the proofs of the death of Tan Ceang with a claim for the payment of the policy but the
defendant refused to pay claiming, by special defense, that the policy was obtained thru fraud and
deceit as follows
It was falsely represented to the defendant that the said Tan Ceang was single and was a
merchant, and that the plaintiff Tan Chai Heng, the beneficiary, was his nephew, whereas in
truth and in fact the said Tan Ceang was not single but was legally married to Marcelina Patalita
with whom he had several children; and that he was not a merchant but was a mere employee
of another Chinaman by the name of Tan Quina from whom he received only a meager salary,
and that the present plaintiff was not a nephew of the said Tan Ceang.
Dr. V. S. Locsin, in his capacity as medical examiner for the defendant insurance company,
prepared and falsified the necessary medical certificate, in which it was made to appear, among
other things, that the said Tan Ceang had never used morphine, cocaine or any other drug; that
he was then in good health and had never consulted any physician; that he had never spit
blood; and that there was no sign of either present or past disease of his lungs; whereas in truth
and in fact, as the plaintiff and his said coconspirators well knew, the said Tan Ceang was
addicted to morphine, cocaine, and opium and had been convicted and imprisoned therefor, and
was then, and for about three year prior thereto had been suffering from pulmonary
tuberculosis.
To this special defense, the plaintiff, claiming that it was a cross-complaint, filed a general demurrer
upon the ground that it does not state facts sufficient to constitute a cause of defense. TC sustained
plaintiffs demurrer and after trial rendered judgment for the plaintiff for P10,000.
Issue: WON the answer with special defense is a cross-complaint asking for the rescission of the
insurance contract.
Held: No. SC Reversed.

By: Elaine Marie G. Laceda

117

INSURANCE LAW

In the instant case, it will be noted that even in its prayer, the defendant does not seek to have the
alleged insurance contract rescinded. It denies that it ever made any contract of insurance on the life of
Tan Ceang or that any such a contract ever existed, and that is the question which it seeks to have
litigated by its special defense. In the very nature of things, if the defendant never made or entered into
the contract in question, there is no contract to rescind, and, hence, section 47 upon which the lower
based its decision in sustaining the demurrer does not apply. As stated, an action to rescind a contract is
founded upon and presupposes the existence of the contract which is sought to be rescinded. If all of the
material matters set forth and alleged in the defendant's special plea are true, there was no valid
contract of insurance, for the simple reason that the minds of the parties never met and never agreed
upon the terms and conditions of the contract. We are clearly of the opinion that, if such matters are
known to exist by a preponderance of the evidence, they would constitute a valid defense to plaintiff's
cause of action. Upon the question as to whether or not they or are not true, we do not at this time have
or express any opinion, but we are clear that section 47 does not apply to the allegations made in the
answer, and that the trial court erred in sustaining the demurrer.
b. Effect of failure to rescind before commencement of action
c. Waiver of right to rescind

2.

Incontestible Clause (for Life Insurance Only) Sec. 48, par. 2, and Secs. 227(b),
228(b), and 230(b), I.C.

Sec. 48. x x x
After a policy of life insurance made payable on the death of the insured shall have been in force
during the lifetime of the insured for a period of two years from the date of its issue or of its last
reinstatement, the insurer cannot prove that the policy is void ab initio or is rescindible by reason of the
fraudulent concealment or misrepresentation of the insured or his agent.
Sec. 227. In the case of individual life or endowment insurance, the policy shall contain in substance
the following conditions:
xxx
(b) A provision that the policy shall be incontestable after it shall have been in force during the
lifetime of the insured for a period of two years from its date of issue as shown in the policy, or date of
approval of last reinstatement, except for non-payment of premium and except for violation of the
conditions of the policy relating to military or naval service in time of war;
Sec. 228. No policy of group life insurance shall be issued and delivered in the Philippines unless it
contains in substance the following provisions, or provisions which in the opinion of the Commissioner
are more favorable to the persons insured, or at least as favorable to the persons insured and more
favorable to the policy-holders:
xxx
(b) A provision that the validity of the policy shall not be contested, except for non-payment of
premiums after it has been in force for two years from its date of issue; and that no statement made by
any insured under the policy relating to his insurability shall be used in contesting the validity of the
insurance with respect to which such statement was made after such insurance has been in force prior
to the contest for a period of two years during such person's lifetime nor unless contained in written
instrument signed by him;
Sec. 230. In the case of industrial life insurance, the policy shall contain in substance the following
provisions:
xxx
(b) A provision that the policy shall be incontestable after it has been in force during the lifetime
of the insured for a specified period, not more than two years from its date of issue, except for nonpayment of premiums and except for violation of the conditions of the policy relating to naval or military
service, or services auxiliary thereto, and except as to provisions relating to benefits in the event of
disability as defined in the policy, and those granting additional insurance specifically against death by
accident or by accidental means, or to additional insurance against loss of, or loss of use of, specific
members of the body;

By: Elaine Marie G. Laceda

118

INSURANCE LAW

a. Requisites of incontestability
Tan vs. Court of Appeals
G.R. No. 48049 June 29, 1989
On September 23,1973, Tan Lee Siong, father of herein petitioners, applied for life insurance in the
amount of P 80,000.00 with respondent company. Said application was approved and was issued
effective November 6, 1973. On April 26,1975, Tan Lee Siong died of hepatoma. Petitioners then filed
with respondent company their claim for the proceeds of the life insurance policy. Respondent company
denied petitioners' claim and rescinded the policy by reason of the alleged misrepresentation and
concealment of material facts made by the deceased Tan Lee Siong in his application for insurance. The
premiums paid on the policy were thereupon refunded. Petitioners filed on November 27, 1975, a
complaint against the former with the Office of the Insurance Commissioner. Commissioner denied
petition. CA affirmed Commissioners decision
Issue: WON according to Sec. 48 of the Insurance Code, insurance company is barred from rescinding
contract
Held:
Section 48. Whenever a right to rescind a contract of insurance is given to the insurer by any provision
of this chapter, such right must be exercised previous to the commencement of an action on the
contract.
After a policy of life insurance made payable on the death of the insured shall have been in force during
the lifetime of the insured for a period of two years from the date of its issue or of its last reinstatement,
the insurer cannot prove that the policy is void ab initio or is rescindable by reason of the fraudulent
concealment or misrepresentation of the insured or his agent.
According to the petitioners, the Insurance Law was amended and the second paragraph of Section 48
added to prevent the insurance company from exercising a right to rescind after the death of the insured
The so-called "incontestability clause" precludes the insurer from raising the defenses of false
representations or concealment of material facts insofar as health and previous diseases are concerned
if the insurance has been in force for at least two years during the insured's lifetime. The phrase "during
the lifetime" found in Section 48 simply means that the policy is no longer considered in force after the
insured has died. The key phrase in the second paragraph of Section 48 is "for a period of two years."
The policy was issued on November 6,1973 and the insured died on April 26,1975. The policy was thus
in force for a period of only one year and five months. Considering that the insured died before the twoyear period had lapsed, respondent company is not, therefore, barred from proving that the policy is
void ab initio by reason of the insured's fraudulent concealment or misrepresentation.
The petitioners contend that there could have been no concealment or misrepresentation by their late
father because Tan Lee Siong did not have to buy insurance. He was only pressured by insistent
salesmen to do so
The legislative answer to the arguments posed by the petitioners is the "incontestability clause" added
by the second paragraph of Section 48. The insurer has two years from the date of issuance of the
insurance contract or of its last reinstatement within which to contest the policy, whether or not, the
insured still lives within such period. After two years, the defenses of concealment or misrepresentation,
no matter how patent or well founded, no longer lie.
PhilamCare Health Systems, Inc. vs. Court of Appeals
G.R. No. 125678 March 18, 2002
Ernani Trinos, deceased husband of respondent Julita Trinos, applied for a health care coverage with
petitioner Philamcare Health Systems, Inc. In the standard application form, he stated that he was

By: Elaine Marie G. Laceda

119

INSURANCE LAW

never treated for high blood pressure, heart trouble, diabetes, cancer, liver disease, asthma or peptic
ulcer.
Under the agreement, respondents husband was entitled to avail of hospitalization benefits, whether
ordinary or emergency, listed therein. He was also entitled to avail of "out-patient benefits" such as
annual physical examinations, preventive health care and other out-patient services.
During the period of his coverage, Ernani suffered a heart attack and was confined at the Manila Medical
Center (MMC) for one month. While her husband was in the hospital, respondent tried to claim the
benefits under the health care agreement. However, petitioner denied her claim saying that the Health
Care Agreement was void. According to petitioner, there was a concealment regarding Ernanis medical
history. Doctors at the MMC allegedly discovered at the time of Ernanis confinement that he was
hypertensive, diabetic and asthmatic, contrary to his answer in the application form.
After Ernanis death, Julita Trinos filed an action for damages where she asked for reimbursement of her
expenses plus moral damages and attorneys fees.
RTC rendered judgment in favor of the plaintiff ordering defendants to pay and reimburse the medical
and hospital coverage of the late Ernani Trinos, and to pay moral and exemplary damages plus
attorneys fees and cost of suit. CA affirmed.
Held: SC Affirmed.
An insurance contract exists where the following elements concur:
1. The insured has an insurable interest;
2. The insured is subject to a risk of loss by the happening of the designated peril;
3. The insurer assumes the risk;
4. Such assumption of risk is part of a general scheme to distribute actual losses among a large
group of persons bearing a similar risk; and
5. In consideration of the insurers promise, the insured pays a premium.
Section 10 provides that: Every person has an insurable interest in the life and health
1. of himself, of his spouse and of his children;
2. of any person on whom he depends wholly or in part for education or support, or in whom he
has a pecuniary interest;
3. of any person under a legal obligation to him for the payment of money, respecting property or
service, of which death or illness might delay or prevent the performance; and
4. of any person upon whose life any estate or interest vested in him depends.
In the case at bar, the insurable interest of respondents husband in obtaining the health care
agreement was his own health. The health care agreement was in the nature of non-life insurance,
which is primarily a contract of indemnity. Once the member incurs hospital, medical or any other
expense arising from sickness, injury or other stipulated contingent, the health care provider must pay
for the same to the extent agreed upon under the contract.
Having assumed a responsibility under the agreement, petitioner is bound to answer the same to the
extent agreed upon. In the end, the liability of the health care provider attaches once the member is
hospitalized for the disease or injury covered by the agreement or whenever he avails of the covered
benefits which he has prepaid.
Under Section 27 of the Insurance Code, "a concealment entitles the injured party to rescind a contract
of insurance." However, the right to rescind should be exercised previous to the commencement of an
action on the contract. In this case, no rescission was made. Besides, the cancellation of health care
agreements as in insurance policies require the concurrence of certain conditions, none of which was
fulfilled in this case.
1. Prior notice of cancellation to insured;
2. Notice must be based on the occurrence after effective date of the policy of one or more of the
grounds mentioned;
3. Must be in writing, mailed or delivered to the insured at the address shown in the policy;

By: Elaine Marie G. Laceda

120

INSURANCE LAW

4.

Must state the grounds relied upon provided in Section 64 of the Insurance Code and upon
request of insured, to furnish facts on which cancellation is based.

The defendant Philamcare Health Systems Inc. had twelve months from the date of issuance of the
Agreement within which to contest the membership of the patient if he had previous ailment of asthma,
and six months from the issuance of the agreement if the patient was sick of diabetes or hypertension.
The periods having expired, the defense of concealment or misrepresentation no longer lie.
Lastly, petitioner alleges that respondent was not the legal wife of the deceased member considering
that at the time of their marriage, the deceased was previously married to another woman who was still
alive. However, the health care agreement is in the nature of a contract of indemnity. Hence, payment
should be made to the party who incurred the expenses.
b. Effects and purpose of incontestability
c. Defenses not barred by incontestability
VII.
A.

LOSS AND NOTICE OF LOSS


LOSS

1. Concepts and Definitions


a. Loss
b. Proximate Cause
c. Remote Cause
d. Immediate Cause
e. Peril Insured Against
2. Losses for which Insurer Liable
a. Loss of which a peril insured against was the proximate cause Sec. 84, I.C.
Sec. 84. Unless otherwise provided by the policy, an insurer is liable for a loss of which a peril insured
against was the proximate cause, although a peril not contemplated by the contract may have been a
remote cause of the loss; but he is not liable for a loss which the peril insured against was only a remote
cause.
b. Loss caused by a peril not insured against to which the thing insured was
exposed in the course of rescuing the same from the peril insured against
Sec. 85, I.C.
Sec. 85. An insurer is liable where the thing insured is rescued from a peril insured against that would
otherwise have caused a loss, if, in the course of such rescue, the thing is exposed to a peril not insured
against, which permanently deprives the insured of its possession, in whole or in part; or where a loss is
caused by efforts to rescue the thing insured from a peril insured against.

c.

Loss caused by efforts to rescue the thing insured from a peril insured
against Sec. 85, I.C., supra

d. Loss, the immediate cause of which was the peril insured against, if the
proximate cause thereof was not excepted in the contract Sec. 86, I.C.
Sec. 86. Where a peril is especially excepted in a contract of insurance, a loss, which would not have
occurred but for such peril, is thereby excepted although the immediate cause of the loss was a peril
which was not excepted.
Paris-Manila Perfume Co. vs. Phoenix Assurance Co.
G.R. No. L-25845 December 17, 1926

By: Elaine Marie G. Laceda

121

INSURANCE LAW

Phoenix issued a fire insurance policy covering the properties of insured. On a relevant date, a fire broke
out and destroyed the properties of the insured. The insured duly made a claim against Phoenix and was
denied. One of the grounds asserted by Phoenix is that the policy was not in the name of the company
but in the name of one Peter Johnson. Another ground raised is that the policy doesn't cover explosions.
The trial court however overruled its defenses and ruled in favor of the insured.
Held:
The factory where the fire occurred was filed with numerous kinds of essences and oils used in the
manufacture of perfumery and with a quantity of alcohol and manufactured perfumes, all of which were
of a highly inflammable nature, and the fire may have started from any one of a number of reasons. But
in the final analysis, the fact remains that there was a fire, and that the plaintiffs property was
destroyed. It is true that it may be that the explosion was the primary cause of the fire, but that is only
a matter of conjecture, and upon that point, the burden of proof was upon the defendant.
It will be noted that section 5 of the subject policy excludes not only the damages which may
immediately result from an earthquake, but also any damage which may follow the earthquake, and that
section 6 excludes only the damages which are the direct result of the explosion itself, and that it does
not except damages which occurred from the fire occuring after the explosion, even though the
explosion may have been the primary cause of the fire. But assuming, without deciding, that if it be a
fact that the fire resulted from an explosion that fact, if proven, would be a complete defense, the
burden of the proof of that fact is upon the defendant, and upon that point, there is a failure of proof.
There is no competent evidence as to whether the explosion caused the fire or the fire caused the
explosion.
e. Loss caused by negligence of the insured Sec. 87, I.C.
Sec. 87. An insurer is not liable for a loss caused by the willful act or through the connivance of the
insured; but he is not exonerated by the negligence of the insured, or of the insurance agents or others.
FGU Insurance Corporation vs. Court of Appeals
G.R. No. 137775 March 31, 2005
ANCO, a partnership between Ang Gui and Co To, was engaged in the shipping business. It owned the
M/T ANCO tugboat and the D/B Lucio barge which were operated as common carriers. On 23
September 1979, SMC shipped from Mandaue City, Cebu, on board the D/B Lucio, for towage by M/T
ANCO, cargoes of beer. When the barge and tugboat arrived at San Jose, Antique, in the afternoon of 30
September 1979, the clouds over the area were dark and the waves were already big. The arrastre
workers unloading the cargoes of SMC on board the D/B Lucio began to complain about their difficulty in
unloading the cargoes. SMCs District Sales Supervisor, Fernando Macabuag, requested ANCOs
representative to transfer the barge to a safer place because the vessel might not be able to withstand
the big waves. But ANCOs representative did not heed the request because he was confident that the
barge could withstand the waves. Thus, with the waves growing bigger and bigger, only 10,790 cases
of beer were discharged into the custody of the arrastre operator. At about 10 pm of 01 October 1979,
the crew of D/B Lucio abandoned the vessel because the barges rope attached to the wharf was cut off
by the big waves. At around midnight, the barge run aground and was broken and the cargoes of beer
in the barge were swept away. As a consequence of the incident, SMC filed a complaint for Breach of
Contract of Carriage and Damages against ANCO. Upon Ang Guis death, ANCO, as a partnership, was
dissolved hence, SMC filed a second amended complaint which was admitted by the Court impleading
the surviving partner, Co To and the Estate of Ang Gui represented by Lucio, Julian and Jaime, all
surnamed Ang.
ANCO admitted that the cases of beer were indeed loaded on the vessel belonging to ANCO. It claimed
however that it had an agreement with SMC that ANCO would not be liable for any losses or damages
resulting to the cargoes by reason of fortuitous event. ANCO further asserted that there was an
agreement between them and SMC to insure the cargoes in order to recover indemnity in case of loss.
Pursuant to that agreement, the cargoes to the extent of 20,000 cases was insured with FGU for the

By: Elaine Marie G. Laceda

122

INSURANCE LAW

total amount of P858,500.00. Thus, ANCO, with leave of court, filed a Third-Party Complaint against
FGU.
In its answer to the Third-Party complaint, FGU admitted the existence of the Insurance Policy but
maintained that the alleged loss of the cargoes covered by the said insurance policy cannot be attributed
directly or indirectly to any of the risks insured against in the said insurance policy. According to FGU, it
is only liable under the policy to Third-party Plaintiff ANCO and/or Plaintiff SMC in case of any of the
following: (a) total loss of the entire shipment; (b) loss of any case as a result of the sinking of the
vessel; or (c) loss as a result of the vessel being on fire. Furthermore, FGU alleged that the Third-Party
Plaintiff ANCO and Plaintiff SMC failed to exercise ordinary diligence or the diligence of a good father of
the family in the care and supervision of the cargoes insured to prevent its loss and/or destruction.
The trial court found that while the cargoes were indeed lost due to fortuitous event, there was failure
on ANCOs part, through their representatives, to observe the degree of diligence required that would
exonerate them from liability. The trial court thus held the Estate of Ang Gui and Co To liable to SMC for
the amount of the lost shipment. With respect to the Third-Party complaint, the court a quo found FGU
liable to bear 53% of the amount of the lost cargoes. CA affirmed in toto.
Issue: WON the negligence of ANCOs representatives was the proximate cause of the loss.
Held: Yes.
A careful study of the records shows no cogent reason to fault the findings of the lower court, as
sustained by the appellate court, that ANCOs representatives failed to exercise the extraordinary degree
of diligence required by the law to exculpate them from liability for the loss of the cargoes. First, ANCO
admitted that they failed to deliver to the designated consignee the 29,210 cases of Pale Pilsen and 550
cases of Cerveza Negra. Second, it is borne out in the testimony of the witnesses on record that the
barge D/B Lucio had no engine of its own and could not maneuver by itself. Yet, the patron of ANCOs
tugboat M/T ANCO left it to fend for itself notwithstanding the fact that as the two vessels arrived at the
port of San Jose, Antique, signs of the impending storm were already manifest. At that precise moment,
since it is the duty of the defendant to exercise and observe extraordinary diligence in the vigilance over
the cargo of the plaintiff, the patron or captain of M/T ANCO, representing the defendant could have
placed D/B Lucio in a very safe location before they left knowing or sensing at that time the coming of a
typhoon. The presence of big waves and dark clouds could have warned the patron or captain of M/T
ANCO to insure the safety of D/B Lucio including its cargo. D/B Lucio being a barge, without its engine,
as the patron or captain of M/T ANCO knew, could not possibly maneuver by itself. Had the patron or
captain of M/T ANCO, the representative of the defendants observed extraordinary diligence in placing
the D/B Lucio in a safe place, the loss to the cargo of the plaintiff could not have occurred.
While the loss of the cargoes was admittedly caused by the typhoon Sisang, a natural disaster, ANCO
could not escape liability to respondent SMC. The records clearly show the failure of petitioners
representatives to exercise the extraordinary degree of diligence mandated by law. To be exempted
from responsibility, the natural disaster should have been the proximate and only cause of the loss.
There must have been no contributory negligence on the part of the common carrier.
Issue: WON FGU can be held liable under the insurance policy to reimburse ANCO for the loss of the
cargoes despite the findings of the respondent court that such loss was occasioned by the blatant
negligence of the latters employees.
Held: No.
One of the purposes for taking out insurance is to protect the insured against the consequences of his
own negligence and that of his agents. Thus, it is a basic rule in insurance that the carelessness and
negligence of the insured or his agents constitute no defense on the part of the insurer. This rule
however presupposes that the loss has occurred due to causes which could not have been prevented by
the insured, despite the exercise of due diligence.
The ordinary negligence of the insured and his agents has long been held as a part of the risk which the
insurer takes upon himself, and the existence of which, where it is the proximate cause of the loss, does

By: Elaine Marie G. Laceda

123

INSURANCE LAW

not absolve the insurer from liability. But willful exposure, gross negligence, negligence amounting to
misconduct, etc., have often been held to release the insurer from such liability. In the case at bar, both
the trial court and the appellate court had concluded from the evidence that the crewmembers of both
the D/B Lucio and the M/T ANCO were blatantly negligent.
3. Losses for which Insurer Not Liable
a. Loss of which a peril insured against was only a remote cause Sec. 84, I.C.
Sec. 84. Unless otherwise provided by the policy, an insurer is liable for a loss of which a peril insured
against was the proximate cause, although a peril not contemplated by the contract may have been a
remote cause of the loss; but he is not liable for a loss which the peril insured against was only a remote
cause.
b. Loss, the immediate cause of which was the peril insured against, if the
proximate cause thereof was excepted in the contract Sec. 86, I.C.
Sec. 86. Where a peril is especially excepted in a contract of insurance, a loss, which would not have
occurred but for such peril, is thereby excepted although the immediate cause of the loss was a peril
which was not excepted.

c.

Loss caused by willful act or through connivance of insured Sec. 87, I.C.,
But See Sec. 180-A, I.C.

Sec. 87. An insurer is not liable for a loss caused by the willful act or through the connivance of the
insured; but he is not exonerated by the negligence of the insured, or of the insurance agents or others.
Sec. 180-A. The insurer in a life insurance contract shall be liable in case of suicides only when it is
committed after the policy has been in force for a period of two years from the date of its issue or of its
last reinstatement, unless the policy provides a shorter period: Provided, however, That suicide
committed in the state of insanity shall be compensable regardless of the date of commission. (As
amended by Batasang Pambansa Blg. 874).
East Furniture Inc. vs. Globe & Rutgers Fire Ins. Co.
G.R. No. L-35848 November 22, 1932
Plaintiff East Furniture Inc. is a duly registered partnership engaged in the sale of furniture. The plaintiff
insured against fire the articles existing in its establishment situated at Nos. 626 and 628 Rizal Avenue,
Manila with Globe & Rutgers, P5,000, in force from July 12, 1928, to July 12, 1929; Commercial Union,
P5,000, in force during the same period; and The Continental, P10,000, in force from August 16, 1928,
to August 16, 1929. On March 2, 1929, a fire broke out in plaintiff's establishment, as a result of which
the insured articles therein found were destroyed by the fire. Thus, within the period marked in the
policies the plaintiff presented to the insurance companies an inventory of the insured furniture which
was destroyed by the fire.
The defendants in their respective answers interposed a general denial and as special defenses alleged
in substance (1) that the fire in question was of intentional origin; (2) that the claims of loss presented
by the plaintiff were false and fraudulent; (3) that the furniture in question had been mortgaged by the
plaintiff to the Manila Finance and Discount Corporation, so that at the time of the fire the plaintiff was
not the only party interested therein, contrary to the representations made in its claims of loss; and (4)
that the plaintiff violated one of the conditions of the policies by refusing to furnish the defendants with
a physical inventory of the contents of its store at the time of the fire.
By agreement of the parties the three cases were tried jointly before Judge Concepcion, who after the
trial found that the claims presented by the plaintiff were notoriously fraudulent, and, accordingly,
sustained defendant's second special defense and dismissed the complaint in each of the three cases,
with costs against the plaintiff.

By: Elaine Marie G. Laceda

124

INSURANCE LAW

The appellant contends that the trial court erred (1) in finding that the claims presented by the plaintiff
to the insurance companies were fraudulent; (2) in giving weight to the testimony of Captain Lorenzo,
deputy chief of the Manila Fire Department, and Isidro Guevara, a furniture manufacturer, as to the
value of the articles found in the premises after the fire; and (3) in dismissing plaintiff's complaints.
Held:
Defendants' first special defense is well founded that the fire in question was of intentional origin and
was caused with the connivance of the plaintiff. It appears from the record that at the time of the fire
the plaintiff was heavily indebted to the Manila Finance & Discount Corporation, to the Bank of the
Philippine Islands, and to Attorney Alfonso E. Mendoza.
We may also consider the damage caused by the fire in relation with defendant's second special defense
that plaintiff's claims of loss were false and fraudulent. Condition 12 of each of the insurance policies
sued upon provides that "if the claim be in any respect fraudulent, or if any false declaration be made or
used in support thereof, or if any fraudulent means or devices are used by the Insured or anyone acting
on his behalf to obtain any benefit under this policy; or, if the loss or damage be occasioned by the
wilful act, or with the connivance of the Insured, all benefit under this policy shall be forfeited."
Prats & Co. vs. Phoenix Insurance Co.
G.R. No. L-28607 February 21, 1929
Prats and Company purchased a building on which it stored its merchandise. The building and
merchandise were covered by several insurance policies and one of them was issued by Phoenix. A fire
broke out and destroyed the building. Prats duly filed its claim but was denied on the ground that the
fire was caused by connivance of the insured with others as well as the claim wasn't in good faith.
Held:
The insurance policy which was the subject of action in this case was held to have been avoided by the
connivance of the insured in setting fire to the insured goods and the submission of the insured of
fraudulent proof of loss. The finding of the trial court in the effect that the plaintiff had submitted false
proof in the support of his claim is also well founded. That conclusion appears to have been based upon
three items of proof. These two facts are, first, that the plaintiff had submitted a claim for jewelry lost in
the fire as of a value of P12,800 when the true value of said jewelry was about P600; and, secondly,
that the plaintiff had sought to recover from the insurance company the value of goods which had been
surreptitiously withdrawn by it from the bodega prior to the fire. Neither of these two facts are
consistent with good faith on the part of the plaintiff, and each constituted a breach of the stipulations of
the policy against the use of fraudulent devices and false proof with respect to the loss.
The other point relied upon to support conclusion that the plaintiff had attempted to deceive the
defendant with respect to the extent of the loss was at least competent in its general bearing on the
good faith of the plaintiff, even if, as is probably true, not alone sufficient to constitute a breach of the
same stipulations.
The point is this: After the fire the plaintiff presented to the adjuster certain cost sheets and copies of
supposed invoices in which the prices and expenses of importation of a quantity of goods were stated at
double the true amount. The adjuster soon discovered the artificial nature of these documents, and, with
his consent, they were withdrawn by Prats and subsequently destroyed. At the hearing Prats stated that
these documents had been fabricated in order that they might be exhibited to intending purchasers of
the goods, thereby making it appear to them that the cost of the merchandise had been much greater
than it in fact was a ruse which is supposed to have been entirely innocent or at least not directed
against the insurer. But a question naturally arises as to the purpose which these documents might have
been made to serve if the fire, as doubtless intended by its designers, had been so destructive as to
remove all vestiges of the stock actually involved. Upon the whole we are forced to state the conclusion,
not only that the plaintiff caused the fire to be set, or connived therein, but also that it submitted
fraudulent proof as the trial judge found.

By: Elaine Marie G. Laceda

125

INSURANCE LAW

4. Void Agreement Sec. 83, I.C.


Sec. 83. An agreement not to transfer the claim of the insured against the insurer after the loss has
happened, is void if made before the loss except as otherwise provided in the case of life insurance.
B.

NOTICE AND PROOF

1. Requisites for Recovery After Loss


a. Give notice of loss without unnecessary delay
b. When required by policy, submit a preliminary proof of loss
2. Notice of Loss Sec. 88, I.C.
Sec. 88. In case of loss upon an insurance against fire, an insurer is exonerated, if notice thereof be not
given to him by an insured, or some person entitled to the benefit of the insurance, without unnecessary
delay.
Bachrach vs. British American Assurance Co.
G.R. No. L-5715 December 20, 1910
E. M. Bachrach was issued a fire insurance policy by defendant insurance company. On the July 13,
1908, the plaintiff commenced an action against the defendant to recover the sum of P9,841.50, the
amount due, deducting the salvage, upon the fire insurance policy issued by the defendant. The
defendant alleged that it was released from all obligations whatever under said policy because the
plaintiff transferred his interest in and to the property covered by the policy to H. W. Peabody & Co. to
secure certain indebtedness due and owing to said company, and also that the plaintiff had transferred
his interest in certain of the goods covered by the said policy to one Macke, to secure certain obligations
assumed by the said Macke for and on behalf of the insured without the consent of the insurer as
required by the policy. TC found the defendant liable to the plaintiff.
Issue: WON the execution of the said chattel mortgage, without the knowledge and consent of the
insurance company and without receiving the sanction of said company, annulled the said policy of
insurance.
Held: No.
It is claimed that the execution of a chattel mortgage on the insured property violated what is known as
the "alienation clause". However it has been held in a line of cases that the interest in property insured
does not pass by the mere execution of a chattel mortgage and that while a chattel mortgage is a
conditional sale, there is no alienation within the meaning of the insurance law until the mortgage
acquires a right to take possession by default under the terms of the mortgage. No such right is claimed
to have accrued in the case at bar, and the alienation clause is therefore inapplicable.
Furthermore, upon reading the policy of insurance issued by the defendant to the plaintiff, it will be
noted that there is no provision in said policy prohibiting the plaintiff from placing a mortgage upon the
property insured.
3. Proof of Loss Secs. 89 and 92, I.C.
Sec. 89. When a preliminary proof of loss is required by a policy, the insured is not bound to give such
proof as would be necessary in a court of justice; but it is sufficient for him to give the best evidence
which he has in his power at the time.
Sec. 92. If the policy requires, by way of preliminary proof of loss, the certificate or testimony of a
person other than the insured, it is sufficient for the insured to use reasonable diligence to procure it,
and in case of the refusal of such person to give it, then to furnish reasonable evidence to the insurer

By: Elaine Marie G. Laceda

126

INSURANCE LAW

that such refusal was not induced by any just grounds of disbelief in the facts necessary to be certified
or testified.
Fernandez vs. National Life Insurance Co.
G.R. No. L-9146 January 27, 1959
National Life Insurance Company (NLIC) insured J. Fernandezs life for P10,000 upon his payment of
P444 from July 15, 1944 to July 14, 1945. The insured died on November 2, 1944, while the policy was
in force. After more than 7 years, in 1952, Atty de la Torre, representing the benficiaries of the policy,
informed the company that Fernandez had died in 1944, and claimed the proceeds of the policy. The
company said that the status of the policies issued during the Japanese occupation was still pending
consideration before the courts. NLIC said that because the policy matured upon the insureds death in
November, 1944, they should compute the value of their claim under the Ballantyne scale of values
(which would amount only to P500).
Beneficiaries commenced suit, and the lower court sustained the stand of the company, dismissed the
complaint. Beneficiaries maintain that the obligation of the company to pay accrued not upon the death
of Fernandez, but only upon the receipt and approval by the company, on proof of death of the insured,
which was in 1954. The policy reads:
National Life Insurance Company of the Philippine hereby agrees to pay at its Home Office,
Manila, Ten Thousand Pesos to Juan D. Fernandez (hereinafter called the insured) on the 15th
day of July, 1964, if the Insured is living and this Policy is in force, or upon receipt and approved
at its Office of due proofs of the title of the claimant and of the prior death of the Insured while
this Policy is in force to Teresa Duat Vda. De Fernandez, Maria T. and Manuela Fernandez,
mother and sisters respectively of the Insured (Hereinafter called the Beneficiary) subject to the
right of the Insured to change the beneficiary as stated on the second page of this Policy.
The above stipulation is apparently based on Sec. 91-A of the Insurance Law which provides as follows:
The proceeds of a life insurance policy shall be paid immediately upon maturity of the policy, unless
such proceeds are made payable in installments or a as an annuity, in which case the installments or
annuities shall be paid as they become due: Provided, however, That in case of a policy maturing by the
death of the insured, the proceeds thereof shall be paid within sixty days after presentation of the claim
and filing of the proof of the death of the insured. Refused to pay the claim within the time prescribed
herein will entitle the beneficiary to collect interest on the proceeds of the policy for the duration of the
delay at the rate of six per centum per annum, unless such failure or refusal to pay is based on the
ground that the claim is fraudulent . . . .
Based on the foregoing provision of law and the aforequoted stipulation as well as on the allegation that
the filing of proof of death by the beneficiaries is a condition precedent of the demandability of the
obligation of the insurer to pay the proceeds, appellants claim that they should be paid P10,000 in
Philippine currency and not under the Ballantyne scale of values.
Issue: WON the policy matured upon the death of the insured.
Held: Yes.
In life insurance, the policy matures either upon the expiration of the term set forth therein, or upon his
death occuring at any time prior to the expiration of such stipulated term, in which case, the proceeds
are payable to his beneficiaries within sixty days after their filing of proof of death.
The sixty day period fixed by law within which to pay the proceeds after presentation of proof of death
Is merely procedural in nature, evidently to determine the exact amount to be paid and the interest
thereon to which the beneficiaries may be entitled to collect in case of unwarranted refusal of the
company to pay, and also to enable the insurer to verify or check on the fact of death which it may even
validly waive. It is the happening of the suspensive condition of death that renders a life policy matured,
and not ht efiling of proof of death which, as above stated, is merely procedural. The insured having
died during the Japanese occupation, the proceeds of his policy should be adjusted accordingly, for The

By: Elaine Marie G. Laceda

127

INSURANCE LAW

rule is already settled that where a debtor could have paid his obligation at any time during the
Japanese occupation, payment after liberation must be adjusted in accordance with the Ballantyne
schedule (De Asis vs. Agdamag, among other cases). (Collaboration is defined as the acts of working
together in a joint project.)
Insular Life Assurance Co. Ltd. v. Fernandez, 109 Phil 530 (1960)
Malayan Insurance Co., Inc. vs. Arnaldo
G.R. No. L-67835 October 12, 1987
Petitioner issued to private respondent, Coronacion Pinca, Fire Insurance Policy No. F-001-17212 on her
property for the amount of P14,000.00 effective July 22, 1981, until July 22, 1982. Howver, on October
15, 1981, MICO allegedly cancelled the policy for non-payment, of the premium and sent the
corresponding notice to Pinca. Payment of the premium was then later received by Domingo Adora,
agent of MICO who remitted this payment to MICO, together with other payments on January 15, 1982.
On January 18, 1982, Pinca's property was completely burned. On February 5, 1982, Pinca's payment
was returned by MICO to Adora on the ground that her policy had been cancelled earlier. But Adora
refused to accept it. In due time, Pinca made the requisite demands for payment, which MICO rejected.
She then went to the Insurance Commission which sustained her claim for compensation for her burned
property.
Issue: WON Adora had the authority to receive payment.
Held: Yes.
MICO's acknowledgment of Adora as its agent defeats its contention that he was not authorized to
receive the premium payment on its behalf. It is clearly provided in Section 306 of the Insurance Code
that: Any insurance company which delivers to an insurance agant or insurance broker a policy or
contract of insurance shall be demmed to have authorized such agent or broker to receive on its behalf
payment of any premium which is due on such policy or contract of insurance at the time of its issuance
or delivery or which becomes due thereon.
It is also a well-known principle under the law of agency that: Payment to an agent having authority to
receive or collect payment is equivalent to payment to the principal himself; such payment is complete
when the money delivered is into the agent's hands and is a discharge of the indebtedness owing to the
principal.
Issue: WON the policy was subsisting at the time of the fire.
Held: Yes.
MICO's claims it cancelled the policy in question on October 15, 1981, for non-payment of premium. To
support this assertion, it presented one of its employees, who testified that "the original of the
endorsement and credit memo" presumably meaning the alleged cancellation "were sent the
assured by mail through our mailing section" However, there is no proof that the notice, assuming it
complied with the other requisites for cancellation, was actually mailed to and received by Pinca. All
MICO's offers to show that the cancellation was communicated to the insured is its employee's testimony
that the said cancellation was sent "by mail through our mailing section." without more.
It stands to reason that if Pinca had really received the said notice, she would not have made payment
on the original policy on December 24, 1981. Instead, she would have asked for a new insurance,
effective on that date and until one year later, and so taken advantage of the extended period. The
Court finds that if she did pay on that date, it was because she honestly believed that the policy issued
on June 7, 1981, was still in effect and she was willing to make her payment retroact to July 22, 1981,
its stipulated commencement date. After all, the premium invoice issued to Pinca at the time of the
delivery of the policy on June 7, 1981 was stamped "Payment Received" of the amount of P930.60 on
"12-24-81" by Domingo Adora. This is important because it suggests an understanding between MICO
and the insured that such payment could be made later, as agent Adora had assured Pinca.

By: Elaine Marie G. Laceda

128

INSURANCE LAW

Pacific Banking Corp. vs. Court of Appeals


G.R. No. L-41014 November 28, 1988
October 21,1963: an open Fire Policy was issued to the Paramount Shirt Manufacturing Co. (insured), by
which Oriental Assurance Corporation bound itself to indemnify the insured for any loss or damage, not
exceeding P61,000.00, caused by fire to its property consisting of stocks, materials and supplies usual to
a shirt factory, including furniture, fixtures, machinery and equipment while contained in the ground,
second and third floors of the building situated at number 256 Jaboneros St., San Nicolas, Manila, for a
period of one year commencing from that date to October 21, 1964.
Insured was at the time of the issuance of the policy and is up to this time, a debtor of Pacific Banking in
the amount of not less P800,000.00 and the goods described in the policy were held in trust by the
insured for the Pacific Banking under thrust receipts.
Said policy was duly endorsed to Pacific Banking as mortgagee/trustor of the properties insured, with
the knowledge and consent of Oriental Assurance to the effect that "loss if any under this policy is
payable to the Pacific Banking Corporation".
While the aforesaid policy was in full force and effect, a fire broke out on the subject premises
destroying the goods contained in its ground and second floors. Counsel for the Pacific Banking sent a
letter of demand to Oriental Assurance for indemnity due to the loss of property by fire. Oriental
Assurance informed counsel that it was not yet ready to accede to the latter's demand as the former is
awaiting the final report of the insurance adjuster, H.H. Bayne Adjustment Company.
Said insurance adjuster notified counsel for the Pacific Banking that the insured under the policy had not
filed any claim with it, nor submitted proof of loss which is a clear violation of Policy Condition No.11,
and for which reason, determination of the liability of Oriental Assurance could not be had. Pacific
Banking's counsel replied asking the insurance adjuster to verify from the records of the Bureau of
Customs the entries of merchandise taken into the customs bonded warehouse razed by fire as a
reliable proof of loss.
For failure of the insurance company to pay the loss as demanded, Pacific Banking field before CFI an
action for a sum of money against the Oriental Assurance, in the principal sum of P61,000.00 issued in
favor of Paramount Shirt Manufacturing Co.
Oriental Assurance defenses: (a) lack of formal claim by insured over the loss and (b) premature filing
of the suit as neither plaintiff nor insured had submitted any proof of loss on the basis of which
defendant would determine its liability and the amount thereof, either to the Oriental Assurance or its
adjuster H.H. Bayne Adjustment Co.
Pacific Banking: presented evidence that insured has undeclared co-insurances with the following:
P30,000.00 with Wellington Insurance; P25,000. 00 with Empire Surety and P250,000.00 with Asian
Surety; undertaken by insured Paramount on the same property covered by its policy with Oriental
Assurance whereas the only co-insurances declared in the subject policy are those of P30,000.00 with
Malayan, P50,000.00 with South Sea, and P25.000.00 with Victory. NOTE: the defense of fraud and/or
violation of non-declaration of co-insurances was not pleaded in the answer, also not pleaded in the
Motion to Dismiss.
CFI denied Oriental Assurance's motion on the ground that since the defense was raised for the first
time, it must be deemed to have waived the requirement of proof of loss. Case was submitted for
decision. But upon MR, Oriental Asurance was allowed to present additional evidence, "in order to prove
that 'insured has committed a violation of condition No. 3 of the policy in relation to the other Insurance
Clause.' " CFI eventually adjudged Oriental Assurance liable to the Pacific Banking under the said
contract of insurance. CA reversed. Pacific Banking's MR denied.
Issue: WON insured is guilty of fraud

By: Elaine Marie G. Laceda

129

INSURANCE LAW

Held: Yes.
The crux of the controversy centers on two points: (a) unrevealed co-insurances which violated policy
conditions No. 3; and (b) failure of the insured to file the required proof of loss prior to court action.
Policy Condition No. 3 explicitly provides: The Insured shall give notice to the Company of any
insurance already effected, or which may subsequently be effected, covering any of the property hereby
insured, and unless such notice be given and the particulars of such insurance or insurances be stated in
or endorsed on this Policy by or on behalf of the Company before the occurrence of any loss or damage,
all benefit under this policy shall be forfeited.
It is not disputed that the insured failed to reveal before the loss three other insurances. By reason of
said unrevealed insurances, the insured had been guilty of a false declaration; a clear misrepresentation
and a vital one because where the insured had been asked to reveal but did not, that was deception.
Otherwise stated, had the insurer known that there were many co-insurances, it could have hesitated or
plainly desisted from entering into such contract. Hence, the insured was guilty of clear fraud.
Pacific Banking's contention that the allegation of fraud is but a mere inference or suspicion is
untenable. Concrete evidence of fraud or false declaration by the insured was furnished by the Pacific
Banking itself when the facts alleged in the policy under clauses "Co-Insurances Declared" and "Other
Insurance Clause" are materially different from the actual number of co-insurances taken over the
subject property. Consequently, the whole foundation of the contract fails, the risk does not attach and
the policy never becomes a contract between the parties. Representations of facts are the foundation of
the contract and if the foundation does not exist, the superstructure does not arise. Falsehood in such
representations is not shown to vary or add to the contract, or to terminate a contract which has once
been made, but to show that no contract has ever existed (Tolentino). A void or inexistent contract is
one which has no force and effect from the very beginning, as if it had never been entered into, and
which cannot be validated either by time or by ratification.
As the insurance policy against fire expressly required that notice should be given by the insured of
other insurance upon the same property, the total absence of such notice nullifies the policy.
Argument that notice of co-insurances may be made orally is preposterous and negates policy condition
No. 20 which requires every notice and other communications to the insurer to be written or printed.
Issue: WON mortgagee/assignee can still claim from the insurance
Held: No.
Subject mortgage clause pecifically provides: Loss, if any, under this policy, shall be payable to the
PACIFIC BANKING CORPORATION Manila mortgagee/trustor as its interest may appear, it being hereby
understood and agreed that this insurance as to the interest of the mortgagee/trustor only herein, shall
not be invalidated by any act or neglect except fraud or misrepresentation, or arson of the mortgagor or
owner/trustee of the property insured; provided, that in case the mortgagor or owner/ trustee neglects
or refuses to pay any premium, the mortgagee/ trustor shall, on demand pay the same.
The paragraph clearly states the exceptions to the general rule that insurance as to the interest of the
mortgagee, cannot be invalidated; namely: fraud, or misrepresentation or arson.
Concealment of the aforecited co-insurances can easily be fraud, or in the very least, misrepresentation.
It is but fair and just that where the insured who is primarily entitled to receive the proceeds of the
policy has by its fraud and/or misrepresentation, forfeited said right, with more reason Pacific Banking
which is merely claiming as indorsee of said insured, cannot be entitled to such proceeds.
The fact of fraud was tried by express or at least implied consent of the parties. Pacific Banking did not
only object to the introduction of evidence but on the contrary, presented the very evidence that proved
its existence.

By: Elaine Marie G. Laceda

130

INSURANCE LAW

Be that as it may, SC has ample authority to give beyond the pleadings where in the interest of justice
and the promotion of public policy, there is a need to make its own finding to support its conclusion.
Otherwise stated, the Court can consider a fact which surfaced only after trial proper.
Generally, the cause of action on the policy accrues when the loss occurs, but when the policy provides
that no action shall be brought unless the claim is first presented extrajudicially in the manner provided
in the policy, the cause of action will accrue from the time the insurer finally rejects the claim for
payment.
In the case at bar, policy condition No. 11 specifically provides that the insured shall on the happening
of any loss or damage give notice to the company and shall within fifteen (15) days after such loss or
damage deliver to the Oriental Assurance (a) a claim in writing giving particular account as to the
articles or goods destroyed and the amount of the loss or damage and (b) particulars of all other
insurances, if any. Likewise, insured was required "at his own expense to produce, procure and give to
the company all such further particulars, plans, specifications, books, vouchers, invoices, duplicates or
copies thereof, documents, proofs and information with respect to the claim".
Evidence adduced shows that 24 days after the fire, Pacific Banking merely wrote letters to Oriental
Assurance to serve as a notice of loss, thereafter, the former did not furnish the latter whatever
pertinent documents were necessary to prove and estimate its loss. Instead, Pacific Banking shifted
upon Oriental Assurance the burden of fishing out the necessary information to ascertain the particular
account of the articles destroyed by fire as well as the amount of loss.
Oriental Assurance and its adjuster notified Pacific Banking that insured had not yet filed a written claim
nor submitted the supporting documents in compliance with the requirements set forth in the policy.
Despite the notice, the latter remained unheedful. Since the required claim by insured, together with the
preliminary submittal of relevant documents had not been complied with, it follows that Oriental
Assurance could not be deemed to have finally rejected Pacific Banking's claim and therefore the latter's
cause of action had not yet arisen. Compliance with condition No. 11 is a requirement sine qua non to
the right to maintain an action as prior thereto no violation of Pacific Banking's right can be attributable
to Oriental Assurance. As before such final rejection, there was no real necessity for bringing suit. Pacific
Banking should have endeavored to file the formal claim and procure all the documents, papers,
inventory needed by Oriental Assurance or its adjuster to ascertain the amount of loss and after
compliance await the final rejection of its claim. Indeed, the law does not encourage unnecessary
litigation.
Pacific Banking prematurely filed the civil case and dismissal thereof was warranted under the
circumstances. While it is a cardinal principle of insurance law that a policy or contract of insurance is to
be construed liberally in favor of the insured and strictly as against the insurer company yet, contracts
of insurance, like other contracts, are to be construed according to the sense and meaning of the terms
which the parties themselves have used. If such terms are clear and unambiguous, they must be taken
and understood in their plain, ordinary and popular sense.
Contracts of insurance are contracts of indemnity upon the terms and conditions specified in the policy.
The parties have a right to impose such reasonable conditions at the time of the making of the contract
as they may deem wise and necessary. The agreement has the force of law between the parties. The
terms of the policy constitute the measure of the insurer's liability, and in order to recover, the insured
must show himself within those terms. The compliance of the insured with the terms of the policy is a
condition precedent to the light of recovery.
It appearing that insured has violated or failed to perform the conditions under No. 3 and 11 of the
contract, and such violation or want of performance has not been waived by the insurer, the insured
cannot recover, much less the herein Pacific Banking. Courts are not permitted to make contracts for the
parties; the function and duty of the courts is simply to enforce and carry out the contracts actually
made.
4. Defects Sec. 90, I.C.

By: Elaine Marie G. Laceda

131

INSURANCE LAW

Sec. 90. All defects in a notice of loss, or in preliminary proof thereof, which the insured might remedy,
and which the insurer omits to specify to him, without unnecessary delay, as grounds of objection, are
waived.
5. Delay Sec. 91, I.C.
Sec. 91. Delay in the presentation to an insurer of notice or proof of loss is waived if caused by any act
of him, or if he omits to take objection promptly and specifically upon that ground.
Pacific Timber vs. Court of Appeals
G.R. No. L-38613 February 25, 1982
On March 19, 1963, the plaintiff secured temporary insurance from the defendant for its exportation of
1,250,000 board feet of Philippine Lauan and Apitong logs to be shipped from the Diapitan Bay, Quezon
Province to Okinawa and Tokyo, Japan. The defendant issued on said date Cover Note No. 1010,
insuring the said cargo of the plaintiff. On March 29, 1963, after the issuance of Cover Note No. 1010,
but before the issuance of the marine policies on April 2, 1963, some of the logs intended to be exported
were lost during loading operations in the Diapitan Bay.
In a letter dated April 4, 1963, the plaintiff informed the defendant about the loss of 'appropriately 32
pieces of log's during loading of the 'SS Woodlock'. Although dated April 4, 1963, the letter was received
in the office of the defendant only on April 15, 1963, as shown by the stamp impression appearing on
the left bottom corner of said letter. The plaintiff subsequently submitted a 'Claim Statement demanding
payment of the loss under Policies Nos. 53 HO 1032 and 53 HO 1033, in the total amount of
P19,286.79. The claim was denied on the ground that the said loss may be considered as covered under
Cover Note No. 1010 which had become 'null and void by virtue of the issuance of Marine Policies.
Defendant later alleged that the cover note is null and void for lack of valuable consideration. In a
complaint filed by plaintiff, CFI ordered payment by defendant. CA reversed. Hence the petition.
Issue: WON insurer was released from liability under the cover note due to unreasonable delay in giving
notice of loss despite failure of the insurer to promptly and specifically object to the claim on the ground
of delay.
Held: No.
The defense of delay as raised by private respondent in resisting the claim cannot be sustained. The law
requires this ground of delay to be promptly and specifically asserted when a claim on the insurance
agreement is made. The undisputed facts show that instead of invoking the ground of delay in objecting
to petitioner's claim of recovery on the cover note, it took steps clearly indicative that this particular
ground for objection to the claim was never in its mind. The nature of this specific ground for resisting a
claim places the insurer on duty to inquire when the loss took place, so that it could determine whether
delay would be a valid ground upon which to object to a claim against it.
As already stated earlier, private respondent's reaction upon receipt of the notice of loss, which was on
April 15, 1963, was to set in motion from July 1963 what would be necessary to determine the cause
and extent of the loss, with a view to the payment thereof on the insurance agreement. Thus it sent its
adjuster to investigate and assess the loss in July, 1963. The adjuster submitted his report on August
23, 1963 and its computation of respondent's liability on September 14, 1963. From April 1963 to July,
1963, enough time was available for private respondent to determine if petitioner was guilty of delay in
communicating the loss to respondent company. In the proceedings that took place later in the Office of
the Insurance Commissioner, private respondent should then have raised this ground of delay to avoid
liability. It did not do so. It must be because it did not find any delay, as this Court fails to find a real
and substantial sign thereof. But even on the assumption that there was delay, this Court is satisfied
and convinced that as expressly provided by law, waiver can successfully be raised against private
respondent.
Philippine Charter Insurance Corporation vs. Chemoil Lighterage Corporation
G.R. No. 136888 June 29, 2005

By: Elaine Marie G. Laceda

132

INSURANCE LAW

On 24 January 1991, Samkyung Chemical Company, Ltd., based in South Korea, shipped 62.06 and
436.70 metric tons of the liquid chemical DOP. The consignee Plastic Group Phils., Inc. (PGP) in Manila
insured the cargo with petitioner Philippine Charter Insurance Corporation against all risks.
The ocean tanker MT "TACHIBANA" unloaded the cargo to Tanker Barge LB-1011 of respondent Chemoil
Lighterage Corporation, which shall transport the same to Del Pan Bridge in Pasig River. Tanker Barge
LB-1011 would unload the cargo to tanker trucks, also owned by the respondent, and haul it by land to
PGPs storage tanks in Calamba, Laguna.
Upon inspection by PGP, the samples taken from the shipment showed discoloration from yellowish to
amber, demonstrating that it was damaged, as DOP is colorless and water clear. PGP then sent a letter
to the petitioner dated 18 February 1991 where it formally made an insurance claim for the loss it
sustained due to the contamination. In the meantime, on 03 April 1991, PGP paid the respondent the
amount of P301,909.50 as full payment for the latters services. On 13 May 1991, the petitioner paid
PGP the amount of P5,000,000.00 as full and final payment for the loss.
Pursuant to the Subrogation Receipt issued by PGP to the petitioner, an action for damages was
instituted by the petitioner-insurer against respondent-carrier.
In respondent-carriers Answer with Compulsory Counterclaim, respondent admitted it undertook to
transport the consignees shipment from MT "TACHIBANA" to the Del Pan Bridge, Pasig River, where it
was transferred to its tanker trucks for hauling to PGPs storage tanks in Calamba, Laguna. The
respondent alleged that before the DOP was loaded into its barge (LB-1011), the
surveyor/representative of PGP, Adjustment Standard Corporation, inspected it and found the same
clean, dry, and fit for loading. The entire loading and unloading of the shipment were also done under
the control and supervision of PGPs surveyor/representative. It was also mentioned by the respondent
that the contract between it and PGP expressly stipulated that it shall be free from any and all claims
arising from contamination, loss of cargo or part thereof; that the consignee accepted the cargo without
any protest or notice; and that the cargo shall be insured by its owner sans recourse against all risks. As
subrogee, the petitioner was bound by this stipulation. As carrier, no fault and negligence can be
attributed against respondent as it exercised extraordinary diligence in handling the cargo.
TC ruled in favor of plaintiff-insurer. On appeal, respondent claimed that PGP failed to file any notice,
claim or protest within the period required by Article 366 of the Code of Commerce, which is a condition
precedent to the accrual of a right of action against the carrier. A telephone call which was supposedly
made by a certain Alfred Chan, an employee of PGP, to one of the Vice Presidents of the respondent,
informing the latter of the discoloration, is not the notice required by Article 366 of the Code of
Commerce. Thus, CA reversed and set aside TC decision.
Issue: WON the notice of claim was filed within the required period.
Held: No.
Article 366 of the Code of Commerce provides: Within twenty-four hours following the receipt of the
merchandise a claim may be made against the carrier on account of damage or average found upon
opening the packages, provided that the indications of the damage or average giving rise to the claim
cannot be ascertained from the exterior of said packages, in which case said claim shall only be
admitted at the time of the receipt of the packages. After the periods mentioned have elapsed, or after
the transportation charges have been paid, no claim whatsoever shall be admitted against the carrier
with regard to the condition in which the goods transported were delivered.
The petitioner contends that the notice of contamination was given by Alfredo Chan, an employee of
PGP, to Ms. Encarnacion Abastillas, Vice President for Administration and Operations of the respondent,
at the time of the delivery of the cargo, and therefore, within the required period. This was done by
telephone. The respondent, however, claims that the supposed notice given by PGP over the telephone
was denied by Ms. Abastillas. Between the testimonies of Alfredo Chan and Encarnacion Abastillas, the
latters testimony is purportedly more credible because it would be quite unbelievable and contrary to

By: Elaine Marie G. Laceda

133

INSURANCE LAW

business practice for Alfredo Chan to merely make a verbal notice of claim that involves millions of
pesos.
The requirement that a notice of claim should be filed within the period stated by Article 366 of the Code
of Commerce is not an empty or worthless proviso. The object sought to be attained by the requirement
of the submission of claims in pursuance of this article is to compel the consignee of goods entrusted to
a carrier to make prompt demand for settlement of alleged damages suffered by the goods while in
transport, so that the carrier will be enabled to verify all such claims at the time of delivery or within
twenty-four hours thereafter, and if necessary fix responsibility and secure evidence as to the nature
and extent of the alleged damages to the goods while the matter is still fresh in the minds of the parties.
Where the contract of shipment contains a reasonable requirement of giving notice of loss of or injury to
the goods, the giving of such notice is a condition precedent to the action for loss or injury or the right
to enforce the carriers liability. Such requirement is not an empty formalism. The fundamental reason
or purpose of such a stipulation is not to relieve the carrier from just liability, but reasonably to inform it
that the shipment has been damaged and that it is charged with liability therefore, and to give it an
opportunity to examine the nature and extent of the injury. This protects the carrier by affording it an
opportunity to make an investigation of a claim while the matter is fresh and easily investigated so as to
safeguard itself from false and fraudulent claims.
The filing of a claim with the carrier within the time limitation therefore actually constitutes a condition
precedent to the accrual of a right of action against a carrier for loss of, or damage to, the goods. The
shipper or consignee must allege and prove the fulfillment of the condition. If it fails to do so, no right of
action against the carrier can accrue in favor of the former. The aforementioned requirement is a
reasonable condition precedent; it does not constitute a limitation of action.
The second paragraph of Article 366 of the Code of Commerce is also edifying. It is not only when the
period to make a claim has elapsed that no claim whatsoever shall be admitted, as no claim may
similarly be admitted after the transportation charges have been paid.
VIII.
A.

DOUBLE INSURANCE AND REINSURANCE


DOUBLE INSURANCE

1. Definition Sec. 93, I.C.


2. Requisites Sec. 93, I.C.
Sec. 93. A double insurance exists where the same person is insured by several insurers separately in
respect to the same subject and interest.
a.
b.
c.
d.
e.

Same person is insured


Two or more insurers that insured the person separately
Insurance is over the same subject
Same interest is involved
Same peril is insured against

Pacific Banking Corp. vs. Court of Appeals (1988), supra


Pioneer Insurance and Surety Corporation vs. Yap
G.R. No. L-36232 December 19, 1974
Yap owned a store in a 2 storey building, where she sold shopping bags and footwear. Her son-in-law
was in charge of the store. On April 19, 1962 Yap took out Fire Insurance Policy No. 4216 from Pioneer
with a face value of P25,000 covering her stocks, office furniture, fixtures, etc. Among the conditions set
forth in the policy:
The Insured shall give notice to the Company of any insurance or insurances already
effected, or which may subsequently be effected, covering any of the property hereby insured,

By: Elaine Marie G. Laceda

134

INSURANCE LAW

and unless such notice be given and the particulars of such insurance or insurances be stated in,
or endorsed on this Policy by or on behalf of the Company before the occurrence of any loss or
damage, all benefits under this Policy shall be forfeited. (emphasis supplied)
It is understood that, except as may be stated on the face of this policy there is no other
insurance on the property hereby covered and no other insurance is allowed except by the
consent of the Company endorsed hereon. Any false declaration or breach or this condition will
render this policy null and void.
At the time of insurance of Policy 4219(April 19, 1962), an insurance policy for P20,000 issued by the
Great American Insurance Company covering the same properties was noted on said policy as coinsurance.
On August 29, 1962 parties executed an endorsement on Policy 4219 stating:
It is hereby declared and agreed that the co-insurance existing at present under this
policy is as follows: P20,000.00 Northwest Ins., and not as originally stated. (emphasis
supplied)
Except as varied by this endorsement, all other terms and conditions remain unchanged.
On September 26, 1962 Yap took out another fire insurance policy for P20,000 covering the same
properties, from Federal Insurance Company. This policy was procured without notice to and the written
consent of Pioneer, and was therefore not noted as a co-insurance in Policy 4219.
On December 19, 1962 fire burned Yaps store.
Issue: WON petitioner should be absolved from liability on Fire insurance Policy No. 4219 on account of
any violation by respondent Yap of the co-insurance clause therein
Held: Yes.
There was a violation by Yap of the co-insurance clause contained in Policy No. 4219 which resulted in
the avoidance of the petitioners liability. By the plain terms of the policy, other insurance without the
consent of petitioner would ipso facto avoid the contract. It required no affirmative act of election on the
part of the company to make operative the clause avoiding the contract, wherever the specified
conditions should occur. Its obligations ceased, unless, being informed of the fact, it consented to the
additional insurance.
The obvious purpose of the aforesaid requirement in the policy is to prevent over-insurance and thus
avert the perpetration of fraud. The public, as well as the insurer, is interested in preventing the
situation in which a fire would be profitable to the insured. According to Justice Story: "The insured has
no right to complain, for he assents to comply with all the stipulation on his side, in order to entitle
himself to the benefit of the contract, which, upon reason or principle, he has no right to ask the court to
dispense with the performance of his own part of the agreement, and yet to bind the other party to
obligations, which, but for those stipulation would not have been entered into."
New Life Enterprises vs. Court of Appeals
G.R. No. 94071 March 31, 1992
Julian Sy and Jose Sy Bang are partners engaged in the business of selling construction materials under
the business name New Life Enterprises. Julian Sy insured against fire the stocks in trade of New Life
Enterprises with Western Guaranty Corporation, Reliance Surety and Insurance Co. Inc., and Equitable
Insurance Corporation in the aggregate amount of P1,550,000.00. When the building where New Life
Enterprises was located, along with the stocks in trade therein, were gutted by fire, petitioners filed an
insurance claim against the three companies. The insurance companies all denied Julian Sys claim on
the ground of breach of policy condition, (i.e., the other insurance clause which required New Life
Enterprises to inform each of the insurance companies in case the former insures with another company
the same property already insured by each of the insurance companies).

By: Elaine Marie G. Laceda

135

INSURANCE LAW

Because of the denial of their claims for payment by the 3 insurance companies, petitioners filed
separate civil actions against the former before the Regional Trial Court of Lucena City, which cases
were consolidated for trial. The trial court ruled in favor of petitioner. However, the Court of Appeals
reversed the trial courts decision, found petitioner to have violated Clauses 3 and 27 of the separate
insurance policies issued by the 3 companies, and exonerated the insurance companies from liability.
Issue: WON petitioners violated the Other Insurance Clause of the insurance policies.
Held: Yes.
Petitioners admit that the respective insurance policies issued by private respondents did not state or
endorse thereon the other insurance coverage obtained or subsequently effected on the same stocks in
trade for the loss of which compensation is claimed by petitioners. It is further admitted by petitioners
that Equitable's policy stated "nil" in the space thereon requiring indication of any co-insurance although
there were 3 policies subsisting on the same stocks in trade at the time of the loss, namely, that of
Western in the amount of P350,000.00 and two 2 policies of Reliance in the total amount of
P1,000,000.00.
The coverage by other insurance or co-insurance effected or subsequently arranged by petitioners were
neither stated nor endorsed in the policies of the 3 private respondents, warranting forfeiture of all
benefits thereunder if we are to follow the express stipulation in Policy Condition No. 3.
The terms of the contract are clear and unambiguous. The insured is specifically required to disclose to
the insurer any other insurance and its particulars which he may have effected on the same subject
matter. The knowledge of such insurance by the insurer's agents, even assuming the acquisition thereof
by the former, is not the "notice" that would stop the insurers from denying the claim. Besides, the socalled theory of imputed knowledge, that is, knowledge of the agent is knowledge of the principal, aside
from being of dubious applicability here has likewise been roundly refuted by respondent court whose
factual findings we find acceptable. The mere fact that Yap Kam Chuan was an agent for both Reliance
and Equitable does not justify the allegation that the two are sister companies. Availment of the services
of the same agents and adjusters by different companies is a common practice in the insurance business
and such facts do not warrant the speculative conclusion of the trial court.
Considering the terms of the policy which required the insured to declare other insurances, the
statement in question must be deemed to be a statement (warranty) binding on both insurer and
insured, that there were no other insurance on the property. The annotation then, must be deemed to
be a warranty that the property was not insured by any other policy. Violation thereof entitled the
insurer to rescind.
The obvious purpose of the aforesaid requirement in the policy is to prevent over-insurance and thus
avert the perpetration of fraud. The public, as well as the insurer, is interested in preventing the
situation in which a fire would be profitable to the insured. The insured has no right to complain, for he
assents to comply with all the stipulations on his side, in order to entitle himself to the benefit of the
contract, which, upon reason or principle, he has no right to ask the court to dispense with the
performance of his own part of the agreement, and yet to bind the other party to obligations, which, but
for those stipulations, would not have been entered into.
It is not disputed that the insured failed to reveal before the loss three other insurances. By reason of
said unrevealed insurances, the insured had been guilty of a false declaration; a clear misrepresentation
and a vital one because where the insured had been asked to reveal but did not, that was deception.
Otherwise stated, had the insurer known that there were many co-insurances, it could have hesitated or
plainly desisted from entering into such contract. Hence, the insured was guilty of clear fraud.
As the insurance policy against fire expressly required that notice should be given by the insured of
other insurance upon the same property, the total absence of such notice nullifies the policy.
Additionally, insofar as the liability of respondent Reliance is concerned, it is not denied that the
complaint for recovery was filed in court by petitioners only on January 31, 1984, or after more than one
(1) year had elapsed from petitioners' receipt of the insurers' letter of denial on November 29, 1982.

By: Elaine Marie G. Laceda

136

INSURANCE LAW

The condition contained in an insurance policy that claims must be presented within one year after
rejection is not merely a procedural requirement but an important matter essential to a prompt
settlement of claims against insurance companies as it demands that insurance suits be brought by the
insured while the evidence as to the origin and cause of destruction have not yet disappeared.
Geagonia vs. Court of Appeals
G.R. No. 114427 February 6, 1995
Geagonia is the owner of Norman's Mart located in the public market of San Francisco, Agusan del Sur.
On 22 Dec 1989, he obtained from the private respondent fire insurance policy for P100,000.00. The
period of the policy was from 22 Dec 1989 to 22 Dec 1990 and covered the ff: "Stock-in-trade consisting
principally of dry goods such as RTW's for men and women wear and other usual to assured's business.
The policy contained the following condition:
"3. The insured shall give notice to the Company of any insurance or insurances already
effected, or which may subsequently be effected, covering any of the property or properties
consisting of stocks in trade, goods in process and/or inventories only hereby insured, and
unless notice be given and the particulars of such insurance or insurances be stated therein or
endorsed in this policy pursuant to Section 50 of the Insurance Code, by or on behalf of the
Company before the occurrence of any loss or damage, all benefits under this policy shall be
deemed forfeited, provided however, that this condition shall not apply when the total insurance
or insurances in force at the time of the loss or damage is not more than P200,000.00."
On 27 May 1990, fire of accidental origin broke out at around 7:30 p.m. at the public market of San
Francisco, Agusan del Sur. The petitioner's insured stocks-in-trade were completely destroyed prompting
him to file w/ the private respondent a claim under the policy. On 28 Dec 1990, the private respondent
denied the claim because it found that at the time of the loss the petitioner's stocks-in-trade were
likewise covered by two fire insurance policies for P100,000.00 each, issued by the Cebu Branch of the
Philippines First Insurance Co., Inc. (PFIC).
The basis of the private respondent's denial was the petitioner's alleged violation of Condition 3 of the
policy.
Geagonia then filed a complaint against the private respondent w/ the Insurance Commission for the
recovery of P100,000.00 under fire insurance policy, for attorney's fees, and costs of litigation. He
claims that the time he obtained the private respondent's fire insurance policy he knew that the two
policies issued by the PFIC were already in existence; however, he had no knowledge of the provision in
the private respondent's policy requiring him to inform it of the prior policies; this requirement was not
mentioned to him by the private respondent's agent; and had it been so mentioned, he would not have
withheld such information. He further asserted that the total of the amounts claimed under the three
policies was below the actual value of his stocks at the time of loss, w/c was P1M.
The Insurance Commission found that the petitioner did not violate Condition 3 as he had no knowledge
of the existence of the two fire insurance policies obtained from the PFIC; that it was Cebu Tesing
Textiles w/c procured the PFIC policies w/o informing him or securing his consent; and that Cebu Tesing
Textile, as his creditor, had insurable interest on the stocks. These findings were based on the
petitioner's testimony that he came to know of the PFIC policies only when he filed his claim with the
private respondent and that Cebu Tesing Textile obtained them and paid for their premiums w/o
informing him. The Insurance Commission then ordered the respondent company to pay complainant
the sum of P100,000.00 with legal interest from the time the complaint was filed until fully satisfied plus
the amount of P10,000.00 as attorney's fees.
CA reversed the decision of the Insurance Commission because it found that the petitioner knew of the
existence of the two other policies issued by the PFIC

By: Elaine Marie G. Laceda

137

INSURANCE LAW

Issue: WON the petitioner had prior knowledge of the two insurance policies issued by the PFIC when he
obtained the fire insurance policy from the private respondent, thereby, for not disclosing such fact,
violating Condition 3 of the policy
Held: Yes.
We agree w/ the CA that the petitioner knew of the prior policies issued by the PFIC. His letter of 18
January 1991 to the private respondent conclusively proves this knowledge. His testimony to the
contrary before the Insurance Commissioner and which the latter relied upon cannot prevail over a
written admission made ante litem motam. It was, indeed, incredible that he did not know about the
prior policies since these policies were not new or original.
Issue: WON he is precluded from recovering therefrom
Held: No
It must, however, be underscored that unlike the "other insurance" clauses involved in General
Insurance and Surety Corp. vs. Ng Hua or in Pioneer Insurance & Surety Corp. vs. Yap, which read:
"The insured shall give notice to the company of any insurance or insurances already effected, or which
may subsequently be effected covering any of the property hereby insured, and unless such notice be
given and the particulars of such insurance or insurances be stated in or endorsed on this Policy by or on
behalf of the Company before the occurrence of any loss or damage, all benefits under this Policy shall
be forfeited." or in the 1930 case of Santa Ana vs. Commercial Union Assurance Co.
which provided
"that any outstanding insurance upon the whole or a portion of the objects thereby assured must be
declared by the insured in writing and he must cause the company to add or insert it in the policy,
without which such policy shall be null and void, and the insured will not be entitled to indemnity in case
of loss," Condition 3 in the private respondent's policy No. F-14622 does not absolutely declare void any
violation thereof. It expressly provides that the condition "shall not apply when the total insurance or
insurances in force at the time of the loss or damage is not more than P200,000.00."
Interpretation: It is a cardinal rule on insurance that a policy or insurance contract is to be interpreted
liberally in favor of the insured and strictly against the company, the reason being, undoubtedly, to
afford the greatest protection which the insured was endeavoring to secure when he applied for
insurance. It is also a cardinal principle of law that forfeitures are not favored and that any construction
which would result in the forfeiture of the policy benefits for the person claiming thereunder, will be
avoided, if it is possible to construe the policy in a manner which would permit recovery, as, for
example, by finding a waiver for such forfeiture.
Stated differently, provisions, conditions or
exceptions in policies which tend to work a forfeiture of insurance policies should be construed most
strictly against those for whose benefits they are inserted, and most favorably toward those against
whom they are intended to operate. The reason for this is that, except for riders which may later be
inserted, the insured sees the contract already in its final form and has had no voice in the selection or
arrangement of the words employed therein. On the other hand, the language of the contract was
carefully chosen and deliberated upon by experts and legal advisers who had acted exclusively in the
interest of the insurers and the technical language employed therein is rarely understood by ordinary
laymen.
With these principles in mind, we are of the opinion that Condition 3 of the subject policy is not totally
free from ambiguity and must be meticulously analyzed. Such analysis leads us to conclude that (a) the
prohibition applies only to double insurance, and (b) the nullity of the policy shall only be to the extent
exceeding P200,000.00 of the total policies obtained.
Furthermore, by stating within Condition 3 itself that such condition shall not apply if the total insurance
in force at the time of loss does not exceed P200,000.00, the private respondent was amenable to
assume a co-insurer's liability up to a loss not exceeding P200,000.00. What it had in mind was to
discourage over-insurance. Indeed, the rationale behind the incorporation of "other insurance" clause in
fire policies is to prevent over-insurance and thus avert the perpetration of fraud. When a property
owner obtains insurance policies from two or more insurers in a total amount that exceeds the
property's value, the insured may have an inducement to destroy the property for the purpose of

By: Elaine Marie G. Laceda

138

INSURANCE LAW

collecting the insurance. The public as well as the insurer is interested in preventing a situation in which
a fire would be profitable to the insured.
3. Other Insurance Clause
a. Alternative forms
b. Rationale
c. Validity
d. Additional insurance
4. Over-Insurance By Double Insurance Sec. 94, I.C.
Sec. 94. Where the insured is overinsured by double insurance:
(a) The insured, unless the policy otherwise provides, may claim payment from the insurers in
such order as he may select, up to the amount for which the insurers are severally liable
under their respective contracts;
(b) Where the policy under which the insured claims is a valued policy, the insured must give
credit as against the valuation for any sum received by him under any other policy without
regard to the actual value of the subject matter insured;
(c) Where the policy under which the insured claims is an unvalued policy he must give credit,
as against the full insurable value, for any sum received by him under any policy;
(d) Where the insured receives any sum in excess of the valuation in the case of valued
policies, or of the insurable value in the case of unvalued policies, he must hold such sum in
trust for the insurers, according to their right of contribution among themselves;
(e) Each insurer is bound, as between himself and the other insurers, to contribute ratably to
the loss in proportion to the amount for which he is liable under his contract.
B.

REINSURANCE

1. Definition Sec. 95, I.C.


Sec. 95. A contract of reinsurance is one by which an insurer procures a third person to insure him
against loss or liability by reason of such original insurance.
Equitable Insurance and Casualty Co., Inc. vs. Rural Insurance and Surety Co., Inc.
G.R. No. L-17436 January 31, 1962
Plaintiff Equitable Insurance file a complaint with the CFI of Manila against defendant Rural Insurance
alleging, as first cause of action, that they entered into a reciprocal facultative reinsurance agreement,
wherein they agreed to cede to each other. Pursuant to said agreement, plaintiff reinsured for P2k with
defendant the stock covered by fire insurance Policy No. 5880 issued by plaintiff which was later burned;
the share of the loss of defendant as per insurance agreement was computed at P2,024 for which
plaintiff sent to defendant a statement of account for payment by the latter. Despite repeated demands
by plaintiff, defendant refused to pay.
On the second cause of action, plaintiff reinsured for P2k with defendant stock covered by fire insurance
Policy No. 6062 which also burned. Again, defendant refused to pay its share of the loss of P1,334 hence
said complaint.
Defendant filed a motion to dismiss on the ground that it states no cause of action, as pursuant to Art
VIII of the Reinsurance Agreement between the parties, before a court action can be brought, the
parties agreed to submit all disputes to a board of arbitrators. The Court denied the motion and required
defendant to answer.
Defendant filed its answer, alleging that the nature of the agreement is self-liquidating between the
parties, the reinsurer becoming the reinsured and vice versa; and that said agreement has not yet been
abrogated so the liability of either to the other is not yet known. Defendant prayed that the complaint be
dismissed and plaintiff filed a motion for judgment on the pleadings which the court denied.

By: Elaine Marie G. Laceda

139

INSURANCE LAW

Instead of going into a formal hearing, the parties submitted their case for decision stipulating the ff
facts: defendant admits the allegations of the complaint and that plaintiff admits that the issues of the
complaint were not submitted to a Board of Arbitrators as provided in par VIII of the complaint, but
instead referred it to the Insurance Commissioner. The CFI rendered judgment in favor of plaintiff.
Hence this appeal.
Issue: WON Equitable had no cause of action as the matter was not referred to the decision of
arbitrators
Held: No
The requirement of submission for decision to 2 arbitrators or an umpire the matter of losses by fire or
the liability of the parties thereto under Art VIII of the agreement arises only if the same is disputed by
one of the parties. In the instant case, there is no dispute between the parties; in the stipulation of facts
defendant admitted that plaintiff had paid its liability and defendant likewise admitted that it ignored
plaintiffs demands for reimbursement for defendants failure to pay its share as reinsurer. As held in
Maligad v United Assurance Co., if in the course of the settlement of a loss, the action of the company or
its agents amounts to refusal to pay, the company will be deemed to have waived the condition
precedent with reference to arbitration and a suit upon the policy will lie.
Issue: WON in a facultative obligation the right to choose an alternative remedy lies only with the debtor
(here the defendant) under Art 1206
Held: No
There is no connection between Art 1206 NCC and the agreement of this action. The term facultative is
used in reinsurance contracts, and it is so used in this particular case, merely to define the right of the
reinsurer to accept or not to accept participation in the risk insured. But once the share is accepted, as it
was in the case at bar, the obligation is absolute and the liability assumed thereunder can be discharged
by only one waythe payment of the share of the losses.
Phil. American Life Ins. Co. vs. Auditor General
G.R. No. L-19255 January 18, 1968
Philamlife, a domestic life insurance corp., and American International Reinsurance Company (Airco), a
corporation organized under the laws of the Republic of Panama, entered into a REINSURANCE TREATY
wherein Philamlife agrees to reinsure with Airco on January 1950. Philamlife agreed to pay premiums for
all reinsurances on an annual premium basis.
In July 16, 1959, the Margin Law was approved and became effective, which exempts certain
obligations from payment of margin fees, particularly contractual obligations calling for payment of
foreign exchange issued, approved and outstanding as of the date this Act takes place.
Central Bank of the Philippines collected P268,747.48 as foreign exchange margin on Philamlife
remittances to Airco purportedly totalling $610,998.63 and made subsequent to July 16, 1959.
Philamlife filed a claim for refund on the ground that the reinsurance premiums remitted were paid in
pursuant to the January 1950 reinsurance treaty, and therefore exempted.
Monetary Board exempted Philamlife from payment of margin fee. However, Auditor of CB refused to
pass in audit Philamlifes claim for refund. Philamlife sought reconsideration but was denied, saying
reinsurance treaty NOT EXEMPTED.
Issue: WON the premia remitted were in pursuance of the reinsurance treaty between Philamlife and
Airco of January 1959, a contract antedating the Margin Law, and therefore, Philamlife exempted from
paying margin fee
Held: No

By: Elaine Marie G. Laceda

140

INSURANCE LAW

For an exemption to come into play, there must be a reinsurance policy or, as in the reinsurance treaty
provided, a "reinsurance cession" which may be automatic or facultative.
Ratio: A reinsurance policy is thus a contract of indemnity one insurer makes with another to protect the
first insurer from a risk it has already assumed. . . . In contradistinction, a reinsurance treaty is merely
an agreement between two insurance companies whereby one agrees to cede and the other to accept
reinsurance business pursuant to provisions specified in the treaty. The practice of issuing policies by
insurance companies includes, among other things, the issuance of reinsurance policies on standard
risks and also on substandard risks under special arrangements. The lumping of the different
agreements under a contract has resulted in the term known to the insurance world as 'treaties.' Such a
treaty is, in fact, an agreement between insurance companies to cover the different situations described.
Reinsurance treaties and reinsurance policies are not synonymous. Treaties are contracts for insurance;
reinsurance policies or cessions are contracts of insurance.
Even if reinsurance treaty preceded the Margin Law by over nine years, nothing in the treaty obligates
Philamlife to remit to Airco a fixed, certain, and obligatory sum by way of reinsurance premiums. The
reinsurance treaty per se cannot give rise to a contractual obligation for the payment of foreign
exchange. Philamlifes obligation to remit reinsurance premiums becomes fixed and definite upon the
execution of the reinsurance cession. It is only after a reinsurance cession is made that payment of
reinsurance premium may be exacted, as it is only after Philamlife seeks to remit that reinsurance
premium that the obligation to pay the margin fee arises.
Issue: WON Margin Law impairs the obligation of contract
Held: No
Existing laws form part of the contract "as the measure of the obligation to perform them by the one
party and the right acquired by the other. If the obligation does not inhere and subsist in the contract
itself, propio vigore, but in the law applicable to the contract.
When petitioner entered into the reinsurance treaty of January 1, 1950 with Airco, it did so with the
understanding that the municipal laws of the Philippines at the time said treaty was executed, became
an unwritten condition thereof. Such municipal laws constitute part of the obligation of contract.
Rationale of Margin Law: to reduce the excessive demand on and prevent further decline of our
international reserves; to provide the Central Bank with an additional instrument for effectively coping
with the problem and achieving domestic and international stability of our currency; to reduce the
excessive demand-for foreign exchange.
Issue: WON reinsurance contracts abroad would be made impractical by the imposition of the 25%
margin fee
Held: No
First, there is no concrete evidence that such imposition of the 25% margin fee is unreasonable, Second,
if really continuance of the existing reinsurance treaty becomes unbearable, that contract itself provides
that petitioner may potestatively write finis thereto on ninety days' written notice. Petitioner is not
forced to continue its reinsurance treaty indefinitely with Airco.
Fieldmens Insurance Co., Inc. vs. Asian Surety & Insruance Co.
G.R. No. L-23447 July 31, 1970
On various dates between April 11, 1960 and Jan. 9, 1961 the Asian Surety & Insurance Company,
Inc. and the Fieldmen's Insurance Company, Inc. entered into 7 reinsurance agreements under which
the former, as the ceding company undertook to cede to the latter, as the reinsuring company, a
specified portion of the amount of insurance underwritten by ASIAN upon payment to FIELDMEN'S of a
proportionate share of the gross rate of the premium applicable with respect to each cession after
deducting a commission. Said agreements were to take effect from certain specific dates and were to be

By: Elaine Marie G. Laceda

141

INSURANCE LAW

in force until cancelled by either party upon previous notice of at least 3 months by registered mail to
the other party, the cancellation to take effect as of Dec. 31 of the year in which the notice was given.
On Sep. 19, 1961 FIELDMEN'S, by means of registered mail, served notice to ASIAN of the former's
desire to be relieved from all participation in its various agreements with the latter effective Dec. 31,
1961. This communication, although admittedly received by ASIAN on Sep. 25, 1961, did not elicit any
reply from ASIAN.
On Dec. 7, 1961 FIELDMEN'S sent another letter to ASIAN expressing regrets at alleged violations
committed by the latter with respect to the various agreements between them and reiterated its position
that it would consider itself "no longer at risk for any reinsurance and/or cession" given by ASIAN which
might be in force on Dec. 31, 1961. Not having received any formal reply from ASIAN, FIELDMEN'S sent
a new a letter on Feb. 17, 1962 reminding ASIAN of the cancellation of all the reinsurance treaties and
cessions as of Dec. 31, 1961 and requested ASIAN to submit its final accounting of all cessions made to
the former for the preceding months when the reinsurance agreements were in force.
Meanwhile one of the risks reinsured with FIELDMEN'S issued in favor of the GSIS became a liability
when the insured property was burned on February 16, 1962. Since the policy was issued on July 1,
1961, it was supposed to expire on July 1, 1962. 2 The next day, Feb. 17, ASIAN immediately notified
FIELDMEN'S of said fire loss.
FIELDMEN'S, relying on the sufficiency of its notice of termination dated September 19, 1961 and
obviously bent on avoiding its liability under the reinsurance agreements with ASIAN, filed a petition for
declaratory relief with the CFI of Manila to seek a declaration that all the reinsurance contracts entered
into between them had terminated as of December 31, 1961 and to obtain an order directing ASIAN to
render final accounting of the transactions between them with respect to said reinsurance treaties as of
the cut-off date.
In its answer below ASIAN denied having received FIELDMEN'S letter dated Sep 19, 1961, and argued
that even assuming it did, FIELDMEN'S could not have terminated the reinsurance treaties as of Dec 31,
1961 because the letter was merely an expression of FIELDMEN'S desire to cancel the treaties and not a
formal notice of cancellation as contemplated in their reinsurance agreements. By way of special defense
Asian contended that even if the Sep. 19 letter were considered sufficient notice of cancellation
thereby rendering the reinsurance agreements terminated as of December 31, 1961 the liability of
FIELDMEN'S with respect to policies or cessions issued under two of the said agreements prior to their
cancellation continued to have full force and effect until the stated expiry dates of such policies or
cessions.
On Dec. 4, 1962, the trial court declared 6 of the 7 reinsurance agreements in question cancelled as of
Dec 31, 1961. At the same time, it upheld ASIAN'S position that all cessions of reinsurance made by it
to FIELDMEN'S prior to the cancellation of the reinsurance treaties continued in full force and effect until
expiry dates and ordered FIELDMEN'S to make an accounting of its business transactions with ASIAN
within 30 days.
On appeal to the CA, the decision of the trial court was substantially affirmed, with the slight
modification that the order for accounting was eliminated, without prejudice to the filing of a proper
action between the parties for that purpose.
Issue: WON the cancellation as of Dec. 31, 1961 of the reinsurance treaties had the effect of terminating
also the liability of FIELDMEN'S as reinsurer with respect to policies or cessions issued prior to the
termination of the principal reinsurance contracts or treaties
Held: NO to the 2 reinsurance contracts.
Of the 6 reinsurance contracts, 2 contain provisions, which clearly and expressly recognize the
continuing effectivity of policies ceded under them for reinsurance notwithstanding the cancellation of
the contracts themselves. The said treaties provide "that in the event of termination of this
Agreement . . ., the liability of the Fieldmen's under current cessions shall continue in full force and

By: Elaine Marie G. Laceda

142

INSURANCE LAW

effect until their natural expiry . . .;" and the 4th paragraph of Article VI of the Personal Accident
Reinsurance Treaty states:
"4. On the termination of this Agreement from any cause whatever, the liability of the
REINSURER (Fieldmen's) under any current cession including any amounts due to be ceded
under the terms of this Agreement and which are not cancelled in the ordinary course of
business shall continue in full force until their expiry unless the COMPANY (Asian) shall, prior to
the thirty-first December next following such notice, elect to withdraw the existing cessions . . ."
Thus, insofar as the two reinsurance agreements are concerned, there is clearly no merit in FIELDMEN'S
claim that their cancellation carried with it ipso facto the termination of all reinsurance cessions
thereunder. Such cessions continued to be in force until their respective dates of expiration. Since it was
under one of said agreements that the reinsurance cession corresponding to the GSIS policy had been
made, FIELDMEN'S cannot avoid liability which arose by reason of the burning of the insured property.
With respect to the other 4 agreements, it would seem that the petition for declaratory relief is moot,
and that no useful purpose would be served by defining the respective rights and obligations of the
parties thereunder. The said agreements have been cancelled, and it does not appear that any claim by
or liability in favor of the insured has actually arisen under any of the reinsurance cessions made prior to
such cancellation. Future conflicts of the same nature as those which have motivated the present action
can of course be obviated by using more precise and definite terminology in the reinsurance agreements
which the parties may enter into henceforth.

2.

Distinguish from Double Insurance


3. When Required Secs. 215-222 and 275, I.C.
Sec. 215. No insurance company other than life, whether foreign or domestic, shall retain any risk on
any one subject of insurance in an amount exceeding twenty per centum of its net worth. For purposes
of this section, the term "subject of insurance" shall include all properties or risks insured by the same
insurer that customarily are considered by non-life company underwriters to be subject to loss or
damage from the same occurrence of any hazard insured against.
Reinsurance ceded as authorized under the succeeding title shall be deducted in determining the
risk retained. As to surety risk, deduction shall also be made of the amount assumed by any other
company authorized to transact surety business and the value of any security mortgage, pledged, or
held subject to the surety's control and for the surety's protection.
Sec. 216. An insurance company doing business in the Philippines may accept reinsurances only of such
risks, and retain risk thereon within such limits, as it is otherwise authorized to insure.
Sec. 217. No insurance company doing business in the Philippines shall cede all or part of any risks
situated in the Philippines by way of reinsurance directly to any foreign insurer not authorized to do
business in the Philippines unless such foreign insurer or, if the services of a non-resident broker are
utilized, such non-resident broker is represented in the Philippines by a resident agent duly registered
with the Commissioner as required in this Code.
The resident agent of such unauthorized foreign insurer or non- resident broker shall
immediately upon registration furnish the Commissioner with the annual statement of such insurer, or of
such company or companies where such broker may place Philippine business as of the year preceding
such registration, and annually thereafter as soon as available.
Sec. 218. All insurance companies, both life and non-life, authorized to do business in the Philippines
shall cede their excess risks to other companies similarly authorized to do business in the Philippines in
such amounts and under such arrangements as would be consistent with sound underwriting practices
before they enter into reinsurance arrangements with unauthorized foreign insurers.
Sec. 219. Any insurance company doing business in the Philippines desiring to cede their excess risks
to foreign insurance or reinsurance companies not authorized to transact business in the Philippines may
do so under the following conditions:

By: Elaine Marie G. Laceda

143

INSURANCE LAW

(1) Except in facultative reinsurance and excess of loss covers, the full amount of the reserve
fund required by law shall be set up in the books of and held by the ceding company for so long as the
risk concerned is in force: Provided, That in case of facultative insurance, the ceding company shall
show to the satisfaction of the Commissioner that the Philippine market cannot provide the facilities
sought abroad.
(2) The reserve fund withheld shall be invested in bonds or other evidences of debt of the
Government of the Philippines or its political subdivisions or instrumentalities, or of government-owned
or controlled corporations and entities, including the Central Bank, and/or other securities acceptable
under section two hundred.
Should any reinsurance agreement be for any reason cancelled or terminated, the ceding
company concerned shall inform the Commissioner in writing of such cancellation or termination within
thirty days from the date of such cancellation or termination or from the date notice or information of
such cancellation or termination is received by such company as the case may be.
Sec. 220. Every insurance company authorized to do business in the Philippines shall report to the
Commissioner on forms prescribed by him the particulars of reinsurance treaties as of the first day of
January of the year following the approval of this Code and shall thereafter similarly report to the
Commissioner particulars of any new treaties or changes in existing treaties.
Sec. 221. No credit shall be allowed as an admitted asset or as a deduction from liability, to any ceding
insurer for reinsurance made, ceded, renewed, or otherwise becoming effective after January first,
nineteen hundred seventy-five, unless the reinsurance shall be payable by the assuming insurer on the
basis of the liability of the ceding insurer under the contract or contracts reinsured without diminution
because of the insolvency of the ceding insurer nor unless under the contract or contracts of reinsurance
the liability for such reinsurance is assumed by the assuming insurer or insurers as of the same effective
date; nor unless the reinsurance agreement provides that payments by the assuming insurer shall be
made directly to the ceding insurer or to its liquidator, receiver, or statutory successor except (a) where
the contract specifically provides another payee of such reinsurance in the event of the insolvency of the
ceding insurer and (b) where the assuming insurer with the consent of the direct insured or insureds has
assumed such policy obligations of the ceding insurer as direct obligations of the assuming insurer to the
payees under such policies and in substitution for the obligations of the ceding insurer to such payees.
Sec. 222. No life insurance company doing business in the Philippines shall reinsure its whole risk on
any individual life or joint lives, or substantially all of its insurance in force, without having first obtained
the written permission of the Commissioner.
Sec. 275. Every foreign insurance company desiring to withdraw from the Philippines shall, prior to
such withdrawal, discharge its liabilities to policyholders and creditors in this country. In case of its
policies insuring residents of the Philippines, it shall cause the primary liabilities under such policies to
be reinsured and assumed by another insurance company authorized to transact business in the
Philippines. In the case of such policies as are subject to cancellation by the withdrawing company, it
may cancel such policies pursuant to the terms thereof in lieu of such reinsurance and assumption of
liabilities.
4. Matters to be Communicated by Reinsured Sec. 96, I.C.
Sec. 96. Where an insurer obtains reinsurance, except under automatic reinsurance treaties, he must
communicate all the representations of the original insured, and also all the knowledge and information
he possesses, whether previously or subsequently acquired, which are material to the risk.
5. Reinsurers Extent of Liability Sec. 87, I.C.
Sec. 87. An insurer is not liable for a loss caused by the willful act or through the connivance of the
insured; but he is not exonerated by the negligence of the insured, or of the insurance agents or others.
6. Original Insured No Interest in Reinsurance Sec. 98, I.C.
Sec. 98. The original insured has no interest in a contract of reinsurance.

By: Elaine Marie G. Laceda

144

INSURANCE LAW

Artex Development Co., Inc. vs. Wellington Insurance Co., Inc.


G.R. No. L-29508 June 27, 1973
Wellington Insurance Co. Inc. insured for P24,346,509.00 the buildings, stocks and machinery of
plaintiff Artex Development Co. Inc. against loss or damage by fire or lighting upon payment of the
plaintiff of the corresponding premiums; that said properties were insured for an additional sum of
P883,034.00; that defendant insured plaintiff against business interruption (use and occupancy) for
P5,200,000.00; Wellington entered into a contract of reinsurance with Alexander and Alexander, Inc. of
New York. USA.
The buildings, stocks and machineries of plaintiffs spinning department were burned. Notice of the loss
and damage was given the defendant. As per report of the adjusters, the total property loss of the
plaintiff was the sum of P10,106,554.40 and the total business interruption loss was P3,000,000.00. The
defendant has paid to the plaintiff the sum of P6,481,870.07 of the property loss suffered by plaintiff
and P1,864,134.08 on its business interruption loss, leaving a balance of P3,624,683.43 and
P1,748,460.00, respectively.
The counsel for Artex filed a Manifestation saying that in view of the Deeds of Discharge and Collateral
Agreement, the only remaining liability subject of litigation shall be the proportion of the loss reinsured
with or through Alexander and Alexander, Inc. of New York, USA, namely, P397,813.00.
The document recited further that Artex acknowledges receipt of the sum of P3.6M paid by the insurer in
full and final settlement of all or any claims of Artex against its insurer. It discharges its insurer from all
actions, proceedings, claims, demands, costs and expenses in respect thereof.
With regard the balance unpaid, Wellington contends that Artex should have been directed against the
reinsurers to cover the liability and not against Wellington.
Issue: WON the insured (Artex) has a cause of action against the reinsurer
Held: No
Unless there is a specific grant in, or assignment of, the reinsurance contract in favor of the insured or a
manifest intention of the contracting parties to the reinsurance contract to grant such benefit or favor to
the insured, the insured, not being privy to the reinsurance contract, has no cause of action against the
reinsurer. It is expressly provided in Section 91 the Insurance Act 1 that "(T)he original insured has no
interest in a contract of insurance."
Coquia vs. Fieldmen's Insurance Co., Inc.
G.R. No. L-23276 November 29, 1968
On December 1, 1961, appellant Fieldmen's Insurance Company, Inc. issued, in favor of the Manila
Yellow Taxicab Co., Inc. a common carrier accident insurance policy, covering the period from December
1, 1961 to December 1, 1962. Under the policy, the Insurer agreed indemnify the Insured in the event
of accident caused by or arising out of the use of Motor Vehicle against all sums which the Insured will
become legally liable to pay in respect of: Death or bodily injury to any fare-paying passenger including
the Driver, Conductor and/or Inspector who is riding in the Motor Vehicle insured at the time of accident
or injury.
While the policy was in force a taxicab of the Insured, driven by Carlito Coquia, met a vehicular accident
at Mangaldan, Pangasinan, in consequence of which Carlito died. The Insured filed therefor a claim for
P5,000.00 to which the Company replied with an offer to pay P2,000.00, by way of compromise. The
Insured rejected the same and made a counter-offer for P4,000.00, but the Company did not accept it.
Thus, the Insured and Carlito's parents filed a complaint against the Company to collect the proceeds of
the aforementioned policy. CFI rendered a decision sentencing the Company to pay to the plaintiffs the
sum of P4,000.00 and the costs.
Issue: WON the Coquias have a cause of action against the Company.

By: Elaine Marie G. Laceda

145

INSURANCE LAW

Held: Yes.
Although, in general, only parties to a contract may bring an action based thereon, this rule is subject to
exceptions, one of which is found in the second paragraph of Article 1311 of the Civil Code of the
Philippines, reading: If a contract should contain some stipulation in favor of a third person, he may
demand its fulfillment provided he communicated his acceptance to the obligor before its revocation. A
mere incidental benefit or interest of a person is not sufficient. The contracting parties must have clearly
and deliberately conferred a favor upon a third person.
Pursuant to the contract, the Company "will indemnify any authorized Driver who is driving the Motor
Vehicle" of the Insured and, in the event of death of said driver, the Company shall, likewise, "indemnify
his personal representatives." In fact, the Company "may, at its option, make indemnity
payable directly to the claimants or heirs of claimants ... it being the true intention of this Policy to
protect ... the liabilities of the Insured towards the passengers of the Motor Vehicle and the Public" in
other words, third parties.
Guingon vs. Del Monte
G.R. No. L-22042 August 17, 1967
Julio Aguilar entered into a contract with the Capital Insurance & Surety Co., Inc. insuring the operation
of his jeepneys against accidents with third-party liability. During the effectivity of such insurance policy
Iluminado del Monte, one of the drivers of the jeepneys operated by Aguilar, bumped with the jeepney
of Gervacio Guingon who had just alighted from another jeepney and as a consequence the latter died
some days thereafter. A corresponding information for homicide thru reckless imprudence was filed
against Iluminado del Monte, who pleaded guilty. A penalty of four months imprisonment was imposed
on him.
The heirs of Gervacio Guingon filed an action for damages praying that the sum of P82,771.80 be paid
to them jointly and severally by the defendants, driver Iluminado del Monte, owner and operator Julio
Aguilar, and the Capital Insurance & Surety Co., Inc. Capital Insurance & Surety Co., Inc. answered,
alleging that the plaintiff has no cause of action against it. CFI granted prayer.
Issue: WON the plaintiff could sue the Insurer jointly with the Insured.
Held: Yes.
The policy in the present case is one whereby the insurer agreed to indemnify the insured "against all
sums . . . which the Insured shall become legally liable to pay in respect of: a. death of or bodily injury
to any person . . . ." Clearly, therefore, it is one for indemnity against liability; from the fact then that
the insured is liable to the third person, such third person is entitled to sue the insurer.
The right of the person injured to sue the insurer of the party at fault (insured), depends on whether the
contract of insurance is intended to benefit third persons also or only the insured. And the test applied
has been this: Where the contract provides for indemnity against liability to third persons, then third
persons to whom the insured is liable, can sue the insurer. Where the contract is for indemnity against
actual loss or payment, then third persons cannot proceed against the insurer, the contract being solely
to reimburse the insured for liability actually discharged by him thru payment to third persons, said third
persons' recourse being thus limited to the insured alone.
The policy requires that suit and final judgment be first obtained against the insured; that only
"thereafter" can the person injured recover on the policy; it expressly disallows suing the insurer as a
co-defendant of the insured in a suit to determine the latter's liability. However, the "no action" clause in
the policy of insurance cannot prevail over the Rules of Court provision aimed at avoiding multiplicity of
suits.
Gibson v. Revilla, 92 SCRA 219 (1979)

By: Elaine Marie G. Laceda

146

You might also like